ΠŸΠΎΠΌΠΎΡ‰ΡŒ Π² написании студСнчСских Ρ€Π°Π±ΠΎΡ‚
АнтистрСссовый сСрвис

Π‘ΠΈΠ»Π΅Ρ‚Ρ‹ ΠΏΠΎ Ρ„ΠΈΠ·ΠΈΠΊΠ΅

Π Π΅Ρ„Π΅Ρ€Π°Ρ‚ΠŸΠΎΠΌΠΎΡ‰ΡŒ Π² Π½Π°ΠΏΠΈΡΠ°Π½ΠΈΠΈΠ£Π·Π½Π°Ρ‚ΡŒ ΡΡ‚ΠΎΠΈΠΌΠΎΡΡ‚ΡŒΠΌΠΎΠ΅ΠΉ Ρ€Π°Π±ΠΎΡ‚Ρ‹

ΠŸΡ€ΠΈ ΠΏΠ΅Ρ€Π΅ΠΌΠ΅Ρ‰Π΅Π½ΠΈΠΈ элСктричСского заряда Π² ΡΠ»Π΅ΠΊΡ‚ростатичСском ΠΏΠΎΠ»Π΅ Ρ€Π°Π±ΠΎΡ‚Π° сил Ρ€Π°Π²Π½Π° ΠΏΡ€ΠΎΠΈΠ·Π²Π΅Π΄Π΅Π½ΠΈΡŽ заряда Π½Π° Ρ€Π°Π·Π½ΠΎΡΡ‚ΡŒ ΠΏΠΎΡ‚Π΅Π½Ρ†ΠΈΠ°Π»ΠΎΠ² Π½Π°Ρ‡Π°Π»ΡŒΠ½ΠΎΠΉ ΠΈ ΠΊΠΎΠ½Π΅Ρ‡Π½ΠΎΠΉ Ρ‚ΠΎΡ‡Π΅ΠΊ Ρ‚Ρ€Π°Π΅ΠΊΡ‚ΠΎΡ€ΠΈΠΈ двиТСния заряда. Π’Π°ΠΊ ΠΊΠ°ΠΊ Ρ€Π°Π±ΠΎΡ‚Π° сил элСктростатичСского поля ΠΏΡ€ΠΈ ΠΏΠ΅Ρ€Π΅ΠΌΠ΅Ρ‰Π΅Π½ΠΈΠΈ заряда ΠΈΠ· ΠΎΠ΄Π½ΠΎΠΉ Ρ‚ΠΎΡ‡ΠΊΠΈ пространства Π² Π΄Ρ€ΡƒΠ³ΡƒΡŽ Π½Π΅ Π·Π°Π²ΠΈΡΠΈΡ‚ ΠΎΡ‚ Ρ‚Ρ€Π°Π΅ΠΊΡ‚ΠΎΡ€ΠΈΠΈ двиТСния заряда ΠΌΠ΅ΠΆΡƒ этими Ρ‚ΠΎΡ‡ΠΊΠ°ΠΌΠΈ, Ρ‚ΠΎ Ρ€Π°Π·Π½ΠΎΡΡ‚ΡŒ ΠΏΠΎΡ‚Π΅Π½Ρ†ΠΈΠ°Π»ΠΎΠ² являСтся Π²Π΅Π»ΠΈΡ‡ΠΈΠ½ΠΎΠΉ… Π§ΠΈΡ‚Π°Ρ‚ΡŒ Π΅Ρ‰Ρ‘ >

Π‘ΠΈΠ»Π΅Ρ‚Ρ‹ ΠΏΠΎ Ρ„ΠΈΠ·ΠΈΠΊΠ΅ (Ρ€Π΅Ρ„Π΅Ρ€Π°Ρ‚, курсовая, Π΄ΠΈΠΏΠ»ΠΎΠΌ, ΠΊΠΎΠ½Ρ‚Ρ€ΠΎΠ»ΡŒΠ½Π°Ρ)

Π‘ΠΈΠ»Π΅Ρ‚ 1.

1. 1. ΠœΠ΅Ρ…Π°Π½ΠΈΡ‡Π΅ΡΠΊΠΎΠ΅ Π΄Π²ΠΈΠΆΠ΅Π½ΠΈΠ΅. ΠžΡ‚Π½ΠΎΡΠΈΡ‚Π΅Π»ΡŒΠ½ΠΎΡΡ‚ΡŒ двиТСния. БистСма отсчСта. ΠœΠ°Ρ‚Π΅Ρ€ΠΈΠ°Π»ΡŒΠ½Π°Ρ Ρ‚ΠΎΡ‡ΠΊΠ°. ВраСктория. ΠŸΡƒΡ‚ΡŒ ΠΈ ΠΏΠ΅Ρ€Π΅ΠΌΠ΅Ρ‰Π΅Π½ΠΈΠ΅. Π Π°Π²Π½ΠΎΠΌΠ΅Ρ€Π½ΠΎΠ΅ Π΄Π²ΠΈΠΆΠ΅Π½ΠΈΠ΅. Π‘Π»ΠΎΠΆΠ΅Π½ΠΈΠ΅ скоростСй.

2. 2. Π˜ΡΠΏΠ°Ρ€Π΅Π½ΠΈΠ΅ ТидкостСй. НасыщСнныС ΠΈ Π½Π΅Π½Π°ΡΡ‹Ρ‰Π΅Π½Π½Ρ‹Π΅ ΠΏΠ°Ρ€Ρ‹. Π’Π»Π°ΠΆΠ½ΠΎΡΡ‚ΡŒ Π²ΠΎΠ·Π΄ΡƒΡ…Π°. Π˜Π·ΠΌΠ΅Ρ€ΡŒΡ‚Π΅ ΠΎΡ‚Π½ΠΎΡΠΈΡ‚Π΅Π»ΡŒΠ½ΡƒΡŽ Π²Π»Π°ΠΆΠ½ΠΎΡΡ‚ΡŒ Π²ΠΎΠ·Π΄ΡƒΡ…Π° Π² ΠΊΠ»Π°ΡΡΠ΅.

3. 3. «ΠŸΠžΠ‘Π›Π•Π”ΠžΠ’ΠΠ’Π•Π›Π¬ΠΠžΠ• Π‘ΠžΠ•Π”Π˜ΠΠ•ΠΠ˜Π• ΠŸΠ ΠžΠ’ΠžΠ”ΠΠ˜ΠšΠžΠ’».

1. 1. ΠœΠ΅Ρ…Π°Π½ΠΈΡ‡Π΅ΡΠΊΠΎΠ΅ Π΄Π²ΠΈΠΆΠ΅Π½ΠΈΠ΅ — это ΠΈΠ·ΠΌΠ΅Π½Π΅Π½ΠΈΠ΅ полоТСния Ρ‚Π΅Π»Π° Π² ΠΏΡ€ΠΎΡΡ‚ранствС с Ρ‚Π΅Ρ‡Π΅Π½ΠΈΠ΅ΠΌ Π²Ρ€Π΅ΠΌΠ΅Π½ΠΈ ΠΎΡ‚Π½ΠΎΡΠΈΡ‚Π΅Π»ΡŒΠ½ΠΎ Π΄Ρ€ΡƒΠ³ΠΈΡ… Ρ‚Π΅Π».

Из Π²ΡΠ΅Ρ… ΠΌΠ½ΠΎΠ³ΠΎΠΎΠ±Ρ€Π°Π·Π½Ρ‹Ρ… Ρ„ΠΎΡ€ΠΌ двиТСния ΠΌΠ°Ρ‚Π΅Ρ€ΠΈΠΈ этот Π²ΠΈΠ΄ двиТСния являСтся самым простым.

НапримСр: ΠΏΠ΅Ρ€Π΅ΠΌΠ΅Ρ‰Π΅Π½ΠΈΠ΅ стрСлки часов ΠΏΠΎ Ρ†ΠΈΡ„Π΅Ρ€Π±Π»Π°Ρ‚Ρƒ, ΠΈΠ΄ΡƒΡ‚ люди, ΠΊΠΎΠ»Ρ‹ΡˆΡƒΡ‚ΡΡ Π²Π΅Ρ‚ΠΊΠΈ Π΄Π΅Ρ€Π΅Π²ΡŒΠ΅Π², ΠΏΠΎΡ€Ρ…Π°ΡŽΡ‚ Π±Π°Π±ΠΎΡ‡ΠΊΠΈ, Π»Π΅Ρ‚ΠΈΡ‚ самолСт ΠΈ Ρ‚. Π΄.

ΠžΠΏΡ€Π΅Π΄Π΅Π»Π΅Π½ΠΈΠ΅ полоТСния Ρ‚Π΅Π»Π° Π² Π»ΡŽΠ±ΠΎΠΉ ΠΌΠΎΠΌΠ΅Π½Ρ‚ Π²Ρ€Π΅ΠΌΠ΅Π½ΠΈ являСтся основной Π·Π°Π΄Π°Ρ‡Π΅ΠΉ ΠΌΠ΅Ρ…Π°Π½ΠΈΠΊΠΈ.

Π”Π²ΠΈΠΆΠ΅Π½ΠΈΠ΅ Ρ‚Π΅Π»Π°, ΠΏΡ€ΠΈ ΠΊΠΎΡ‚ΠΎΡ€ΠΎΠΌ всС Ρ‚ΠΎΡ‡ΠΊΠΈ двиТутся ΠΎΠ΄ΠΈΠ½Π°ΠΊΠΎΠ²ΠΎ, называСтся ΠΏΠΎΡΡ‚ΡƒΠΏΠ°Ρ‚Π΅Π»ΡŒΠ½Ρ‹ΠΌ.

* * ΠœΠ°Ρ‚Π΅Ρ€ΠΈΠ°Π»ΡŒΠ½Π°Ρ Ρ‚ΠΎΡ‡ΠΊΠ° — это физичСскоС Ρ‚Π΅Π»ΠΎ, Ρ€Π°Π·ΠΌΠ΅Ρ€Π°ΠΌΠΈ ΠΊΠΎΡ‚ΠΎΡ€ΠΎΠ³ΠΎ Π² Π΄Π°Π½Π½Ρ‹Ρ… условиях двиТСния ΠΌΠΎΠΆΠ½ΠΎ ΠΏΡ€Π΅Π½Π΅Π±Ρ€Π΅Ρ‡ΡŒ, считая, Ρ‡Ρ‚ΠΎ вся Π΅Π³ΠΎ масса сосрСдоточСнны Π² ΠΎΠ΄Π½ΠΎΠΉ Ρ‚ΠΎΡ‡ΠΊΠ΅.

* * ВраСктория — это линия ΠΊΠΎΡ‚ΠΎΡ€ΡƒΡŽ описываСт ΠΌΠ°Ρ‚Π΅Ρ€ΠΈΠ°Π»ΡŒΠ½Π°Ρ Ρ‚ΠΎΡ‡ΠΊΠ° ΠΏΡ€ΠΈ своСм Π΄Π²ΠΈΠΆΠ΅Π½ΠΈΠΈ.

* * ΠŸΡƒΡ‚ΡŒ — это Π΄Π»ΠΈΠ½Π° Ρ‚Ρ€Π°Π΅ΠΊΡ‚ΠΎΡ€ΠΈΠΈ двиТСния ΠΌΠ°Ρ‚Π΅Ρ€ΠΈΠ°Π»ΡŒΠ½ΠΎΠΉ Ρ‚ΠΎΡ‡ΠΊΠΈ.

* * ΠŸΠ΅Ρ€Π΅ΠΌΠ΅Ρ‰Π΅Π½ΠΈΠ΅ — это Π½Π°ΠΏΡ€Π°Π²Π»Π΅Π½Π½Ρ‹ΠΉ ΠΎΡ‚Ρ€Π΅Π·ΠΎΠΊ прямой (Π²Π΅ΠΊΡ‚ΠΎΡ€), ΡΠΎΠ΅Π΄ΠΈΠ½ΡΡŽΡ‰ΠΈΠΉ Π½Π°Ρ‡Π°Π»ΡŒΠ½ΠΎΠ΅ ΠΏΠΎΠ»ΠΎΠΆΠ΅Π½ΠΈΠ΅ Ρ‚Π΅Π»Π° с Π΅Π³ΠΎ ΠΏΠΎΡΠ»Π΅Π΄ΡƒΡŽΡ‰ΠΈΠΌ ΠΏΠΎΠ»ΠΎΠΆΠ΅Π½ΠΈΠ΅ΠΌ.

Π“Π»Π°Π²Π½ΠΎΠ΅ ΠΎΡ‚Π»ΠΈΡ‡ΠΈΠ΅ ΠΏΡƒΡ‚ΠΈ Ρ‚ΠΎ ΠΏΠ΅Ρ€Π΅ΠΌΠ΅Ρ‰Π΅Π½ΠΈΡ.

ΠŸΡƒΡ‚ΡŒ — Π²Π΅Π»ΠΈΡ‡ΠΈΠ½Π° скалярная (Π½Π° Ρ€ΠΈΡ. — ΡΡ‚ΠΎ расстояниС [ММ1], Π° ΠΏΠ΅Ρ€Π΅ΠΌΠ΅Ρ‰Π΅Π½ΠΈΠ΅ — Π²Π΅Π»ΠΈΡ‡ΠΈΠ½Π° вСкторная (Π½Π° Ρ€ΠΈΡ. — ΡΡ‚ΠΎ r).

ΠŸΡƒΡ‚ΡŒ ΠΈ ΠΏΠ΅Ρ€Π΅ΠΌΠ΅Ρ‰Π΅Π½ΠΈΠ΅ ΠΌΠΎΠ³ΡƒΡ‚ ΡΠΎΠ²ΠΏΠ°Π΄Π°Ρ‚ΡŒ Ρ‚ΠΎΠ»ΡŒΠΊΠΎ Ρ‚ΠΎΠ³Π΄Π°, ΠΊΠΎΠ³Π΄Π° Ρ‚Π΅Π»ΠΎ двиТСтся Ρ€Π°Π²Π½ΠΎΠΌΠ΅Ρ€Π½ΠΎ, прямолинСйно.

* * БистСма отсчСта — это: Ρ‚Π΅Π»ΠΎ отсчСта, связанная с Π½ΠΈΠΌ систСма ΠΊΠΎΠΎΡ€Π΄ΠΈΠ½Π°Ρ‚, Π° Ρ‚Π°ΠΊΠΆΠ΅ ΠΏΡ€ΠΈΠ±ΠΎΡ€ для отсчСта Π²Ρ€Π΅ΠΌΠ΅Π½ΠΈ.

ВаТная ΠΎΡΠΎΠ±Π΅Π½Π½ΠΎΡΡ‚ΡŒ ΠΌΠ΅Ρ…. ДвиТСния — Π΅Π³ΠΎ ΠΎΡ‚Π½ΠΎΡΠΈΡ‚Π΅Π»ΡŒΠ½ΠΎΡΡ‚ΡŒ.

* * ΠžΡ‚Π½ΠΎΡΠΈΡ‚Π΅Π»ΡŒΠ½ΠΎΡΡ‚ΡŒ двиТСния — это Π·Π°Π²ΠΈΡΠΈΠΌΠΎΡΡ‚ΡŒ Ρ‚Ρ€Π°Π΅ΠΊΡ‚ΠΎΡ€ΠΈΠΈ, ΠΏΡƒΡ‚ΠΈ, пСрСмСщСния ΠΈ ΡΠΊΠΎΡ€ΠΎΡΡ‚ΠΈ ΠΎΠ΄Π½ΠΎΠΉ ΠΈ Ρ‚ΠΎΠΉ ΠΆΠ΅ ΠΌΠ°Ρ‚Π΅Ρ€ΠΈΠ°Π»ΡŒΠ½ΠΎΠΉ Ρ‚ΠΎΡ‡ΠΊΠΈ ΠΎΡ‚ Π²Ρ‹Π±ΠΎΡ€Π° систСмы отсчСта.

НапримСр: Π²Ρ‹ ΡΠΈΠ΄ΠΈΡ‚Π΅ Π² Π½Π΅ΠΏΠΎΠ΄Π²ΠΈΠΆΠ½ΠΎΠΌ Π²Π°Π³ΠΎΠ½Π΅, Π° Ρ€ΡΠ΄ΠΎΠΌ Π½Π°Ρ‡ΠΈΠ½Π°Π΅Ρ‚ Π΄Π²ΠΈΠΆΠ΅Π½ΠΈΠ΅ Π΄Ρ€ΡƒΠ³ΠΎΠΉ ΠΏΠΎΠ΅Π·Π΄. Π’Π°ΠΊ Π²ΠΎΡ‚, Π²Π°ΠΌ каТСтся, Ρ‡Ρ‚ΠΎ двиТСтся ΠΈΠΌΠ΅Π½Π½ΠΎ ваш Π²Π°Π³ΠΎΠ½, Π° Π½Π΅ Ρ‚Π΅, Ρ‡Ρ‚ΠΎ ΠΏΡ€ΠΎΠ΅Π·ΠΆΠ°ΡŽΡ‚ ΠΌΠΈΠΌΠΎ, Π½ΠΎ ΠΏΠΎΡ‚ΠΎΠΌ ΠΌΡ‹ ΠΎΠ±Π½Π°Ρ€ΡƒΠΆΠΈΠ²Π°Π΅ΠΌ, Ρ‡Ρ‚ΠΎ наш состав стоит Π½Π° ΠΌΠ΅ΡΡ‚Π΅.

По ΡΠ²ΠΎΠ΅ΠΌΡƒ Ρ…Π°Ρ€Π°ΠΊΡ‚Π΅Ρ€Ρƒ Π΄Π²ΠΈΠΆΠ΅Π½ΠΈΠ΅ ΠΌΠΎΠΆΠ΅Ρ‚ Π±Ρ‹Ρ‚ΡŒ Ρ€Π°Π²Π½ΠΎΠΌΠ΅Ρ€Π½Ρ‹ΠΌ, ΠΊΠΎΠ³Π΄Π° двиТущаяся Ρ‚ΠΎΡ‡ΠΊΠ° Π² Π»ΡŽΠ±Ρ‹Π΅ Ρ€Π°Π²Π½Ρ‹Π΅ ΠΏΡ€ΠΎΠΌΠ΅ΠΆΡƒΡ‚ΠΊΠΈ Π²Ρ€Π΅ΠΌΠ΅Π½ΠΈ ΠΏΡ€ΠΎΡ…ΠΎΠ΄ΠΈΡ‚ Ρ€Π°Π²Π½Ρ‹Π΅ расстояния, ΠΈ Π΅Π³ΠΎ ΠΏΡ€ΠΎΡ‚ΠΈΠ²ΠΎΠΏΠΎΠ»ΠΎΠΆΠ½ΠΎΡΡ‚ΡŒΡŽ — Π½Π΅Ρ€Π°Π²Π½ΠΎΠΌΠ΅Ρ€Π½Ρ‹ΠΌ (ΠΈΠ»ΠΈ ΠΏΠ΅Ρ€Π΅ΠΌΠ΅Π½Π½Ρ‹ΠΌ).

* * Π Π°Π²Π½ΠΎΠΌΠ΅Ρ€Π½ΠΎΠ΅ прямолинСйноС Π΄Π²ΠΈΠΆΠ΅Π½ΠΈΠ΅ Π½Π°Π·Ρ‹Π²Π°ΡŽΡ‚ Ρ‚Π°ΠΊΠΎΠ΅ происходящСС ΠΏΠΎ ΠΏΡ€ΡΠΌΠΎΠ»ΠΈΠ½Π΅ΠΉΠ½ΠΎΠΉ Ρ‚Ρ€Π°Π΅ΠΊΡ‚ΠΎΡ€ΠΈΠΈ двиТСния ΠΏΡ€ΠΈ ΠΊΠΎΡ‚ΠΎΡ€ΠΎΠΌ ΠΌΠ°Ρ‚Π΅Ρ€ΠΈΠ°Π»ΡŒΠ½Π°Ρ Ρ‚ΠΎΡ‡ΠΊΠ° Π·Π° Π»ΡŽΠ±Ρ‹Π΅ Ρ€Π°Π²Π½Ρ‹Π΅ ΠΏΡ€ΠΎΠΌΠ΅ΠΆΡƒΡ‚ΠΊΠΈ Π²Ρ€Π΅ΠΌΠ΅Π½ΠΈ ΡΠΎΠ²Π΅Ρ€ΡˆΠ°Π΅Ρ‚ Π΅Π΄ΠΈΠ½ΠΈΡ‡Π½Ρ‹Π΅ пСрСмСщСния.

Π’.ΠΊ. ΡΠΊΠΎΡ€ΠΎΡΡ‚ΡŒ постоянная, Ρ‚ΠΎ Π³Ρ€Π°Ρ„ΠΈΠΊ скорости прСдставляСт собой линию, ΠΏΠ°Ρ€Π°Π»Π»Π΅Π»ΡŒΠ½ΡƒΡŽ оси Π²Ρ€Π΅ΠΌΠ΅Π½ΠΈ.

2. Π˜ΡΠΏΠ°Ρ€Π΅Π½ΠΈΠ΅ ТидкостСй. НасыщСнныС ΠΈ Π½Π΅Π½Π°ΡΡ‹Ρ‰Π΅Π½Π½Ρ‹Π΅ ΠΏΠ°Ρ€Ρ‹. Π’Π»Π°ΠΆΠ½ΠΎΡΡ‚ΡŒ Π²ΠΎΠ·Π΄ΡƒΡ…Π°. Π˜Π·ΠΌΠ΅Ρ€ΡŒΡ‚Π΅ ΠΎΡ‚Π½ΠΎΡΠΈΡ‚Π΅Π»ΡŒΠ½ΡƒΡŽ Π²Π»Π°ΠΆΠ½ΠΎΡΡ‚ΡŒ Π²ΠΎΠ·Π΄ΡƒΡ…Π° Π² ΠΊΠ»Π°ΡΡΠ΅.

Π‘ΡƒΡ‰Π΅ΡΡ‚Π²ΡƒΡŽΡ‚ Π΄Π²Π° способа ΠΏΠ΅Ρ€Π΅Ρ…ΠΎΠ΄Π° Тидкости Π² Π³Π°Π·ΠΎΠΎΠ±Ρ€Π°Π·Π½ΠΎΠ΅ состояниС: испарСниС ΠΈ ΠΊΠΈΠΏΠ΅Π½ΠΈΠ΅.

Π˜ΡΠΏΠ°Ρ€Π΅Π½ΠΈΠ΅ происходит с ΠΎΡ‚ΠΊΡ€Ρ‹Ρ‚ΠΎΠΉ, свободной повСрхности, ΠΎΡ‚Π΄Π΅Π»ΡΡŽΡ‰Π΅ΠΉ ΠΆΠΈΠ΄ΠΊΠΎΡΡ‚ΡŒ ΠΎΡ‚ Π³Π°Π·Π°, Π½Π°ΠΏΡ€ΠΈΠΌΠ΅Ρ€ с ΠΏΠΎΠ²Π΅Ρ€Ρ…ности ΠΎΡ‚ΠΊΡ€Ρ‹Ρ‚ΠΎΠ³ΠΎ сосуда, с ΠΏΠΎΠ²Π΅Ρ€Ρ…ности Π²ΠΎΠ΄ΠΎΠ΅ΠΌΠ° ΠΈ Ρ‚. Π΄. Π˜ΡΠΏΠ°Ρ€Π΅Π½ΠΈΠ΅ происходит ΠΏΡ€ΠΈ любой Ρ‚Π΅ΠΌΠΏΠ΅Ρ€Π°Ρ‚ΡƒΡ€Π΅, Π½ΠΎ Π΄Π»Ρ всякой Тидкости ΠΏΠΎΠ²Ρ‹ΡˆΠ΅Π½ΠΈΠ΅ΠΌ Ρ‚Π΅ΠΌΠΏΠ΅Ρ€Π°Ρ‚ΡƒΡ€Ρ‹ ΡΠΊΠΎΡ€ΠΎΡΡ‚ΡŒ Π΅Π³ΠΎ увСличиваСтся. ОбъСм, Π·Π°Π½ΠΈΠΌΠ°Π΅ΠΌΡ‹ΠΉ Π΄Π°Π½Π½ΠΎΠΉ массой вСщСства, ΠΏΡ€ΠΈ испарСнии возрастаСт скачком. Π‘Π»Π΅Π΄ΡƒΠ΅Ρ‚ Ρ€Π°Π·Π»ΠΈΡ‡Π°Ρ‚ΡŒ Π΄Π²Π° основных случая. ΠŸΠ΅Ρ€Π²Ρ‹ΠΉ, ΠΊΠΎΠ³Π΄Π° испарСниС происходит Π² Π·Π°ΠΊΡ€Ρ‹Ρ‚ΠΎΠΌ сосудС ΠΈ Ρ‚Π΅ΠΌΠΏΠ΅Ρ€Π°Ρ‚ΡƒΡ€Π° Π²ΠΎ Π²ΡΠ΅Ρ… Ρ‚ΠΎΡ‡ΠΊΠ°Ρ… ΠΎΠ΄ΠΈΠ½Π°ΠΊΠΎΠ²Π°. Π’Π°ΠΊ, Π½Π°ΠΏΡ€ΠΈΠΌΠ΅Ρ€, испаряСтся Π²ΠΎΠ΄Π° Π²Π½ΡƒΡ‚Ρ€ΠΈ ΠΏΠ°Ρ€ΠΎΠ²ΠΎΠ³ΠΎ ΠΊΠΎΡ‚Π»Π° ΠΈΠ»ΠΈ Ρ‡Π°ΠΉΠ½ΠΈΠΊΠ΅, Π·Π°ΠΊΡ€Ρ‹Ρ‚ΠΎΠΌ ΠΊΡ€Ρ‹ΡˆΠΊΠΎΠΉ, Ссли Ρ‚Π΅ΠΌΠΏΠ΅Ρ€Π°Ρ‚ΡƒΡ€Π° Π²ΠΎΠ΄Ρ‹ ΠΈ ΠΏΠ°Ρ€Π° Π½ΠΈΠΆΠ΅ Ρ‚Π΅ΠΌΠΏΠ΅Ρ€Π°Ρ‚ΡƒΡ€Ρ‹ кипСния. Π’ ΡΡ‚ΠΎΠΌ случаС объСм ΠΎΠ±Ρ€Π°Π·ΡƒΡŽΡ‰Π΅Π³ΠΎΡΡ Π³Π°Π·Π° ΠΎΠ³Ρ€Π°Π½ΠΈΡ‡Π΅Π½ пространством сосуда. Π”Π°Π²Π»Π΅Π½ΠΈΠ΅ ΠΏΠ°Ρ€Π° достигаСт Π½Π΅ΠΊΠΎΡ‚ΠΎΡ€ΠΎΠ³ΠΎ ΠΏΡ€Π΅Π΄Π΅Π»ΡŒΠ½ΠΎΠ³ΠΎ значСния, ΠΏΡ€ΠΈ ΠΊΠΎΡ‚ΠΎΡ€ΠΎΠΌ ΠΎΠ½ Π½Π°Ρ…одится Π² Ρ‚Π΅ΠΏΠ»ΠΎΠ²ΠΎΠΌ равновСсии с ΠΆΠΈΠ΄ΠΊΠΎΡΡ‚ΡŒΡŽ; Ρ‚Π°ΠΊΠΎΠΉ ΠΏΠ°Ρ€ называСтся насыщСнным, Π° Π΅Π³ΠΎ Π΄Π°Π²Π»Π΅Π½ΠΈΠ΅ — ΡƒΠΏΡ€ΡƒΠ³ΠΎΡΡ‚ΡŒΡŽ ΠΏΠ°Ρ€Π°. Π’Ρ‚ΠΎΡ€ΠΎΠΉ случай, ΠΊΠΎΠ³Π΄Π° пространство Π½Π°Π΄ ΠΆΠΈΠ΄ΠΊΠΎΡΡ‚ΡŒΡŽ Π½Π΅Π·Π°ΠΌΠΊΠ½ΡƒΡ‚ΠΎΠ΅; Ρ‚Π°ΠΊ испаряСтся Π²ΠΎΠ΄Π° с ΠΏΠΎΠ²Π΅Ρ€Ρ…ности ΠΏΡ€ΡƒΠ΄Π°. Π—Π΄Π΅ΡΡŒ равновСсиС Π½Π΅ Π΄ΠΎΡΡ‚игаСтся практичСски Π½ΠΈΠΊΠΎΠ³Π΄Π° ΠΈ ΠΏΠ°Ρ€ являСтся нСнасыщСнным, Π° ΡΠΊΠΎΡ€ΠΎΡΡ‚ΡŒ испарСния зависит ΠΎΡ‚ ΠΌΠ½ΠΎΠ³ΠΈΡ… Ρ„Π°ΠΊΡ‚ΠΎΡ€ΠΎΠ². ΠœΠ΅Ρ€ΠΎΠΉ скорости испарСния являСтся количСство вСщСства, ΡƒΠ»Π΅Ρ‚Π°ΡŽΡ‰Π΅Π³ΠΎ Π² Π΅Π΄ΠΈΠ½ΠΈΡ†Ρƒ Π²Ρ€Π΅ΠΌΠ΅Π½ΠΈ с Π΅Π΄ΠΈΠ½ΠΈΡ†Ρ‹ свободной повСрхности Тидкости. Если ΠΆΠΈΠ΄ΠΊΠΎΡΡ‚ΡŒ ΠΈ ΠΏΠ°Ρ€ находятся Π² Ρ€Π°Π²Π½ΠΎΠ²Π΅ΡΠΈΠΈ, Ρ‚ΠΎ ΡΠΊΠΎΡ€ΠΎΡΡ‚ΡŒ испарСния Ρ€Π°Π²Π½Π° Π½ΡƒΠ»ΡŽ. Π’ΠΎΡ‡Π½Π΅Π΅, ΠΎΠ½ΠΎ всС ΠΆΠ΅ происходит, Π½ΠΎ Ρ Ρ‚ΠΎΠΉ ΠΆΠ΅ ΡΠΊΠΎΡ€ΠΎΡΡ‚ΡŒΡŽ ΠΈΠ΄Π΅Ρ‚ Π² ΠΎΠ±Ρ€Π°Ρ‚Π½ΠΎΠΌ процСсс — кондСнсация (ΠΏΠ΅Ρ€Π΅Ρ…ΠΎΠ΄ вСщСства ΠΈΠ· Π³Π°Π·ΠΎΠΎΠ±Ρ€Π°Π·Π½ΠΎΠ³ΠΎ состояния Π² ΠΆΠΈΠ΄ΠΊΠΎΠ΅).

Π’Π΅ΠΏΠ»ΠΎΡ‚Π° испарСния — количСства Ρ‚Π΅ΠΏΠ»Π°, Π½Π΅ΠΎΠ±Ρ…ΠΎΠ΄ΠΈΠΌΠΎΠ΅ ΡΠΎΠΎΠ±Ρ‰ΠΈΡ‚ΡŒ Тидкости, находящСгося ΠΏΡ€ΠΈ Π΄Π°Π½Π½ΠΎΠΉ Ρ‚Π΅ΠΌΠΏΠ΅Ρ€Π°Ρ‚ΡƒΡ€Π΅ ΠΈ Ρ„иксированном Π΄Π°Π²Π»Π΅Π½ΠΈΠΈ, Ρ‡Ρ‚ΠΎΠ±Ρ‹ пСрСвСсти Π΅Π΅ Π² ΠΏΠ°Ρ€ ΠΏΡ€ΠΈ этих ΠΆΠ΅ Ρ‚Π΅ΠΌΠΏΠ΅Ρ€Π°Ρ‚ΡƒΡ€Π΅ ΠΈ Π΄Π°Π²Π»Π΅Π½ΠΈΡ.

Иногда испарСниС Π½Π°Π·Ρ‹Π²Π°ΡŽΡ‚ Ρ‚Π°ΠΊΠΆΠ΅ ΡΡƒΠ±Π»ΠΈΠΌΠ°Ρ†ΠΈΡŽ, ΠΈΠ»ΠΈ Π²ΠΎΠ·Π³ΠΎΠ½ΠΊΡƒ, Ρ‚. Π΅. ΠΏΠ΅Ρ€Π΅Ρ…ΠΎΠ΄ Ρ‚Π²Π΅Ρ€Π΄ΠΎΠ³ΠΎ вСщСства Π² Π³Π°Π·ΠΎΠΎΠ±Ρ€Π°Π·Π½ΠΎΠ΅ состояниС, минуя ΠΆΠΈΠ΄ΠΊΡƒΡŽ ΡΡ‚Π°Π΄ΠΈΡŽ. Π’Π΅ΠΏΠ»ΠΎΡ‚Π° сублимации большС Ρ‚Π΅ΠΏΠ»ΠΎΡ‚Ρ‹ испарСния ΠΏΡ€ΠΈΠ±Π»ΠΈΠ·ΠΈΡ‚Π΅Π»ΡŒΠ½ΠΎ Π½Π° Ρ‚Π΅ΠΏΠ»ΠΎΡ‚Ρƒ плавлСния.

ΠŸΠ΅Ρ€Π΅Π³ΠΎΠ½ΠΊΠ° — это процСсс раздСлСния ΠΌΠ½ΠΎΠ³ΠΎΠΊΠΎΠΌΠΏΠΎΠ½Π΅Π½Ρ‚Π½Ρ‹Ρ… ΠΆΠΈΠ΄ΠΊΠΈΡ… смСсСй ΠΏΡƒΡ‚Π΅ΠΌ частичного испарСния ΠΈ ΠΏΠΎΡΠ»Π΅Π΄ΡƒΡŽΡ‰Π΅ΠΉ кондСнсации ΠΏΠ°Ρ€ΠΎΠ².

ВслСдствиС всСвозмоТных испарСнии содСрТится ΠΎΠ³Ρ€ΠΎΠΌΠ½ΠΎΠ΅ количСство водяных ΠΏΠ°Ρ€ΠΎΠ². Π’Π΅Π»ΠΈΡ‡ΠΈΠ½Ρƒ, ΠΈΠ·ΠΌΠ΅Ρ€ΡΠ΅ΠΌΡƒΡŽ количСством водяного ΠΏΠ°Ρ€Π° (Π² Π³Ρ€Π°ΠΌΠΌΠ°Ρ…), содСрТащСгося Π² 1 ΠΌ³ Π²ΠΎΠ·Π΄ΡƒΡ…Π°. ΠΠ°Π·Ρ‹Π²Π°ΡŽΡ‚ Π°Π±ΡΠΎΠ»ΡŽΡ‚Π½ΠΎΠΉ Π²Π»Π°ΠΆΠ½ΠΎΡΡ‚ΡŒΡŽ Π²ΠΎΠ·Π΄ΡƒΡ…Π°. (ΠΏΠ°Ρ€Ρ†ΠΈΠ°Π»ΡŒΠ½ΠΎΠ΅ Π΄Π°Π²Π»Π΅Π½ΠΈΠ΅ измСряСтся Π² ΠΌΠΌ Ρ€Ρ‚. ст.). ΠΠ±ΡΠΎΠ»ΡŽΡ‚Π½ΠΎΠΉ Π²Π»Π°ΠΆΠ½ΠΎΡΡ‚ΡŒΡŽ Π²ΠΎΠ·Π΄ΡƒΡ…Π° принято Π½Π°Π·Ρ‹Π²Π°Ρ‚ΡŒ ΠΏΠ°Ρ€Ρ†ΠΈΠ°Π»ΡŒΠ½ΠΎΠ΅ Π΄Π°Π²Π»Π΅Π½ΠΈΠ΅ водяного ΠΏΠ°Ρ€, содСрТащСгося Π² Π½Π΅ΠΌ ΠΏΡ€ΠΈ Ρ‚Π΅ΠΌΠΏΠ΅Ρ€Π°Ρ‚ΡƒΡ€Π΅, ΠΈΠ·ΠΌΠ΅Ρ€Π΅Π½Π½ΠΎΠΉ Π² ΠΌΠΌ Ρ€Ρ‚. ст. Для суТдСния ΠΎ ΡΡ‚Π΅ΠΏΠ΅Π½ΠΈ влаТности Π²ΠΎΠ·Π΄ΡƒΡ…Π° Π²Π°ΠΆΠ½ΠΎ Π·Π½Π°Ρ‚ΡŒ, Π±Π»ΠΈΠ·ΠΎΠΊ ΠΈΠ»ΠΈ Π΄Π°Π»Π΅ΠΊ водяной ΠΏΠ°Ρ€, находящийся Π² Π½Π΅ΠΌ, ΠΎΡ‚ ΡΠΎΡΡ‚ояния насыщСнности. Π‘ ΡΡ‚ΠΎΠΉ Ρ†Π΅Π»ΡŒΡŽ вводят понятиС ΠΎΡ‚Π½ΠΎΡΠΈΡ‚Π΅Π»ΡŒΠ½ΠΎΠΉ влаТности. О.Π². — Π²Π΅Π»ΠΈΡ‡ΠΈΠ½Π°, измСряСмая ΠΎΡ‚Π½ΠΎΡˆΠ΅Π½ΠΈΠ΅ΠΌ абс. ВлаТности ΠΊ ΠΊΠΎΠ»ΠΈΡ‡Π΅ΡΡ‚Π²Ρƒ ΠΏΠ°Ρ€Π°, Π½Π΅ΠΎΠ±Ρ…ΠΎΠ΄ΠΈΠΌΠΎΠ³ΠΎ для насыщСния 1 ΠΌ³ Π²ΠΎΠ·Π΄ΡƒΡ…Π° ΠΏΡ€ΠΈ Ρ‚ΠΎΠΉ ΠΆΠ΅ Ρ‚Π΅ΠΌΠΏΠ΅Ρ€Π°Ρ‚ΡƒΡ€Π΅.

Π‘ΠΈΠ»Π΅Ρ‚ 10.

1. 1. Π˜Π΄Π΅Π°Π»ΡŒΠ½Ρ‹ΠΉ Π³Π°Π·. ОсновноС ΡƒΡ€Π°Π²Π½Π΅Π½ΠΈΠ΅ МКВ Π³Π°Π·Π°. ИспользованиС свойств Π³Π°Π·ΠΎΠ² Π² Ρ‚Π΅Ρ…Π½ΠΈΠΊΠ΅.

2. 2. ΠœΠ°Π³Π½ΠΈΡ‚Π½Ρ‹Π΅ свойства вСщСства. Π€Π΅Ρ€Ρ€ΠΎΠΌΠ°Π³Π½Π΅Ρ‚ΠΈΠΊΠΈ ΠΈ ΠΈΡ… ΠΏΡ€ΠΈΠΌΠ΅Π½Π΅Π½ΠΈΠ΅.

3. 3. (Π·Π°Π΄Π°Ρ‡Π° Π½Π° ΠΏΡ€ΠΈΠΌΠ΅Π½Π΅Π½ΠΈΠ΅ Ρ„ΠΎΡ€ΠΌΡƒΠ» мСханичСской Ρ€Π°Π±ΠΎΡ‚Ρ‹).

1. 1. ΠžΡΠ½ΠΎΠ²Π½Ρ‹Π΅ полоТСния ΠœΠΎΠ»Π΅ΠΊΡƒΠ»ΡΡ€Π½ΠΎ кинСтичСской Ρ‚Π΅ΠΎΡ€ΠΈΠΈ ΠΏΠΎΠ·Π²ΠΎΠ»ΡΡŽΡ‚ ΠΎΠ±ΡŠΡΡΠ½ΠΈΡ‚ΡŒ Ρ†Π΅Π»Ρ‹ΠΉ ряд физичСских явлСний ΠΈ ΡΠ²ΠΎΠΉΡΡ‚Π² Ρ‚Π΅Π» Ρ‚Π΅ΠΌ Ρ‚ΠΎΡ‡Π½Π΅Π΅ ΠΈ ΠΏΠΎΠ΄Ρ€ΠΎΠ±Π½Π΅Π΅, Ρ‡Π΅ΠΌ Π±Π»ΠΈΠΆΠ΅ построСнная модСль Π΄Π΅ΠΉΡΡ‚Π²ΠΈΡ‚Π΅Π»ΡŒΠ½ΠΎΠΌΡƒ корпускулярного ΡΡ‚Ρ€ΠΎΠ΅Π½ΠΈΡŽ Ρ‚Π΅Π». МаксимальноС совпадСниС получаСтся Π² ΡΠ»ΡƒΡ‡Π°Π΅ использования ΠΏΡ€ΠΎΡΡ‚Π΅ΠΉΡˆΠΈΡ… систСмы — разряТСнных Π³Π°Π·ΠΎΠ². Π₯ΠΎΡ€ΠΎΡˆΠ΅ΠΉ модСлью разряТСнного Π³Π°Π·Π° являСтся ΠΈΠ΄Π΅Π°Π»ΡŒΠ½Ρ‹ΠΉ Π³Π°Π·, ΠΊΠΎΡ‚ΠΎΡ€Ρ‹ΠΉ Π½Π°Π΄Π΅Π»ΡΡŽΡ‚ ΡΠ»Π΅Π΄ΡƒΡŽΡ‰ΠΈΠΌΠΈ свойствами:

1. 1. Π Π°Π·ΠΌΠ΅Ρ€Ρ‹ ΠΌΠΎΠ»Π΅ΠΊΡƒΠ» ΠΌΠ°Π»Ρ‹ ΠΏΠΎ ΡΡ€Π°Π²Π½Π΅Π½ΠΈΡŽ с Ρ€Π°ΡΡΡ‚ояниями ΠΌΠ΅ΠΆΠ΄Ρƒ Π½ΠΈΠΌΠΈ.

2. 2. ΠœΠΎΠ»Π΅ΠΊΡƒΠ»Ρ‹ Π²Π·Π°ΠΈΠΌΠΎΠ΄Π΅ΠΉΡΡ‚Π²ΡƒΡŽΡ‚ Π΄Ρ€ΡƒΠ³ с Π΄Ρ€ΡƒΠ³ΠΎΠΌ ΠΈ ΡΠΎ ΡΡ‚Π΅Π½ΠΊΠ°ΠΌΠΈ сосуда Ρ‚ΠΎΠ»ΡŒΠΊΠΎ Π² ΠΌΠΎΠΌΠ΅Π½Ρ‚ соударСния.

3. 3. ВсС соударСния Π°Π±ΡΠΎΠ»ΡŽΡ‚Π½ΠΎ ΡƒΠΏΡ€ΡƒΠ³ΠΈ.

4. 4. Π Π°ΡΡΠΌΠ°Ρ‚Ρ€ΠΈΠ²Π°ΡŽΡ‚ΡΡ Π»ΡŽΠ±Ρ‹Π΅ Π³Π°Π·Ρ‹, Π² ΠΊΠΎΡ‚ΠΎΡ€Ρ‹Ρ… число ΠΌΠΎΠ»Π΅ΠΊΡƒΠ» ΠΎΡ‡Π΅Π½ΡŒ Π²Π΅Π»ΠΈΠΊΠΎ.

5. 5. ΠœΠΎΠ»Π΅ΠΊΡƒΠ»Ρ‹ распрСдСлСны ΠΏΠΎ Π²ΡΠ΅ΠΌΡƒ ΠΎΠ±ΡŠΠ΅ΠΌΡƒ Ρ€Π°Π²Π½ΠΎΠΌΠ΅Ρ€Π½ΠΎ.

6. 6. ΠœΠΎΠ»Π΅ΠΊΡƒΠ»Ρ‹ двиТутся Ρ…Π°ΠΎΡ‚ΠΈΡ‡Π½ΠΎ.

7. 7. Бкорости ΠΌΠΎΠ»Π΅ΠΊΡƒΠ» ΠΌΠΎΠ³ΡƒΡ‚ ΠΏΡ€ΠΈΠ½ΠΈΠΌΠ°Ρ‚ΡŒ Π»ΡŽΠ±Ρ‹Π΅ значСния.

8. 8. К Π΄Π²ΠΈΠΆΠ΅Π½ΠΈΡŽ ΠΎΡ‚Π΄Π΅Π»ΡŒΠ½ΠΎΠΉ ΠΌΠΎΠ»Π΅ΠΊΡƒΠ»Ρ‹ ΠΏΡ€ΠΈΠΌΠ΅Π½ΠΈΠΌΡ‹ Π·Π°ΠΊΠΎΠ½Ρ‹ классичСской ΠΌΠ΅Ρ…Π°Π½ΠΈΠΊΠΈ.

ПолоТСниС 1 ΠΈ 2 ΡΠ»Π΅Π΄ΡƒΡŽΡ‚ ΠΈΠ· ΡΠ²Π΅Π΄Π΅Π½ΠΈΠΉ ΠΎ Ρ€Π°Π·ΠΌΠ΅Ρ€Π°Ρ… ΠΈ Π²Π·Π°ΠΈΠΌΠΎΠ΄Π΅ΠΉΡΡ‚виях ΠΌΠΎΠ»Π΅ΠΊΡƒΠ».

ПолоТСниС 3 опираСтся Π½Π° ΡΠ²ΠΎΠΉΡΡ‚Π²Π° броуновского двиТСния, самой ΡƒΠ΄ΠΈΠ²ΠΈΡ‚Π΅Π»ΡŒΠ½ΠΎΠΉ Ρ‡Π΅Ρ€Ρ‚ΠΎΠΉ ΠΊΠΎΡ‚ΠΎΡ€ΠΎΠ³ΠΎ являСтся Π΅Π³ΠΎ постоянство, ΡΠ²ΠΈΠ΄Π΅Ρ‚Π΅Π»ΡŒΡΡ‚Π²ΡƒΡŽΡ‰Π΅Π΅ ΠΎΠ± Π°Π±ΡΠΎΠ»ΡŽΡ‚Π½ΠΎ ΡƒΠΏΡ€ΡƒΠ³ΠΎΠΌ соударСнии ΠΌΠΎΠ»Π΅ΠΊΡƒΠ».

ПолоТСниС 4 ΠΌΠΎΠΆΠ½ΠΎ ΠΏΠΎΡΡΠ½ΠΈΡ‚ΡŒ ΠΏΡ€ΠΈΠΌΠ΅Ρ€ΠΎΠΌ: Π² ΠΌΠΈΠ»Π»ΠΈΠΎΠ½Π½ΠΎΠΉ Π΄ΠΎΠ»Π΅ 1 ΠΌΠΌ3 ΠΊΠΎΠΌΠ½Π°Ρ‚Π½ΠΎΠ³ΠΎ Π²ΠΎΠ·Π΄ΡƒΡ…Π° содСрТится 2,7 * 1010 ΠΌΠΎΠ»Π΅ΠΊΡƒΠ».

ПолоТСния 5 ΠΈ 6 ΠΈΠΌΠ΅ΡŽΡ‚ своС ΠΎΠΏΡ‹Ρ‚Π½ΠΎΠ΅ обоснованиС Π² ΠΎΠ΄ΠΈΠ½Π°ΠΊΠΎΠ²ΠΎΠΌ Π΄Π°Π²Π»Π΅Π½ΠΈΠΈ Π³Π°Π·Π° Π² Ρ€Π°Π·Π½Ρ‹Ρ… мСстах сосуда, Π² ΠΊΠΎΡ‚ΠΎΡ€ΠΎΠΌ ΠΎΠ½ Π½Π°Ρ…одится, Π·Π°ΠΊΠΎΠ½ Паскаля ΠΈ Π±Ρ€ΠΎΡƒΠ½ΠΎΠ²ΡΠΊΠΎΠΌ Π΄Π²ΠΈΠΆΠ΅Π½ΠΈΠΈ. А Π²ΠΎΡ‚ ΡΠΏΡ€Π°Π²Π΅Π΄Π»ΠΈΠ²ΠΎΡΡ‚ΡŒ постулатов 7ΠΈ 8, Π° Ρ‚Π°ΠΊΠΆΠ΅ Π³Ρ€Π°Π½ΠΈΡ†Ρ‹ ΠΈΡ… ΠΏΡ€ΠΈΠΌΠ΅Π½Π΅Π½ΠΈΡ ΠΌΠΎΠΆΠ΅Ρ‚ ΠΏΠΎΠ΄Ρ‚Π²Π΅Ρ€Π΄ΠΈΡ‚ΡŒ Ρ‚ΠΎΠ»ΡŒΠΊΠΎ ΠΊΠΎΠ½Ρ‚Ρ€ΠΎΠ»ΡŒΠ½Ρ‹ΠΉ экспСримСнт. Π‘ Ρ‚ΠΎΡ‡ΠΊΠΈ зрСния соврСмСнной Ρ„ΠΈΠ·ΠΈΠΊΠΈ скорости Ρ‚Π΅Π» Π½Π΅ ΠΌΠΎΠ³ΡƒΡ‚ Π±Ρ‹Ρ‚ΡŒ сколь ΡƒΠ³ΠΎΠ΄Π½ΠΎ большими (ΠΏΡ€Π΅Π΄Π΅Π»ΡŒΠ½ΠΎΠΉ ΡΠΊΠΎΡ€ΠΎΡΡ‚ΡŒΡŽ являСтся ΡΠΊΠΎΡ€ΠΎΡΡ‚ΡŒ свСта Π² Π²Π°ΠΊΡƒΡƒΠΌΠ΅ 3*108 ΠΌ/с). Однако ΠΌΠΎΠ΄Π½ΠΎ ΠΏΡ€Π΅Π΄ΠΏΠΎΠ»ΠΎΠΆΠΈΡ‚ΡŒ, Ρ‡Ρ‚ΠΎ ΠΌΠΎΠ»Π΅ΠΊΡƒΠ» с ΠΎΡ‡Π΅Π½ΡŒ большими ΠΈ ΠΎΡ‡Π΅Π½ΡŒ малСнькими скоростями Π² Π³Π°Π·Π΅ ΠΎΡ‚Π½ΠΎΡΠΈΡ‚Π΅Π»ΡŒΠ½ΠΎ Π½Π΅ΠΌΠ½ΠΎΠ³ΠΎ.

ВнутрСнняя энСргия идСального Π³Π°Π·Π°. Если ΠΏΠΎΡ‚Π΅Π½Ρ†ΠΈΠ°Π»ΡŒΠ½Π°Ρ энСргия взаимодСйствия ΠΌΠΎΠ»Π΅ΠΊΡƒΠ» Ρ€Π°Π²Π½Π° 0, внутрСнняя энСргия идСального Π³Π°Π·Π° Ρ€Π°Π²Π½Π° суммС кинСтичСских энСргий хаотичСского Ρ‚Π΅ΠΏΠ»ΠΎΠ²ΠΎΠ³ΠΎ двиТСния Π²Π΅Ρ… ΠΌΠΎΠ»Π΅ΠΊΡƒΠ»: U=N*E=v*Na*3/2 kT=3m/2M RT.

ВнутрСнняя энСргия идСального Π³Π°Π·Π° ΠΏΡ€ΡΠΌΠΎΠΏΡ€ΠΎΠΏΠΎΡ€Ρ†ΠΈΠΎΠ½Π°Π»ΡŒΠ½ΠΎ Π΅Π³ΠΎ Π°Π±ΡΠΎΠ»ΡŽΡ‚Π½ΠΎΠΉ Ρ‚Π΅ΠΌΠΏΠ΅Ρ€Π°Ρ‚ΡƒΡ€Π΅. Π‘Π»Π΅Π΄ΠΎΠ²Π°Ρ‚Π΅Π»ΡŒΠ½ΠΎ, ΠΏΡ€ΠΈ ΠΈΠ·ΠΌΠ΅Π½Π΅Π½ΠΈΠΈ Ρ‚Π΅ΠΌΠΏΠ΅Ρ€Π°Ρ‚ΡƒΡ€Π° идСального Π³Π°Π·Π° измСняСтся Π΅Π³ΠΎ внутрСнняя энСргия; Ссли Ρ‚Π΅ΠΌΠΏΠ΅Ρ€Π°Ρ‚ΡƒΡ€Π° остаСтся постоянной, Ρ‚ΠΎ Π²Π½ΡƒΡ‚рСнняя энСргия идСального Π³Π°Π·Π° Π½Π΅ ΠΈΠ·ΠΌΠ΅Π½ΡΠ΅Ρ‚ся. Π˜ΡΠΏΠΎΠ»ΡŒΠ·ΡƒΡ ΡƒΡ€Π°Π²Π½Π΅Π½ΠΈΠ΅ состояния идСального Π³Π°Π·Π° ΠΌΠΎΠΆΠ½ΠΎ ΠΏΠΎΠ»ΡƒΡ‡ΠΈΡ‚ΡŒ ΠΎΠ΄Π½ΠΎ Π²Ρ‹Ρ€Π°ΠΆΠ΅Π½ΠΈΠ΅ для вычислСния Π²Π½ΡƒΡ‚Ρ€Π΅Π½Π½Π΅ΠΉ энСргии идСального ΠΎΠ΄Π½ΠΎΠ°Ρ‚ΠΎΠΌΠ½ΠΎΠ³ΠΎ Π³Π°Π·Π°: p V = m/MRT => U=3/2 pV. Π’Π°ΠΊΠΈΠΌ ΠΎΠ±Ρ€Π°Π·ΠΎΠΌ, внутрСнняя энСргия идСального прямо ΠΏΡ€ΠΎΠΏΠΎΡ€Ρ†ΠΈΠΎΠ½Π°Π»ΡŒΠ½Π° ΠΏΡ€ΠΎΠΈΠ·Π²Π΅Π΄Π΅Π½ΠΈΡŽ давлСния Ρ€ Π½Π° ΠΎΠ±ΡŠΠ΅ΠΌ V, Π·Π°Π½ΠΈΠΌΠ°Π΅ΠΌΡ‹ΠΉ Π³Π°Π·ΠΎΠΌ.

Π£Ρ€Π°Π²Π½Π΅Π½ΠΈΠ΅ состояния идСального Π³Π°Π·Π° (ΡƒΡ€Π°Π²Π½Π΅Π½ΠΈΠ΅ МСндСлССва — ΠšΠ»Π°ΠΉΠΏΠ΅Ρ€ΠΎΠ½Π°). БостояниС идСально Π³Π°Π·Π° Ρ…Π°Ρ€Π°ΠΊΡ‚Π΅Ρ€ΠΈΠ·ΡƒΠ΅Ρ‚ Ρ‚Ρ€ΠΈ макроскопичСскиС Π²Π΅Π»ΠΈΡ‡ΠΈΠ½Ρ‹: Ρ€Π΄Π°Π²Π»Π΅Π½ΠΈΠ΅, V — объСм, Π’ — Ρ‚Π΅ΠΌΠΏΠ΅Ρ€Π°Ρ‚ΡƒΡ€Π°.

Π =n k T n =N/V N =v* N a Ρ‚ΠΎΠ³Π΄Π° p=nkT=v*Na/V * kT, R=8,31.

PV= m /MRT.

2. 2. ΠœΠ°Π³Π½ΠΈΡ‚Π½Ρ‹Π΅ свойства вСщСства. Π€Π΅Ρ€Ρ€ΠΎΠΌΠ°Π³Π½Π΅Ρ‚ΠΈΠΊΠΈ ΠΈ ΠΈΡ… ΠΏΡ€ΠΈΠΌΠ΅Π½Π΅Π½ΠΈΠ΅.

Π’ΠΎ Π²ΡΠ΅Ρ… Ρ‚Π΅Π»Π°Ρ…, ΠΏΠΎΠΌΠ΅Ρ‰Π΅Π½Π½Ρ‹Ρ… Π² ΠΌΠ°Π³Π½ΠΈΡ‚Π½ΠΎΠ΅ ΠΏΠΎΠ»Π΅, Π²ΠΎΠ·Π½ΠΈΠΊΠ°Π΅Ρ‚ ΠΌΠ°Π³Π½ΠΈΡ‚Π½Ρ‹ΠΉ ΠΌΠΎΠΌΠ΅Π½Ρ‚. Π­Ρ‚ΠΎ явлСниС называСтся Π½Π°ΠΌΠ°Π³Π½ΠΈΡ‡ΠΈΠ²Π°Π½ΠΈΠ΅ΠΌ. ΠšΠΎΠ»ΠΈΡ‡Π΅ΡΡ‚Π²Π΅Π½Π½ΠΎΠΉ характСристикой ΠΌΠ°Π³Π½ΠΈΡ‚Π½Ρ‹Ρ… свойств вСщСства являСтся ΠΌΠ°Π³Π½ΠΈΡ‚Π½ΠΎΠΉ ΠΏΡ€ΠΎΠ½ΠΈΡ†Π°Π΅ΠΌΠΎΡΡ‚ΡŒΡŽ, которая числСнно Ρ€Π°Π²Π½Π° ΠΎΡ‚Π½ΠΎΡˆΠ΅Π½ΠΈΡŽ Π²Π΅ΠΊΡ‚ΠΎΡ€Π° ΠΌΠ°Π³Π½ΠΈΡ‚Π½ΠΎΠΉ ΠΈΠ½Π΄ΡƒΠΊΡ†ΠΈΠΈ Π² Π΄Π°Π½Π½ΠΎΠΉ ΠΎΠΏΡ€Π΅Π΄Π΅Π»Π΅Π½Π½ΠΎΠΉ срСдС ΠΊ Π²Π΅ΠΊΡ‚ΠΎΡ€Ρƒ ΠΌΠ°Π³Π½ΠΈΡ‚Π½ΠΎΠΉ ΠΈΠ½Π΄ΡƒΠΊΡ†ΠΈΠΈ Π² Ρ‚ΠΎΠΉ ΠΆΠ΅ Ρ‚ΠΎΡ‡ΠΊΠ΅ пространства Π² Π²Π°ΠΊΡƒΡƒΠΌΡƒ: m= Π²Π΅ΠΊΡ‚ΠΎΡ€ B/Π²Π΅ΠΊΡ‚ΠΎΡ€B0. Π’ Π·Π°Π²ΠΈΡΠΈΠΌΠΎΡΡ‚ΠΈ ΠΎΡ‚ Π·Π½Π°Ρ‡Π΅Π½ΠΈΡ ΠΌΠ°Π³Π½ΠΈΡ‚Π½ΠΎΠΉ проницаСмости, вСщСства ΠΌΠΎΠΆΠ½ΠΎ Ρ€Π°Π·Π΄Π΅Π»ΠΈΡ‚ΡŒ Π½Π° Ρ„Π΅Ρ€Ρ€ΠΎΠΌΠ°Π³Π½Π΅Ρ‚ΠΈΠΊΠΈ m>>1 — ΠΆΠ΅Π»Π΅Π·ΠΎ, ΠΊΠΎΠ±Π°Π»ΡŒΡ‚, никСль.

ΠŸΠ°Ρ€Π°ΠΌΠ°Π³Π½Π΅Ρ‚ΠΈΠΊΠΈ — Ρ‡ΡƒΡ‚ΡŒ большС 1, ΠΏΠ»Π°Ρ‚ΠΈΠ½Π°, ΠΆΠΈΠ΄ΠΊΠΈΠΉ кислород.

Π”ΠΈΠ°ΠΌΠ°Π³Π½Π΅Ρ‚ΠΈΠΊΠΈ M Ρ‡ΡƒΡ‚ΡŒ мСньшС 1, висмут.

Π€Π΅Ρ€Ρ€ΠΎΠΌΠ°Π³Π½Π΅Ρ‚ΠΈΠ·ΠΌ ΠΎΠ±ΡŠΡΡΠ½ΡΠ΅Ρ‚ΡΡ ΠΌΠ°Π³Π½ΠΈΡ‚Π½Ρ‹ΠΌΠΈ свойствами элСктронов. Π­Π»Π΅ΠΊΡ‚Ρ€ΠΎΠ½ эквивалСнтСн ΠΊΡ€ΡƒΠ³ΠΎΠ²ΠΎΠΌΡƒ Ρ‚ΠΎΠΊΡƒ ΠΈΠ»ΠΈ Π²Ρ€Π°Ρ‰Π°ΡŽΡ‰Π΅ΠΌΡƒΡΡ заряТСнному Ρ‚Π΅Π»Ρƒ ΠΈ ΠΏΠΎΡΡ‚ΠΎΠΌΡƒ ΠΎΠ±Π»Π°Π΄Π°Π΅Ρ‚ собствСнным ΠΌΠ°Π³Π½ΠΈΡ‚Π½Ρ‹ΠΌ ΠΏΠΎΠ»Π΅ΠΌ. Π‘ ΡƒΠ²Π΅Π»ΠΈΡ‡Π΅Π½ΠΈΠ΅ΠΌ ΠΌΠ°Π³Π½ΠΈΡ‚Π½ΠΎΠΉ ΠΈΠ½Π΄ΡƒΠΊΡ†ΠΈΠΈ внСшнСго поля возрастаСт ΡΡ‚Π΅ΠΏΠ΅Π½ΡŒ упорядочСнности ΠΎΡ€ΠΈΠ΅Π½Ρ‚Π°Ρ†ΠΈΠΈ ΠΎΡ‚Π΄Π΅Π»ΡŒΠ½Ρ‹Ρ… Π΄ΠΎΠΌΠ΅Π½ΠΎΠ² — магнитная индукция возрастаСт. ΠŸΡ€ΠΈ Π½Π΅ΠΊΠΎΡ‚ΠΎΡ€ΠΎΠΌ Π·Π½Π°Ρ‡Π΅Π½ΠΈΠΈ ΠΈΠ½Π΄ΡƒΠΊΡ†ΠΈΠΈ внСшнСго поля наступаСт ΠΏΠΎΠ»Π½ΠΎΠ΅ упорядочСниС ΠΎΡ€ΠΈΠ΅Π½Ρ‚Π°Ρ†ΠΈΠΈ Π΄ΠΎΠΌΠ΅Π½ΠΎΠ², ΠΈ Π²ΠΎΠ·Ρ€Π°ΡΡ‚Π°Π½ΠΈΠ΅ ΠΌΠ°Π³Π½ΠΈΡ‚Π½ΠΎΠΉ ΠΈΠ½Π΄ΡƒΠΊΡ†ΠΈΠΈ прСкращаСтся. Π­Ρ‚ΠΎ явлСниС называСтся ΠΌΠ°Π³Π½ΠΈΡ‚Π½Ρ‹ΠΌ насыщСниСм. ΠŸΡ€ΠΈ вынСсСнии Ρ„Π΅Ρ€Ρ€ΠΎΠΌΠ°Π³Π½ΠΈΡ‚Π½ΠΎΠ³ΠΎ ΠΎΠ±Ρ€Π°Π·Ρ†Π° ΠΈΠ· Π²Π½Π΅ΡˆΠ½Π΅Π³ΠΎ ΠΌΠ°Π³Π½ΠΈΡ‚Π½ΠΎΠ³ΠΎ поля Π·Π½Π°Ρ‡ΠΈΡ‚Π΅Π»ΡŒΠ½Π°Ρ Ρ‡Π°ΡΡ‚ΡŒ Π΄ΠΎΠΌΠ΅Π½ΠΎΠ² сохраняСт ΡƒΠΏΠΎΡ€ΡΠ΄ΠΎΡ‡Π΅Π½Π½ΡƒΡŽ ΠΎΡ€ΠΈΠ΅Π½Ρ‚Π°Ρ†ΠΈΡŽ — ΠΎΠ±Ρ€Π°Π·Π΅Ρ† становится постоянным ΠΌΠ°Π³Π½ΠΈΡ‚ΠΎΠΌ. Π€Π΅Ρ€Ρ€ΠΎΠΌΠ°Π³Π½ΠΈΡ‚Π½Ρ‹Π΅ ΠΌΠ°Ρ‚Π΅Ρ€ΠΈΠ°Π»Ρ‹, способныС ΡƒΡΠΈΠ»ΠΈΠ²Π°Ρ‚ΡŒ ΠΌΠ°Π³Π½ΠΈΡ‚Π½Ρ‹Π΅ поля Π² Π΄Π΅ΡΡΡ‚ΠΊΠΈ Ρ€Π°Π·, ΡˆΠΈΡ€ΠΎΠΊΠΎ ΠΏΡ€ΠΈΠΌΠ΅Π½ΡΡŽΡ‚ΡΡ Π² ΡΠΎΠ²Ρ€Π΅ΠΌΠ΅Π½Π½ΠΎΠΉ Ρ‚Π΅Ρ…Π½ΠΈΠΊΠ΅. Π‘Ρ‚Π°Π»ΡŒΠ½ΠΎΠΉ сСрдСчник являСтся ΠΎΠ΄Π½ΠΎΠΉ ΠΈΠ· ΠΎΡΠ½ΠΎΠ²Π½Ρ‹Ρ… Π΄Π΅Ρ‚Π°Π»Π΅ΠΉ Π² ΡΠΎΠ²Ρ€Π΅ΠΌΠ΅Π½Π½ΠΎΠΉ Ρ‚Π΅Ρ…Π½ΠΈΠΊΠ΅. Π‘Ρ‚Π°Π»ΡŒΠ½ΠΎΠΉ сСрдСчник являСтся ΠΎΠ΄Π½ΠΎΠΉ ΠΈΠ· ΠΎΡΠ½ΠΎΠ²Π½Ρ‹Ρ… Π΄Π΅Ρ‚Π°Π»Π΅ΠΉ элСктрогСнСратора ΠΈ ΡΠ»Π΅ΠΊΡ‚родвигатСля, элСктромагнита ΠΈ Ρ‚рансформатора. Π’ΠΎΠ½ΠΊΠΈΠΉ слой Ρ„Π΅Ρ€Ρ€ΠΎΠΌΠ°Π³Π½ΠΈΡ‚Π½ΠΎΠ³ΠΎ ΠΏΠΎΡ€ΠΎΡˆΠΊΠ° Π½Π° Π³ΠΈΠ±ΠΊΠΎΠΉ ΠΏΠ»Π΅Π½ΠΊΠ΅ ΠΈΡΠΏΠΎΠ»ΡŒΠ·ΡƒΠ΅Ρ‚ΡΡ для ΠΌΠ°Π³Π½ΠΈΡ‚Π½ΠΎΠΉ записи ΠΈ Π²ΠΎΡΠΏΡ€ΠΎΠΈΠ·Π²Π΅Π΄Π΅Π½ΠΈΡ Π·Π²ΡƒΠΊΠ°.

Π‘ΠΈΠ»Π΅Ρ‚ 11.

1. 1. АгрСгатныС состояния вСщСства. Π˜Ρ… ΠΎΠ±ΡŠΡΡΠ½Π΅Π½ΠΈΠ΅ Π½Π° ΠΎΡΠ½ΠΎΠ²Π΅ МКВ. Π€Π°Π·ΠΎΠ²Ρ‹Π΅ ΠΏΠ΅Ρ€Π΅Ρ…ΠΎΠ΄Ρ‹ вСщСства.

2. 2. Π—Π²ΡƒΠΊΠΎΠ²Ρ‹Π΅ Π²ΠΎΠ»Π½Ρ‹. Π‘ΠΊΠΎΡ€ΠΎΡΡ‚ΡŒ Π·Π²ΡƒΠΊΠ°. Π“Ρ€ΠΎΠΌΠΊΠΎΡΡ‚ΡŒ ΠΈ Π²Ρ‹ΡΠΎΡ‚Π° Π·Π²ΡƒΠΊΠ°. Π­Ρ…ΠΎ. Π£Π»ΡŒΡ‚Ρ€Π°Π·Π²ΡƒΠΊ ΠΈ Π΅Π³ΠΎ использованиС.

3. 3. (Π—Π°Π΄Π°Ρ‡Π° Π½Π° ΡΠΎΠ΅Π΄ΠΈΠ½Π΅Π½ΠΈΠ΅ ΠΏΡ€ΠΎΠ²ΠΎΠ΄Π½ΠΈΠΊΠΎΠ²).

Π—Π²ΡƒΠΊ — Ρ‡Π°ΡΡ‚ΡŒ ΡƒΠΏΡ€ΡƒΠ³ΠΈΡ… Π²ΠΎΠ»Π½, воспринимаСмых нашими ΡƒΡˆΠ°ΠΌΠΈ. Высота являСтся частотой Π·Π²ΡƒΠΊΠ°. Π’ Π·Π°Π²ΠΈΡΠΈΠΌΠΎΡΡ‚ΠΈ ΠΎΡ‚ ΡƒΡΠ»ΠΎΠ²ΠΈΠΉ ΠΎΠ΄Π½ΠΎ ΠΈ Ρ‚ΠΎΠΆΠ΅ вСщСство ΠΌΠΎΠΆΠ΅Ρ‚ Π½Π°Ρ…ΠΎΠ΄ΠΈΡ‚ΡŒΡΡ Π² Ρ€Π°Π·Π»ΠΈΡ‡Π½Ρ‹Ρ… состояниях: Π² Ρ‚Π²Π΅Ρ€Π΄ΠΎΠΌ, ΠΆΠΈΠ΄ΠΊΠΎΠΌ ΠΈΠ»ΠΈ Π³Π°Π·ΠΎΠΎΠ±Ρ€Π°Π·Π½ΠΎΠΌ. Π­Ρ‚ΠΈ состояния Π½Π°Π·Ρ‹Π²Π°ΡŽΡ‚ΡΡ Π°Π³Ρ€Π΅Π³Π°Ρ‚Π½Ρ‹ΠΌΠΈ. ΠœΠΎΠ»Π΅ΠΊΡƒΠ»Ρ‹ ΠΎΠ΄Π½ΠΎΠ³ΠΎ ΠΈ Ρ‚ΠΎΠ³ΠΎ ΠΆΠ΅ вСщСства Π² Ρ‚Π²Π΅Ρ€Π΄ΠΎΠΌ, ΠΆΠΈΠ΄ΠΊΠΎΠΌ ΠΈΠ»ΠΈ Π³Π°Π·ΠΎΠΎΠ±Ρ€Π°Π·Π½ΠΎΠΌ состоянии ΠΎΠ΄Π½ΠΈ ΠΈ Ρ‚ΠΎΠΆΠ΅, Π½ΠΈΡ‡Π΅ΠΌ Π½Π΅ ΠΎΡ‚Π»ΠΈΡ‡Π°ΡŽΡ‚ΡΡ Π΄Ρ€ΡƒΠ³ ΠΎΡ‚ Π΄Ρ€ΡƒΠ³Π°, мСняСтся Ρ‚ΠΎΠ»ΡŒΠΊΠΎ ΠΈΡ… Π²Π·Π°ΠΈΠΌΠ½ΠΎΠ΅ располоТСниС. ИзмСнСниС Π²Π½ΡƒΡ‚Ρ€Π΅Π½Π½Π΅ΠΉ энСргии ΠΌΠΎΠΆΠ΅Ρ‚ ΠΏΡ€ΠΈΠ²ΠΎΠ΄ΠΈΡ‚ΡŒ ΠΊ ΠΈΠ·ΠΌΠ΅Π½Π΅Π½ΠΈΡŽ Π°Π³Ρ€Π΅Π³Π°Ρ‚Π½ΠΎΠ³ΠΎ состояния. ΠŸΠ΅Ρ€Π΅Ρ…ΠΎΠ΄ вСщСства ΠΏΡ€ΠΈ ΠΎΠΏΡ€Π΅Π΄Π΅Π»Π΅Π½Π½ΠΎΠΉ Ρ‚Π΅ΠΌΠΏΠ΅Ρ€Π°Ρ‚ΡƒΡ€Π΅ ΠΈΠ· Ρ‚Π²Π΅Ρ€Π΄ΠΎΠ³ΠΎ состояния Π² ΠΆΠΈΠ΄ΠΊΠΎΠ΅ называСтся ΠΏΠ»Π°Π²Π»Π΅Π½ΠΈΠ΅ΠΌ. Из ΠΆΠΈΠ΄ΠΊΠΎΠ³ΠΎ Π² Ρ‚Π²Π΅Ρ€Π΄ΠΎΠ΅ — ΠΎΡ‚Π²Π΅Ρ€Π΄Π΅Π²Π°Π½ΠΈΠ΅ ΠΈΠ»ΠΈ кристаллизация. Из ΠΆΠΈΠ΄ΠΊΠΎΠ³ΠΎ Π² Π³Π°Π·ΠΎΠΎΠ±Ρ€Π°Π·Π½ΠΎΠ΅ — испарСниС. ΠžΠ±Ρ€Π°Ρ‚Π½ΠΎ — кондСнсация.

2. 2. Π—Π²ΡƒΠΊΠΎΠ²Ρ‹Π΅ Π²ΠΎΠ»Π½Ρ‹. Π‘ΠΊΠΎΡ€ΠΎΡΡ‚ΡŒ Π·Π²ΡƒΠΊΠ°. Π“Ρ€ΠΎΠΌΠΊΠΎΡΡ‚ΡŒ ΠΈ Π²Ρ‹ΡΠΎΡ‚Π° Π·Π²ΡƒΠΊΠ°. Π­Ρ…ΠΎ. Π£Π»ΡŒΡ‚Ρ€Π°Π·Π²ΡƒΠΊ ΠΈ Π΅Π³ΠΎ использованиС.

ΠœΠΈΡ€ Π½Π°ΠΏΠΎΠ»Π½Π΅Π½ самыми Ρ€Π°Π·Π½ΠΎΠΎΠ±Ρ€Π°Π·Π½Ρ‹ΠΌΠΈ Π·Π²ΡƒΠΊΠ°ΠΌΠΈ: Ρ‚ΠΈΠΊΠ°Π½ΡŒΠ΅ΠΌ часов ΠΈ Π³ΡƒΠ»ΠΎΠΌ ΠΌΠΎΡ‚ΠΎΡ€ΠΎΠ², ΡˆΠ΅Π»Π΅ΡΡ‚ΠΎΠΌ листов ΠΈ Π·Π°Π²Ρ‹Π²Π°Π½ΠΈΠ΅ΠΌ Π²Π΅Ρ‚Ρ€Π°, ΠΏΠ΅Π½ΠΈΠ΅ΠΌ ΠΏΡ‚ΠΈΡ† ΠΈ Π³ΠΎΠ»ΠΎΡΠ°ΠΌΠΈ людСй. О Ρ‚ΠΎΠΌ, ΠΊΠ°ΠΊ Ρ€ΠΎΠΆΠ΄Π°ΡŽΡ‚ΡΡ Π·Π²ΡƒΠΊΠΈ ΠΈ Ρ‡Ρ‚ΠΎ ΠΎΠ½ΠΈ собой ΠΏΡ€Π΅Π΄ΡΡ‚Π°Π²Π»ΡΡŽΡ‚, люди Π½Π°Ρ‡Π°Π»ΠΈ Π΄ΠΎΠ³Π°Π΄Ρ‹Π²Π°Ρ‚ΡŒΡΡ ΠΎΡ‡Π΅Π½ΡŒ Π΄Π°Π²Π½ΠΎ. Достигая ΡƒΡ…Π°, Π·Π²ΡƒΠΊ воздСйствуСт Π½Π° Π±Π°Ρ€Π°Π±Π°Π½Π½Ρ‹Π΅ ΠΏΠ΅Ρ€Π΅ΠΏΠΎΠ½ΠΊΠΈ ΠΈ Π²Ρ‹Π·Ρ‹Π²Π°Π΅Ρ‚ ΠΎΡ‰ΡƒΡ‰Π΅Π½ΠΈΠ΅ Π·Π²ΡƒΠΊΠ°. На ΡΠ»ΡƒΡ… Ρ‡Π΅Π»ΠΎΠ²Π΅ΠΊ воспринимаСт ΡƒΠΏΡ€ΡƒΠ³ΠΈΠ΅ Π²ΠΎΠ»Π½Ρ‹, ΠΈΠΌΠ΅ΡŽΡ‰ΠΈΠ΅ частоту Π² ΠΏΠ΅Ρ€Π΅Π΄Π΅Π»Π°Ρ… ΠΎΡ‚ 16 Π“Ρ† Π΄ΠΎ 20 ΠΊΠ“Ρ† (1 Π“Ρ† — ΠΎΠ΄Π½ΠΎ ΠΊΠΎΠ»Π΅Π±Π°Π½ΠΈΠ΅ Π² ΡΠ΅ΠΊΡƒΠ½Π΄Ρƒ). Π’ΠΎΡ‚ ΠΏΠΎΡ‡Π΅ΠΌΡƒ ΡƒΠΏΡ€ΡƒΠ³ΠΈΠ΅ Π²ΠΎΠ»Π½Ρ‹ Π² Π»ΡŽΠ±ΠΎΠΉ срСдС, частоты ΠΊΠΎΡ‚ΠΎΡ€Ρ‹Ρ… Π»Π΅ΠΆΠ°Ρ‚ Π² ΡƒΠΊΠ°Π·Π°Π½Π½Ρ‹Ρ… ΠΏΡ€Π΅Π΄Π΅Π»Π°Ρ…, Π½Π°Π·Ρ‹Π²Π°ΡŽΡ‚ Π·Π²ΡƒΠΊΠΎΠ²Ρ‹ΠΌΠΈ Π²ΠΎΠ»Π½Π°ΠΌΠΈ ΠΈΠ»ΠΈ просто Π·Π²ΡƒΠΊΠΎΠΌ. Π’ Π²ΠΎΠ·Π΄ΡƒΡ…Π΅ ΠΏΡ€ΠΈ Ρ‚Π΅ΠΌΠΏΠ΅Ρ€Π°Ρ‚ΡƒΡ€Π΅ 0 ΠΈ Π½ΠΎΡ€ΠΌΠ°Π»ΡŒΠ½ΠΎΠΌ атмосфСрном Π΄Π°Π²Π»Π΅Π½ΠΈΠΈ Π·Π²ΡƒΠΊ распространяСтся со ΡΠΊΠΎΡ€ΠΎΡΡ‚ΡŒΡŽ 330 ΠΌ/с, Π° Π² ΠΌΠΎΡ€ΡΠΊΠΎΠΉ Π²ΠΎΠ΄Π΅ — ΠΎΠΊΠΎΠ»ΠΎ 1500 ΠΌ/с, Π° Π² Π½Π΅ΠΊΠΎΡ‚ΠΎΡ€Ρ‹Ρ… ΠΌΠ΅Ρ‚Π°Π»Π»Π°Ρ… Π΅Π³ΠΎ ΡΠΊΠΎΡ€ΠΎΡΡ‚ΡŒ достигаСт 700 ΠΌ/с. Π£ΠΏΡ€ΡƒΠ³ΠΈΠ΅ Π²ΠΎΠ»Π½Ρ‹ с Ρ‡Π°ΡΡ‚ΠΎΡ‚ΠΎΠΉ мСньшС 16 Π“Ρ† Π½Π°Π·Ρ‹Π²Π°ΡŽΡ‚ ΠΈΠ½Ρ„Ρ€Π°Π·Π²ΡƒΠΊΠΎΠΌ, Π° Ρ Ρ‡Π°ΡΡ‚ΠΎΡ‚ΠΎΠΉ ΠΏΡ€Π΅Π²Ρ‹ΡˆΠ°ΡŽΡ‰Π΅ΠΉ 20 ΠΊΠ“Ρ† — ΡƒΠ»ΡŒΡ‚Ρ€Π°Π·Π²ΡƒΠΊΠΎΠΌ. Π—Π²ΡƒΠΊ ΠΌΠΎΠΆΠ΅Ρ‚ Ρ€Π°ΡΠΏΡ€ΠΎΡΡ‚Ρ€Π°Π½ΡΡ‚ΡŒΡΡ Π² Π²ΠΈΠ΄Π΅ ΠΏΡ€ΠΎΠ΄ΠΎΠ»ΡŒΠ½Ρ‹Ρ… ΠΈ ΠΏΠΎΠΏΠ΅Ρ€Π΅Ρ‡Π½Ρ‹Ρ… Π²ΠΎΠ»Π½. Π’ Π³Π°Π·ΠΎΠΎΠ±Ρ€Π°Π·Π½ΠΎΠΌ состоянии Π²ΠΎΠ·Π½ΠΈΠΊΠ°ΡŽΡ‚ Ρ‚ΠΎΠ»ΡŒΠΊΠΎ ΠΏΡ€ΠΎΠ΄ΠΎΠ»ΡŒΠ½Ρ‹Π΅ Π²ΠΎΠ»Π½Ρ‹, ΠΊΠΎΠ³Π΄Π° ΠΊΠΎΠ»Π΅Π±Π°Ρ‚Π΅Π»ΡŒΠ½ΠΎΠ΅ Π΄Π²ΠΈΠΆΠ΅Π½ΠΈΠ΅ частиц происходит лишь Π² Ρ‚ΠΎΠΌ Π½Π°ΠΏΡ€Π°Π²Π»Π΅Π½ΠΈΠΈ, Π² ΠΊΠΎΡ‚ΠΎΡ€ΠΎΠΌ распространяСтся Π²ΠΎΠ»Π½Π°. Π’ Ρ‚Π²Π΅Ρ€Π΄Ρ‹Ρ… Ρ‚Π΅Π»Π° ΠΏΠΎΠΌΠΈΠΌΠΎ ΠΏΡ€ΠΎΠ΄ΠΎΠ»ΡŒΠ½Ρ‹Ρ… Π²ΠΎΠ·Π½ΠΈΠΊΠ°Π΅Ρ‚ ΠΈ ΠΏΠΎΠΏΠ΅Ρ€Π΅Ρ‡Π½Ρ‹Π΅, ΠΊΠΎΠ³Π΄Π° частицы срСды ΠΊΠΎΠ»Π΅Π±Π»ΡŽΡ‚ΡΡ Π² Π½Π°ΠΏΡ€Π°Π²Π»Π΅Π½ΠΈΠΈ, пСрпСндикулярных Π½Π°ΠΏΡ€Π°Π²Π»Π΅Π½ΠΈΡŽ Π²ΠΎΠ»Π½Ρ‹. Π—Π²ΡƒΠΊΠΎΠ²Ρ‹Π΅ Π²ΠΎΠ»Π½Ρ‹ нСсут с ΡΠΎΠ±ΠΎΠΉ ΡΠ½Π΅Ρ€Π³ΠΈΡŽ, ΠΊΠΎΡ‚ΠΎΡ€ΡƒΡŽ ΡΠΎΠΎΠ±Ρ‰Π°ΡŽΡ‚ ΠΈΠΌ ΠΈΡΡ‚ΠΎΡ‡Π½ΠΈΠΊ Π·Π²ΡƒΠΊΠ°. Π’Π΅Π»ΠΈΡ‡ΠΈΠ½Ρƒ кинСтичСской энСргии, ΠΏΡ€ΠΎΡ‚Π΅ΠΊΠ°ΡŽΡ‰Π΅ΠΉ Π·Π° ΠΎΠ΄Ρƒ сСкунду Ρ‡Π΅Ρ€Π΅Π· ΠΊΠ²Π°Π΄Ρ€Π°Ρ‚Π½Ρ‹ΠΉ сантимСтр повСрхности, пСрпСндикулярной Π½Π°ΠΏΡ€Π°Π²Π»Π΅Π½ΠΈΡŽ распространСния Π²ΠΎΠ»Π½Ρ‹, вычислил Николай АлСксССвич Наумов. Π­Ρ‚Ρƒ Π²Π΅Π»ΠΈΡ‡ΠΈΠ½Ρƒ Π½Π°Π·Π²Π°Π»ΠΈ ΠΏΠΎΡ‚ΠΎΠΊΠΎΠΌ энСргии. Она Π²Ρ‹Ρ€Π°ΠΆΠ°Π΅Ρ‚ ΠΌΠ΅Ρ€Ρƒ интСнсивности, ΠΈΠ»ΠΈ, ΠΊΠ°ΠΊ Π΅Ρ‰Π΅ говорят, силы Π·Π²ΡƒΠΊΠ°. Всякий Ρ€Π΅Π°Π»ΡŒΠ½Ρ‹ΠΉ Π·Π²ΡƒΠΊ — это нСпросто гармоничСскоС ΠΊΠΎΠ»Π΅Π±Π°Π½ΠΈΠ΅, Π° ΡΠ²ΠΎΠ΅ΠΎΠ±Ρ€Π°Π·Π½Π°Ρ смСсь ΠΌΠ½ΠΎΠ³ΠΈΡ… гармоничСских ΠΊΠΎΠ»Π΅Π±Π°Π½ΠΈΠΉ с ΠΎΠΏΡ€Π΅Π΄Π΅Π»Π΅Π½Π½Ρ‹ΠΌ Π½Π°Π±ΠΎΡ€ΠΎΠΌ частот.

ΠœΡƒΠ·Ρ‹ΠΊΠ°Π»ΡŒΠ½Ρ‹ΠΉ Π·Π²ΡƒΠΊ характСризуСтся трСмя качСствами: высотой (ΠΎΠΏΡ€Π΅Π΄Π΅Π»ΡΡŽΡ‰Π΅2йся чистом ΠΊΠΎΠ»Π΅Π±Π°Π½ΠΈΠΉ Π² ΡΠ΅ΠΊΡƒΠ½Π΄Ρƒ — частотой), Π³Ρ€ΠΎΠΌΠΊΠΎΡΡ‚ΡŒΡŽ (зависящСй ΠΎΡ‚ ΠΈΠ½Ρ‚Снсивности ΠΊΠΎΠ»Π΅Π±Π°Π½ΠΈΠΉ) ΠΈ Ρ‚Π΅ΠΌΠ±Ρ€ΠΎΠΌ — окраской Π·Π²ΡƒΠΊΠ° (зависящСй ΠΎΡ‚ Ρ„ΠΎΡ€ΠΌΡ‹ ΠΊΠΎΠ»Π΅Π±Π°Π½ΠΈΠΉ).

Из-Π·Π° ΠΊΠΎΠ½Π΅Ρ‡Π½ΠΎΠΉ скорости Π·Π²ΡƒΠΊΠ° появляСтся эхо. Π§Ρ‚ΠΎΠ±Ρ‹ Π΅Π³ΠΎ ΡƒΡΠ»Ρ‹ΡˆΠ°Ρ‚ΡŒ, ΠΌΠΎΠΆΠ½ΠΎ произнСсти Π³Ρ€ΠΎΠΌΠΊΠΈΠΉ Π·Π²ΡƒΠΊ ΠΏΠ΅Ρ€Π΅Π΄ ΠΊΡ€ΡƒΠΏΠ½Ρ‹ΠΌ Π·Π΄Π°Π½ΠΈΠ΅ΠΌ, отстоящим ΠΎΡ‚ Π²Π°Ρ Π½Π° 20 -30 ΠΌΠ΅Ρ‚Ρ€ΠΎΠ². Π Π°ΡΠΏΡ€ΠΎΡΡ‚Ρ€Π°Π½ΡΡŽΡ‰Π°ΡΡΡ звуковая Π²ΠΎΠ»Π½Π°, встрСтив Π½Π° ΡΠ²ΠΎΠ΅ΠΌ ΠΏΡƒΡ‚ΠΈ Π±ΠΎΠ»ΡŒΡˆΡƒΡŽ ΠΏΡ€Π΅Π³Ρ€Π°Π΄Ρƒ — стСну здания, отраТаСтся ΠΎΡ‚ Π½Π΅Π΅. Когда отраТСнная Π²ΠΎΠ»Π½Π° достигаСт нашСго ΡƒΡ…Π°, ΠΌΡ‹ ΡΠ»Ρ‹ΡˆΠΈΠΌ отголосок ΠΈΠ»ΠΈ эхо. Π­Ρ…ΠΎ — это звуковая Π²ΠΎΠ»Π½Π°, отраТСнная ΠΊΠ°ΠΊΠΎΠΉ — Π»ΠΈΠ±ΠΎ ΠΏΡ€Π΅Π³Ρ€Π°Π΄ΠΎΠΉ ΠΈ Π²ΠΎΠ·Π²Ρ€Π°Ρ‚ΠΈΠ²ΡˆΠ°ΡΡΡ Π² Ρ‚ΠΎ ΠΌΠ΅ΡΡ‚ΠΎ, ΠΎΡ‚ΠΊΡƒΠ΄Π° ΠΎΠ½Π° Π½Π°Ρ‡Π°Π»Π° Ρ€Π°ΡΠΏΡ€ΠΎΡΡ‚Ρ€Π°Π½ΡΡ‚ΡŒΡΡ. Π›Π΅Π³ΠΊΠΎ ΠΏΠΎΠ½ΡΡ‚ΡŒ, Ρ‡Ρ‚ΠΎ ΠΌΡ‹ ΡΠ»Ρ‹ΡˆΠΈΠΌ эхо Ρ‡Π΅Ρ€Π΅Π· Ρ‚Π°ΠΊΠΎΠΉ ΠΏΡ€ΠΎΠΌΠ΅ΠΆΡƒΡ‚ΠΎΠΊ Π²Ρ€Π΅ΠΌΠ΅Π½ΠΈ. Π’ Ρ‚Π΅Ρ‡Π΅Π½ΠΈΠΈ ΠΊΠΎΡ‚ΠΎΡ€ΠΎΠ³ΠΎ звуковая Π²ΠΎΠ»Π½Π° ΠΏΡ€ΠΎΡ…ΠΎΠ΄ΠΈΡ‚ ΠΏΡƒΡ‚ΡŒ Π΄ΠΎ ΠΏΡ€Π΅Π³Ρ€Π°Π΄Ρ‹ ΠΈ ΠΎΠ±Ρ€Π°Ρ‚Π½ΠΎ, Ρ‚Π΅ ΠΏΡ€ΠΎΡ…ΠΎΠ΄ΠΈΡ‚ Π΄Π²ΠΎΠΉΠ½ΠΎΠ΅ расстояниС ΠΌΠ΅ΠΆΠ΄Ρƒ источником Π·Π²ΡƒΠΊΠ° ΠΈ ΠΏΡ€Π΅Π³Ρ€Π°Π΄ΠΎΠΉ. S=V*t/2. Π˜Π·Π»ΡƒΡ‡Π°Ρ ΠΊΠΎΡ€ΠΎΡ‚ΠΊΠΈΠ΅ ΠΈΠΌΠΏΡƒΠ»ΡŒΡΡ‹ Π²ΠΎΠ»Π½ ΠΈ ΡƒΠ»Π°Π²Π»ΠΈΠ²Π°Ρ ΠΈΡ… ΡΡ…ΠΎ, ΠΈΠ·ΠΌΠ΅Ρ€ΡΡŽΡ‚ врСмя двиТСния Π²ΠΎΠ»Π½Ρ‹ ΠΎΡ‚ ΠΏΡ€Π΅Π³Ρ€Π°Π΄Ρ‹ ΠΈ ΠΎΠ±Ρ€Π°Ρ‚Π½ΠΎ, Π° ΠΏΠΎΡ‚ΠΎΠΌ ΠΎΠΏΡ€Π΅Π΄Π΅Π»ΠΈΡ‚ расстояниС Π΄ΠΎ ΠΏΡ€Π΅Π³Ρ€Π°Π΄Ρ‹. Π’ ΡΡ‚ΠΎΠΌ ΡΡƒΡ‚ΡŒ эхолокации.

Π‘ΠΈΠ»Π΅Ρ‚ 12.

1. 1. ЭлСктризация Ρ‚Π΅Π». ЭлСктричСский заряд, Π΅Π³ΠΎ Π΄ΠΈΡΠΊΡ€Π΅Ρ‚Π½ΠΎΡΡ‚ΡŒ. Π—Π°ΠΊΠΎΠ½ сохранСния элСктричСского заряда. ВзаимодСйствиС заряТСнных Ρ‚Π΅Π». Π—Π°ΠΊΠΎΠ½ ΠšΡƒΠ»ΠΎΠ½Π°.

2. 2. Π’ΠΎΠ»Π½Ρ‹. ΠŸΠΎΠΏΠ΅Ρ€Π΅Ρ‡Π½Ρ‹Π΅ ΠΈ ΠΏΡ€ΠΎΠ΄ΠΎΠ»ΡŒΠ½Ρ‹Π΅ Π²ΠΎΠ»Π½Ρ‹. Π”Π»ΠΈΠ½Π° Π²ΠΎΠ»Π½Ρ‹, Π΅Π΅ ΡΠ²ΡΠ·ΡŒ со ΡΠΊΠΎΡ€ΠΎΡΡ‚ΡŒΡŽ распространСния ΠΈ Ρ‡Π°ΡΡ‚ΠΎΡ‚ΠΎΠΉ (ΠΏΠ΅Ρ€ΠΈΠΎΠ΄ΠΎΠΌ) ΠΊΠΎΠ»Π΅Π±Π°Π½ΠΈΠΉ.

3. 3. (Π—Π°Π΄Π°Ρ‡Π° Π½Π° ΠΏΠΎΡΡ‚ΡƒΠ»Π°Ρ‚Ρ‹ Π‘ΠΎΡ€Π°).

ЭлСктричСский заряд — это физичСская Π²Π΅Π»ΠΈΡ‡ΠΈΠ½Π°, Ρ…Π°Ρ€Π°ΠΊΡ‚Π΅Ρ€ΠΈΠ·ΡƒΡŽΡ‰Π°Ρ элСктромагнитноС взаимодСйствиС. Π’Π΅Π»ΠΎ заряТСно ΠΎΡ‚Ρ€ΠΈΡ†Π°Ρ‚Π΅Π»ΡŒΠ½ΠΎ, Ссли Π½Π° Π½Π΅ΠΌ ΠΈΠ·Π±Ρ‹Ρ‚ΠΎΠΊ элСктронов, ΠΏΠΎΠ»ΠΎΠΆΠΈΡ‚Π΅Π»ΡŒΠ½ΠΎ — Π΄Π΅Ρ„ΠΈΡ†ΠΈΡ‚. Π—Π°ΠΊΠΎΠ½ сохранСния зарядов — Π² Π·Π°ΠΌΠΊΠ½ΡƒΡ‚ΠΎΠΉ систСмС алгСбраичСская сумма зарядов Π½Π΅ ΠΈΠ·ΠΌΠ΅Π½ΡΠ΅Ρ‚ся.

Π—Π°ΠΊΠΎΠ½ ΠšΡƒΠ»ΠΎΠ½Π°: Π΄Π²Π° заряда Π²Π·Π°ΠΈΠΌΠΎΠ΄Π΅ΠΉΡΡ‚Π²ΡƒΡŽΡ‚ Π΄Ρ€ΡƒΠ³ с Π΄Ρ€ΡƒΠ³ΠΎΠΌ с ΡΠΈΠ»ΠΎΠΉ ΠΏΡ€ΡΠΌΠΎΠΏΡ€ΠΎΠΏΠΎΡ€Ρ†ΠΈΠΎΠ½Π°Π»ΡŒΠ½ΠΎ ΠΏΡ€ΠΎΠΈΠ·Π²Π΅Π΄Π΅Π½ΠΈΡŽ Π²Π΅Π»ΠΈΡ‡ΠΈΠ½ этих зарядов ΠΈ ΠΎΠ±Ρ€Π°Ρ‚Π½ΠΎ ΠΏΡ€ΠΎΠΏΠΎΡ€Ρ†ΠΈΠΎΠ½Π°Π»ΡŒΠ½ΠΎΠΉ ΠΊΠ²Π°Π΄Ρ€Π°Ρ‚Ρƒ расстояния ΠΌΠ΅ΠΆΠ΄Ρƒ Π½ΠΈΠΌΠΈ F=kq1*q2/r2, Π³Π΄Π΅ k=9*109 Нм2/Кл2. Π‘ΠΈΠ»Π° Π½Π°ΠΏΡ€Π°Π²Π»Π΅Π½Π° вдоль прямой, ΡΠΎΠ΅Π΄ΠΈΠ½ΡΡŽΡ‰Π΅ΠΉ заряТСнныС Ρ‚Π΅Π»Π°. Она являСтся силой отталкивания ΠΏΡ€ΠΈ ΠΎΠ΄ΠΈΠ½Π°ΠΊΠΎΠ²Ρ‹Ρ… Π·Π½Π°ΠΊΠΎΠ² зарядов, ΠΈΠ»ΠΈ притяТСниС ΠΏΡ€ΠΈ Ρ€Π°Π·Π½Ρ‹Ρ… Π·Π½Π°ΠΊΠΎΠ². ВзаимодСйствиС Π½Π΅ΠΏΠΎΠ΄Π²ΠΈΠΆΠ½Ρ‹Ρ… элСктричСских зарядов — элСктростатичСскоС ΠΈΠ»ΠΈ кулоновскоС. БоотвСтствСнно ΠΏΠΎΠ»Π΅ — элСктростатичСским. K=9*109 ΠΈΠ»ΠΈ k=¼ΠŸΠ•0 =>Π•0=¼ΠŸΠΊ = 8,85 *10^-12. Один ΠΊΡƒΠ»ΠΎΠ½ это заряд проходящий Ρ‡Π΅Ρ€Π΅Π· ΠΏΠΎΠΏΠ΅Ρ€Π΅Ρ‡Π½ΠΎΠ΅ сСчСниС ΠΏΡ€ΠΎΠ²ΠΎΠ΄Π½ΠΈΠΊΠ° Π·Π° 1 с ΠΏΡ€ΠΈ силС Ρ‚ΠΎΠΊΠ° Π² 1А. ЭлСктризация — это процСсс индуцирования зарядов Π² Ρ‚Π΅Π»Π΅ (Ρ€Π°Π·Π΄Π΅Π»Π΅Π½ΠΈΠ΅ зарядов Π½Π° ΠΏΠΎΠ»ΠΎΠΆ. ΠΈ ΠΎΡ‚Ρ€ΠΈΡ†Π°Ρ‚Π΅Π»ΡŒΠ½Ρ‹Π΅). ЭлСктричСский заряд ΠΌΠΎΠΆΠ΅Ρ‚ дСлится ΠΈ Π½Π°ΠΈΠΌΠ΅Π½ΡŒΡˆΠ°Ρ порция ΠΎΡ‚Ρ€ΠΈΡ†Π°Ρ‚Π΅Π»ΡŒΠ½ΠΎΠ³ΠΎ заряда — элСктрон Π•Ρ‰Π΅ мСньшСй ΠΎΡ‚Ρ€ΠΈΡ†Π°Ρ‚Π΅Π»ΡŒΠ½ΠΎΠΉ частицСй ΡΠ²Π»ΡΡŽΡ‚ΡΡ ΠΊΠ²Π°Ρ€ΠΊΠΈ, ΡΠΎΡΡ‚Π°Π²Π»ΡΡŽΡ‰ΠΈΠ΅ 1/3, 2/3 ΠΎΡ‚ Π·Π°Ρ€ΡΠ΄Π° элСктрона. Заряд элСктрона обозначаСтся Π±ΡƒΠΊΠ²ΠΎΠΉ Π΅. Он ΠΎΡ‚Ρ€ΠΈΡ†Π°Ρ‚Π΅Π»Π΅Π½ ΠΈ Ρ€Π°Π²Π΅Π½ -1,6*10^-19. Заряд Π±Π΅Π· частицы Π½Π΅ ΡΡƒΡ‰Π΅ΡΡ‚Π²ΡƒΠ΅Ρ‚, Π° Ρ‡Π°ΡΡ‚ΠΈΡ†Π° Π±Π΅Π· заряда ΠΌΠΎΠΆΠ΅Ρ‚ ΡΡƒΡ‰Π΅ΡΡ‚Π²ΠΎΠ²Π°Ρ‚ΡŒ. ΠŸΠ΅Ρ€Π΅Π½ΠΎΡ зарядов — это ΠΏΠ΅Ρ€Π΅Ρ…ΠΎΠ΄ элСктронов с ΠΎΠ΄Π½ΠΎΠ³ΠΎ Ρ‚Π΅Π»Π° Π½Π° Π΄Ρ€ΡƒΠ³ΠΎΠ΅.

2. 2. Π’ΠΎΠ»Π½Ρ‹. ΠŸΠΎΠΏΠ΅Ρ€Π΅Ρ‡Π½Ρ‹Π΅ ΠΈ ΠΏΡ€ΠΎΠ΄ΠΎΠ»ΡŒΠ½Ρ‹Π΅ Π²ΠΎΠ»Π½Ρ‹. Π”Π»ΠΈΠ½Π° Π²ΠΎΠ»Π½Ρ‹, Π΅Π΅ ΡΠ²ΡΠ·ΡŒ со ΡΠΊΠΎΡ€ΠΎΡΡ‚ΡŒΡŽ распространСния ΠΈ Ρ‡Π°ΡΡ‚ΠΎΡ‚ΠΎΠΉ (ΠΏΠ΅Ρ€ΠΈΠΎΠ΄ΠΎΠΌ) ΠΊΠΎΠ»Π΅Π±Π°Π½ΠΈΠΉ.

Π’ΠΎΠ»Π½Π° — это процСсс распространСния ΠΊΠΎΠ»Π΅Π±Π°Π½ΠΈΠΉ (Π²Ρ‹Π½ΡƒΠΆΠ΄Π΅Π½Π½Ρ‹Ρ…) Π² ΠΏΡ€ΠΎΡΡ‚ранствС с Ρ‚Π΅Ρ‡Π΅Π½ΠΈΠ΅ΠΌ Π²Ρ€Π΅ΠΌΠ΅Π½ΠΈ. УсловиС сущСствования Π²ΠΎΠ»Π½Ρ‹: источник ΠΊΠΎΠ»Π΅Π±Π°Π½ΠΈΠΉ, взаимодСйствиС ΠΌΠ΅ΠΆΠ΄Ρƒ частицами срСды. ΠžΡΠΎΠ±Π΅Π½Π½ΠΎΡΡ‚ΠΈ Π²ΠΎΠ»Π½ΠΎΠ²ΠΎΠ³ΠΎ двиТСния — пСрСнос энСргии Π±Π΅Π· пСрСноса вСщСства. ΠŸΠ°Ρ€Π°ΠΌΠ΅Ρ‚Ρ€Ρ‹ — ΡΠΊΠΎΡ€ΠΎΡΡ‚ΡŒ Π²ΠΎΠ»Π½Ρ‹ — ΡΠΊΠΎΡ€ΠΎΡΡ‚ΡŒ пСрСмСщСния энСргии; Π΄Π»ΠΈΠ½Π° Π²ΠΎΠ»Π½Ρ‹ — минимальноС расстояниС ΠΌΠ΅ΠΆΠ΄Ρƒ Ρ‚ΠΎΡ‡ΠΊΠ°ΠΌΠΈ, ΠΊΠΎΠ»Π΅Π±Π»ΡŽΡ‰ΠΈΠΌΠΈΡΡ Π² ΠΎΠ΄ΠΈΠ½Π°ΠΊΠΎΠ²Ρ‹Ρ… Ρ„Π°Π·Π°Ρ…. ΠŸΠ΅Ρ€ΠΈΠΎΠ΄ — врСмя повторСния внСшнСго Π²ΠΈΠ΄Π° (ΠΏΠ΅Ρ€ΠΈΠΎΠ΄ Π²ΠΎΠ»Π½Ρ‹=ΠΏΠ΅Ρ€ΠΈΠΎΠ΄ ΠΊΠΎΠ»Π΅Π±Π°Π½ΠΈΠΉ). V=Y/T V=Yv. Π²ΠΈΠ΄Ρ‹ Π²ΠΎΠ»Π½Ρ‹: попСрСчная — Π²ΠΎΠ»Π½Π° Π² ΠΊΠΎΡ‚ΠΎΡ€ΠΎΠΉ ΠΊΠΎΠ»Π΅Π±Π°Π½ΠΈΠ΅ частиц происходит ΠΏΠΎΠΏΠ΅Ρ€Π΅Ρ‡Π½ΠΎ Π½Π°ΠΏΡ€Π°Π²Π»Π΅Π½ΠΈΡŽ Ρ€Π°ΡΠΏΡ€ΠΎΡΡ‚Ρ€Π°Π½Π΅Π½ΠΈΡŽ Π²ΠΎΠ»Π½Ρ‹ (пСрпСндикулярно скорости). ΠŸΡ€ΠΎΠ΄ΠΎΠ»ΡŒΠ½Π°Ρ — вдоль. Π’ΠΎΠ»Π½ΠΎΠ²ΠΎΠΉ Ρ„Ρ€ΠΎΠ½Ρ‚ — волновая ΠΏΠΎΠ²Π΅Ρ€Ρ…Π½ΠΎΡΡ‚ΡŒ — это мноТСство Ρ‚ΠΎΡ‡Π΅ΠΊ, ΠΊΠΎΠ»Π΅Π±Π»ΡŽΡ‰ΠΈΡ…ΡΡ Π² ΠΎΠ΄ΠΈΠ½Π°ΠΊΠΎΠ²Ρ‹Ρ… Ρ„Π°Π·Π°Ρ… (Π»ΠΈΠ½Π΅ΠΉΠ½Ρ‹ΠΉ, плоский, ΠΊΡ€ΡƒΠ³ΠΎΠ²ΠΎΠΉ, сфСричСский). Π›ΡƒΡ‡ ΠΈ Π²ΠΎΠ»Π½ΠΎΠ²ΠΎΠΉ Ρ„Ρ€ΠΎΠ½Ρ‚ всСгда пСрпСндикулярны.

Π‘ΠΈΠ»Π΅Ρ‚ 13.

1. 1. ЭлСктричСскоС ΠΏΠΎΠ»Π΅. ΠΠ°ΠΏΡ€ΡΠΆΠ΅Π½Π½ΠΎΡΡ‚ΡŒ элСктричСского поля.

2. 2. УскорСниС, ΡΠΊΠΎΡ€ΠΎΡΡ‚ΡŒ ΠΈ ΠΏΠ΅Ρ€Π΅ΠΌΠ΅Ρ‰Π΅Π½ΠΈΠ΅ ΠΏΡ€ΠΈ равноускорСнном прямолинСйном Π΄Π²ΠΈΠΆΠ΅Π½ΠΈΠΈ.

3. 3. «Π˜Π—ΠœΠ•Π Π•ΠΠ˜Π• Π”Π›Π˜ΠΠ« Π‘Π’Π•Π’ΠžΠ’ΠžΠ™ Π’ΠžΠ›ΠΠ«».

1. 1. ВзаимодСйствиС зарядов ΠΏΠΎ Π·Π°ΠΊΠΎΠ½Ρƒ ΠšΡƒΠ»ΠΎΠ½Π° являСтся установлСнным Ρ„Π°ΠΊΡ‚ΠΎΠΌ. Однако матСматичСскоС Π²Ρ‹Ρ€Π°ΠΆΠ΅Π½ΠΈΠ΅ Π·Π°ΠΊΠΎΠ½Π° взаимодСйствия зарядов раскрываСт физичСской ΠΊΠ°Ρ€Ρ‚ΠΈΠ½ΠΎΠΉ самого процСсса взаимодСйствия, Π½Π΅ ΠΎΡ‚Π²Π΅Ρ‡Π°Π΅Ρ‚ Π½Π° Π²ΠΎΠΏΡ€ΠΎΡ: ΠΊΠ°ΠΊΠΈΠΌ ΠΏΡƒΡ‚Π΅ΠΌ осущСствляСтся дСйствиС зарядов q1 Π½Π° Π·Π°Ρ€ΡΠ΄ q2? Π’ΠΎΠ·ΠΌΠΎΠΆΠ½Ρ‹ΠΉ ΠΎΡ‚Π²Π΅Ρ‚ Π½Π° ΡΡ‚ΠΎΡ‚ вопрос Π΄Π°Π²Π°Π»Π° тСория Π΄Π°Π»ΡŒΠ½ΠΎΠ΄Π΅ΠΉΡΡ‚Π²ΠΈΡ, которая ΡƒΡ‚Π²Π΅Ρ€ΠΆΠ΄Π°Π», Ρ‡Ρ‚ΠΎ элСктричСскиС заряды ΠΎΠ±Π»Π°Π΄Π°ΡŽΡ‚ ΡΠΏΠΎΡΠΎΠ±Π½ΠΎΡΡ‚ΡŒΡŽ ΠΌΠ³Π½ΠΎΠ²Π΅Π½Π½ΠΎ Π΄Π΅ΠΉΡΡ‚Π²ΠΎΠ²Π°Ρ‚ΡŒ Π΄Ρ€ΡƒΠ³ Π½Π° Π΄Ρ€ΡƒΠ³Π° Π½Π° Ρ€Π°ΡΡΡ‚оянии. ВСория близкодСйствия, созданная Π½Π° ΠΎΡΠ½ΠΎΠ²Π΅ Ρ€Π°Π±ΠΎΡ‚ ЀарадСя, ΠΎΠ±ΡŠΡΡΠ½ΡΠ΅Ρ‚ взаимодСйствиС элСктричСских зарядов Ρ‚Π΅ΠΌ, Ρ‡Ρ‚ΠΎ Π²ΠΎΠΊΡ€ΡƒΠ³ ΠΊΠ°ΠΆΠ΄ΠΎΠ³ΠΎ элСктричСского заряда ΡΡƒΡ‰Π΅ΡΡ‚Π²ΡƒΡŽΡ‚ нСкая срСда, Ρ‡Π΅Ρ€Π΅Π· ΠΊΠΎΡ‚ΠΎΡ€ΡƒΡŽ ΠΏΠ΅Ρ€Π΅Π΄Π°ΡŽΡ‚ΡΡ элСктричСскиС притяТСния ΠΈΠ»ΠΈ отталкивания, Ρ‚Π΅ ΡΠ»Π΅ΠΊΡ‚ричСскоС ΠΏΠΎΠ»Π΅. ЭлСктричСскоС ΠΏΠΎΠ»Π΅ заряда — ΠΌΠ°Ρ‚Π΅Ρ€ΠΈΠ°Π»ΡŒΠ½Ρ‹ΠΉ ΠΎΠ±ΡŠΠ΅ΠΊΡ‚, ΠΎΠ½ΠΎ Π½Π΅ΠΏΡ€Π΅Ρ€Ρ‹Π²Π½ΠΎ Π²Ρ‹ ΠΏΡ€ΠΎΡΡ‚ранствС ΠΈ ΡΠΏΠΎΡΠΎΠ±Π½Π° Π΄Π΅ΠΉΡΡ‚Π²ΠΎΠ²Π°Ρ‚ΡŒ Π½Π° Π΄Ρ€ΡƒΠ³ΠΈΠ΅ элСктричСскиС заряды. Если ΠΊ ΡΠ»Π΅ΠΊΡ‚роскопу, Π½Π΅ ΠΊΠ°ΡΠ°ΡΡΡŒ Π΅Π³ΠΎ оси, поднСсти Π½Π° Π½Π΅ΠΊΠΎΡ‚ΠΎΡ€ΠΎΠΌ расстоянии Π·Π°Ρ€ΡΠΆΠ΅Π½Π½ΡƒΡŽ ΠΏΠ°Π»ΠΎΡ‡ΠΊΡƒ, Ρ‚ΠΎ ΡΡ‚Ρ€Π΅Π»ΠΊΠ° всС Ρ€Π°Π²Π½ΠΎ Π±ΡƒΠ΄Π΅Ρ‚ ΠΎΡ‚ΠΊΠ»Π°Π½ΡΡ‚ΡŒΡΡ. Π­Ρ‚ΠΎ ΠΈ Π΅ΡΡ‚ΡŒ дСйствиС элСктричСского поля. ΠŸΡ€ΠΈ ΡƒΠ΄Π°Π»Π΅Π½ΠΈΠΈ ΠΏΠ°Π»ΠΎΡ‡ΠΊΠΈ ΠΎΡ‚ ΠΎΡΠΈ элСктроскопа Π΅Π³ΠΎ дСйствиС Π±ΡƒΠ΄Π΅Ρ‚ ΠΏΡ€ΠΎΡΠ²Π»ΡΡ‚ΡŒΡΡ слабСС. Богласно Ρ‚Π΅ΠΎΡ€ΠΈΠΈ близкодСйствия, взаимодСйствиС элСктричСских зарядов q1 ΠΈ q2 Π΅ΡΡ‚ΡŒ Ρ€Π΅Π·ΡƒΠ»ΡŒΡ‚Π°Ρ‚ дСйствия поля заряда q1 Π½Π° Π·Π°Ρ€ΡΠ΄ q2 ΠΈ Π½Π°ΠΎΠ±ΠΎΡ€ΠΎΡ‚. Π›ΡŽΠ±ΠΎΠ΅ элСктричСскоС ΠΏΠΎΠ»Π΅ ΠΎΠ±Π»Π°Π΄Π°Π΅Ρ‚ Π½Π°ΠΏΡ€ΡΠΆΠ΅Π½Π½ΠΎΡΡ‚ΡŒΡŽ. ΠΠ°ΠΏΡ€ΡΠΆΠ΅Π½Π½ΠΎΡΡ‚ΡŒ элСктричСского поля — это физичСская Π²Π΅Π»ΠΈΡ‡ΠΈΠ½Π°, равная ΠΎΡ‚Π½ΠΎΡˆΠ΅Π½ΠΈΡŽ силы, с ΠΊΠΎΡ‚ΠΎΡ€ΠΎΠΉ ΠΏΠΎΠ»Π΅ дСйствуСт Π½Π° Ρ‚ΠΎΡ‡Π΅Ρ‡Π½Ρ‹ΠΉ элСктричСский заряд, ΠΊ Π·Π½Π°Ρ‡Π΅Π½ΠΈΡŽ этого заряда. Π’Π΅ΠΊΡ‚ΠΎΡ€ E=Π²Π΅ΠΊΡ‚ΠΎΡ€ΡƒF/q1. Π˜ΡΠΏΠΎΠ»ΡŒΠ·ΡƒΡ Π·Π°ΠΊΠΎΠ½ ΠšΡƒΠ»ΠΎΠ½Π° ΠΈ ΠΎΠΏΡ€Π΅Π΄Π΅Π»Π΅Π½ΠΈΠ΅ понятия напряТСнности поля, ΠΏΠΎΠ»ΡƒΡ‡ΠΈΠΌ Π²Ρ‹Ρ€Π°ΠΆΠ΅Π½ΠΈΠ΅ для модуля напряТСнности элСктричСского поля Π² Π½Π΅ΠΊΠΎΡ‚ΠΎΡ€ΠΎΠΉ Ρ‚ΠΎΡ‡ΠΊΠ΅, А Π½Π° Ρ€Π°ΡΡΡ‚оянии r ΠΎΡ‚ Ρ‚ΠΎΡ‡Π΅Ρ‡Π½ΠΎΠ³ΠΎ заряда. Если Π² Ρ‚ΠΎΡ‡ΠΊΡƒ, А ΠΏΠΎΠΌΠ΅ΡΡ‚ΠΈΡ‚ΡŒ Ρ‚ΠΎΡ‡Π΅Ρ‡Π½Ρ‹ΠΉ заряд, Ρ‚ΠΎ Π½Π° Π½Π΅Π³ΠΎ Π±ΡƒΠ΄Π΅Ρ‚ Π΄Π΅ΠΉΡΡ‚Π²ΠΎΠ²Π°Ρ‚ΡŒ сила F=kq1q2/r2 ΠžΡ‚ ΠΊΡƒΠ΄Π° слСдуСт E=kq/r2. Π²Π΅ΠΊΡ‚ΠΎΡ€F= q* Π²Π΅ΠΊΡ‚ΠΎΡ€ E.

2. 2. УскорСниС, ΡΠΊΠΎΡ€ΠΎΡΡ‚ΡŒ ΠΈ ΠΏΠ΅Ρ€Π΅ΠΌΠ΅Ρ‰Π΅Π½ΠΈΠ΅ ΠΏΡ€ΠΈ равноускорСнном прямолинСйном Π΄Π²ΠΈΠΆΠ΅Π½ΠΈΠΈ.

РавноускорСнноС прямолинСйноС, Ρ€Π°Π²Π½ΠΎΠ·Π°ΠΌΠ΅Π΄Π»Π΅Π½Π½ΠΎΠ΅.

ΠŸΡ€ΠΈ Ρ€Π°Π²Π½ΠΎΠΌΠ΅Ρ€Π½ΠΎΠΌ прямолинСйном Π΄Π²ΠΈΠΆΠ΅Π½ΠΈΠΈ с ΠΏΠΎΡΡ‚оянной ΡΠΊΠΎΡ€ΠΎΡΡ‚ΡŒΡŽ U Π²Π΅ΠΊΡ‚ΠΎΡ€ скорости Π² ΠΊΠ°ΠΆΠ΄ΠΎΠΉ Ρ‚ΠΎΡ‡ΠΊΠ΅ Π½Π°ΠΏΡ€Π°Π²Π»Π΅Π½ вдоль Ρ‚Ρ€Π°Π΅ΠΊΡ‚ΠΎΡ€ΠΈΠΈ.

БрСдняя ΡΠΊΠΎΡ€ΠΎΡΡ‚ΡŒ ΠΈ Ρ‡ΠΈΡΠ»Π΅Π½Π½ΠΎΠ΅ Π·Π½Π°Ρ‡Π΅Π½ΠΈΠ΅ ΠΌΠ³Π½ΠΎΠ²Π΅Π½Π½ΠΎΠΉ — Ρ€Π°Π²Π½Ρ‹, ΠΏΡ€ΠΈ Ρ‚Π°ΠΊΠΎΠΌ Π΄Π²ΠΈΠΆΠ΅Π½ΠΈΠΈ ускорСниС, Π° ΠΎΡΡ‚аСтся Π²Π΅Π»ΠΈΡ‡ΠΈΠ½ΠΎΠΉ постоянной, ΠΏΡ€ΠΈΡ‡Π΅ΠΌ Π½ΠΎΡ€ΠΌΠ°Π»ΡŒΠ½Π°Ρ ΡΠΎΡΡ‚Π°Π²Π»ΡΡŽΡ‰Π°Ρ Ρ€Π°Π²Π½Π° 0.

Если Π½Π°ΠΏΡ€Π°Π²Π»Π΅Π½ΠΈΠ΅ ускорСния совпадаСт с Π½Π°ΠΏΡ€Π°Π²Π»Π΅Π½ΠΈΠ΅ΠΌ скорости, Ρ‚ΠΎ Π΄Π²ΠΈΠΆΠ΅Π½ΠΈΠ΅ называСтся — равноускорСнным, Π° Π΅ΡΠ»ΠΈ Π½Π΅ ΡΠΎΠ²ΠΏΠ°Π΄Π°Π΅Ρ‚ — Ρ‚ΠΎ, Ρ€Π°Π²Π½ΠΎΠ·Π°ΠΌΠ΅Π΄Π»Π΅Π½Π½Ρ‹ΠΌ.

* * ΠŸΡ€ΡΠΌΠΎΠ»ΠΈΠ½Π΅ΠΉΠ½ΠΎΠ΅ Π΄Π²ΠΈΠΆΠ΅Π½ΠΈΠ΅, ΠΏΡ€ΠΈ ΠΊΠΎΡ‚ΠΎΡ€ΠΎΠΌ ΡΠΊΠΎΡ€ΠΎΡΡ‚ΡŒ Ρ‚Π΅Π»Π° Π·Π° Π»ΡŽΠ±Ρ‹Π΅ Ρ€Π°Π²Π½Ρ‹Π΅ ΠΏΡ€ΠΎΠΌΠ΅ΠΆΡƒΡ‚ΠΊΠΈ Π²Ρ€Π΅ΠΌΠ΅Π½ΠΈ измСняСтся Π½Π° ΠΎΠ΄ΠΈΠ½Π°ΠΊΠΎΠ²ΡƒΡŽ Π²Π΅Π»ΠΈΡ‡ΠΈΠ½Ρƒ, называСтся равноускорСнным прямолинСйным Π΄Π²ΠΈΠΆΠ΅Π½ΠΈΠ΅ΠΌ.

ΠžΠ½ΠΎΠ»ΠΎΠ³ΠΈΡ‡Π½ΠΎ для Ρ€Π°Π²Π½ΠΎΠ·Π°ΠΌΠ΅Π΄Π»Π΅Π½Π½ΠΎΠ³ΠΎ двиТСния с ΡƒΡΠΊΠΎΡ€Π΅Π½ΠΈΠ΅ΠΌ Π°.

Π‘ΠΈΠ»Π΅Ρ‚ 14.

1. 1. Π Π°Π±ΠΎΡ‚Π° ΠΏΡ€ΠΈ ΠΏΠ΅Ρ€Π΅ΠΌΠ΅Ρ‰Π΅Π½ΠΈΠΈ заряда Π² ΡΠ»Π΅ΠΊΡ‚ричСском ΠΏΠΎΠ»Π΅. Π Π°Π·Π½ΠΎΡΡ‚ΡŒ ΠΏΠΎΡ‚Π΅Π½Ρ†ΠΈΠ°Π»ΠΎΠ². НапряТСниС.

2. 2. ДСформация, растяТСниС ΠΈ ΡΠΆΠ°Ρ‚ΠΈΠ΅. Π‘ΠΈΠ»Π° упругости. Π—Π°ΠΊΠΎΠ½ Π“ΡƒΠΊΠ°.

3. 3. (Π—Π°Π΄Π°Ρ‡Π° Π½Π° Ρ€Π°ΡΡ‡Π΅Ρ‚ ΠΈΠΌΠΏΡƒΠ»ΡŒΡΠ° ΠΈ ΡΠ½Π΅Ρ€Π³ΠΈΠΈ Ρ„ΠΎΡ‚ΠΎΠ½Π°).

ΠŸΡƒΡΡ‚ΡŒ Π² ΡΠ»Π΅ΠΊΡ‚ричСском ΠΎΠ΄Π½ΠΎΡ€ΠΎΠ΄Π½ΠΎΠΌ ΠΏΠΎΠ»Π΅ с Π½Π°ΠΏΡ€ΡΠΆΠ΅Π½Π½ΠΎΡΡ‚ΡŒΡŽ Π²Π΅ΠΊΡ‚ΠΎΡ€ Π• ΠΏΡ€ΠΎΠΈΡΡ…ΠΎΠ΄ΠΈΡ‚ ΠΏΠ΅Ρ€Π΅ΠΌΠ΅Ρ‰Π΅Π½ΠΈΠ΅ заряда ΠΏΠΎ Π»ΠΈΠ½ΠΈΠΈ напряТСнности Π½Π° Ρ€Π°ΡΡΡ‚ояниС Π΄Π΅Π»ΡŒΡ‚Π° d=d1-d2, Ρ‚ΠΎΠ³Π΄Π° Ρ€Π°Π±ΠΎΡ‚Π° Ρ€Π°Π²Π½Π° A=F (d1-d2)=qE (d1-d2). Из ΠΌΠ΅Ρ…Π°Π½ΠΈΠΊΠΈ извСстно, Ρ‡Ρ‚ΠΎ ΠΏΡ€ΠΈ ΠΏΠ΅Ρ€Π΅ΠΌΠ΅Ρ‰Π΅Π½ΠΈΠΈ ΠΌΠ΅ΠΆΠ΄Ρƒ двумя Ρ‚ΠΎΡ‡ΠΊΠ°ΠΌΠΈ Π² Π³Ρ€Π°Π²ΠΈΡ‚Π°Ρ†ΠΈΠΎΠ½Π½ΠΎΠΌ ΠΏΠΎΠ»Π΅ Ρ€Π°Π±ΠΎΡ‚Π° силы тяТСсти Π½Π΅ Π·Π°Π²ΠΈΡΠΈΡ‚ ΠΎΡ‚ Ρ‚Ρ€Π°Π΅ΠΊΡ‚ΠΎΡ€ΠΈΠΈ двиТСния Ρ‚Π΅Π»Π°. Π‘ΠΈΠ»Ρ‹ Π³Ρ€Π°Π²ΠΈΡ‚Π°Ρ†ΠΈΠΎΠ½Π½ΠΎΠ³ΠΎ ΠΈ ΡΠ»Π΅ΠΊΡ‚ростатичСского взаимодСйствий ΠΈΠΌΠ΅ΡŽΡ‚ ΠΎΠ΄ΠΈΠ½Π°ΠΊΠΎΠ²ΡƒΡŽ Π·Π°Π²ΠΈΡΠΈΠΌΠΎΡΡ‚ΡŒ ΠΎΡ‚ Ρ€Π°ΡΡΡ‚ояния. Π’Π΅ΠΊΡ‚ΠΎΡ€Ρ‹ силы Π½Π°ΠΏΡ€Π°Π²Π»Π΅Π½Ρ‹ ΠΏΠΎ ΠΏΡ€ΡΠΌΠΎΠΉ, ΡΠΎΠ΅Π΄ΠΈΠ½ΡΡŽΡ‰Π΅ΠΉ Ρ‚ΠΎΡ‡Π΅Ρ‡Π½Ρ‹Π΅ Ρ‚Π΅Π»Π°. ΠžΡ‚ΡΡŽΠ΄Π° слСдуСт, Ρ‡Ρ‚ΠΎ ΠΈ ΠΏΡ€ΠΈ ΠΏΠ΅Ρ€Π΅ΠΌΠ΅Ρ‰Π΅Π½ΠΈΠΈ заряда Π² ΡΠ»Π΅ΠΊΡ‚ричСском ΠΏΠΎΠ»Π΅ ΠΈΠ· ΠΎΠ΄Π½ΠΎΠΉ Ρ‚ΠΎΡ‡ΠΊΠΈ Π² Π΄Ρ€ΡƒΠ³ΡƒΡŽ Ρ€Π°Π±ΠΎΡ‚Π° сил элСктричСского поля Π½Π΅ Π·Π°Π²ΠΈΡΠΈΡ‚ ΠΎΡ‚ Ρ‚Ρ€Π°Π΅ΠΊΡ‚ΠΎΡ€ΠΈΠΈ двиТСния. Π Π°Π±ΠΎΡ‚Ρ‹ сил элСктростатичСского поля ΠΏΠΎ Π·Π°ΠΌΠΊΠ½ΡƒΡ‚ΠΎΠΉ Ρ‚Ρ€Π°Π΅ΠΊΡ‚ΠΎΡ€ΠΈΠΈ Ρ€Π°Π²Π½Π° Π½ΡƒΠ»ΡŽ. ПолС, Ρ€Π°Π±ΠΎΡ‚Π° сил ΠΊΠΎΡ‚ΠΎΡ€ΠΎΠ³ΠΎ ΠΏΠΎ Π·Π°ΠΌΠΊΠ½ΡƒΡ‚ΠΎΠΉ Ρ‚Ρ€Π°Π΅ΠΊΡ‚ΠΎΡ€ΠΈΠΈ Ρ€Π°Π²Π½Π° Π½ΡƒΠ»ΡŽ, называСтся ΠΏΠΎΡ‚Π΅Π½Ρ†ΠΈΠ°Π»ΡŒΠ½Ρ‹ΠΌ. И Π³Ρ€Π°Π²ΠΈΡ‚Π°Ρ†ΠΈΠΎΠ½Π½ΠΎΠ΅ ΠΏΠΎΠ»Π΅, ΠΈ ΠΏΠΎΠ»Π΅ элСктростатичСскоС ΡΠ²Π»ΡΡŽΡ‚ΡΡ ΠΏΠΎΡ‚Π΅Π½Ρ†ΠΈΠ°Π»ΡŒΠ½Ρ‹ΠΌΠΈ полями.

ΠŸΡ€ΠΈ ΠΏΠ΅Ρ€Π΅ΠΌΠ΅Ρ‰Π΅Π½ΠΈΠΈ элСктричСского заряда Π² ΡΠ»Π΅ΠΊΡ‚ростатичСском ΠΏΠΎΠ»Π΅ Ρ€Π°Π±ΠΎΡ‚Π° сил Ρ€Π°Π²Π½Π° ΠΏΡ€ΠΎΠΈΠ·Π²Π΅Π΄Π΅Π½ΠΈΡŽ заряда Π½Π° Ρ€Π°Π·Π½ΠΎΡΡ‚ΡŒ ΠΏΠΎΡ‚Π΅Π½Ρ†ΠΈΠ°Π»ΠΎΠ² Π½Π°Ρ‡Π°Π»ΡŒΠ½ΠΎΠΉ ΠΈ ΠΊΠΎΠ½Π΅Ρ‡Π½ΠΎΠΉ Ρ‚ΠΎΡ‡Π΅ΠΊ Ρ‚Ρ€Π°Π΅ΠΊΡ‚ΠΎΡ€ΠΈΠΈ двиТСния заряда. Π’Π°ΠΊ ΠΊΠ°ΠΊ Ρ€Π°Π±ΠΎΡ‚Π° сил элСктростатичСского поля ΠΏΡ€ΠΈ ΠΏΠ΅Ρ€Π΅ΠΌΠ΅Ρ‰Π΅Π½ΠΈΠΈ заряда ΠΈΠ· ΠΎΠ΄Π½ΠΎΠΉ Ρ‚ΠΎΡ‡ΠΊΠΈ пространства Π² Π΄Ρ€ΡƒΠ³ΡƒΡŽ Π½Π΅ Π·Π°Π²ΠΈΡΠΈΡ‚ ΠΎΡ‚ Ρ‚Ρ€Π°Π΅ΠΊΡ‚ΠΎΡ€ΠΈΠΈ двиТСния заряда ΠΌΠ΅ΠΆΡƒ этими Ρ‚ΠΎΡ‡ΠΊΠ°ΠΌΠΈ, Ρ‚ΠΎ Ρ€Π°Π·Π½ΠΎΡΡ‚ΡŒ ΠΏΠΎΡ‚Π΅Π½Ρ†ΠΈΠ°Π»ΠΎΠ² являСтся Π²Π΅Π»ΠΈΡ‡ΠΈΠ½ΠΎΠΉ, Π½Π΅ Π·Π°Π²ΠΈΡΡΡ‰Π΅ΠΉ ΠΎΡ‚ Ρ‚Ρ€Π°Π΅ΠΊΡ‚ΠΎΡ€ΠΈΠΈ двиТСния заряда. Π‘Π»Π΅Π΄ΠΎΠ²Π°Ρ‚Π΅Π»ΡŒΠ½ΠΎ, Ρ€Π°Π·Π½ΠΎΡΡ‚ΡŒ ΠΏΠΎΡ‚Π΅Π½Ρ†ΠΈΠ°Π»ΠΎΠ² ΠΌΠΎΠΆΠ΅Ρ‚ ΡΠ»ΡƒΠΆΠΈΡ‚ΡŒ энСргСтичСской характСристикой элСктростатичСского поля. Π•Π΄ΠΈΠ½ΠΈΡ†Π° разности ΠΏΠΎΡ‚Π΅Π½Ρ†ΠΈΠ°Π»ΠΎΠ² называСтся Π²ΠΎΠ»ΡŒΡ‚ΠΎΠΌ. Если ΠΏΠΎΡ‚Π΅Π½Ρ†ΠΈΠ°Π» элСктростатичСского поля Π½Π° Π±Π΅ΡΠΊΠΎΠ½Π΅Ρ‡Π½ΠΎ большом расстоянии ΠΎΡ‚ Ρ‚ΠΎΡ‡Π΅Ρ‡Π½ΠΎΠ³ΠΎ элСктричСского заряда Π² Π²Π°ΠΊΡƒΡƒΠΌΠ΅ принимаСтся Ρ€Π°Π²Π½Ρ‹ΠΌ Π½ΡƒΠ»ΡŽ, Ρ‚ΠΎ Π½Π° Ρ€Π°ΡΡΡ‚оянии r ΠΎΡ‚ Π·Π°Ρ€ΡΠ΄Π° ΠΎΠ½ ΠΎΠΏΡ€Π΅Π΄Π΅Π»ΡΠ΅Ρ‚ся ΠΏΠΎ Ρ„ΠΎΡ€ΠΌΡƒΠ»Π΅ Ρ„ΠΈ=k/q/r. ΠžΡ‚Π½ΠΎΡˆΠ΅Π½ΠΈΠ΅ Ρ€Π°Π±ΠΎΡ‚Ρ‹ А, ΡΠΎΠ²Π΅Ρ€ΡˆΠ°Π΅ΠΌΡ‹ΠΉ Π»ΡŽΠ±Ρ‹ΠΌ элСктричСским ΠΏΠΎΠ»Π΅ΠΌ ΠΏΡ€ΠΈ ΠΏΠ΅Ρ€Π΅ΠΌΠ΅Ρ‰Π΅Π½ΠΈΠΈ заряда ΠΈΠ· ΠΎΠ΄Π½ΠΎΠΉ Ρ‚ΠΎΡ‡ΠΊΠΈ поля Π² Π΄Ρ€ΡƒΠ³ΡƒΡŽ, ΠΊ Π·Π½Π°Ρ‡Π΅Π½ΠΈΡŽ этого заряда называСтся напряТСниС ΠΌΠ΅ΠΆΠ΄Ρƒ этими Ρ‚ΠΎΡ‡ΠΊΠ°ΠΌΠΈ U=A/q. ΠžΡ‚ΡΡŽΠ΄Π° Ρ€Π°Π±ΠΎΡ‚Π° сил элСктричСского поля ΠΏΡ€ΠΈ ΠΏΠ΅Ρ€Π΅ΠΌΠ΅Ρ‰Π΅Π½ΠΈΠΈ заряда Ρ€Π°Π²Π½Π° ΠΏΡ€ΠΎΠΈΠ·Π²Π΅Π΄Π΅Π½ΠΈΡŽ напряТСния U ΠΌΠ΅ΠΆΠ΄Ρƒ Ρ‚ΠΎΡ‡ΠΊΠ°ΠΌΠΈ Π½Π° Π·Π°Ρ€ΡΠ΄ q А=q*U. Π’ ΡΠ»Π΅ΠΊΡ‚ростатичСском ΠΏΠΎΠ»Π΅ напряТСниС ΠΌΠ΅ΠΆΠ΄Ρƒ двумя Π»ΡŽΠ±Ρ‹ΠΌΠΈ Ρ‚ΠΎΡ‡ΠΊΠ°ΠΌΠΈ Ρ€Π°Π²Π½ΠΎ разности ΠΏΠΎΡ‚Π΅Π½Ρ†ΠΈΠ°Π»ΠΎΠ² этих Ρ‚ΠΎΡ‡Π΅ΠΊ. U12=Ρ„ΠΈ1-Ρ„ΠΈ2. НапряТСниС Ρ…Π°Ρ€Π°ΠΊΡ‚Π΅Ρ€ΠΈΠ·ΡƒΠ΅Ρ‚ элСктричСскоС ΠΏΠΎΠ»Π΅, ΠΊΠΎΡ‚ΠΎΡ€ΠΎΠ΅ создаСт Ρ‚ΠΎΠΊ. НапряТСниС ΠΏΠΎΠΊΠ°Π·Ρ‹Π²Π°Π΅Ρ‚, ΠΊΠ°ΠΊΡƒΡŽ Ρ€Π°Π±ΠΎΡ‚Ρƒ ΡΠΎΠ²Π΅Ρ€ΡˆΠΈΡ‚ элСктричСскоС ΠΏΠΎΠ»Π΅ ΠΏΡ€ΠΈ ΠΏΠ΅Ρ€Π΅ΠΌΠ΅Ρ‰Π΅Π½ΠΈΠΈ ΠΌΠ΅ΠΆΡƒ Ρ‚ΠΎΡ‡ΠΊΠ°ΠΌΠΈ поля заряда Π² 1 Кл. N=A/t A=N*t U=N*t/q = N/I. Бвязь напряТСния с Π½Π°ΠΏΡ€ΡΠΆΠ΅Π½Π½ΠΎΡΡ‚ΡŒΡŽ поля. ΠŸΡ€ΠΈ ΠΏΠ΅Ρ€Π΅ΠΌΠ΅Ρ‰Π΅Π½ΠΈΠΈ ΠΏΠΎΠ»ΠΎΠΆΠΈΡ‚Π΅Π»ΡŒΠ½ΠΎΠ³ΠΎ заряда ΠΏΠΎ Π»ΠΈΠ½ΠΈΠΈ напряТСнности ΠΎΠ΄Π½ΠΎΡ€ΠΎΠ΄Π½ΠΎΠ³ΠΎ поля Π½Π° Ρ€Π°ΡΡΡ‚ояниС Π΄ ΠΊΡƒΠ»ΠΎΠ½ΠΎΠ²ΡΠΊΠ°Ρ сила ΡΠΎΠ²Π΅Ρ€ΡˆΠ°Π΅Ρ‚ Ρ€Π°Π±ΠΎΡ‚Ρƒ A-F*d=qEd A=U*q qEd=Uq U=Ed E=U/d.

2. 2. ДСформация, растяТСниС ΠΈ ΡΠΆΠ°Ρ‚ΠΈΠ΅. Π‘ΠΈΠ»Π° упругости. Π—Π°ΠΊΠΎΠ½ Π“ΡƒΠΊΠ°.

ДСформация это процСсс измСнСния Ρ„ΠΎΡ€ΠΌΡ‹ ΠΈ Ρ€Π°Π·ΠΌΠ΅Ρ€ΠΎΠ² Ρ‚Π΅Π»Π°. ДСформация Π• — это бСзразмСрная Π²Π΅Π»ΠΈΡ‡ΠΈΠ½Π°, равная ΠΎΡ‚Π½ΠΎΡˆΠ΅Π½ΠΈΡŽ Ρ€Π°Π·ΠΌΠ΅Ρ€Π° издСлия Π΄Π΅Π»ΡŒΡ‚Π° эль ΠΊ ΠΈΡΡ…ΠΎΠ΄Π½ΠΎΠΌΡƒ Ρ€Π°Π·ΠΌΠ΅Ρ€Ρƒ эль Π½ΡƒΠ»Π΅Π²ΠΎΠ΅. ΠœΠ΅Ρ…Π°Π½ΠΈΡ‡Π΅ΡΠΊΠΎΠ΅ напряТСниС — Π²Π΅Π»ΠΈΡ‡ΠΈΠ½Π°, Ρ…Π°Ρ€Π°ΠΊΡ‚Π΅Ρ€ΠΈΠ·ΡƒΡŽΡ‰Π°Ρ ΡƒΠΏΡ€ΡƒΠ³ΠΈΠ΅ силы Π½Π° Π΅Π΄ΠΈΠ½ΠΈΡ†Ρƒ ΠΏΠ»ΠΎΡ‰Π°Π΄ΠΈ, числСнно раная ΠΎΡ‚Π½ΠΎΡˆΠ΅Π½ΠΈΡŽ силы упругости ΠΊ ΠΏΠ»ΠΎΡ‰Π°Π΄ΠΈ ΠΏΠΎΠΏΠ΅Ρ€Π΅Ρ‡Π½ΠΎΠ³ΠΎ сСчСния ΠΎΠ±Ρ€Π°Π·Ρ†Π°.

Π—Π°ΠΊΠΎΠ½ Π“ΡƒΠΊΠ°. Ряд растяТСния ΠΈΠ»ΠΈ сТатия, Ρ…Π°Ρ€Π°ΠΊΡ‚Π΅Ρ€ΠΈΠ·ΡƒΡŽΡ‰Π΅Π³ΠΎΡΡ Π²Π΅ΠΊΡ‚ΠΎΡ€ΠΎΠΌ Π΄Π΅Ρ„ΠΎΡ€ΠΌΠ°Ρ†ΠΈΠΈ (удлинСния ΠΈΠ»ΠΈ сТатия) Π΄Π΅Π»ΡŒΡ‚Π° l: сила упругости ΠΏΡ€ΠΎΠΏΠΎΡ€Ρ†ΠΈΠΎΠ½Π°Π»ΡŒΠ½Π° Π²Π΅ΠΊΡ‚ΠΎΡ€Ρƒ Π΄Π΅Ρ„ΠΎΡ€ΠΌΠ°Ρ†ΠΈΠΈ ΠΈ ΠΏΡ€ΠΎΡ‚ΠΈΠ²ΠΎΠΏΠΎΠ»ΠΎΠΆΠ½Π° Π΅ΠΌΡƒ ΠΏΠΎ Π½Π°ΠΏΡ€Π°Π²Π»Π΅Π½ΠΈΡŽ. ΠœΠ΅Ρ…Π°Π½ΠΈΡ‡Π΅ΡΠΊΠΎΠ΅ напряТСниС Π²ΠΎΠ·Π½ΠΈΠΊΠ°ΡŽΡ‰Π°Ρ Π² ΠΎΠ±Ρ€Π°Π·Ρ†Π΅ ΠΏΡ€ΠΎΠΏΠΎΡ€Ρ†ΠΈΠΎΠ½Π°Π»ΡŒΠ½Π° ΠΎΡ‚Π½ΠΎΡΠΈΡ‚Π΅Π»ΡŒΠ½ΠΎΠΌΡƒ ΡƒΠ΄Π»ΠΈΠ½Π΅Π½ΠΈΡŽ сигма=EE.

Π‘ΠΈΠ³ΠΌΠ°=F/S, F/S=E*Π΄Π΅Π»ΡŒΡ‚Π°l/l0 F=(ES/l0)*Π΄Π΅Π»ΡŒΡ‚Π° l. F=k*Π΄Π΅Π»ΡŒΡ‚Π° l.

Π–Π΅ΡΡ‚ΠΊΠΎΡΡ‚ΡŒ K=ES/l0. Упругая Π΄Π΅Ρ„. — Π΄Π΅Ρ„, ΠΏΡ€ΠΈ ΠΊΠΎΡ‚ΠΎΡ€ΠΎΠΌ ΠΏΡ€ΠΈ снятии Π½Π°Π³Ρ€ΡƒΠ·ΠΊΠΈ ΠΎΠ±Ρ€Π°Π·Π΅Ρ† восстанавливаСт свою Ρ„ΠΎΡ€ΠΌΡƒ. ΠŸΠ»Π°ΡΡ‚ΠΈΡ‡Π½Π°Ρ Π½Π°ΠΎΠ±ΠΎΡ€ΠΎΡ‚. ΠŸΠ»Π°ΡΡ‚ΠΈΡ‡Π½Π°Ρ дСформация происходит ΠΏΡƒΡ‚Π΅ΠΌ Π²Π·Π°ΠΈΠΌΠ½Ρ‹Ρ… сдвигов сосСдних слоСв ΠΌΠ°Ρ‚Π΅Ρ€ΠΈΠ°Π»Π°, ΠΏΡ€ΠΈΡ‡Π΅ΠΌ эти сдвиги ΠΈΠΌΠ΅ΡŽΡ‚ Π½Π΅ΠΎΠ±Ρ€Π°Ρ‚ΠΈΠΌΡ‹ΠΉ Ρ…Π°Ρ€Π°ΠΊΡ‚Π΅Ρ€. Запас прочности Π²Π΅Π»ΠΈΡ‡ΠΈΠ½Π°, ΠΏΠΎΠΊΠ°Π·Ρ‹Π²Π°ΡŽΡ‰Π°Ρ Π²ΠΎ ΡΠΊΠΎΠ»ΡŒΠΊΠΎ Ρ€Π°Π· ΠΏΡ€Π΅Π΄Π΅Π» прочности большС допустимой Π½Π°Π³Ρ€ΡƒΠ·ΠΊΠΈ. ДСформация: растяТСниС, сТатиС, сдвиг, ΠΈΠ·Π³ΠΈΠ±, ΠΊΡ€ΡƒΡ‡Π΅Π½ΠΈΠ΅.

Π‘ΠΈΠ»Π΅Ρ‚ 15.

1. 1. Π­Π”Π‘. Π—Π°ΠΊΠΎΠ½ Ома для ΠΏΠΎΠ»Π½ΠΎΠΉ Ρ†Π΅ΠΏΠΈ. ΠžΠΏΡ€Π΅Π΄Π΅Π»Π΅Π½ΠΈΠ΅ сопротивлСния ΠΏΡ€ΠΎΠ²ΠΎΠ΄Π½ΠΈΠΊΠΎΠ².

2. 2. Π‘Π²ΠΎΠ±ΠΎΠ΄Π½Ρ‹Π΅ колСбания Π² ΠΌΠ΅Ρ…аничСских ΠΈ ΡΠ»Π΅ΠΊΡ‚ричСских ΠΊΠΎΠ»Π΅Π±Π°Ρ‚Π΅Π»ΡŒΠ½Ρ‹Ρ… систСмах. Частота свободных ΠΊΠΎΠ»Π΅Π±Π°Π½ΠΈΠΉ. Π—Π°Ρ‚ΡƒΡ…Π°Π½ΠΈΠ΅ свободных ΠΊΠΎΠ»Π΅Π±Π°Π½ΠΈΠΉ.

3. 3. (Π—Π°Π΄Π°Ρ‡Π° Π½Π° ΠΈΡΠΏΠΎΠ»ΡŒΠ·ΠΎΠ²Π°Π½ΠΈΠ΅ Π³Ρ€Π°Ρ„ΠΈΠΊΠΎΠ² изопроцСссов Π² Π³Π°Π·Π°Ρ…).

1. 1. Полная Ρ€Π°Π±ΠΎΡ‚Π° сил элСктростатичСского поля ΠΏΡ€ΠΈ Π΄Π²ΠΈΠΆΠ΅Π½ΠΈΠΈ зарядов ΠΏΠΎ Π·Π°ΠΌΠΊΠ½ΡƒΡ‚ΠΎΠΉ Ρ†Π΅ΠΏΠΈ постоянного Ρ‚ΠΎΠΊΠ° Ρ€Π°Π²Π½Π° Π½ΡƒΠ»ΡŽ. Π‘Π»Π΅Π΄ΠΎΠ²Π°Ρ‚Π΅Π»ΡŒΠ½ΠΎ, вся Ρ€Π°Π±ΠΎΡ‚Π° элСктричСского Ρ‚ΠΎΠΊΠ° Π² Π·Π°ΠΌΠΊΠ½ΡƒΡ‚ΠΎΠΉ элСктричСской Ρ†Π΅ΠΏΠΈ ΡΠΎΠ²Π΅Ρ€ΡˆΠ°Π΅Ρ‚ΡΡ Π·Π° ΡΠ΅Ρ‚ сторонних сил, Π²Ρ‹Π·Ρ‹Π²Π°ΡŽΡ‰ΠΈΡ… Ρ€Π°Π·Π΄Π΅Π»Π΅Π½ΠΈΠ΅ зарядов Π²Π½ΡƒΡ‚Ρ€ΠΈ источника Ρ‚ΠΎΠΊΠ° ΠΈ ΠΏΠΎΠ΄Π΄Π΅Ρ€ΠΆΠΈΠ²Π°ΡŽΡ‰ΠΈΡ… Π½Π° Π΅Π³ΠΎ Π²Ρ‹Ρ…ΠΎΠ΄Π΅ постоянноС напряТСниС. Π­Π”Π‘ — Π²Π΅Π»ΠΈΡ‡ΠΈΠ½Π°, Ρ…Π°Ρ€Π°ΠΊΡ‚Π΅Ρ€ΠΈΠ·ΡƒΡŽΡ‰Π°Ρ ΡΠΏΠΎΡΠΎΠ±Π½ΠΎΡΡ‚ΡŒ источника Ρ‚ΠΎΠΊΠ° ΡΠΎΠ²Π΅Ρ€ΡˆΠ°Ρ‚ΡŒ Ρ€Π°Π±ΠΎΡ‚Ρƒ ΠΏΠΎ Ρ€Π°Π·Π΄Π΅Π»Π΅Π½ΠΈΡŽ заряда, Ρ‡.Ρ€. ΠΎΡ‚Π½ΠΎΡˆΠ΅Π½ΠΈΡŽ Ρ€Π°Π±ΠΎΡ‚Ρ‹ сторонних сил, ΡΠΎΠ²Π΅Ρ€ΡˆΠ°Π΅ΠΌΠΎΠΉ сторонними силами ΠΏΠΎ ΠΏΠ΅Ρ€Π΅ΠΌΠ΅Ρ‰Π΅Π½ΠΈΡŽ заряда q Π²Π΄ΠΎΠ»ΡŒ Ρ†Π΅ΠΏΠΈ, ΠΊ Π·Π½Π°Ρ‡Π΅Π½ΠΈΡŽ этого заряда. E=A/Π΄Π΅Π»ΡŒΡ‚Π°q. ВыраТаСтся Π² Ρ‚Π΅Ρ… ΠΆΠ΅ Π΅Π΄ΠΈΠ½ΠΈΡ†Π°Ρ…, Ρ‡Ρ‚ΠΎ ΠΈ Π½Π°ΠΏΡ€ΡΠΆΠ΅Π½ΠΈΠ΅.

Если Π² Ρ€Π΅Π·Ρ‰ΡƒΠ»ΡŒΡ‚Π°Ρ‚Π΅ прохоТдСния Ρ‚ΠΎΠΊΠ° Π² Π·Π°ΠΌΠΊΠ½ΡƒΡ‚ΠΎΠΉ элСктричСской Ρ†Π΅ΠΏΠΈ происходит Ρ‚ΠΎΠ»ΡŒΠΊΠΎ Π½Π°Π³Ρ€Π΅Π²Π°Π½ΠΈΠ΅ ΠΏΡ€ΠΎΠ²ΠΎΠ΄Π½ΠΈΠΊΠΎ, Ρ‚ΠΎ ΠΏΠΎ Π·Π°ΠΊΠΎΠ½Ρƒ сохранСния энСргии полная Ρ€Π°Π±ΠΎΡ‚Π° элСктричСского Ρ‚ΠΎΠΊΠ° Π² Π·Π°ΠΌΠΊΠ½ΡƒΡ‚ΠΎΠΉ Ρ†Π΅ΠΏΠΈ, равная Ρ€Π°Π±ΠΎΡ‚Π΅ сторонних сил источника Ρ‚ΠΎΠΊΠ°, Ρ€Π°Π²Π½Π° количСству Ρ‚Π΅ΠΏΠ»ΠΎΡ‚Ρ‹, Π²Ρ‹Π΄Π΅Π»ΠΈΠ²ΡˆΠ΅ΠΉΡΡ Π½Π° Π²Π½Π΅ΡˆΠ½Π΅ΠΌ ΠΈ Π²Π½ΡƒΡ‚Ρ€Π΅Π½Π½Π΅ΠΌ участках Ρ†Π΅ΠΏΠΈ: А=Аст=Аполн. Π’Π°ΠΊ ΠΊΠ°ΠΊ Аст=Π΄Π΅Π»ΡŒΡ‚Π°q*(Π­Π”Π‘), Π° ΠΏΠΎΠ»Π½Π°Ρ QΠΏΠΎΠ»Π½=I2(R+r)*Π΄Π΅Π»ΡŒΡ‚Π°t, Ρ‚ΠΎ Π΄Π΅Π»ΡŒΡ‚Π°q*e=I2*(R+r)*Π΄Π΅Π»ΡŒΡ‚Π° t; Π΄Π΅Π»ΡŒΡ‚Π°q=I*Π΄Π΅Π»ΡŒΡ‚Π° t. Π—ΠΠΠ§Π˜Π’ E=I (R+r) I=E/R+r. Π‘ΠΈΠ»Π° Ρ‚ΠΎΠΊΠ° Π² ΡΠ»Π΅ΠΊΡ‚ричСской Ρ†Π΅ΠΏΠΈ прямо ΠΏΡ€ΠΎΠΏΠΎΡ€Ρ†ΠΈΠΎΠ½Π°Π»ΡŒΠ½Π° элСктродвиТущСй силС источника Ρ‚ΠΎΠΊΠ° ΠΈ ΠΎΠ±Ρ€Π°Ρ‚Π½ΠΎ ΠΏΠΎΠ»Π½ΠΎΠΌΡƒ ΡΠΎΠΏΡ€ΠΎΡ‚ΠΈΠ²Π»Π΅Π½ΠΈΡŽ Ρ†Π΅ΠΏΠΈ.

R=Ρ€ΠΎl/s, Π³Π΄Π΅ Ρ€ΠΎ — ΡƒΠ΄Π΅Π»ΡŒΠ½ΠΎΠ΅ сопротивлСниС ΠΏΡ€ΠΎΠ²ΠΎΠ΄Π½ΠΈΠΊΠ°.

2. 2. Π‘Π²ΠΎΠ±ΠΎΠ΄Π½Ρ‹Π΅ колСбания Π² ΠΌΠ΅Ρ…аничСских ΠΈ ΡΠ»Π΅ΠΊΡ‚ричСских ΠΊΠΎΠ»Π΅Π±Π°Ρ‚Π΅Π»ΡŒΠ½Ρ‹Ρ… систСмах. Частота свободных ΠΊΠΎΠ»Π΅Π±Π°Π½ΠΈΠΉ. Π—Π°Ρ‚ΡƒΡ…Π°Π½ΠΈΠ΅ свободных ΠΊΠΎΠ»Π΅Π±Π°Π½ΠΈΠΉ.

ΠŸΡ€ΠΎΡΡ‚Π΅ΠΉΡˆΠ°Ρ элСктричСская Ρ†Π΅ΠΏΡŒ, Π² ΠΊΠΎΡ‚ΠΎΡ€ΠΎΠΉ ΠΌΠΎΠ³ΡƒΡ‚ Π²ΠΎΠ·Π½ΠΈΠΊΠ°Ρ‚ΡŒ свободныС элСктромагнитныС колСбания Π·Π° ΡΡ‡Π΅Ρ‚ ΠΏΠ΅Ρ€Π²ΠΎΠ½Π°Ρ‡Π°Π»ΡŒΠ½ΠΎΠ³ΠΎ сообщСния ΠΉ ΡΠ½Π΅Ρ€Π³ΠΈΠΈ, состоит ΠΈΠ· ΠΊΠΎΠ½Π΄Π΅Π½ΡΠ°Ρ‚ΠΎΡ€Π° ΠΈ ΠΊΠ°Ρ‚ΡƒΡˆΠΊΠΈ индуктивности. Π’Π°ΠΊΡƒΡŽ систСму Π½Π°Π·Ρ‹Π²Π°ΡŽΡ‚ ΠΊΠΎΠ»Π΅Π±Π°Ρ‚Π΅Π»ΡŒΠ½Ρ‹ΠΌ ΠΊΠΎΠ½Ρ‚ΡƒΡ€ΠΎΠΌ. ΠŸΡ€ΠΈ присоСдинСнии ΠΊ ΠΊΠΎΠ½Π΄Π΅Π½ΡΠ°Ρ‚ΠΎΡ€Ρƒ ΠΊΠ°Ρ‚ΡƒΡˆΠΊΠΈ индуктивности ΠΎΠ½ Π½Π°Ρ‡ΠΈΠ°Π΅Ρ‚ Ρ€Π°Π·Ρ€ΡΠΆΠ°Ρ‚ΡŒΡΡ, Π² Ρ†Π΅ΠΏΠΈ появляСтся элСктричСский Ρ‚ΠΎΠΊ. Π‘ΠΈΠ»Π° Ρ‚ΠΎΠΊΠ° возрастаСт постСпСнно, это обусловлСно явлСниСм самоиндкции. ΠŸΡ€ΠΈ появлСнии Ρ‚ΠΎΠΊΠ° Π²ΠΎΠ·Π½ΠΈΠΊΠ°Π΅Ρ‚ ΠΏΠ΅Ρ€Π΅ΠΌΠ΅Π½Π½ΠΎΠ΅ ΠΌΠ°Π³Π½ΠΈΡ‚Π½ΠΎΠ΅ ΠΏΠΎΠ»Π΅. Оно ΠΏΠΎΡ€Π°ΠΆΠ΄Π°Π΅Ρ‚ Π² ΠΏΡ€ΠΎΠ²ΠΎΠ΄Π½ΠΈΠΊΠ΅ Π²ΠΈΡ…Ρ€Π΅Π²ΠΎΠ΅ элСктричСскоС ΠΏΠΎΠ»Π΅. Оно ΠΏΡ€ΠΈ нарастании ΠΌΠ°Π³Π½ΠΈΡ‚Π½ΠΎΠ³ΠΎ поля Π½Π°ΠΏΡ€Π°Π²Π»Π΅Π½ΠΎ ΠΏΡ€ΠΎΡ‚ΠΈΠ² Ρ‚ΠΎΠΊΠ° ΠΈ ΠΈ ΠΏΡ€Π΅ΠΏΡΡ‚ствуСт Π΅Π³ΠΎ ΠΌΠ³Π½ΠΎΠ²Π΅Π½Π½ΠΎΠΌΡƒ ΡƒΠ²Π΅Π»ΠΈΡ‡Π΅Π½ΠΈΡŽ. По ΠΌΠ΅Ρ€Π΅ разрядки кондСнсатора энСргия элСктричСского поля ΡƒΠΌΠ΅Π½ΡŒΡˆΠ°Π΅Ρ‚ΡΡ, Π½ΠΎ ΠΎΠ΄Π½ΠΎΠ²Ρ€Π΅ΠΌΠ΅Π½Π½ΠΎ возрастаСт энСргия ΠΌΠ°Π³Π½ΠΈΡ‚Π½ΠΎΠ³ΠΎ поля Ρ‚ΠΎΠΊΠ°. ΠšΠΎΠ½Π΄Π΅Π½ΡΠ°Ρ‚ΠΎΡ€ ΠΏΠΎΠ»Π½ΠΎΡΡ‚ΡŒΡŽ разрядился, вся энСргия Π² ΠΊΠ°Ρ‚ΡƒΡˆΠΊΠ΅. Но Π½Π΅ ΡΠΌΠΎΡ‚ря Π½Π° Ρ‚ΠΎ, Ρ‡Ρ‚ΠΎ напряТСниС Ρ€Π°Π²Π½ΠΎ Π½ΡƒΠ»ΡŽ, элСктричСский Ρ‚ΠΎΠΊ Π½Π΅ ΠΏΡ€Π΅ΠΊΡ€Π°Ρ‰Π°Π΅Ρ‚ся, прСпятставуСт явлСниС самоиндукции. Как Ρ‚ΠΎΠ»ΡŒΠΊΠΎ сила Ρ‚ΠΎΠΊΠ° ΠΈ ΡΠΎΠ·Π΄Π°Π½Π½ΠΎΠ΅ ΠΈΠΌ ΠΌΠ°Π³Π½ΠΈΡ‚Π½ΠΎΠ΅ ΠΏΠΎΠ»Π΅ Π½Π°Ρ‡Π½ΡƒΡ‚ ΡƒΠΌΠ΅Π½ΡŒΡˆΠ°Ρ‚ΡŒΡΡ, Π²ΠΎΠ·Π½ΠΈΠΊΠ½Π΅Ρ‚ Π²ΠΈΡ…Ρ€Π΅Π²ΠΎΠ΅ элСктричСскоС, ΠΊΠΎΡ‚ΠΎΡ€ΠΎΠ΅ Π±ΡƒΠ΄ΡƒΡ‡ΠΈ Π½Π°ΠΏΡ€Π°Π²Π»Π΅Π½Π½ΠΎΠ΅ ΠΏΠΎ Ρ‚ΠΎΠΊΡƒ, Π½Π°Ρ‡Π½Π΅Ρ‚ Π΅Π³ΠΎ ΠΏΠΎΠ΄Π΄Π΅Ρ€ΠΆΠΈΠ²Π°Ρ‚ΡŒ. Π’ Ρ€Π΅Π·ΡƒΠ»ΡŒΡ‚Π°Ρ‚Π΅ кондСнсатор пСрСзаряТаСтся Π΄ΠΎ Ρ‚Π΅Ρ… ΠΏΠΎΡ€, ΠΏΠΎΠΊΠ° Ρ‚ΠΎΠΊ, постСпСнно ΡƒΠΌΠ΅Π½ΡŒΡˆΠ°ΡΡΡŒ, Π½Π΅ ΡΡ‚Π°Π½Π΅Ρ‚ Ρ€Π°Π²Π½Ρ‹ΠΌ Π½ΡƒΠ»ΡŽ. ΠŸΡ€ΠΈΠΌΠ΅Ρ€ΠΎΠΌ ΠΌΠΎΠΆΠ΅Ρ‚ ΡΠ»ΡƒΠΆΠΈΡ‚ΡŒ ΠΏΠ΅Ρ€Π΅ΠΌΠ΅Π½Π½Ρ‹ΠΉ Ρ‚ΠΎΠΊ Π² ΠΎΡΠ²Π΅Ρ‚ΠΈΡ‚Π΅Π»ΡŒΠ½ΠΎΠΉ эСктросСтях. ΠœΠ΅Ρ…Π°Π½ΠΈΡ‡Π΅ΡΠΊΠΈΠ΅ колСбания — это ΠΏΠΎΠΎΡ‡Π΅Ρ€Π΅Π΄Π½Ρ‹Π΅ пСриоичСскиС виТСния Ρ‚ΠΎΠ΅Π»Π° Π² Π΄Π²ΡƒΡ… ΠΏΡ€ΠΎΡ‚ΠΈΠ²ΠΎΠΏΠΎΠ»ΠΎΠΆΠ½Ρ‹ полоТСниях. Π§Π°Ρ‚ΠΎΡ‚ΠΎΠΉ ΠΊΠΎΠ»Π΅Π±Π°ΠΈΠΉ Π½Π°Π·Ρ‹Π²Π°ΡŽΡ‚ число ΠΊΠΎΠ»Π΅Π±Π°Π½ΠΈΠΉ, ΡΠΎΠ²Π΅Ρ€ΡˆΠ°Π΅ΠΌΡ‹Ρ… Ρ‚Π΅Π»ΠΎΠΌ Π·Π° ΠΎΠ΄Π½Ρƒ сСкунду — cобствСнная частота. Ню=1/T. Π’=2ΠΏ*ΠΊΠΎΡ€Π΅Π½ΡŒ ΠΈΠ· LC.

ВсС свободныС колСбания — Π·Π°Ρ‚ΡƒΡ…Π°ΡŽΡ‰ΠΈΠ΅.

Π‘ΠΈΠ»Π΅Ρ‚ 16.

1. 1. ΠœΠ°Π³Π½ΠΈΡ‚Π½ΠΎΠ΅ ΠΏΠΎΠ»Π΅ Ρ‚ΠΎΠΊΠ°. ΠœΠ°Π³Π½ΠΈΡ‚Π½Π°Ρ индукция. Π‘ΠΈΠ»Π° Π›ΠΎΡ€Π΅Π½Ρ†Π°. Π‘ΠΈΠ»Π° АмпСра.

2. 2. ВСрмоядСрная рСакция. ΠŸΠ΅Ρ€ΡΠΏΠ΅ΠΊΡ‚ΠΈΠ²Ρ‹ ΠΈ ΠΏΡ€ΠΎΠ±Π»Π΅ΠΌΡ‹ развития ядСрной энСргСтики.

3. 3. (Π—Π°Π΄Π°Ρ‡Π° Π½Π° ΠΈΡΠΏΠΎΠ»ΡŒΠ·ΠΎΠ²Π°Π½ΠΈΠ΅ зависимостСй кинСматичСских Π²Π΅Π»ΠΈΡ‡ΠΈΠ½).

Подобно Ρ‚ΠΎΠΌΡƒ, ΠΊΠ°ΠΊ Π² ΠΏΡ€ΠΎΡΡ‚ранствС, ΠΎΠΊΡ€ΡƒΠΆΠ°ΡŽΡ‰Π΅ΠΌ Π½Π΅ΠΏΠΎΠ΄Π²ΠΈΠΆΠ½Ρ‹Π΅ элСктричСскиС заряды, Π²ΠΎΠ·Π½ΠΈΠΊΠ°Π΅Ρ‚ элСктричСскоС ΠΏΠΎΠ»Π΅, Π² ΠΏΡ€ΠΎΡΡ‚ранствС, ΠΎΠΊΡ€ΡƒΠΆΠ°ΡŽΡ‰Π΅ΠΌ ΠΏΡ€ΠΎΠ²ΠΎΠ΄Π½ΠΈΠΊΠΈ с Ρ‚ΠΎΠΊΠΎΠΌ, Π²ΠΎΠ·Π½ΠΈΠΊΠ°Π΅Ρ‚ ΠΌΠ°Π³Π½ΠΈΡ‚Π½ΠΎΠ΅ ΠΏΠΎΠ»Π΅. ΠœΠ°Π³Π½ΠΈΡ‚Π½ΠΎΠ΅ ΠΏΠΎΠ»Π΅ прСдставляСт собой особый Π²ΠΈΠ΄ ΠΌΠ°Ρ‚Π΅Ρ€ΠΈΠΈ, посрСдством ΠΊΠΎΡ‚ΠΎΡ€ΠΎΠ³ΠΎ осущСствляСтся взаимодСйствиС ΠΌΠ΅ΠΆΠ΄Ρƒ двиТущимися элСктричСскими заряТСнными частицами. ΠžΡΠ½ΠΎΠ²Π½Ρ‹Π΅ свойства ΠΌΠ°Π³Π½ΠΈΡ‚Π½ΠΎΠ³ΠΎ поля: ΠΌΠ°Π³Π½ΠΈΡ‚Π½ΠΎΠ΅ ΠΏΠΎΠ»Π΅ пороТдаСтся элСктричСским ΠΏΠΎΠ»Π΅ΠΌ. ΠœΠ°Π³Π½ΠΈΡ‚Π½ΠΎΠ΅ ΠΏΠΎΠ»Π΅ обнаруТиваСтся ΠΏΠΎ Π΄Π΅ΠΉΡΡ‚Π²ΠΈΡŽ Π½Π° Ρ‚ΠΎΠΊ. ΠœΠ°Π³Π½ΠΈΡ‚Π½ΠΎΠ΅ ΠΏΠΎΠ»Π΅ ΠΌΠ°Ρ‚Π΅Ρ€ΠΈΠ°Π»ΡŒΠ½ΠΎ, ΠΎΠ½ΠΎ дСйствуСт Π½Π° Ρ‚Π΅Π»Π°, Π° ΡΠ»Π΅Π΄ΠΎΠ²Π°Ρ‚Π΅Π»ΡŒΠ½ΠΎ, ΠΎΠ±Π»Π°Π΄Π°Π΅Ρ‚ энСргиСй. Π­ΠΊΡΠΏΠ΅Ρ€ΠΈΠΌΠ΅Π½Ρ‚Π°Π»ΡŒΠ½Ρ‹ΠΌ Π΄ΠΎΠΊΠ°Π·Π°Ρ‚Π΅Π»ΡŒΡΡ‚Π²ΠΎΠΌ Ρ€Π΅Π°Π»ΡŒΠ½ΠΎΡΡ‚ΠΈ ΠΌΠ°Π³Π½ΠΈΡ‚Π½ΠΎΠ³ΠΎ поля являСтся Ρ„Π°ΠΊΡ‚ сущСствования элСктромагнитных Π²ΠΎΠ»Π½. Π‘ΠΈΠ»Π°, с ΠΊΠΎΡ‚ΠΎΡ€ΠΎΠΉ ΠΌΠ°Π³Π½ΠΈΡ‚Π½ΠΎΠ΅ ΠΏΠΎΠ»Π΅ дСйствуСт Π½Π° ΠΏΡ€ΠΎΠ²ΠΎΠ΄Π½ΠΈΠΊ с Ρ‚ΠΎΠΊΠΎΠΌ, называСтся силой АмпСра. Для характСристики способности ΠΌΠ°Π³Π½ΠΈΡ‚Π½ΠΎΠ³ΠΎ поля ΠΎΠΊΠ°Π·Ρ‹Π²Π°Ρ‚ΡŒ силовоС воздСйствиС Π½Π° ΠΏΡ€ΠΎΠ²ΠΎΠ΄Π½ΠΈΠΊ с Ρ‚ΠΎΠΊΠΎΠΌ вводится вСкторная Π²Π΅Π»ΠΈΡ‡ΠΈΠ½Π° — магнитная индукция Π²Π΅ΠΊΡ‚ΠΎΡ€ Π’ [Π’Π»]. F=BILsin (Π°Π»ΡŒΡ„Π°). Π‘ΠΈΠ»Ρƒ, Π΄Π΅ΠΉΡΡ‚Π²ΡƒΡŽΡ‰ΡƒΡŽ Π½Π° Π΄Π²ΠΈΠΆΡƒΡ‰ΡƒΡŽΡΡ Π·Π°Ρ€ΡΠΆΠ΅Π½Π½ΡƒΡŽ частицу со ΡΡ‚ΠΎΡ€ΠΎΠ½Ρ‹ ΠΌΠ°Π³Π½ΠΈΡ‚Π½ΠΎΠ³ΠΎ поля, Π½Π°Π·Ρ‹Π²Π°ΡŽΡ‚ — силой Π›ΠΎΡ€Π΅Π½Ρ†Π°. F=B*q*V*sin (Π°Π»ΡŒΡ„Π°).

2. ВСрмоядСрная рСакция. ΠŸΠ΅Ρ€ΡΠΏΠ΅ΠΊΡ‚ΠΈΠ²Ρ‹ ΠΈ ΠΏΡ€ΠΎΠ±Π»Π΅ΠΌΡ‹ развития ядСрной энСргСтики.

Π―Π΄Π΅Ρ€Π½ΡƒΡŽ ΡΠ½Π΅Ρ€Π³ΠΈΡŽ ΠΌΠΎΠΆΠ½ΠΎ ΠΏΠΎΠ»ΡƒΡ‡ΠΈΡ‚ΡŒ двумя способами: Π΄Π΅Π»Π΅Π½ΠΈΠ΅ΠΌ тяТСлых ядСр ΠΈ ΡΠΈΠ½Ρ‚Π΅Π·ΠΎΠΌ Π»Π΅Π³ΠΊΠΈΡ… ядСр. Π―Π΄Π΅Ρ€Π½Ρ‹ΠΉ синтСз, происходящий Π² Ρ€Π°Π·ΠΎΠ³Ρ€Π΅Ρ‚ΠΎΠΌ вСщСствС, Π½Π°Π·Ρ‹Π²Π°ΡŽΡ‚ тСрмоядСрной Ρ€Π΅Π°ΠΊΡ†ΠΈΠ΅ΠΉ. ΠŸΡ€Π΅ΠΆΠ΄Π΅ всСго Π½ΡƒΠΆΠ½ΠΎ Π½Π°Π³Ρ€Π΅Ρ‚ΡŒ тСрмоядСрноС Π³ΠΎΡ€ΡŽΡ‡ΠΈΠ΅ Π΄ΠΎ Ρ‚Π΅ΠΌΠΏΠ΅Ρ€Π°Ρ‚ΡƒΡ€Ρ‹, ΠΊΠΎΠ³Π΄Π° Ρ€Π΅Π°ΠΊΡ†ΠΈΠΈ синтСза ΠΌΠΎΠ³ΡƒΡ‚ ΠΏΡ€ΠΎΠΈΡΡ…ΠΎΠ΄ΠΈΡ‚ΡŒ с Π·Π°ΠΌΠ΅Ρ‚Π½ΠΎΠΉ Π²Π΅Ρ€ΠΎΡΡ‚Π½ΠΎΡΡ‚ΡŒΡŽ. НСобходимо, Ρ‡Ρ‚ΠΎΠ±Ρ‹ ΠΏΡ€ΠΈ синтСзС Π²Ρ‹Π΄Π΅Π»ΡΠ»ΠΎΡΡŒ большС энСргии, Ρ‡Π΅ΠΌ затрачиваСтся Π΅Π΅ Π½Π° Π½Π°Π³Ρ€Π΅Π² вСщСства, ΠΈΠ»ΠΈ Ρ‡Ρ‚ΠΎΠ±Ρ‹ Ρ€ΠΎΠΆΠ΄Π°ΡŽΡ‰ΠΈΠ΅ΡΡ ΠΏΡ€ΠΈ синтСзС быстрыС частицы сами ΠΏΠΎΠ΄Π΄Π΅Ρ€ΠΆΠΈΠ²Π°Π»ΠΈ Ρ‚Ρ€Π΅Π±ΡƒΠ΅ΠΌΡƒΡŽ Ρ‚Π΅ΠΌΠΏΠ΅Ρ€Π°Ρ‚ΡƒΡ€Ρƒ Π³ΠΎΡ€ΡŽΡ‡Π΅Π³ΠΎ. Для этого Π½ΡƒΠΆΠ½ΠΎ, Ρ‡Ρ‚ΠΎΠ±Ρ‹ Π²ΡΡ‚ΡƒΠΏΠ°ΡŽΡ‰Π΅Π΅ Π² ΡΠΈΠ½Ρ‚Π΅Π· вСщСство Π±Ρ‹Π»ΠΎ Π½Π°Π΄Π΅ΠΆΠ½ΠΎ Ρ‚Π΅ΠΏΠ»ΠΎΠΈΠ·ΠΎΠ»ΠΈΡ€ΠΎΠ²Π°Π½Π½ΠΎ ΠΎΡ‚ ΠΎΠΊΡ€ΡƒΠΆΠ°ΡŽΡ‰Π΅ΠΉ Ρ…ΠΎΠ»ΠΎΠ΄Π½ΠΎΠΉ срСды, Ρ‚Π΅ Ρ‡Ρ‚ΠΎΠ±Ρ‹ врСмя остывания Π±Ρ‹Π»ΠΎ достаточно Π²Π΅Π»ΠΈΠΊΠΎ. Π›Π΅Π³Ρ‡Π΅ всСго ΠΎΡΡƒΡ‰Π΅ΡΡ‚Π²Π»ΡΡ‚ΡŒ синтСз ΠΌΠ΅ΠΆΠ΄Ρƒ тяТСлыми ΠΈΠ·ΠΎΡ‚ΠΎΠΏΠ°ΠΌΠΈ Π²ΠΎΠ΄ΠΎΡ€ΠΎΠ΄Π° — Π΄Π΅ΠΉΡ‚Π΅Ρ€ΠΈΠ΅ΠΌ 2/1Н ΠΈ Ρ‚Ρ€ΠΈΡ‚ΠΈΠ΅ΠΌ 3/1Н. ΠŸΡ€ΠΈ этом получаСтся ядро гСлия ΠΈ Π½Π΅ΠΉΡ‚Ρ€ΠΎΠ½ 4/2Н + 1/0n. Π’Π°ΠΊ ΠΊΠ°ΠΊ тСпловая ΡΠΊΠΎΡ€ΠΎΡΡ‚ΡŒ ΠΈΠΎΠ½ΠΎΠ² Π²ΠΎΠ΄ΠΎΡ€ΠΎΠ΄Π° составляСт 108 см/с, Π·Π° 1 с ΠΈΠΎΠ½Ρ‹ ΠΏΡ€ΠΎΠ»Π΅Ρ‚Π°ΡŽΡ‚ 1000 ΠΊΠΌ, Ρ‚Π΅ Π½ΡƒΠΆΠ½Ρ‹ ΡΠΏΠ΅Ρ†ΠΈΠ°Π»ΡŒΠ½Ρ‹Π΅ устройства, ΠΏΡ€Π΅Π΄ΠΎΡ‚Π²Ρ€Π°Ρ‰Π°ΡŽΡ‰ΠΈΠ΅ ΠΏΠΎΠΏΠ°Π΄Π°Π½ΠΈΠ΅ ΠΏΠ»Π°Π·ΠΌΡ‹ Π½Π° ΡΡ‚Π΅Π½ΠΊΠΈ. Плазма — Π³Π°Π·, состоящий ΠΈΠ· ΡΠΌΠ΅ΡΠΈ элСктронов ΠΈ ΠΈΠΎΠ½ΠΎΠ². На Π·Π°Ρ€ΡΠΆΠ΅Π½Π½Ρ‹Π΅ частицы, двиТущиСся ΠΏΠΎΠΏΠ΅Ρ€Π΅ΠΊ ΠΌΠ°Π³Π½ΠΈΡ‚Π½ΠΎΠ³ΠΎ поля, дСйствуСт сила, ΠΈΡΠΊΡ€ΠΈΠ²Π»ΡΡŽΡ‰Π°Ρ ΠΈΡ… Ρ‚Ρ€Π°Π΅ΠΊΡ‚ΠΎΡ€ΠΈΡŽ. Π’Π΅ ΠΌΠ°Π³Π½ΠΈΡ‚Π½ΠΎΠ΅ ΠΏΠΎΠ»Π΅ ΠΌΠΎΠΆΠ΅Ρ‚ ΠΏΡ€Π΅Π΄ΠΎΡ‚Π²Ρ€Π°Ρ‚ΠΈΡ‚ΡŒ ΡƒΡ…ΠΎΠ΄ заряТСнных частиц, Π½ΠΎ ΡΡ‚ΠΎΠ³ΠΎ ΠΌΠ°Π»ΠΎ. МоТно ΡΠ΄Π΅Π»Π°Ρ‚ΡŒ «ΠΌΠ°Π³Π½ΠΈΡ‚Π½Ρ‹Π΅ ΠΏΡ€ΠΎΠ±ΠΊΠΈ» — области с Π±ΠΎΠ»Π΅Π΅ ΡΠΈΠ»ΡŒΠ½Ρ‹ΠΌ ΠΌΠ°Π³Π½ΠΈΡ‚Π½Ρ‹ΠΌ ΠΏΠΎΠ»Π΅ΠΌ, ΠΎΡ‚Ρ€Π°ΠΆΠ°ΡŽΡ‰ΠΈΠ΅ Ρ‡Π°ΡΡ‚ΡŒ частиц. Но Π»ΡƒΡ‡ΡˆΠ΅ всСго «ΡΠ²Π΅Ρ€Π½ΡƒΡ‚ΡŒ» силовыС Π»ΠΈΠ½ΠΈΠΈ Π² ΠΊΠΎΠ»ΡŒΡ†ΠΎ, ΠΈΡΠΏΠΎΠ»ΡŒΠ·ΠΎΠ²Π°Ρ‚ΡŒ Ρ‚ΠΎΡ€ΠΎΠΈΠ΄Π°Π»ΡŒΠ½ΠΎΠ΅ ΠΌΠ°Π³Π½ΠΈΡ‚Π½ΠΎΠ΅ ΠΏΠΎΠ»Π΅. Оно Π½Π΅ΠΎΠ΄Π½ΠΎΡ€ΠΎΠ΄Π½ΠΎ Π² ΠΏΡ€ΠΎΡΡ‚ранствС — Π΅Π³ΠΎ Π½Π°ΠΏΡ€ΡΠΆΠ΅Π½Π½ΠΎΡΡ‚ΡŒ спадаСт ΠΏΠΎ Ρ€Π°Π΄ΠΈΡƒΡΡƒ, Π° Π² Π½Π΅ΠΎΠ΄Π½ΠΎΡ€ΠΎΠ΄Π½ΠΎΠΌ ΠΏΠΎΠ»Π΅ Π²ΠΎΠ·Π½ΠΈΠΊΠ°Π΅Ρ‚ ΠΌΠ΅Π΄Π»Π΅Π½Π½ΠΎΠ΅ Π΄Π²ΠΈΠΆΠ΅Π½ΠΈΠ΅ заряТСнных частиц — Ρ‚Π°ΠΊ Π½Π°Π·Ρ‹Π²Π°Π΅ΠΌΡ‹ΠΉ Π΄Ρ€Π΅ΠΉΡ„. Π•Π³ΠΎ ΠΌΠΎΠΆΠ½ΠΎ Π»ΠΈΠΊΠ²ΠΈΠ΄ΠΈΡ€ΠΎΠ²Π°Ρ‚ΡŒ, пропустив Ρ‡Π΅Ρ€Π΅Π· ΠΏΠ»Π°Π·ΠΌΡƒ Ρ‚ΠΎΠΊ вдоль ΠΎΠ±Ρ…ΠΎΠ΄Π° Ρ‚ΠΎΡ€Π°. Π’ΠΎΠ³Π΄Π° ΠΏΠΎΠ»Π΅ получится Π²ΠΈΠ½Ρ‚ΠΎΠ²Ρ‹ΠΌ. Π”Π²ΠΈΠ³Π°ΡΡΡŒ ΠΏΠΎ ΡΠΏΠΈΡ€Π°Π»ΠΈ вдоль силовых Π»ΠΈΠ½ΠΈΠΉ, заряТСнныС частицы Π±ΡƒΠ΄ΡƒΡ‚ ΠΏΠ΅Ρ€Π΅Ρ…ΠΎΠ΄ΠΈΡ‚ΡŒ ΠΈΠ· Π²Π΅Ρ€Ρ…Π½Π΅ΠΉ полуплоскости Ρ‚ΠΎΡ€Π° Π² Π½ΠΈΠΆΠ½ΡŽΡŽ ΠΈ ΠΎΠ±Ρ€Π°Ρ‚Π½ΠΎ. ИмСнно Ρ‚Π°ΠΊ устроСна магнитная систСма установок Π’ΠΎΠΊΠ°ΠΌΠ°ΠΊ. ΠšΡ€ΠΎΠΌΠ΅ тСрмоизоляции ΠΏΠ»Π°Π·ΠΌΡ‹ Π½Π΅ΠΎΠ±Ρ…ΠΎΠ΄ΠΈΠΌΠΎ ΠΎΠ±Π΅ΡΠΏΠ΅Ρ‡ΠΈΡ‚ΡŒ Π΅Π΅ Π½Π°Π³Ρ€Π΅Π². Π’ Π’ΠΎΠΊΠ°ΠΌΠ°ΠΊΠ΅ для этой Ρ†Π΅Π»ΠΈ ΠΈΡΠΏΠΎΠ»ΡŒΠ·ΡƒΠ΅Ρ‚ΡΡ Ρ‚ΠΎΠΊ, ΠΏΡ€ΠΎΡ‚Π΅ΠΊΠ°ΡŽΡ‰ΠΈΠΉ ΠΏΠΎ ΠΏΠ»Π°Π·ΠΌΠ΅Π½Π½ΠΎΠΌΡƒ ΡˆΠ½ΡƒΡ€Ρƒ.

Π‘ΠΈΠ»Π΅Ρ‚ 17.

1. 1. Π―Π²Π»Π΅Π½ΠΈΠ΅ элСктромагнитной ΠΈΠ½Π΄ΡƒΠΊΡ†ΠΈΠΈ. Π—Π°ΠΊΠΎΠ½ элСктромагнитной ΠΈΠ½Π΄ΡƒΠΊΡ†ΠΈΠΈ. ΠŸΡ€Π°Π²ΠΈΠ»ΠΎ Π›Π΅Π½Ρ†Π°.

2. 2. ΠŸΡ€ΠΈΠ½Ρ†ΠΈΠΏ дСйствия Ρ‚Π΅ΠΏΠ»ΠΎΠ²Ρ‹Ρ… Π΄Π²ΠΈΠ³Π°Ρ‚Π΅Π»Π΅ΠΉ. ΠšΠŸΠ” Ρ‚Π΅ΠΏΠ»ΠΎΠ²ΠΎΠ³ΠΎ двигатСля. Роль Ρ‚Π΅ΠΏΠ»ΠΎΠ²Ρ‹Ρ… машин.

3. 3. (Π—Π°Π΄Π°Ρ‡Π° Π½Π° Π²Ρ‹Ρ…ΠΎΠ΄ ядСрной Ρ€Π΅Π°ΠΊΡ†ΠΈΠΈ).

1. 1. Π―Π²Π»Π΅Π½ΠΈΠ΅ элСктромагнитной ΠΈΠ½Π΄ΡƒΠΊΡ†ΠΈΠΈ Π·Π°ΠΊΠ»ΡŽΡ‡Π°Π΅Ρ‚ΡΡ Π² Π²ΠΎΠ·Π½ΠΈΠΊΠ½ΠΎΠ²Π΅Π½ΠΈΠΈ элСктричСского Ρ‚ΠΎΠΊΠ° Π² Π·Π°ΠΌΠΊΠ½ΡƒΡ‚ΠΎΠΌ элСктропроводящСм ΠΊΠΎΠ½Ρ‚ΡƒΡ€Π΅ ΠΏΡ€ΠΈ ΠΈΠ·ΠΌΠ΅Π½Π΅Π½ΠΈΠΈ ΠΌΠ°Π³Π½ΠΈΡ‚Π½ΠΎΠ³ΠΎ ΠΏΠΎΡ‚ΠΎΠΊΠ° Ρ‡Π΅Ρ€Π΅Π· ΠΏΠ»ΠΎΡ‰Π°Π΄ΡŒ этого ΠΊΠΎΠ½Ρ‚ΡƒΡ€Π°. По ΠΏΡ€Π°Π²ΠΈΠ»Ρƒ Π›Π΅Π½Ρ†Π°, Π²ΠΎΠ·Π½ΠΈΠΊΠ°ΡŽΡ‰ΠΈΠΉ Π² Π·Π°ΠΌΠΊΠ½ΡƒΡ‚ΠΎΠΌ ΠΊΠΎΠ½Ρ‚ΡƒΡ€Π΅ ΠΈΠ½Π΄ΡƒΠΊΡ†ΠΈΠΎΠ½Π½Ρ‹ΠΉ Ρ‚ΠΎΠΊ Π½Π°ΠΏΡ€Π°Π²Π»Π΅Π½ Ρ‚Π°ΠΊ, Ρ‡Ρ‚ΠΎ создаваСмый ΠΈΠΌ ΠΌΠ°Π³Π½ΠΈΡ‚Π½Ρ‹ΠΉ ΠΏΠΎΡ‚ΠΎΠΊ Ρ‡Π΅Ρ€Π΅Π· ΠΏΠ»ΠΎΡ‰Π°Π΄ΡŒ, ΠΎΠ³Ρ€Π°Π½ΠΈΡ‡Π΅Π½Π½ΡƒΡŽ ΠΊΠΎΠ½Ρ‚ΡƒΡ€ΠΎΠΌ, ΡΡ‚Ρ€Π΅ΠΌΠΈΡ‚ΡŒΡΡ ΠΏΡ€Π΅ΠΏΡΡ‚ΡΡ‚Π²ΠΎΠ²Π°Ρ‚ΡŒ Ρ‚ΠΎΠΌΡƒ измСнСнию ΠΏΠΎΡ‚ΠΎΠΊΠ°, ΠΊΠΎΡ‚ΠΎΡ€ΠΎΠ΅ Π²Ρ‹Π·Ρ‹Π²Π°Π΅Ρ‚ Π΄Π°Π½Π½Ρ‹ΠΉ Ρ‚ΠΎΠΊ. Π―Π²Π»Π΅Π½ΠΈΠ΅ ЭИ Π½Π°Ρ…ΠΎΠ΄ΠΈΡ‚ ΡˆΠΈΡ€ΠΎΠΊΠΎΠ΅ ΠΏΡ€ΠΈΠΌΠ΅Π½Π΅Π½ΠΈΠ΅ Π² Ρ‚Π΅Ρ…Π½ΠΈΠΊΠ΅. Оно ΠΈΡΠΏΠΎΠ»ΡŒΠ·ΡƒΠ΅Ρ‚ΡΡ Π² ΠΈΠ½Π΄ΡƒΠΊΡ†ΠΈΠΎΠ½Π½Ρ‹Ρ… Π³Π΅Π½Π΅Ρ€Π°Ρ‚ΠΎΡ€Π°Ρ… Ρ‚ΠΎΠΊΠ°, ΠΈΠ½Π΄ΡƒΠΊΡ†ΠΈΠΎΠ½Π½Ρ‹Ρ… ΠΏΠ»Π°Π²ΠΈΠ»ΡŒΠ½Ρ‹Ρ… ΠΏΠ΅Ρ‡Π°Ρ…, трансформаторах, Π² ΡΡ‡Π΅Ρ‚Ρ‡ΠΈΠΊΠ°Ρ… элСктроэнСргии ΠΈ Π΄Ρ€.

Π—Π°ΠΊΠΎΠ½ Π­Π˜ формулируСтся ΠΈΠΌΠ΅Π½Π½ΠΎ для Π­Π”Π‘ ΠΈΠ½Π΄ΡƒΠΊΡ†ΠΈΠΈ, Π° Π½Π΅ Π΄Π»Ρ силы ΠΈΠ½Π΄ΡƒΠΊΡ†ΠΈΠΎΠ½Π½ΠΎΠ³ΠΎ Ρ‚ΠΎΠΊΠ°: Π­Π”Π‘ ΠΈΠ½Π΄ΡƒΠΊΡ†ΠΈΠΈ Π² Π·Π°ΠΌΠΊΠ½ΡƒΡ‚ΠΎΠΌ ΠΊΠΎΠ½Ρ‚ΡƒΡ€Π΅ Ρ€Π°Π²Π½Π° ΠΏΠΎ ΠΌΠΎΠ΄ΡƒΠ»ΡŽ скорости измСнСния ΠΌΠ°Π³Π½ΠΈΡ‚Π½ΠΎΠ³ΠΎ ΠΏΠΎΡ‚ΠΎΠΊΠ° Ρ‡Π΅Ρ€Π΅Π· ΠΏΠΎΠ²Π΅Ρ€Ρ…Π½ΠΎΡΡ‚ΡŒ, ΠΎΠ³Ρ€Π°Π½ΠΈΡ‡Π΅Π½Π½ΡƒΡŽ ΠΊΠΎΠ½Ρ‚ΡƒΡ€ΠΎΠΌ: Ei=ΠΌΠΎΠ΄ΡƒΠ»ΡŒ (Π΄Π΅Π»ΡŒΡ‚Π° Π€/Π΄Π΅Π»ΡŒΡ‚Π° t). Π‘ ΡƒΡ‡Π΅Ρ‚ΠΎΠΌ ΠΏΡ€Π°Π²ΠΈΠ»Π° Π›Π΅Π½Ρ†Π°: Ei=-Π΄Π΅Π»ΡŒΡ‚Π° Π€/Π΄Π΅Π»ΡŒΡ‚Π° t. ΠŸΡ€ΠΈ ΠΈΠ·ΠΌΠ΅Π½Π΅Π½ΠΈΠΈ ΠΌΠ°Π³Π½ΠΈΡ‚Π½ΠΎΠ³ΠΎ ΠΏΠΎΡ‚ΠΎΠΊΠ° Π² ΠΊΠ°Ρ‚ΡƒΡˆΠΊΠ΅, ΠΈΠΌΠ΅ΡŽΡ‰Π΅ΠΉ n ΠΎΠ΄ΠΈΠ½Π°ΠΊΠΎΠ²Ρ‹Ρ… Π²ΠΈΡ‚ΠΊΠΎΠ² ΠΏΡ€ΠΎΠ²ΠΎΠ΄Π°, общая Π­Π”Π‘ ΠΈΠ½Π΄ΡƒΠΊΡ†ΠΈΠΈ Π² n Ρ€Π°Π· большС Π­Π”Π‘ ΠΎΠ΄Π½ΠΎΠ³ΠΎ Π²ΠΈΡ‚ΠΊΠ°.

2. ΠŸΡ€ΠΈΠ½Ρ†ΠΈΠΏ дСйствия Ρ‚Π΅ΠΏΠ»ΠΎΠ²Ρ‹Ρ… Π΄Π²ΠΈΠ³Π°Ρ‚Π΅Π»Π΅ΠΉ. ΠšΠŸΠ” Ρ‚Π΅ΠΏΠ»ΠΎΠ²ΠΎΠ³ΠΎ двигатСля. Роль Ρ‚Π΅ΠΏΠ»ΠΎΠ²Ρ‹Ρ… машин.

Π’Π΅ΠΏΠ»ΠΎΠ²Ρ‹ΠΌ двигатСлями Π½Π°Π·Ρ‹Π²Π°ΡŽΡ‚ ΠΌΠ°ΡˆΠΈΠ½Ρ‹, Π² ΠΊΠΎΡ‚ΠΎΡ€Ρ‹Ρ… внутрСнняя энСргия Ρ‚ΠΎΠΏΠ»ΠΈΠ²Π° прСвращаСтся Π² ΠΌΠ΅Ρ…Π°Π½ΠΈΡ‡Π΅ΡΠΊΡƒΡŽ ΡΠ½Π΅Ρ€Π³ΠΈΡŽ. БСгодня ΠΎΠ΄ΠΈΠ½ ΠΈΠ· ΡΠ°ΠΌΡ‹Ρ… распространСнных Ρ‚Π΅ΠΏΠ»ΠΎΠ²Ρ‹Ρ… Π΄Π²ΠΈΠ³Π°Ρ‚Π΅Π»Π΅ΠΉ являСтся Π”Π’Π‘. ΠŸΡ€ΠΈΠ½Ρ†ΠΈΠΏ дСйствия Π·Π°ΠΊΠ»ΡŽΡ‡Π°Π΅Ρ‚ΡΡ Π² Ρ‚ΠΎΠΌ, Ρ‡Ρ‚ΠΎ энСргия Ρ‚ΠΎΠΏΠ»ΠΈΠ²Π° ΠΏΠ΅Ρ€Π΅Ρ…ΠΎΠ΄ΠΈΡ‚ Π²ΠΎ Π²Π½ΡƒΡ‚Ρ€Π΅Π½Π½ΡŽΡŽ ΡΠ½Π΅Ρ€Π³ΠΈΡŽ ΠΏΠ°Ρ€Π°, Π° ΠΏΠ°Ρ€, Ρ€Π°ΡΡˆΠΈΡ€ΡΡΡΡŒ, ΡΠΎΠ²Π΅Ρ€ΡˆΠ°Π΅Ρ‚ Ρ€Π°Π±ΠΎΡ‚Ρƒ. Π’Π°ΠΊ внутрСнняя энСргия ΠΏΠ°Ρ€Π° прСвращаСтся Π² ΠΊΠΈΠ½Π΅Ρ‚ΠΈΡ‡Π΅ΡΠΊΡƒΡŽ ΡΠ½Π΅Ρ€Π³ΠΈΡŽ ΠΏΠΎΡ€ΡˆΠ½Ρ.

Волько Π² ΠΈΠ΄Π΅Π°Π»ΡŒΠ½Ρ‹Ρ… условиях поная Ρ€Π°Π±ΠΎΡ‚Π° Ρ€Π°Π²Π½Π° Ρ€Π°Π±ΠΎΡ‚Π΅ ΠΏΠΎΠ»Π΅Π·Π½ΠΎΠΉ. ΠžΡ‚Π½ΠΎΡˆΠ΅Π½ΠΈΠ΅ ΠΏΠΎΠ»Π΅Π·Π½ΠΎΠΉ Ρ€Π°Π±ΠΎΡ‚Ρ‹ ΠΊ ΠΏΠΎΠ»Π½ΠΎΠΉ называСтся ΠšΠŸΠ”. ΠšΠŸΠ” любого ΠΌΡ…Π°Π½ΠΈΠ·ΠΌΠ° всСгда мСньшС 100%.

Π‘ΠΈΠ»Π΅Ρ‚ 18.

1. 1. Π­Π»Π΅ΠΊΡ‚Ρ€ΠΎΠΌΠ°Π³Π½ΠΈΡ‚Π½ΠΎΠ΅ ΠΏΠΎΠ»Π΅ ΠΈ Π΅Π³ΠΎ ΠΌΠ°Ρ‚Π΅Ρ€ΠΈΠ°Π»ΡŒΠ½ΠΎΡΡ‚ΡŒ. Π­Π»Π΅ΠΊΡ‚Ρ€ΠΎΠΌΠ°Π³Π½ΠΈΡ‚Π½Ρ‹Π΅ Π²ΠΎΠ»Π½Ρ‹, ΠΈΡ… ΡΠ²ΠΎΠΉΡΡ‚Π²Π°. Радиолокация ΠΈ Π΅Π΅ ΠΏΡ€ΠΈΠΌΠ΅Π½Π΅Π½ΠΈΠ΅.

2. 2. АрхимСдова сила. УсловиС плавания Ρ‚Π΅Π». ПлаваниС судов. Π’ΠΎΠ·Π΄ΡƒΡ…ΠΎΠΏΠ»Π°Π²Π°Π½ΠΈΠ΅. Π˜Π·ΠΌΠ΅Ρ€ΡŒΡ‚Π΅ Π²Ρ‹Ρ‚Π°Π»ΠΊΠΈΠ²Π°ΡŽΡ‰ΡƒΡŽ силу ΠΈΡΠΏΠΎΠ»ΡŒΠ·ΡƒΡ Π΄ΠΈΠ½Π°ΠΌΠΎΠΌΠ΅Ρ‚Ρ€.

3. 3. Π—Π°Π΄Π°Ρ‡Π° Π½Π° ΡƒΡ€Π°Π²Π½Π΅Π½ΠΈΠ΅ состояния.

Π”Π²Π° полоТСния: 1. ИзмСнСниС элСктричСского поля всСгда сопровоТдаСтся ΠΌΠ°Π³Π½ΠΈΡ‚Π½Ρ‹ΠΌ ΠΏΠΎΠ»Π΅ΠΌ. 2. Π˜Π·ΠΌΠ΅Π½ΡΡŽΡ‰Π΅Π΅ΡΡ ΠΌΠ°Π³Π½ΠΈΡ‚Π½ΠΎΠ΅ ΠΏΠΎΠ»Π΅ всСгда сопровоТдаСтся элСктричСским ΠΏΠΎΠ»Π΅ΠΌ. Π—Π°ΠΌΠΊΠ½ΡƒΡ‚ΠΎΡΡ‚ΡŒ ΠΌΠ°Π³Π½ΠΈΡ‚Π½Ρ‹Ρ… ΠΈ ΡΠ»Π΅ΠΊΡ‚ричСских силовых Π»ΠΈΠ½ΠΈΠΉ элСктромагнитного поля — вСсьма Π²Π°ΠΆΠ½ΠΎΠ΅ ΠΏΠΎΠ»ΠΎΠΆΠ΅Π½ΠΈΠ΅ Ρ‚Π΅ΠΎΡ€ΠΈΠΈ МаксвСлла. Π­Π»Π΅ΠΊΡ‚Ρ€ΠΎΠΌΠ°Π³Π½ΠΈΡ‚Π½ΠΎΠ΅ ΠΏΠΎΠ»Π΅ ΠΌΠ°Ρ‚Π΅Ρ€ΠΈΠ°Π»ΡŒΠ½ΠΎ. Π€ΠΈΠ·ΠΈΠΊΠΈ Π·Π½Π°ΡŽΡ‚ Π΄Π²Π΅ Ρ„ΠΎΡ€ΠΌΡ‹ ΠΌΠ°Ρ‚Π΅Ρ€ΠΈΠΈ — вСщСство ΠΈ ΠΏΠΎΠ»Π΅ (элСктромагнитноС, Π³Ρ€Π°Π²ΠΈΡ‚Π°Ρ†ΠΈΠΎΠ½Π½ΠΎΠ΅, внутриядСрноС). Π‘ΠΊΠΎΡ€ΠΎΡΡ‚ΡŒ распространСния элСктромагнитного поля Ρ€Π°Π²Π½Π° скорости распространСния свСта. ΠžΡ‚ΡΡŽΠ΄Π° Π²ΠΎΠ·Π½ΠΈΠΊΠ»Π° идСя, Ρ‡Ρ‚ΠΎ ΠΈ ΡΠ²Π΅Ρ‚ прСдставляСт собой элСктромагнитноС ΠΏΠΎΠ»Π΅. Π­Π»Π΅ΠΊΡ‚Ρ€ΠΎΠΌΠ°Π³Π½ΠΈΡ‚Π½ΠΎΠ΅ ΠΏΠΎΠ»Π΅ — это Ρ‚Π° Ρ‡Π°ΡΡ‚ΡŒ пространства, которая содСрТит Π² ΡΠ΅Π±Π΅ ΠΈ ΠΎΠΊΡ€ΡƒΠΆΠ°Π΅Ρ‚ Ρ‚Π΅Π»Π°, находящиСся Π² ΡΠ»Π΅ΠΊΡ‚ричСском ΠΈΠ»ΠΈ ΠΌΠ°Π³Π½ΠΈΡ‚Π½ΠΎΠΌ состоянии. ΠœΠ°Ρ‚Π΅Ρ€ΠΈΠ°Π»ΡŒΠ½ΠΎΡΡ‚ΡŒ элСктромагнитного поля подтвСрТдаСтся Ρ‚Π΅ΠΌ, Ρ‡Ρ‚ΠΎ Π² Π½Π΅ΠΌ Π½Π°Π±Π»ΡŽΠ΄Π°Π΅Ρ‚ΡΡ дСйствиС сил, Ρ‡Ρ‚ΠΎ ΠΎΠ½ΠΎ являСтся носитСлСм ΠΈ ΠΏΠ΅Ρ€Π΅Π΄Π°Ρ‚Ρ‡ΠΈΠΊΠΎΠΌ энСргии.

ВсС пространство ΠΏΡ€ΠΎΠ½ΠΈΠ·Π°Π½ΠΎ элСктромагнитным ΠΈΠ·Π»ΡƒΡ‡Π΅Π½ΠΈΠ΅ΠΌ. Π­Π»Π΅ΠΊΡ‚Ρ€ΠΎΠΌΠ°Π³Π½ΠΈΡ‚Π½Ρ‹Π΅ Π²ΠΎΠ»Π½Ρ‹ — это процСсс распространСния элСктромагнитных ΠΊΠΎΠ»Π΅Π±Π°Π½ΠΈΠΉ Π² ΠΏΡ€ΠΎΡΡ‚ранствС с ΠΊΠΎΠ½Π΅Ρ‡Π½ΠΎΠΉ ΡΠΊΠΎΡ€ΠΎΡΡ‚ΡŒΡŽ. ΠŸΡ€Π΅Π΄ΡΡ‚Π°Π²ΡŒΡ‚Π΅ сСбС, Ρ‡Ρ‚ΠΎ элСктричСский заряд ΠΏΡ€ΠΈΠ²Π΅Π΄Π΅Π½ Π² Π±Ρ‹ΡΡ‚Ρ€Ρ‹Π΅ колСбания вдоль Π½Π΅ΠΊΠΎΡ‚ΠΎΡ€ΠΎΠΉ прямой. Π’ΠΎΠ³Π΄Π° Π½Π°Ρ‡Π½Π΅Ρ‚ пСриодичСски ΠΈΠ·ΠΌΠ΅Π½ΡΡ‚ΡŒΡΡ ΠΈ ΡΠ»Π΅ΠΊΡ‚ричСскоС ΠΏΠΎΠ»Π΅ Π²ΠΎΠΊΡ€ΡƒΠ³ заряда. ΠŸΡ€ΠΈΡ‡Π΅ΠΌ ΠΏΠ΅Ρ€ΠΈΠΎΠ΄ ΠΈΠ·ΠΌΠ΅Π½Π΅Π½ΠΈΠΉ Π±ΡƒΠ΄Π΅Ρ‚ Ρ€Π°Π²Π΅Π½ ΠΏΠ΅Ρ€ΠΈΠΎΠ΄Ρƒ ΠΊΠΎΠ»Π΅Π±Π°Π½ΠΈΠΉ заряда. ΠŸΠ΅Ρ€Π΅ΠΌΠ΅Π½Π½ΠΎΠ΅ элСктричСскоС ΠΏΠΎΠ»Π΅ Π±ΡƒΠ΄Π΅Ρ‚ ΠΏΠΎΡ€ΠΎΠΆΠ΄Π°Ρ‚ΡŒ пСриодичСски ΠΌΠ΅Π½ΡΡŽΡ‰Π΅Π΅ΡΡ ΠΌΠ°Π³Π½ΠΈΡ‚Π½ΠΎΠ΅ ΠΏΠΎΠ»Π΅, Π° ΠΏΠΎΡΠ»Π΅Π΄Π½Π΅Π΅ Π²Ρ‹Π·ΠΎΠ²Π΅Ρ‚ появлСниС элСктричСского поля ΡƒΠΆΠ΅ Π½Π° Π±ΠΎΠ»ΡŒΡˆΠ΅ΠΌ расстоянии ΠΎΡ‚ Π·Π°Ρ€ΡΠ΄Π°. Π˜Ρ… ΡΠ²ΠΎΠΉΡΡ‚Π²Π°: ΠΏΠΎΠ³Π»ΠΎΡ‰Π΅Π½ΠΈΠ΅, ΠΎΡ‚Ρ€Π°ΠΆΠ΅Π½ΠΈΠ΅, ΠΏΡ€Π΅Π»ΠΎΠΌΠ»Π΅Π½ΠΈΠ΅. Π₯Π°Ρ€Π°ΠΊΡ‚Π΅Ρ€Π½ΠΎ явлСниС ΠΈΠ½Ρ‚Π΅Ρ€Ρ„Π΅Ρ€Π΅Π½Ρ†ΠΈΠΈ (слоТСниС Π²ΠΎΠ»Π½) ΠΈ Π΄ΠΈΡ„Ρ€Π°ΠΊΡ†ΠΈΠΈ (ΠΎΠ³ΠΈΠ±Π°Π½ΠΈΠ΅ прСпятствий).

Радиолокация — ΠΎΠ±Π»Π°ΡΡ‚ΡŒ Π½Π°ΡƒΠΊΠΈ ΠΈ Ρ‚Π΅Ρ…Π½ΠΈΠΊΠΈ, Π·Π°Π½ΠΈΠΌΠ°ΡŽΡ‰Π°ΡΡΡ наблюдСниСм Ρ€Π°Π·Π»ΠΈΡ‡Π½Ρ‹Ρ… ΠΎΠ±ΡŠΠ΅ΠΊΡ‚ΠΎΠ² Π²ΠΎΠ·Π΄ΡƒΡ…Π΅, Π½Π° Π²ΠΎΠ΄Π΅, Π½Π΅ Π·Π΅ΠΌΠ»Π΅ ΠΈ ΠΎΠΏΡ€Π΅Π΄Π΅Π»Π΅Π½ΠΈΠ΅ΠΌ ΠΈΡ… ΠΌΠ΅ΡΡ‚ополоТСния, Π° Ρ‚Π°ΠΊΠΆΠ΅ расстояния Π΄ΠΎ Π½ΠΈΡ… ΠΏΡ€ΠΈ ΠΏΠΎΠΌΠΎΡ‰ΠΈ Ρ€Π°Π΄ΠΈΠΎ. ВсСм Ρ…ΠΎΡ€ΠΎΡˆΠΎ извСстно эхо: Π·Π²ΡƒΠΊ, ΠΎΡ‚Ρ€Π°ΠΆΠ΅Π½Π½Ρ‹ΠΉ ΠΎΡ‚ ΠΏΡ€Π΅ΠΏΡΡ‚ствия. Π’ Ρ€Π°Π΄ΠΈΠΎΠ»ΠΎΠΊΠ°Ρ†ΠΈΠΈ происходит всС Ρ‚Π°ΠΊΠΆΠ΅. Π Π°Π΄ΠΈΠΎΠ»ΠΎΠΊΠ°Ρ‚ΠΎΡ€ посылаСт ΠΈΠΌΠΏΡƒΠ»ΡŒΡ Ρ€Π°Π΄ΠΈΠΎΠ²ΠΎΠ»Π½ Π² ΡΡ‚ΠΎΡ€ΠΎΠ½Ρƒ ΠΎΠ±ΡŠΠ΅ΠΊΡ‚Π° ΠΈ ΠΎΠ±Ρ€Π°Ρ‚Π½ΠΎ. ΠŸΠΎΡΠΊΠΎΠ»ΡŒΠΊΡƒ ΡΠΊΠΎΡ€ΠΎΡΡ‚ΡŒ распространСния Ρ€Π°Π΄ΠΈΠΎΠ²ΠΎΠ»Π½ извСстна, Ρ€Π°Π²Π½Π° скорости свСта, Ρ‚ΠΎ ΠΏΠΎ ΠΈΠ½Ρ‚Π΅Ρ€Π²Π°Π»Ρƒ ΠΌΠ΅ΠΆΠ΄Ρƒ всплСсками элСктронного Π»ΡƒΡ‡Π° Π½Π° ΡΠΊΡ€Π°Π½Π΅ Ρ‚Ρ€ΡƒΠ±ΠΊΠΈ ΠΌΠΎΠΆΠ½ΠΎ ΠΎΠΏΡ€Π΅Π΄Π΅Π»ΠΈΡ‚ΡŒ расстояниС Π΄ΠΎ ΠΎΠ±ΡŠΠ΅ΠΊΡ‚Π°. R=ct/2. Π‘=3*108 ΠΌ/с.

2. АрхимСдова сила. УсловиС плавания Ρ‚Π΅Π». ПлаваниС судов. Π’ΠΎΠ·Π΄ΡƒΡ…ΠΎΠΏΠ»Π°Π²Π°Π½ΠΈΠ΅. Π˜Π·ΠΌΠ΅Ρ€ΡŒΡ‚Π΅ Π²Ρ‹Ρ‚Π°Π»ΠΊΠΈΠ²Π°ΡŽΡ‰ΡƒΡŽ силу ΠΈΡΠΏΠΎΠ»ΡŒΠ·ΡƒΡ Π΄ΠΈΠ½Π°ΠΌΠΎΠΌΠ΅Ρ‚Ρ€.

АрхимСдова сила — сила, Π²Ρ‹Ρ‚Π°Π»ΠΊΠΈΠ²Π°ΡŽΡ‰Π°Ρ ΠΏΠΎΠ³Ρ€ΡƒΠΆΠ΅Π½Π½ΠΎΠ΅ Π² ΠΆΠΈΠ΄ΠΊΠΎΡΡ‚ΡŒ Ρ‚Π΅Π»ΠΎ. FΠ²Ρ‹Ρ‚.=Π ΠΆΠΈΠ΄=g*mΠΆ. Π—Π°ΠΊΠΎΠ½ АрхимСда: На Ρ‚Π΅Π»ΠΎ, ΠΏΠΎΠ³Ρ€ΡƒΠΆΠ΅Π½Π½ΠΎΠ΅ Π² ΠΆΠΈΠ΄ΠΊΠΎΡΡ‚ΡŒ, дСйствуСт Π²Ρ‹Ρ‚Π°Π»ΠΊΠΈΠ²Π°ΡŽΡ‰Π°Ρ сила, равная вСсу вытСснСнной этим Ρ‚Π΅Π»ΠΎΠΌ Тидкости. FΠ°=Π ΠΆ=gm=gVΡ€ΠΎ.

ПолоТСниС Ρ‚Π΅Π»Π° Π² ΠΆΠΈΠ΄ΠΊΠΎΡΡ‚ΠΈ зависит ΠΎΡ‚ ΡΠΎΠΎΡ‚Π½ΠΎΡˆΠ΅Π½ΠΈΡ Π΄Π²ΡƒΡ… сил — тяТСсти ΠΈ ΠΡ€Ρ…ΠΈΠΌΠ΅Π΄ΠΎΠ²ΠΎΠΉ, ΠΏΡ€ΠΈ этом Π²ΠΎΠ·ΠΌΠΎΠΆΠ½Ρ‹ Ρ‚Ρ€ΠΈ случая. Π‘ΠΈΠ»Π° тяТСсти>Π°Ρ€Ρ…ΠΈΠΌΠ΅Π΄ΠΎΠ²ΠΎΠΉ — Ρ‚ΠΎΠ½Π΅Ρ‚. Π Π°Π²Π½Ρ‹ — ΠΏΠ»Π°Π²Π°Π΅Ρ‚. МСньшС — всплываСт. Когда ΠΏΡ€ΠΈ всплывании Ρ‚Π΅Π»Π° сила тяТСсти сравниваСтся с ΠΡ€Ρ…ΠΈΠΌΠ΅Π΄ΠΎΠ²ΠΎΠΉ, Ρ‚Π΅Π»ΠΎ остановится ΠΈ Π±ΡƒΠ΄Π΅Ρ‚ ΠΏΠ»Π°Π²Π°Ρ‚ΡŒ Π½Π° ΠΏΠΎΠ²Π΅Ρ€Ρ…ности Тидкости. ΠžΠΏΡ‹Ρ‚Π½Ρ‹ΠΌ ΠΏΡƒΡ‚Π΅ΠΌ Π΄ΠΎΠΊΠ°Π·Π°Π½ΠΎ, Ρ‡Ρ‚ΠΎ Ссли Ρ‚Π΅Π»ΠΎ ΠΏΠ»Π°Π²Π°Π΅Ρ‚ Π² ΠΆΠΈΠ΄ΠΊΠΎΡΡ‚ΠΈ, Ρ‚ΠΎ Π²Π΅Ρ вытСснСнной ΠΈΠΌ ΠΆΠΈΠ΄ΠΊΠΎΡΡ‚ΠΈ Ρ€Π°Π²Π΅Π½ вСсу этого Ρ‚Π΅Π»Π° Π² Π²ΠΎΠ·Π΄ΡƒΡ…Π΅. Π“Π»ΡƒΠ±ΠΈΠ½Ρƒ, Π½Π° ΠΊΠΎΡ‚ΠΎΡ€ΡƒΡŽ суд6Π½ΠΎ погруТаСтся Π² Π²ΠΎΠ΄Ρƒ, называСтся осадкой. ВатСрлиния. Π Π°Π·ΠΌΠ΅Ρ€Ρ‹ судна Ρ…Π°Ρ€Π°ΠΊΡ‚Π΅Ρ€ΠΈΠ·ΡƒΡŽΡ‚ΡΡ Π² ΠΏΡ€Π΅ΠΆΠ΄Π΅ всСго Π΅Π³ΠΎ Π²ΠΎΠ΄ΠΎΠΈΠ·ΠΌΠ΅Ρ‰Π΅Π½ΠΈΠ΅ΠΌ — вСсом вытСсняСмой судном Π²ΠΎΠ΄Ρ‹ ΠΏΡ€ΠΈ ΠΏΠΎΠ³Ρ€ΡƒΠΆΠ΅Π½ΠΈΠΈ Π΄ΠΎ Π²Π°Ρ‚Π΅Ρ€Π»ΠΈΠ½ΠΈΠΈ, Ρ€Π°Π²Π½Ρ‹ΠΌ силС тяТСсти, Π΄Π΅ΠΉΡΡ‚Π²ΡƒΡŽΡ‰Π΅Π΅ Π½Π° ΡΡƒΠ΄Π½ΠΎ с Π³Ρ€ΡƒΠ·ΠΎΠΌ. Если ΠΈΠ· Π²ΠΎΠ΄ΠΎΠΈΠ·ΠΌΠ΅Ρ‰Π΅Π½ΠΈΡ Π²Ρ‹Ρ‡Π΅ΡΡ‚ΡŒ вСс самого судна, Ρ‚ΠΎ ΠΏΠΎΠ»ΡƒΡ‡ΠΈΡ‚ся Π΅Π³ΠΎ Π³Ρ€ΡƒΠ·ΠΎΠΏΠΎΠ΄ΡŠΠ΅ΠΌΠ½ΠΎΡΡ‚ΡŒ.

Π‘ΠΈΠ»Π΅Ρ‚ 19.

1. 1. Π‘ΠΏΠ΅ΠΊΡ‚Ρ€ элСктромагнитных ΠΈΠ·Π»ΡƒΡ‡Π΅Π½ΠΈΠΉ. Π—Π°Π²ΠΈΡΠΈΠΌΠΎΡΡ‚ΡŒ свойств элСктромагнитных ΠΈΠ·Π»ΡƒΡ‡Π΅Π½ΠΈΠΉ ΠΎΡ‚ Ρ‡Π°ΡΡ‚ΠΎΡ‚Ρ‹. ΠŸΡ€ΠΈΠΌΠ΅Π½Π΅Π½ΠΈΠ΅ элСктромагнитных ΠΈΠ·Π»ΡƒΡ‡Π΅Π½ΠΈΠΉ.

2. 2. ДиспСрсия свСта. Π‘ΠΏΠ΅ΠΊΡ‚Ρ€. БпСктроскоп.

3. «ΠŸΠ ΠžΠ’Π•Π ΠšΠ Π£Π‘Π›ΠžΠ’Π˜Π― Π ΠΠ’ΠΠžΠ’Π•Π‘Π˜Π― РЫЧАГА».

1. 1. Π’ΠΈΠ΄ΠΈΠΌΠΎΠ΅ ΠΈΠ·Π»ΡƒΡ‡Π΅Π½ΠΈΠ΅ (свСт) составляСт Π½ΠΈΡ‡Ρ‚ΠΎΠΆΠ½ΡƒΡŽ Ρ‡Π°ΡΡ‚ΡŒ ΡˆΠΈΡ€ΠΎΠΊΠΎΠ³ΠΎ спСктра элСктромагнитных Π²ΠΎΠ»Π½ ΠΈΠ»ΠΈ ΡˆΠΊΠ°Π»Ρ‹ элСктромагнитных ΠΈΠ·Π»ΡƒΡ‡Π΅Π½ΠΈΠΉ. ΠŸΡ€ΠΈΠ½ΡΡ‚ΠΎ Π²Ρ‹Π΄Π΅Π»ΡΡ‚ΡŒ низкочастотноС ΠΈΠ·Π»ΡƒΡ‡Π΅Π½ΠΈΠ΅, Ρ€Π°Π΄ΠΈΠΎΠΈΠ·Π»ΡƒΡ‡Π΅Π½ΠΈΠ΅, инфракрасныС Π»ΡƒΡ‡ΠΈ, Π²ΠΈΠ΄ΠΈΠΌΡ‹ΠΉ свСт (оптичСский Π΄ΠΈΠ°ΠΏΠ°Π·ΠΎΠ½), ΡƒΠ»ΡŒΡ‚Ρ€Π°Ρ„ΠΈΠΎΠ»Π΅Ρ‚ΠΎΠ²Ρ‹Π΅ Π»ΡƒΡ‡ΠΈ, рСнтгСновскиС Π»ΡƒΡ‡ΠΈ ΠΈ Π³Π°ΠΌΠΌΠ° — ΠΈΠ·Π»ΡƒΡ‡Π΅Π½ΠΈΠ΅. НаиболСС ΠΊΠΎΡ€ΠΎΡ‚ΠΊΠΎΠ²ΠΎΠ»Π½ΠΎΠ²ΠΎΠ΅ Π³Π°ΠΌΠΌΠ°ΠΈΠ·Π»ΡƒΡ‡Π΅Π½ΠΈΠ΅ испускаСтся Π°Ρ‚ΠΎΠΌΠ°ΠΌΠΈ ядрами. ВсС ΠΎΠ½ΠΈ ΠΏΡ€Π΅Π΄ΡΡ‚Π°Π²Π»ΡΡŽΡ‚ собой элСктромагнитныС Π²ΠΎΠ»Π½Ρ‹, ΠΏΠΎΡ€ΠΎΠΆΠ΄Π°Π΅ΠΌΡ‹Π΅ двиТущимися заряТСнными частицами. По ΠΌΠ΅Ρ€Π΅ ΡƒΠΌΠ΅Π½ΡŒΡˆΠ΅Π½ΠΈΡ Π΄Π»ΠΈΠ½Ρ‹ Π²ΠΎΠ»Π½Ρ‹ ΠΏΡ€ΠΎΡΠ²Π»ΡΡŽΡ‚ΡΡ ΠΈ ΡΡƒΡ‰Π΅ΡΡ‚Π²Π΅Π½Π½Ρ‹Π΅ качСствСнныС различия элСктромагнитных Π²ΠΎΠ»Π½. Π˜Π·Π»ΡƒΡ‡Π΅Π½ΠΈΡ Ρ€Π°Π·Π»ΠΈΡ‡Π½Ρ‹Ρ… Π΄Π»ΠΈΠ½ Π²ΠΎΠ»Π½ ΠΎΡ‚Π»ΠΈΡ‡Π°ΡŽΡ‚ΡΡ Π΄Ρ€ΡƒΠ³ ΠΎΡ‚ Π΄Ρ€ΡƒΠ³Π° ΠΏΠΎ ΡΠΏΠΎΡΠΎΠ±Ρƒ ΠΈΡ… ΠΏΠΎΠ»ΡƒΡ‡Π΅Π½ΠΈΡ (ΠΈΠ·Π»ΡƒΡ‡Π΅Π½ΠΈΠ΅ Π°Π½Ρ‚Π΅Π½Π½Ρ‹, Ρ‚Π΅ΠΏΠ»ΠΎΠ²ΠΎΠ΅ ΠΈΠ·Π»ΡƒΡ‡Π΅Π½ΠΈΠ΅, ΠΈΠ·Π»ΡƒΡ‡Π΅Π½ΠΈΠ΅ ΠΏΡ€ΠΈ Ρ‚ΠΎΡ€ΠΌΠΎΠΆΠ΅Π½ΠΈΠΈ быстрых элСктронов) ΠΈ ΠΌΠ΅Ρ‚ΠΎΠ΄ΠΎΠΌ рСгистрации, Ρ‚Π΅ ΠΏΠΎ Ρ…Π°Ρ€Π°ΠΊΡ‚Π΅Ρ€Ρƒ взаимодСйствия с Π²Π΅Ρ‰Π΅ΡΡ‚Π²ΠΎΠΌ. ΠŸΡ€ΠΈΠΌΠ΅Π½Π΅Π½ΠΈΠ΅: ΠΌΠ΅Π΄ΠΈΡ†ΠΈΠ½Π°, диагностика, Π»Π΅Ρ‡Π΅Π½ΠΈΠ΅ Π·Π°Π±ΠΎΠ»Π΅Π²Π°Π½ΠΈΠΉ, Π² Π΄Ρ€ΡƒΠ³ΠΈΡ… Π½Π°ΡƒΠΊΠ°Ρ…, криминалистика, искусствовСдСниС.

2. 2. ДиспСрсия свСта. Π‘ΠΏΠ΅ΠΊΡ‚Ρ€. БпСктроскоп.

ДиспСрсияэто Π·Π°Π²ΠΈΡΠΈΠΌΠΎΡΡ‚ΡŒ показатСля прСломлСния свСта ΠΎΡ‚ Ρ‡Π°ΡΡ‚ΠΎΡ‚Ρ‹ ΠΊΠΎΠ»Π΅Π±Π°Π½ΠΈΠΉ ΠΈΠ»ΠΈ Π΄Π»ΠΈΠ½Ρ‹ Π²ΠΎΠ»Π½Ρ‹. Π‘ΠΏΠ΅ΠΊΡ‚Ρ€ — это Ρ†Π²Π΅Ρ‚Π½Ρ‹Π΅ полосы, ΠΏΠΎΠ»ΡƒΡ‡Π°ΡŽΡ‰ΠΈΠ΅ΡΡ Π² Ρ€Π΅Π·ΡƒΠ»ΡŒΡ‚Π°Ρ‚Π΅ разлоТСния свСта ΠΏΡ€ΠΈΠ·ΠΌΠΎΠΉ ΠΏΠΎ Π΄Π»ΠΈΠ½Π°ΠΌ Π²ΠΎΠ»Π½. Π‘Ρ‹Π²Π°ΡŽΡ‚: Π½Π΅ΠΏΡ€Π΅Ρ€Ρ‹Π²Π½Ρ‹Π΅, Π»ΠΈΠ½Π΅ΠΉΡ‡Π°Ρ‚Ρ‹Π΅ ΠΈ ΡΠΏΠ΅ΠΊΡ‚Ρ€Ρ‹ поглощСния.

НСпрСрывныС спСктры. Π‘ΠΎΠ»Π½Π΅Ρ‡Π½Ρ‹ΠΉ спСктр, спСктр элСктричСской Π»Π°ΠΌΠΏΡ‹ ΡΠ²Π»ΡΡŽΡ‚ΡΡ Π½Π΅ΠΏΡ€Π΅Ρ€Ρ‹Π²Π½Ρ‹ΠΌΠΈ. Π’ ΡΠΏΠ΅ΠΊΡ‚Ρ€Π΅ Π½Π΅Ρ‚ Ρ€Π°Π·Ρ€Ρ‹Π²ΠΎΠ², ΠΈ Π½Π° ΡΠΊΡ€Π°Π½Π΅ ΠΌΠΎΠΆΠ½ΠΎ Π²ΠΈΠ΄Π΅Ρ‚ΡŒ ΡΠΏΠ»ΠΎΡˆΠ½ΡƒΡŽ Ρ†Π²Π΅Ρ‚Π½ΡƒΡŽ полоску. Π’Π°ΠΊΠΈΠ΅ спСктры Π΄Π°ΡŽΡ‚ Ρ‚Π΅Π»Π°, находящиСся Π² Ρ‚Π²Π΅Ρ€Π΄ΠΎΠΌ ΠΈΠ»ΠΈ ΠΆΠΈΠ΄ΠΊΠΎΠΌ состоянии. Для получСния Π²ΠΈΠ΄ΠΈΠΌΠΎΠ³ΠΎ Π½Π΅ΠΏΡ€Π΅Ρ€Ρ‹Π²Π½ΠΎΠ³ΠΎ спСктра Π½ΡƒΠΆΠ½ΠΎ Π½Π°Π³Ρ€Π΅Ρ‚ΡŒ Ρ‚Π΅Π»ΠΎ Π΄ΠΎ Π²Ρ‹ΡΠΎΠΊΠΎΠΉ Ρ‚Π΅ΠΌΠΏΠ΅Ρ€Π°Ρ‚ΡƒΡ€Ρ‹. Π₯Π°Ρ€Π°ΠΊΡ‚Π΅Ρ€ Π½Π΅ΠΏΡ€Π΅Ρ€Ρ‹Π²Π½ΠΎΠ³ΠΎ спСктра Π² ΡΠΈΠ»ΡŒΠ½ΠΎΠΉ стСпСни зависит ΠΎΡ‚ Π²Π·Π°ΠΈΠΌΠΎΠ΄Π΅ΠΉΡΡ‚вия Π°Ρ‚ΠΎΠΌΠΎΠ² Π΄Ρ€ΡƒΠ³ с Π΄Ρ€ΡƒΠ³ΠΎΠΌ.

Π›ΠΈΠ½Π΅ΠΉΡ‡Π°Ρ‚Ρ‹Π΅ спСктры. Π˜Ρ… Π΄Π°ΡŽΡ‚ всС вСщСства, находящиСся Π² Π³Π°Π·ΠΎΠΎΠ±Ρ€Π°Π·Π½ΠΎΠΌ Π°Ρ‚ΠΎΠΌΠ°Ρ€Π½ΠΎΠΌ (Π½ΠΎ Π½Π΅ ΠΌΠΎΠ»Π΅ΠΊΡƒΠ»ΡΡ€Π½ΠΎΠΌ) состоянии. Π’ ΡΡ‚ΠΎΠΌ случаС свСт ΠΈΠ·Π»ΡƒΡ‡Π°ΡŽΡ‚ Π°Ρ‚ΠΎΠΌΡ‹, ΠΊΠΎΡ‚ΠΎΡ€Ρ‹Π΅ практичСски Π½Π΅ Π²Π·Π°ΠΈΠΌΠΎΠ΄Π΅ΠΉΡΡ‚Π²ΡƒΡŽΡ‚ Π΄Ρ€ΡƒΠ³ с Π΄Ρ€ΡƒΠ³ΠΎΠΌ. Π­Ρ‚ΠΎ самый Ρ„ΡƒΠ½Π΄Π°ΠΌΠ΅Π½Ρ‚Π°Π»ΡŒΠ½Ρ‹ΠΉ Ρ‚ΠΈΠΏ спСктров. ΠžΠ±Ρ‹Ρ‡Π½ΠΎ для наблюдСния Π»ΠΈΠ½Π΅ΠΉΡ‡Π°Ρ‚Ρ‹Ρ… спСктров ΠΈΡΠΏΠΎΠ»ΡŒΠ·ΡƒΡŽΡ‚ свСчСниС ΠΏΠ°Ρ€ΠΎΠ² вСщСства Π² ΠΏΠ»Π°ΠΌΠ΅Π½ΠΈ ΠΈΠ»ΠΈ свСчСниС Π³Π°Π·ΠΎΠ²ΠΎΠ³ΠΎ разряда Π² Ρ‚Ρ€ΡƒΠ±ΠΊΠ΅, Π½Π°ΠΏΠΎΠ»Π½Π΅Π½Π½ΠΎΠΉ исслСдуСмым Π³Π°Π·ΠΎΠΌ.

Π‘ΠΏΠ΅ΠΊΡ‚Ρ€Ρ‹ поглощСния. Если ΠΏΡ€ΠΎΠΏΡƒΡΠΊΠ°Ρ‚ΡŒ Π±Π΅Π»Ρ‹ΠΉ свСт сквозь Ρ…ΠΎΠ»ΠΎΠ΄Π½Ρ‹ΠΉ Π½Π΅ΠΈΠ·Π»ΡƒΡ‡Π°ΡŽΡ‰ΠΈΠΉ Π³Π°Π·, Ρ‚ΠΎ Π½Π° Ρ„ΠΎΠ½Π΅ Π½Π΅ΠΏΡ€Π΅Ρ€Ρ‹Π²Π½ΠΎΠ³ΠΎ спСктра источника ΠΏΠΎΡΠ²Π»ΡΡŽΡ‚ΡΡ Ρ‚Π΅ΠΌΠ½Ρ‹Π΅ Π»ΠΈΠ½ΠΈΠΈ. Π“Π°Π· ΠΏΠΎΠ³Π»ΠΎΡ‰Π°Π΅Ρ‚ Π½Π°ΠΈΠ±ΠΎΠ»Π΅Π΅ свСт ΠΊΠ°ΠΊ Ρ€Π°Π· Ρ‚Π΅Ρ… Π΄Π»ΠΈΠ½ Π²ΠΎΠ»Π½, ΠΊΠΎΡ‚ΠΎΡ€Ρ‹Π΅ ΠΎΠ½ ΠΈΡΠΏΡƒΡΠΊΠ°Π΅Ρ‚ Π² ΡΠΈΠ»ΡŒΠ½ΠΎ Π½Π°Π³Ρ€Π΅Ρ‚ΠΎΠΌ состоянии. Π’Π΅ΠΌΠ½Ρ‹Π΅ Π»ΠΈΠ½ΠΈΠΈ Π½Π° Ρ„ΠΎΠ½Π΅ Π½Π΅ΠΏΡ€Π΅Ρ€Ρ‹Π²Π½ΠΎΠ³ΠΎ спСктра — это Π»ΠΈΠ½ΠΈΠΈ поглощСния, ΠΎΠ±Ρ€Π°Π·ΡƒΡŽΡ‰ΠΈΠ΅ Π² ΡΠΎΠ²ΠΎΠΊΡƒΠΏΠ½ΠΎΡΡ‚ΠΈ спСктр поглощСния.

ΠžΠΏΡ‹Ρ‚Π½Ρ‹ΠΌ ΠΏΡƒΡ‚Π΅ΠΌ оказалось, Ρ‡Ρ‚ΠΎ свСтящиСся ΠΏΠ°Ρ€Ρ‹ любого химичСского элСмСнта ΠΈΠ·Π»ΡƒΡ‡Π°ΡŽΡ‚ Ρ‚ΠΎΠ»ΡŒΠΊΠΎ ΠΎΠ΄Π½ΠΎΠΌΡƒ Π΅ΠΌΡƒ свойствСнный спСктр — Π½Π°Π±ΠΎΡ€ монохроматичСских ΠΈΠ·Π»ΡƒΡ‡Π΅Π½ΠΈΠΈ, ΠΊΠ°ΠΆΠ΄ΠΎΠΌΡƒ ΠΈΠ· ΠΊΠΎΡ‚ΠΎΡ€Ρ‹Ρ… Π² ΡΠΏΠ΅ΠΊΡ‚Ρ€Π΅ ΠΏΡ€ΠΈΠ½Π°Π΄Π»Π΅ΠΆΠΈΡ‚ своя линия.

БпСктроскоп — это ΠΏΡ€ΠΈΠ±ΠΎΡ€ ΠΏΠΎΠ·Π²ΠΎΠ»ΡΡŽΡ‰ΠΈΠΉ ΠΏΠΎΠ»ΡƒΡ‡ΠΈΡ‚ΡŒ ΠΈΠ½Ρ„ΠΎΡ€ΠΌΠ°Ρ†ΠΈΡŽ ΠΎ ΡΠΎΡΡ‚Π°Π²Π΅ ΠΈ ΡΠ²ΠΎΠΉΡΡ‚Π²Π°Ρ… вСщСства, ΠΏΡ€ΠΈ ΠΏΠΎΠΌΠΎΡ‰ΠΈ ΠΈΡ… ΡΠΏΠ΅ΠΊΡ‚Ρ€Π° (ΠΏΡ€ΠΈΠ±ΠΎΡ€, Π²Ρ‹ΡΠ²Π»ΡΡŽΡ‰ΠΈΠΉ спСктр вСщСства).

Π‘ΠΈΠ»Π΅Ρ‚ 2.

1. 1. ΠŸΠ΅Ρ€Π²Ρ‹ΠΉ Π·-Π½ ΠΡŒΡŽΡ‚ΠΎΠ½Π°. Π˜Π½Π΅Ρ€Ρ†ΠΈΠ°Π»ΡŒΠ½Ρ‹Π΅ систСмы отсчСта. ΠŸΡ€ΠΈΠ½Ρ†ΠΈΠΏ ΠΎΡ‚Π½ΠΎΡΠΈΡ‚Π΅Π»ΡŒΠ½ΠΎΡΡ‚ΠΈ Π² ΠΊΠ»Π°ΡΡΠΈΡ‡Π΅ΡΠΊΠΎΠΉ ΠΈ Ρ€Π΅Π»ΡΡ‚ивистской ΠΌΠ΅Ρ…Π°Π½ΠΈΠΊΠ΅.

2. 2. ΠžΡΠ½ΠΎΠ²Π½Ρ‹Π΅ ΠΏΡ€ΠΈΠ½Ρ†ΠΈΠΏΡ‹ радиосвязи. ΠœΠΎΠ΄ΡƒΠ»ΡΡ†ΠΈΡ ΠΈ Π΄Π΅Ρ‚Π΅ΠΊΡ‚ΠΈΡ€ΠΎΠ²Π°Π½ΠΈΠ΅. ΠŸΡ€ΠΎΡΡ‚Π΅ΠΉΡˆΠΈΠΉ Ρ€Π°Π΄ΠΈΠΎΠΏΡ€ΠΈΠ΅ΠΌΠ½ΠΈΠΊ. Π˜Π·ΠΎΠ±Ρ€Π΅Ρ‚Π΅Π½ΠΈΠ΅ Ρ€Π°Π΄ΠΈΠΎ А. Π‘. ΠŸΠΎΠΏΠΎΠ²Ρ‹ΠΌ. Π Π°Π·Π²ΠΈΡ‚ΠΈΠ΅ срСдств связи.

3. 3. (К=1/3 n mv2).

I Π·Π°ΠΊΠΎΠ½ ΠΡŒΡŽΡ‚ΠΎΠ½Π°.

* * ВсякоС Ρ‚Π΅Π»ΠΎ сохраняСт состояниС покоя ΠΈΠ»ΠΈ Ρ€Π°Π²Π½ΠΎΠΌΠ΅Ρ€Π½ΠΎΠ³ΠΎ прямолинСйного двиТСния Π΄ΠΎ Ρ‚Π΅Ρ… ΠΏΠΎΡ€ ΠΏΠΎΠΊΠ° внСшнСС воздСйствиС Π½Π΅ Π·Π°ΡΡ‚Π°Π²ΠΈΡ‚ Π΅Π³ΠΎ ΠΈΠ·ΠΌΠ΅Π½ΠΈΡ‚ΡŒ это состояниС.

* * Π˜Π½Π΅Ρ€Ρ†ΠΈΡ — стрСмлСниС Ρ‚Π΅Π»Π° ΡΠΎΡ…Ρ€Π°Π½ΡΡ‚ΡŒ состояниС покоя ΠΈΠ»ΠΈ Ρ€Π°Π²Π½ΠΎΠΌΠ΅Ρ€Π½ΠΎΠ³ΠΎ прямолинСйного двиТСния.

* * Π˜Π½Π΅Ρ€Ρ†ΠΈΠ°Π»ΡŒΠ½Ρ‹Π΅ систСмы отсчСта — систСмы ΠΏΠΎ ΠΎΡ‚Π½ΠΎΡˆΠ΅Π½ΠΈΡŽ ΠΊ ΠΊΠΎΡ‚ΠΎΡ€Ρ‹ΠΌ выполняСтся I Π·Π°ΠΊΠΎΠ½ ΠΡŒΡŽΡ‚ΠΎΠ½Π°.

I Π·Π°ΠΊΠΎΠ½ ΠΡŒΡŽΡ‚ΠΎΠ½Π° ΡƒΡ‚Π²Π΅Ρ€ΠΆΠ΄Π°Π΅Ρ‚ сущСствованиС ΠΈ с.ΠΎ.

! М.Ρ‚. сохраняСт состояниС покоя ΠΈΠ»ΠΈ Ρ€Π°Π²Π½ΠΎΠΌΠ΅Ρ€Π½ΠΎΠ³ΠΎ прямолинСйного двиТСния Π΄ΠΎ Ρ‚Π΅Ρ… ΠΏΠΎΡ€, ΠΏΠΎΠΊΠ° внСшнСС воздСйствиС Π½Π΅ Π²Ρ‹Π²Π΅Π΄Π΅Ρ‚ Π΅Π³ΠΎ ΠΈΠ· ΡΡ‚ΠΎΠ³ΠΎ состояния.

Π˜Π½Π΅Ρ€Ρ†ΠΈΠ°Π»ΡŒΠ½ΠΎΠΉ систСмой отсчСта ΠΌΠΎΠΆΠ½ΠΎ ΡΡ‡ΠΈΡ‚Π°Ρ‚ΡŒ Π³Π΅ΠΌΠ΅ΠΎΡ†Π΅Π½Ρ‚Ρ€ΠΈΡ‡Π΅ΡΠΊΡƒΡŽ с.ΠΎ.

ВсякоС ΠΈΠ·ΠΌΠ΅Π½Π΅Π½ΠΈΠ΅ состояния, любоС ускорСниС, Π΅ΡΡ‚ΡŒ Ρ€Π΅Π·ΡƒΠ»ΡŒΡ‚Π°Ρ‚ дСйствия Π½Π° Π΄Π²ΠΈΠΆΡƒΡ‰Π΅Π΅ΡΡ Ρ‚Π΅Π»ΠΎ со ΡΡ‚ΠΎΡ€ΠΎΠ½Ρ‹ Π΄Ρ€ΡƒΠ³ΠΈΡ… Ρ‚Π΅Π».

* * Π‘ΠΈΠ»Π° — это вСкторная физичСская Π²Π΅Π»ΠΈΡ‡ΠΈΠ½Π°, ΡΠ²Π»ΡΡŽΡ‰Π°ΡΡΡ ΠΌΠ΅Ρ€ΠΎΠΉ мСханичСского воздСйствия Π½Π° Ρ‚Π΅Π»ΠΎ со ΡΡ‚ΠΎΡ€ΠΎΠ½Ρ‹ Π΄Ρ€ΡƒΠ³ΠΈΡ… Ρ‚Π΅Π» ΠΈΠ»ΠΈ ΠΏΠΎΠ»Π΅ΠΉ, Π² Ρ€Π΅Π·ΡƒΠ»ΡŒΡ‚Π°Ρ‚Π΅ ΠΊΠΎΡ‚ΠΎΡ€ΠΎΠ³ΠΎ Ρ‚Π΅Π»ΠΎ ΠΏΡ€ΠΈΠΎΠ±Ρ€Π΅Ρ‚Π°Π΅Ρ‚ ускорСниС ΠΈΠ»ΠΈ измСняСт свою Ρ„ΠΎΡ€ΠΌΡƒ ΠΈ Ρ€Π°Π·ΠΌΠ΅Ρ€Ρ‹.

* * Масса Ρ‚Π΅Π»Π° — физичСская Π²Π΅Π»ΠΈΡ‡ΠΈΠ½Π°, ΡΠ²Π»ΡΡŽΡ‰Π°ΡΡΡ ΠΎΠ΄Π½ΠΎΠΉ ΠΈΠ· ΠΎΡΠ½ΠΎΠ²Π½Ρ‹Ρ… характСристик ΠΌΠ°Ρ‚Π΅Ρ€ΠΈ, ΠΎΠΏΡ€Π΅Π΄Π΅Π»ΡΡŽΡ‰Π°Ρ Π΅Π΅ ΠΈΠ½Π΅Ρ€Ρ†ΠΈΠ°Π»ΡŒΠ½Ρ‹Π΅ ΠΈ Π³Ρ€Π°Π²ΠΈΡ‚Π°Ρ†ΠΈΠΎΠ½Π½Ρ‹Π΅ свойства.

4 Π²ΠΈΠ΄Π° воздСйствия.

1. 1. Π“Ρ€Π°Π²ΠΈΡ‚Π°Ρ†ΠΈΠΎΠ½Π½ΠΎΠ΅ (обусловлСнноС всСмирным тяготСниСм).

2. 2. Π­Π»Π΅ΠΊΡ‚Ρ€ΠΎΠΌΠ°Π³Π½ΠΈΡ‚Π½ΠΎΠ΅ (осущСствляСтся Ρ‡Π΅Ρ€Π΅Π· ΠΌΠ°Π³Π½ΠΈΡ‚Π½ΠΎΠ΅ ΠΈΠ»ΠΈ элСктричСскоС ΠΏΠΎΠ»Π΅).

3. 3. БильноС ΠΈΠ»ΠΈ ядСрноС (ΠΎΠ±Π΅ΡΠΏΠ΅Ρ‡ΠΈΠ²Π°ΡŽΡ‰Π΅Π΅ связь части Π² Π°Ρ‚ΠΎΠΌΠ½ΠΎΠΌ ядрС).

4. 4. Π‘Π»Π°Π±ΠΎΠ΅ взаимодСйствиС (отвСтствСнныС Π·Π° ΠΌΠ½ΠΎΠ³ΠΈΠ΅ процСссы распада элСмСнт. частиц).

* * ЀизичСскоС ΠΏΠΎΠ»Π΅ — особая Ρ„ΠΎΡ€ΠΌΠ° ΠΌΠ°Ρ‚Π΅Ρ€ΠΈΠΈ, ΡΠ²ΡΠ·Ρ‹Π²Π°ΡŽΡ‰Π°Ρ частицы вСщСства Π² Π΅Π΄ΠΈΠ½Ρ‹Π΅ систСмы ΠΈ ΠΏΠ΅Ρ€Π΅Π΄Π°ΡŽΡ‰ΠΈΠ΅ΡΡ с ΠΊΠΎΠ½Π΅Ρ‡Π½ΠΎΠΉ ΡΠΊΠΎΡ€ΠΎΡΡ‚ΡŒΡŽ дСйствия ΠΎΠ΄Π½ΠΈΡ… частиц Π½Π° Π΄Ρ€ΡƒΠ³ΠΈΠ΅.

Π‘ΠΈΠ»Π° F ΠΏΠΎΠ»Π½ΠΎΡΡ‚ΡŒΡŽ Π·Π°Π΄Π°Π½Π°, Ссли ΡƒΠΊΠ°Π·Π°Π½Ρ‹ Π΅Π΅ ΠΌΠΎΠ΄ΡƒΠ»ΡŒ, Π½Π°ΠΏΡ€Π°Π²Π»Π΅Π½ΠΈΠ΅ Π² ΠΏΡ€ΠΎΡΡ‚ранствС ΠΈ Ρ‚ΠΎΡ‡ΠΊΠΈ прилоТСния. ΠŸΡ€ΡΠΌΠ°Ρ вдоль ΠΊΠΎΡ‚ΠΎΡ€ΠΎΠΉ Π½Π°ΠΏΡ€Π°Π²Π»Π΅Π½Π° сила, называСтся Π»ΠΈΠ½ΠΈΠ΅ΠΉ дСйствия силы.

* * ПолС, Π΄Π΅ΠΉΡΡ‚Π²ΡƒΡŽΡ‰Π΅Π΅ Π½Π° ΠΌΡ‚ с ΡΠΈΠ»ΠΎΠΉ F, называСтся стационарным ΠΏΠΎΠ»Π΅ΠΌ, Ссли ΠΎΠ½ΠΎ Π½Π΅ ΠΈΠ·ΠΌΠ΅Π½ΡΠ΅Ρ‚ся с Ρ‚Π΅Ρ‡Π΅Π½ΠΈΠ΅ΠΌ Π²Ρ€Π΅ΠΌΠ΅Π½ΠΈ.

Для стационарного поля Π½Π΅ΠΎΠ±Ρ…ΠΎΠ΄ΠΈΠΌΠΎ, Ρ‡Ρ‚ΠΎΠ±Ρ‹ ΡΠΎΠ·Π΄Π°ΡŽΡ‰ΠΈΠ΅ Π΅Π³ΠΎ Ρ‚Π΅Π»Π° покоились ΠΎΡ‚Π½ΠΎΡΠΈΡ‚Π΅Π»ΡŒΠ½ΠΎ ΠΈΠ½Π΅Ρ€Ρ†ΠΈΠ°Π»ΡŒΠ½ΠΎΠΉ систСмы отсчСта, использованной Π² Π΄Π°Π½Π½ΠΎΠΉ Π·Π°Π΄Π°Ρ‡Π΅.

2. 2. ΠžΡΠ½ΠΎΠ²Π½Ρ‹Π΅ ΠΏΡ€ΠΈΠ½Ρ†ΠΈΠΏΡ‹ радиосвязи. ΠœΠΎΠ΄ΡƒΠ»ΡΡ†ΠΈΡ ΠΈ Π΄Π΅Ρ‚Π΅ΠΊΡ‚ΠΈΡ€ΠΎΠ²Π°Π½ΠΈΠ΅. ΠŸΡ€ΠΎΡΡ‚Π΅ΠΉΡˆΠΈΠΉ Ρ€Π°Π΄ΠΈΠΎΠΏΡ€ΠΈΠ΅ΠΌΠ½ΠΈΠΊ. Π˜Π·ΠΎΠ±Ρ€Π΅Ρ‚Π΅Π½ΠΈΠ΅ Ρ€Π°Π΄ΠΈΠΎ А. Π‘. ΠŸΠΎΠΏΠΎΠ²Ρ‹ΠΌ. Π Π°Π·Π²ΠΈΡ‚ΠΈΠ΅ срСдств связи.

Π’Π°ΠΆΠ½Π΅ΠΉΡˆΠΈΠΌ этапом Π² Ρ€Π°Π·Π²ΠΈΡ‚ΠΈΠΈ радиосвязи Π±Ρ‹Π»ΠΎ созданиС Π² 1913 Π³ΠΎΠ΄Ρƒ Π³Π΅Π½Π΅Ρ€Π°Ρ‚ΠΎΡ€Π° Π½Π΅Π·Π°Ρ‚ΡƒΡ…Π°ΡŽΡ‰ΠΈΡ… элСктромагнитных ΠΊΠΎΠ»Π΅Π±Π°Π½ΠΈΠΉ). ΠšΡ€ΠΎΠΌΠ΅ ΠΏΠ΅Ρ€Π΅Π΄Π°Ρ‡ΠΈ Ρ‚Π΅Π»Π΅Π³Ρ€Π°Ρ„Π½Ρ‹Ρ… сигналов, состоящих ΠΈΠ· ΠΊΠΎΡ€ΠΎΡ‚ΠΊΠΈΡ… ΠΈ Π±ΠΎΠ»Π΅Π΅ ΠΏΡ€ΠΎΠ΄ΠΎΠ»ΠΆΠΈΡ‚Π΅Π»ΡŒΠ½Ρ‹Ρ… ΠΈΠΌΠΏΡƒΠ»ΡŒΡΠΎΠ² элСктромагнитных Π²ΠΎΠ»Π½, стала Π²ΠΎΠ·ΠΌΠΎΠΆΠ½Π° надСТная ΠΈ Π²Ρ‹ΡΠΎΠΊΠΎΠΊΠ°Ρ‡Π΅ΡΡ‚вСнная радиотСлСфонная связь — ΠΏΠ΅Ρ€Π΅Π΄Π°Ρ‡Π° Ρ€Π΅Ρ‡ΠΈ ΠΈΠ»ΠΈ ΠΌΡƒΠ·Ρ‹ΠΊΠΈ с ΠΏΠΎΠΌΠΎΡ‰ΡŒΡŽ элСктромагнитных Π²ΠΎΠ»Π½. ΠŸΡ€ΠΈΠ½Ρ†ΠΈΠΏ радиосвязи Π·Π°ΠΊΠ»ΡŽΡ‡Π°Π΅Ρ‚ΡΡ Π² ΡΠ»Π΅Π΄ΡƒΡŽΡ‰Π΅ΠΌ. ΠŸΠ΅Ρ€Π΅ΠΌΠ΅Π½Π½Ρ‹ΠΉ элСктричСский Ρ‚ΠΎΠΊ высокой частоты, созданный Π² ΠΏΠ΅Ρ€Π΅Π΄Π°ΡŽΡ‰Π΅ΠΉ Π°Π½Ρ‚Π΅Π½Π½Π΅, Π²Ρ‹Π·Ρ‹Π²Π°Π΅Ρ‚ Π² ΠΎΠΊΡ€ΡƒΠΆΠ°ΡŽΡ‰Π΅ΠΌ пространствС быстро ΠΌΠ΅Π½ΡΡŽΡ‰Π΅Π΅ΡΡ элСктричСскоС ΠΏΠΎΠ»Π΅, ΠΊΠΎΡ‚ΠΎΡ€ΠΎΠ΅ распространяСтся Π² Π²ΠΈΠ΄Π΅ элСктромагнитной Π²ΠΎΠ»Π½Ρ‹. Достигая ΠΏΡ€ΠΈΠ΅ΠΌΠ½ΠΎΠΉ Π°Π½Ρ‚Π΅Π½Π½Ρ‹, элСктромагнитная Π²ΠΎΠ»Π½Π° Π²Ρ‹Π·Ρ‹Π²Π°Π΅Ρ‚ Π² Π½Π΅ΠΉ ΠΏΠ΅Ρ€Π΅ΠΌΠ΅Π½Π½Ρ‹ΠΉ Ρ‚ΠΎΠΊ Ρ‚ΠΎΠΉ ΠΆΠ΅ частоты, Π½Π° ΠΊΠΎΡ‚ΠΎΡ€ΠΎΠΉ Ρ€Π°Π±ΠΎΡ‚Π°Π΅Ρ‚ ΠΏΠ΅Ρ€Π΅Π΄Π°Ρ‚Ρ‡ΠΈΠΊ.

ΠŸΡ€ΠΈ Ρ€Π°Π΄ΠΈΠΎΡ‚Π΅Π»Π΅Ρ„ΠΎΠ½Π½ΠΎΠΉ связи колСбания давлСния Π²ΠΎΠ·Π΄ΡƒΡ…Π° Π² Π·Π²ΡƒΠΊΠΎΠ²ΠΎΠΉ Π²ΠΎΠ»Π½Π΅ с ΠΏΠΎΠΌΠΎΡ‰ΡŒΡŽ ΠΌΠΈΠΊΡ€ΠΎΡ„ΠΎΠ½Π° ΠΏΡ€Π΅Π²Ρ€Π°Ρ‰Π°ΡŽΡ‚ΡΡ Π² ΡΠ»Π΅ΠΊΡ‚ричСскиС колСбания Ρ‚ΠΎΠΉ ΠΆΠ΅ Ρ„ΠΎΡ€ΠΌΡ‹. КолСбания Π·Π²ΡƒΠΊΠΎΠ²ΠΎΠΉ частоты ΠΏΡ€Π΅Π΄ΡΡ‚Π°Π²Π»ΡΡŽΡ‚ собой ΡΡ€Π°Π²Π½ΠΈΡ‚Π΅Π»ΡŒΠ½ΠΎ ΠΌΠ΅Π΄Π»Π΅Π½Π½Ρ‹Π΅ колСбания, Π° ΡΠ»Π΅ΠΊΡ‚Ρ€ΠΎΠΌΠ°Π³Π½ΠΈΡ‚Π½Ρ‹Π΅ Π²ΠΎΠ»Π½Ρ‹ Π½ΠΈΠ·ΠΊΠΎΠΉ (Π·Π²ΡƒΠΊΠΎΠ²ΠΎΠΉ) частоты ΠΏΠΎΡ‡Ρ‚ΠΈ совсСм Π½Π΅ ΠΈΠ·Π»ΡƒΡ‡Π°ΡŽΡ‚ся.

ΠžΠ±Π½Π°Ρ€ΡƒΠΆΠΈΡ‚ΡŒ Ρ€Π°Π΄ΠΈΠΎΠ²ΠΎΠ»Π½Ρ‹ ΠΈ ΠΈΠ·Π²Π»Π΅Ρ‡ΡŒ ΠΈΠ· Π½ΠΈΡ… ΠΏΠ΅Ρ€Π΅Π΄Π°Π²Π°Π΅ΠΌΡƒΡŽ ΠΈΠ½Ρ„ΠΎΡ€ΠΌΠ°Ρ†ΠΈΡŽ ΠΌΠΎΠΆΠ½ΠΎ с ΠΏΠΎΠΌΠΎΡ‰ΡŒΡŽ Ρ€Π°Π΄ΠΈΠΎΠΏΡ€ΠΈΠ΅ΠΌΠ½ΠΈΠΊΠ°.

Достигая Π°Π½Ρ‚Π΅Π½Π½Ρ‹ ΠΏΡ€ΠΈΠ΅ΠΌΠ½ΠΈΠΊΠ°, Ρ€Π°Π΄ΠΈΠΎΠ²ΠΎΠ»Π½Ρ‹ ΠΏΠ΅Ρ€Π΅ΡΠ΅ΠΊΠ°ΡŽΡ‚ Π΅Π΅ ΠΏΡ€ΠΎΠ²ΠΎΠ΄ ΠΈ Π²ΠΎΠ·Π±ΡƒΠΆΠ΄Π°ΡŽΡ‚ (ΠΈΠ½Π΄ΡƒΡ†ΠΈΡ€ΡƒΡŽΡ‚) Π² Π½Π΅ΠΉ ΠΎΡ‡Π΅Π½ΡŒ слабыС радиочастоты. Π’ ΠΏΡ€ΠΈΠ΅ΠΌΠ½ΠΎΠΉ Π°Π½Ρ‚Π΅Π½Π½Π΅ ΠΎΠ΄Π½ΠΎΠ²Ρ€Π΅ΠΌΠ΅Π½Π½ΠΎ находятся высокочастотныС колСбания ΠΎΡ‚ ΠΌΠ½ΠΎΠ³ΠΈΡ… Ρ€Π°Π΄ΠΈΠΎΠΏΠ΅Ρ€Π΅Π΄Π°Ρ‚Ρ‡ΠΈΠΊΠΎΠ². ΠŸΠΎΡΡ‚ΠΎΠΌΡƒ ΠΎΠ΄ΠΈΠ½ ΠΈΠ· Π²Π°ΠΆΠ½Π΅ΠΉΡˆΠΈΡ… элСмСнтов Ρ€Π°Π΄ΠΈΠΎΠΏΡ€ΠΈΠ΅ΠΌΠ½ΠΈΠΊΠ° — сСлСктивноС (ΠΈΠ·Π±ΠΈΡ€Π°Ρ‚Π΅Π»ΡŒΠ½ΠΎΠ΅) устройство, ΠΊΠΎΡ‚ΠΎΡ€ΠΎΠ΅ ΠΈΠ· Π²ΡΠ΅Ρ… принятых сигналов ΠΌΠΎΠΆΠ΅Ρ‚ ΠΎΡ‚ΠΎΠ±Ρ€Π°Ρ‚ΡŒ Π½ΡƒΠΆΠ½Ρ‹Π΅. Π’Π°ΠΊΠΈΠΌ устройством являСтся ΠΊΠΎΠ»Π΅Π±Π°Ρ‚Π΅Π»ΡŒΠ½Ρ‹ΠΉ ΠΊΠΎΠ½Ρ‚ΡƒΡ€, ΠΏΠΎΠ·Π²ΠΎΠ»ΡΡŽΡ‰ΠΈΠΉ Π½Π°ΡΡ‚Ρ€Π°ΠΈΠ²Π°Ρ‚ΡŒ ΠΏΡ€ΠΈΠ΅ΠΌΠ½ΠΈΠΊ Π½Π° Ρ€Π°Π΄ΠΈΠΎΠ²ΠΎΠ»Π½Ρ‹ ΠΎΠΏΡ€Π΅Π΄Π΅Π»Π΅Π½Π½ΠΎΠΉ Π΄Π»ΠΈΠ½Ρ‹.

КолСбания Ρ‚ΠΎΠΊΠ° Π² ΠΊΠΎΠ½Ρ‚ΡƒΡ€Π΅ Π±ΡƒΠ΄ΡƒΡ‚ Π½Π°ΠΈΠ±ΠΎΠ»Π΅Π΅ ΡΠΈΠ»ΡŒΠ½Ρ‹ΠΌΠΈ, Ссли частота ΠΊΠΎΠ»Π΅Π±Π°Π½ΠΈΠΉ ΠΏΠΎΠ΄Π²Π΅Π΄Π΅Π½Π½ΠΎΠ³ΠΎ сигнала совпадаСт с Ρ‡Π°ΡΡ‚ΠΎΡ‚ΠΎΠΉ ΠΊΠΎΠ»Π΅Π±Π°Π½ΠΈΠΉ ΠΊΠΎΠ½Ρ‚ΡƒΡ€Π°. НазначСниС Π΄Ρ€ΡƒΠ³ΠΈΡ… элСмСнтов Ρ€Π°Π΄ΠΈΠΎΠΏΡ€ΠΈΠ΅ΠΌΠ½ΠΈΠΊΠ° Π·Π°ΠΊΠ»ΡŽΡ‡Π°Π΅Ρ‚ΡΡ Π² Ρ‚ΠΎΠΌ, Ρ‡Ρ‚ΠΎ Π±Ρ‹ ΡƒΡΠΈΠ»ΠΈΡ‚ΡŒ принятыС ΠΈΠ»ΠΈ ΠΎΡ‚Ρ€Π°ΠΆΠ΅Π½Π½Ρ‹Π΅ ΠΊΠΎΠ»Π΅Π±Π°Ρ‚Π΅Π»ΡŒΠ½Ρ‹ΠΌ ΠΊΠΎΠ½Ρ‚ΡƒΡ€ΠΎΠΌ высокочастотныС ΠΌΠΎΠ΄ΡƒΠ»ΠΈΡ€ΠΎΠ²Π°Π½Π½Ρ‹Π΅ колСбания, Π²Ρ‹Π΄Π΅Π»ΠΈΡ‚ΡŒ ΠΈΠ· Π½ΠΈΡ… колСбания Π·Π²ΡƒΠΊΠΎΠ²ΠΎΠΉ частоты, ΡƒΠΌΠ΅Π½ΡŒΡˆΠΈΡ‚ΡŒ ΠΈΡ… ΠΈ ΠΏΡ€Π΅ΠΎΠ±Ρ€Π°Π·ΠΎΠ²Π°Ρ‚ΡŒ Π² ΡΠΈΠ³Π½Π°Π»Ρ‹ ΠΈΠ½Ρ„ΠΎΡ€ΠΌΠ°Ρ†ΠΈΠΈ. ΠŸΠ΅Ρ€Π²ΡƒΡŽ ΠΈΠ· ΡΡ‚ΠΈΡ… Ρ„ΡƒΠ½ΠΊΡ†ΠΈΠΉ выполняСт ΡƒΡΠΈΠ»ΠΈΡ‚Π΅Π»ΡŒ ΠΊΠΎΠ»Π΅Π±Π°Π½ΠΈΠΉ радиочастоты, Π²Ρ‚ΠΎΡ€ΡƒΡŽ — Π΄Π΅Ρ‚Π΅ΠΊΡ‚ΠΎΡ€, Ρ‚Ρ€Π΅Ρ‚ΡŒΡŽ — ΡƒΡΠΈΠ»ΠΈΡ‚Π΅Π»ΡŒ ΠΊΠΎΠ»Π΅Π±Π°Π½ΠΈΠΉ Π·Π²ΡƒΠΊΠΎΠ²ΠΎΠΉ частоты, Ρ‡Π΅Ρ‚Π²Π΅Ρ€Ρ‚ΡƒΡŽ — динамичСская Π³ΠΎΠ»ΠΎΠ²ΠΊΠ° громкоговоритСля ΠΈΠ»ΠΈ ΠΏΡ€ΠΈΠ΅ΠΌΠ½Ρ‹ΠΉ Ρ‚Π΅Π»Π΅Π³Ρ€Π°Ρ„Π½Ρ‹ΠΉ Π°ΠΏΠΏΠ°Ρ€Π°Ρ‚.

Π‘ΠΈΠ»Π΅Ρ‚ 20.

1. 1. Π—Π°ΠΊΠΎΠ½Ρ‹ отраТСния ΠΈ ΠΏΡ€Π΅Π»ΠΎΠΌΠ»Π΅Π½ΠΈΡ свСта. ПолноС ΠΎΡ‚Ρ€Π°ΠΆΠ΅Π½ΠΈΠ΅ ΠΈ Π΅Π³ΠΎ ΠΏΡ€ΠΈΠΌΠ΅Π½Π΅Π½ΠΈΠ΅.

2. 2. ЭлСктричСский Ρ‚ΠΎΠΊ Π² ΠΌΠ΅Ρ‚Π°Π»Π»Π°Ρ…. Π‘ΠΎΠΏΡ€ΠΎΡ‚ΠΈΠ²Π»Π΅Π½ΠΈΠ΅. УдСльноС сопротивлСниС. Π—Π°Π²ΠΈΡΠΈΠΌΠΎΡΡ‚ΡŒ сопротивлСния ΠΎΡ‚ Ρ‚Π΅ΠΌΠΏΠ΅Ρ€Π°Ρ‚ΡƒΡ€Ρ‹. Π‘Π²Π΅Ρ€Ρ…ΠΏΡ€ΠΎΠ²ΠΎΠ΄ΠΈΠΌΠΎΡΡ‚ΡŒ.

«Π˜Π—ΠœΠ•Π Π•ΠΠ˜Π• Π£Π‘ΠšΠžΠ Π•ΠΠ˜Π― Π‘Π’ΠžΠ‘ΠžΠ”ΠΠžΠ“Πž ΠŸΠΠ”Π•ΠΠ˜Π―».

1. 1. Π›ΡƒΡ‡ свСта Π² ΠΎΠ΄Π½ΠΎΡ€ΠΎΠ΄Π½ΠΎΠΉ срСдС прямолинССн. На Π³Ρ€Π°Π½ΠΈΡ†Π΅ Π΄Π²ΡƒΡ… срСд ΠΎΠ½ ΠΌΠ΅Π½ΡΠ΅Ρ‚ своС Π½Π°ΠΏΡ€Π°Π²Π»Π΅Π½ΠΈΠ΅: Ρ‡Π°ΡΡ‚ΡŒ свСта ΠΈΠ»ΠΈ вСсь возвращаСтся Π² ΠΏΠ΅Ρ€Π²ΡƒΡŽ срСду. Π­Ρ‚ΠΎ явлСниС называСтся ΠΎΡ‚Ρ€Π°ΠΆΠ΅Π½ΠΈΠ΅ΠΌ свСта. Π—Π°ΠΊΠΎΠ½ отраТСния: ΠΏΠ°Π΄Π°ΡŽΡ‰ΠΈΠΉ Π»ΡƒΡ‡, ΠΎΡ‚Ρ€Π°ΠΆΠ΅Π½Π½Ρ‹ΠΉ ΠΈ ΠΏΠ΅Ρ€ΠΏΠ΅Π½Π΄ΠΈΠΊΡƒΠ»ΡΡ€ Π½Π° Π³Ρ€Π°Π½ΠΈΡ†Π΅ Π΄Π²ΡƒΡ… срСд, Π»Π΅ΠΆΠ°Ρ‚ Π² ΠΎΠ΄Π½ΠΎΠΉ плоскости. Π£Π³ΠΎΠ» падСния Ρ€Π°Π²Π΅Π½ ΡƒΠ³Π»Ρƒ отраТСния.

Π—Π°ΠΊΠΎΠ½ прСломлСния: ΠΏΠ°Π΄Π°ΡŽΡ‰ΠΈΠΉ Π»ΡƒΡ‡, ΠΏΡ€Π΅Π»ΠΎΠΌΠ»Π΅Π½Π½Ρ‹ΠΉ ΠΈ ΠΏΠ΅Ρ€ΠΏΠ΅Π½Π΄ΠΈΠΊΡƒΠ»ΡΡ€ ΠΊ Π³Ρ€Π°Π½ΠΈΡ†Π΅ Ρ€Π°Π·Π΄Π΅Π»Π° Π΄Π²ΡƒΡ… срСд, восстановлСнный Π² Ρ‚ΠΎΡ‡ΠΊΠ΅ падСния Π»ΡƒΡ‡Π°, Π»Π΅ΠΆΠ°Ρ‚ Π² ΠΎΠ΄Π½ΠΎΠΉ плоскости. ΠžΡ‚Π½ΠΎΡˆΠ΅Π½ΠΈΠ΅ синуса ΡƒΠ³Π»Π° падСния ΠΊ ΡΠΈΠ½ΡƒΡΠ° ΡƒΠ³Π»Π° прСломлСния Π΅ΡΡ‚ΡŒ Π²Π΅Π»ΠΈΡ‡ΠΈΠ½Π° постоянная для Π΄Π²ΡƒΡ… Π΄Π°Π½Π½Ρ‹Ρ… срСд. sinΠ°Π»ΡŒΡ„Π°/sinΠ±Π΅Ρ‚Ρ‚Π°=n. ΠŸΠΎΡΡ‚ΠΎΡΠ½Π½ΡƒΡŽ Π²Π΅Π»ΠΈΡ‡ΠΈΠ½Ρƒ n Π½Π°Π·Ρ‹Π²Π°ΡŽΡ‚ ΠΎΡ‚Π½ΠΎΡΠΈΡ‚Π΅Π»ΡŒΠ½Ρ‹ΠΌ ΠΏΠΎΠΊΠ°Π·Π°Ρ‚Π΅Π»Π΅ΠΌ прСломлСния, ΠΈΠ»ΠΈ ΠΏΠΎΠΊΠ°Π·Π°Ρ‚Π΅Π»Π΅ΠΌ прСломлСния Π²Ρ‚ΠΎΡ€ΠΎΠΉ срСды ΠΎΡ‚Π½ΠΎΡΠΈΡ‚Π΅Π»ΡŒΠ½ΠΎ ΠΏΠ΅Ρ€Π²ΠΎΠΉ. ΠŸΠΎΠΊΠ°Π·Π°Ρ‚Π΅Π»ΡŒ прСломлСния срСды ΠΎΡ‚Π½ΠΎΡΠΈΡ‚Π΅Π»ΡŒΠ½ΠΎ Π²Π°ΠΊΡƒΡƒΠΌΠ° Π½Π°Π·Ρ‹Π²Π°ΡŽΡ‚ Π°Π±ΡΠΎΠ»ΡŽΡ‚Π½Ρ‹ΠΌ ΠΏΠΎΠΊΠ°Π·Π°Ρ‚Π΅Π»Π΅ΠΌ прСломлСния этой срСды. Он Ρ€Π°Π²Π΅Π½ ΠΎΡ‚Π½ΠΎΡˆΠ΅Π½ΠΈΡŽ синуса ΡƒΠ³Π»Π° падСния ΠΊ ΡΠΈΠ½ΡƒΡΡƒ ΡƒΠ³Π»Π° прСломлСния ΠΏΡ€ΠΈ ΠΏΠ΅Ρ€Π΅Ρ…ΠΎΠ΄Π΅ вСщСства ΠΈΠ· Π²Π°ΠΊΡƒΡƒΠΌΠ° Π² Π΄Π°Π½Π½ΡƒΡŽ срСду. Π‘Ρ€Π΅Π΄Ρƒ с ΠΌΠ΅Π½ΡŒΡˆΠΈΠΌ Π°Π±ΡΠΎΠ»ΡŽΡ‚Π½Ρ‹ΠΌ ΠΏΠΎΠΊΠ°Π·Π°Ρ‚Π΅Π»Π΅ΠΌ прСломлСния принято Π½Π°Π·Ρ‹Π²Π°Ρ‚ΡŒ оптичСски ΠΌΠ΅Π½Π΅Π΅ ΠΏΠ»ΠΎΡ‚Π½ΠΎΠΉ.

ПолноС ΠΎΡ‚Ρ€Π°ΠΆΠ΅Π½ΠΈΠ΅ — явлСниС, ΠΊΠΎΠ³Π΄Π° свСт ΠΈΠ΄Π΅Ρ‚ ΠΈΠ· Π±ΠΎΠ»Π΅Π΅ оптичСски ΠΏΠ»ΠΎΡ‚Π½ΠΎΠΉ срСды ΠΌ Π² ΠΌΠ΅Π½Π΅Π΅. Π£Π³ΠΎΠ» падСния Π°Π»ΡŒΡ„Π° Π½ΡƒΠ»Π΅Π²ΠΎΠ΅, ΡΠΎΠΎΡ‚Π²Π΅Ρ‚ΡΡ‚Π²ΡƒΡŽΡ‰ΠΈΠΉ ΡƒΠ³Π»Ρƒ прСломлСния 90 градусов, Π½Π°Π·Ρ‹Π²Π°ΡŽΡ‚ ΠΏΡ€Π΅Π΄Π΅Π»ΡŒΠ½Ρ‹ΠΌ ΡƒΠ³Π»ΠΎΠΌ отраТСния. Π―Π²Π»Π΅Π½ΠΈΠ΅ ΠΏΠΎΠ»Π½ΠΎΠ³ΠΎ отраТСния Π»Π΅ΠΆΠΈΡ‚ Π² ΠΎΡΠ½ΠΎΠ²Π΅ Ρ€Π°Π±ΠΎΡ‚Ρ‹ стСклянных Π²ΠΎΠ»ΠΎΠΊΠΎΠ½Π½Ρ‹Ρ… свСтоводов. Оно ΠΈΡΠΏΠΎΠ»ΡŒΠ·ΡƒΠ΅Ρ‚ΡΡ Π² ΠΏΡ€ΠΈΠ·ΠΌΠ°Ρ‚ичСских биноклях ΠΈ ΠΏΠ΅Ρ€ΠΈΡΠΊΠΎΠΏΠ°Ρ…, этим явлСниСм ΠΎΠ±ΡŠΡΡΠ½ΡΡŽΡ‚ΡΡ ΠΌΠΈΡ€Π°ΠΆΠΈ Π² ΠΏΡ€ΠΈΡ€ΠΎΠ΄Π΅.

2. 2. ЭлСктричСский Ρ‚ΠΎΠΊ Π² ΠΌΠ΅Ρ‚Π°Π»Π»Π°Ρ…. Π‘ΠΎΠΏΡ€ΠΎΡ‚ΠΈΠ²Π»Π΅Π½ΠΈΠ΅. УдСльноС сопротивлСниС. Π—Π°Π²ΠΈΡΠΈΠΌΠΎΡΡ‚ΡŒ сопротивлСния ΠΎΡ‚ Ρ‚Π΅ΠΌΠΏΠ΅Ρ€Π°Ρ‚ΡƒΡ€Ρ‹. Π‘Π²Π΅Ρ€Ρ…ΠΏΡ€ΠΎΠ²ΠΎΠ΄ΠΈΠΌΠΎΡΡ‚ΡŒ.

ВсС ΠΌΠ΅Ρ‚Π°Π»Π»Ρ‹ Π² Ρ‚Π²Π΅Ρ€Π΄ΠΎΠΌ ΠΈ ΠΆΠΈΠ΄ΠΊΠΎΠΌ состоянии ΡΠ²Π»ΡΡŽΡ‚ΡΡ ΠΏΡ€ΠΎΠ²ΠΎΠ΄Π½ΠΈΠΊΠ°ΠΌΠΈ элСктричСского Ρ‚ΠΎΠΊΠ°. ΠŸΡ€ΠΈ ΠΏΡ€ΠΎΡ…ΠΎΠΆΠ΄Π΅Π½ΠΈΠΈ элСктричСского Ρ‚ΠΎΠΊΠ° ΠΏΠΎ ΠΏΡ€ΠΎΠ²ΠΎΠ΄Π½ΠΈΠΊΡƒ Π΅Π³ΠΎ масса Π½Π΅ ΠΌΠ΅Π½ΡΠ΅Ρ‚ся. Π˜ΠΎΠ½Ρ‹ ΠΌΠ΅Ρ‚Π°Π»Π»Π° Π½Π΅ ΠΏΡ€ΠΈΠ½ΠΈΠΌΠ°ΡŽΡ‚ участия Π² ΠΏΠ΅Ρ€Π΅Π½ΠΎΡΠ΅ элСктричСского заряда. Π’ ΡΠΎΠ·Π΄Π°Π½ΠΈΠΈ элСктричСского Ρ‚ΠΎΠΊΠ° ΡƒΡ‡Π°ΡΡ‚Π²ΡƒΡŽΡ‚ Ρ‚ΠΎΠ»ΡŒΠΊΠΎ элСктроны. Π‘Ρ‹Π»ΠΎ ΠΎΠ±Π½Π°Ρ€ΡƒΠΆΠ΅Π½ΠΎ, Ρ‡Ρ‚ΠΎ ΠΏΡ€ΠΈ Ρ€Π΅Π·ΠΊΠΎΠΉ остановкС быстро Π²Ρ€Π°Ρ‰Π°ΡŽΡ‰Π΅ΠΉΡΡ ΠΊΠ°Ρ‚ΡƒΡˆΠΊΠΈ Π² Π΅Π΅ ΠΏΡ€ΠΎΠ²ΠΎΠ΄Π΅ Π²ΠΎΠ·Π½ΠΈΠΊΠ°Π΅Ρ‚ элСктричСский Ρ‚ΠΎΠΊ, создаваСмый элСктронами. Π’ ΠΎΡ‚сутствии элСктричСского поля свободныС элСктроны ΠΏΠ΅Ρ€Π΅ΠΌΠ΅Ρ‰Π°ΡŽΡ‚ΡΡ Π² ΠΊΡ€ΠΈΡΡ‚Π°Π»Π»Π΅ ΠΌΠ΅Ρ‚Π°Π»Π»Π° хаотичСски. Под дСйствиСм элСктричСского поля свободныС элСктроны ΠΎΠ±Ρ€Π΅Ρ‚Π°ΡŽΡ‚ упорядочСнноС Π΄Π²ΠΈΠΆΠ΅Π½ΠΈΠ΅ Π² ΠΎΠ΄Π½ΠΎΠΌ Π½Π°ΠΏΡ€Π°Π²Π»Π΅Π½ΠΈΠΈ, ΠΈ Π² ΠΏΡ€ΠΎΠ²ΠΎΠ΄Π½ΠΈΠΊΠ°Ρ… Π²ΠΎΠ·Π½ΠΈΠΊΠ°Π΅Ρ‚ элСктричСский Ρ‚ΠΎΠΊ. Π‘Π²ΠΎΠ±ΠΎΠ΄Π½Ρ‹Π΅ элСктроны ΡΡ‚Π°Π»ΠΊΠΈΠ²Π°ΡŽΡ‚ΡΡ с ΠΈΠΎΠ½Π°ΠΌΠΈ кристалличСской Ρ€Π΅ΡˆΠ΅Ρ‚ΠΊΠΈ ΠΈ ΠΎΡ‚Π΄Π°ΡŽΡ‚ ΠΈΠΌ Ρ‡Π°ΡΡ‚ΡŒ кинСтичСской энСргии, ΠΏΡ€ΠΎΠ²ΠΎΠ΄Π½ΠΈΠΊ нагрСваСтся. ΠŸΡ€ΠΈ Π½Π°Π³Ρ€Π΅Π²Π°Π½ΠΈΠΈ ΡƒΠ΄Π΅Π»ΡŒΠ½ΠΎΠ΅ элСктричСс4ΠΊΠΎΠ΅ сопротивлСниС ΠΏΡ€ΠΎΠ²ΠΎΠ΄Π½ΠΈΠΊΠ° увСличиваСтся Ρ€ΠΎ=Ρ€ΠΎ Π½ΡƒΠ»Π΅Π²ΠΎΠ΅*(1-Π°Π»ΡŒΡ„Π°t), Π³Π΄Π΅ Ρ€ΠΎ — ΡƒΠ΄Π΅Π»ΡŒΠ½ΠΎΠ΅ элСктричСскоС сопротивлСниС ΠΏΡ€ΠΈ Ρ‚Π΅ΠΌΠΏΠ΅Ρ€Π°Ρ‚ΡƒΡ€Π΅ t, Ρ€ΠΎ Π½ΡƒΠ»Π΅Π²ΠΎΠ΅ ΠΏΡ€ΠΈ Ρ‚Π΅ΠΌΠΏΠ΅Ρ€Π°Ρ‚ΡƒΡ€Π΅ 0, Π°Π»ΡŒΡ„Π° — Ρ‚Π΅ΠΌΠΏΠ΅Ρ€Π°Ρ‚ΡƒΡ€Π½Ρ‹ΠΉ коэффициСнт сопротивлСния. Π‘ ΠΏΡ€ΠΈΠ±Π»ΠΈΠΆΠ΅Π½ΠΈΠ΅ΠΌ Ρ‚Π΅ΠΌΠΏΠ΅Ρ€Π°Ρ‚ΡƒΡ€Ρ‹ ΠΊ Π°Π±ΡΠΎΠ»ΡŽΡ‚Π½ΠΎΠΌΡƒ Π½ΡƒΠ»ΡŽ ΡƒΠ΄Π΅Π»ΡŒΠ½ΠΎΠ΅ сопротивлСниС монокристаллов становится ΠΎΡ‡Π΅Π½ΡŒ малСньким.

R=Ρ€ΠΎl/s, Π³Π΄Π΅ Ρ€ΠΎ — ΡƒΠ΄Π΅Π»ΡŒΠ½ΠΎΠ΅ сопративлСни5Π΅ ΠΏΡ€ΠΎΠ²ΠΎΠ΄Π½ΠΈΠΊΠ° (Ом*ΠΌ).

Π―Π²Π»Π΅Π½ΠΈΠ΅ ΡƒΠΌΠ΅Π½ΡŒΡˆΠ΅Π½ΠΈΡ ΡƒΠ΄Π΅Π»ΡŒΠ½ΠΎΠ³ΠΎ сопротивлСния Π΄ΠΎ Π½ΡƒΠ»Ρ ΠΏΡ€ΠΈ Ρ‚Π΅ΠΌΠΏΠ΅Ρ€Π°Ρ‚ΡƒΡ€Π΅, ΠΎΡ‚Π»ΠΈΡ‡Π½ΠΎΠΉ ΠΎΡ‚ Π½ΡƒΠ»Ρ, называСтся ΡΠ²Π΅Ρ€Ρ…ΠΏΡ€ΠΎΠ²ΠΎΠ΄ΠΈΠΌΠΎΡΡ‚ΡŒΡŽ. ΠŸΡ€ΠΎΡ…ΠΎΠΆΠ΄Π΅Π½ΠΈΠ΅ Ρ‚ΠΎΠΊΠ° Π² ΡΠ²Π΅Ρ€Ρ…ΠΏΡ€ΠΎΠ²ΠΎΠ΄Π½ΠΈΠΊΠ΅ происходит Π±Π΅Π· ΠΏΠΎΡ‚Π΅Ρ€ΡŒ энСргии, поэтому ΠΎΠ΄Π½Π°ΠΆΠ΄Ρ‹ Π²ΠΎΠ·Π±ΡƒΠΆΠ΄Π΅Π½Π½Ρ‹ΠΉ Π² ΡΠ²Π΅Ρ€Ρ…проводящСм ΠΊΠΎΠ»ΡŒΡ†Π΅ элСктричСский Ρ‚ΠΎΠΊ ΠΌΠΎΠΆΠ΅Ρ‚ ΡΡƒΡ‰Π΅ΡΡ‚Π²ΠΎΠ²Π°Ρ‚ΡŒ Π½Π΅ΠΎΠ³Ρ€Π°Π½ΠΈΡ‡Π΅Π½Π½ΠΎ Π΄ΠΎΠ»Π³ΠΎ Π±Π΅Π· ΠΈΠ·ΠΌΠ΅Π½Π΅Π½ΠΈΠΉ.

Π‘ΠΈΠ»Π΅Ρ‚ 21.

1. 1. Π’ΠΎΠ»Π½ΠΎΠ²Ρ‹Π΅ свойства свСта.

2. 2. Π’Ρ‹Π½ΡƒΠΆΠ΄Π΅Π½Π½Ρ‹Π΅ колСбания. РСзонанс. Π—Π°Π²ΠΈΡΠΈΠΌΠΎΡΡ‚ΡŒ Π°ΠΌΠΏΠ»ΠΈΡ‚ΡƒΠ΄Ρ‹ ΠΊΠΎΠ»Π΅Π±Π°Π½ΠΈΠΉ ΠΎΡ‚ Ρ‡Π°ΡΡ‚ΠΎΡ‚Ρ‹ Π²Ρ‹Π½ΡƒΠΆΠ΄Π°ΡŽΡ‰Π΅ΠΉ силы.

3. 3. «Π˜Π—ΠœΠ•Π Π•ΠΠ˜Π• Π­Π”Π‘ И Π‘ΠžΠŸΠ ΠΠ’Π˜Π’Π›Π•ΠΠ˜Π― ИБВОЧНИКА ВОКА».

1. 1. ΠžΡΠ½ΠΎΠ²Π½Ρ‹ΠΌΠΈ Π²ΠΎΠ»Π½ΠΎΠ²Ρ‹ΠΌΠΈ свойствами свСта являСтся интСрфСрСнция, супСрпозиция ΠΈ Π΄ΠΈΡ„ракция. ΠŸΡ€ΠΎΠ½ΠΈΠΊΠ½ΠΎΠ²Π΅Π½ΠΈΠ΅ Π²ΠΎΠ»Π½Ρ‹ Π² ΠΎΠ±Π»Π°ΡΡ‚ΡŒ гСомСтричСской Ρ‚Π΅Π½ΠΈ называСтся Π΄ΠΈΡ„Ρ€Π°ΠΊΡ†ΠΈΠ΅ΠΉ. Дифракция проявляСтся, ΠΊΠΎΠ³Π΄Π° Π΄Π»ΠΈΠ½Π° Π²ΠΎΠ»Π½Ρ‹ сравнима с Ρ€Π°Π·ΠΌΠ΅Ρ€ΠΎΠΌ прСпятствия. Дифракция Π΄ΠΎΠΊΠ°Π·Π°Π½Π° ΠΎΠΏΡ‹Ρ‚Π½Ρ‹ΠΌ ΠΏΡƒΡ‚Π΅ΠΌ. Π˜Π½Ρ‚Π΅Ρ€Ρ„Π΅Ρ€Π΅Π½Ρ†ΠΈΡ свСта — слоТСниС Π΄Π²ΡƒΡ… ΠΊΠΎΠ³Π΅Ρ€Π΅Π½Ρ‚Π½Ρ‹Ρ… Π²ΠΎΠ»Π½, вслСдствиС ΠΊΠΎΡ‚ΠΎΡ€ΠΎΠ³ΠΎ Π½Π°Π±Π»ΡŽΠ΄Π°Π΅Ρ‚ΡΡ устойчивая Π²ΠΎ Π²Ρ€Π΅ΠΌΠ΅Π½ΠΈ ΠΊΠ°Ρ€Ρ‚ΠΈΠ½Π° усилСния ΠΈΠ»ΠΈ ослаблСния Ρ€Π΅Π·ΡƒΠ»ΡŒΡ‚ΠΈΡ€ΡƒΡŽΡ‰ΠΈΡ… свСтовых ΠΊΠΎΠ»Π΅Π±Π°Π½ΠΈΠΉ Π² Ρ€Π°Π·Π»ΠΈΡ‡Π½Ρ‹Ρ… Ρ‚ΠΎΡ‡ΠΊΠ°Ρ… пространства. (это явлСниС возникновСния устойчивой ΠΊΠ°Ρ€Ρ‚ΠΈΠ½Ρ‹ пСрСраспрСдСлСния энСргии ΠΏΡ€ΠΈ слоТСнии Π΄Π²ΡƒΡ… ΠΈΠ»ΠΈ Π±ΠΎΠ»Π΅Π΅ ΠΊΠΎΠ³Π΅Ρ€Π΅Π½Ρ‚Π½Ρ‹Ρ… Π²ΠΎΠ»Π½). Π Π΅Π·ΡƒΠ»ΡŒΡ‚Π°Ρ‚ ΠΈΠ½Ρ‚Π΅Ρ€Ρ„Π΅Ρ€Π΅Π½Ρ†ΠΈΠΈ 1. УсловиС максимума Π΄Π΅Π»ΡŒΡ‚Π° d=2ΠΊ*x/2. 2. ΠœΠΈΠ½ΠΈΠΌΡƒΠΌΠ° Π΄Π΅Π»ΡŒΡ‚Π° d=(2ΠΊ+1)x/2. ΠšΠΎΠ³Π΅Ρ€Π΅Π½Ρ‚Π½Ρ‹Π΅ Π²ΠΎΠ»Π½Ρ‹: 1 ΠΎΠ΄ΠΈΠ½Π°ΠΊΠΎΠ²ΠΎΠΉ частоты 2. ΠŸΠΎΡΡ‚ΠΎΡΠ½Π½Π°Ρ Ρ€Π°Π·Π½ΠΎΡΡ‚ΡŒ Ρ„Π°Π·. БупСрпозиция — явлСниС прохоТдСния Π²ΠΎΠ»Π½ΠΎΠΉ Ρ‡Π΅Ρ€Π΅Π· ΠΎΠ΄Π½ΠΎ пространство Π±Π΅Π· искаТСния.

2. 2. Π’Ρ‹Π½ΡƒΠΆΠ΄Π΅Π½Π½Ρ‹Π΅ колСбания. РСзонанс. Π—Π°Π²ΠΈΡΠΈΠΌΠΎΡΡ‚ΡŒ Π°ΠΌΠΏΠ»ΠΈΡ‚ΡƒΠ΄Ρ‹ ΠΊΠΎΠ»Π΅Π±Π°Π½ΠΈΠΉ ΠΎΡ‚ Ρ‡Π°ΡΡ‚ΠΎΡ‚Ρ‹ Π²Ρ‹Π½ΡƒΠΆΠ΄Π°ΡŽΡ‰Π΅ΠΉ силы.

Π­Ρ‚ΠΎ колСбания Π²Ρ‹Π·Π²Π°Π½Π½Ρ‹Π΅ воздСйствиСм Π½Π° Ρ‚Π΅Π»ΠΎ пСриодичСской внСшнСй силы. РСзонанс — явлСниС Ρ€Π΅Π·ΠΊΠΎΠ³ΠΎ возрастания Π°ΠΌΠΏΠ»ΠΈΡ‚ΡƒΠ΄Ρ‹ Π²Ρ‹Π½ΡƒΠΆΠ΄Π΅Π½Π½Ρ‹Ρ… ΠΊΠΎΠ»Π΅Π±Π°Π½ΠΈΠΉ Ρ‚Π΅Π»Π° ΠΏΡ€ΠΈ совпадСнии частоты Π²Ρ‹Π½ΡƒΠΆΠ΄Π°ΡŽΡ‰Π΅ΠΉ пСриодичСской силы с ΡΠΎΠ±ΡΡ‚Π²Π΅Π½Π½ΠΎΠΉ частотой ΠΊΠΎΠ»Π΅Π±Π°Π½ΠΈΠΉ Ρ‚Π΅Π»Π°. ΠŸΡ€ΠΈ рСзонансС Π²Ρ‹Π½ΡƒΠΆΠ΄Π°ΡŽΡ‰Π°Ρ сила Π² Ρ‚Π΅Ρ‡Π΅Π½ΠΈΠΈ всСго ΠΏΠ΅Ρ€ΠΈΠΎΠ΄Π° колСбания Π½Π°ΠΏΡ€Π°Π²Π»Π΅Π½Π° Π² Ρ‚ΡƒΠΆΠ΅ сторону, Ρ‡Ρ‚ΠΎ ΠΈ Π²Π΅ΠΊΡ‚ΠΎΡ€ скорости ΠΊΠΎΠ»Π΅Π±Π»ΡŽΡ‰Π΅Π³ΠΎΡΡ Ρ‚Π΅Π»Π°. ΠŸΡ€ΠΈ отсутствии трСния ΠΈ ΡΠΎΠΏΡ€ΠΎΡ‚ивлСния Π²ΠΎΠ·Π΄ΡƒΡ…Π° Π°ΠΌΠΏΠ»ΠΈΡ‚ΡƒΠ΄Π° ΠΊΠΎΠ»Π΅Π±Π°Π½ΠΈΠΉ ΠΌΠΎΠ³Π»ΠΈ Π±Ρ‹ Π²ΠΎΠ·Ρ€Π°ΡΡ‚Π°Ρ‚ΡŒ Π½Π΅ΠΎΠ³Ρ€Π°Π½ΠΈΡ‡Π΅Π½Π½ΠΎ. Амплитуда — это ΠΌΠΎΠ΄ΡƒΠ»ΡŒ максимального значСния ΠΈΠ·ΠΌΠ΅Π½ΡΡŽΡ‰Π΅ΠΉΡΡ частоты.

Π‘ΠΈΠ»Π΅Ρ‚ 22.

1. 1. ЀотоэффСкт ΠΈ Π΅Π³ΠΎ Π·Π°ΠΊΠΎΠ½Ρ‹. Π£Ρ€Π°Π²Π½Π΅Π½ΠΈΠ΅ Π­ΠΉΠ½ΡˆΡ‚Π΅ΠΉΠ½Π° для фотоэффСкта. ΠšΠ²Π°Π½Ρ‚Ρ‹ свСта. ΠŸΡ€ΠΈΠΌΠ΅Π½Π΅Π½ΠΈΠ΅ фотоэффСкта.

2. 2. Π­Π»Π΅ΠΊΡ‚Ρ€ΠΎΠ΅ΠΌΠΊΠΎΡΡ‚ΡŒ. ΠšΠΎΠ½Π΄Π΅Π½ΡΠ°Ρ‚ΠΎΡ€Ρ‹. ЭнСргия заряТСнного кондСнсатора. Π‘ΠΎΠ΅Π΄ΠΈΠ½Π΅Π½ΠΈΠ΅ кондСнсаторов.

3. 3. (Π—Π°Π΄Π°Ρ‡Π° Π½Π° Π·Π°ΠΊΠΎΠ½ сохранСния ΠΌΠ΅Ρ…. Π­Π½Π΅Ρ€Π³ΠΈΠΈ).

1. 1. ΠŸΡ€ΠΈ распространСнии свСта ΠΏΡ€ΠΎΡΠ²Π»ΡΡŽΡ‚ΡΡ Π΅Π³ΠΎ Π²ΠΎΠ»Π½ΠΎΠ²Ρ‹Π΅ свойства, Π° ΠΏΡ€ΠΈ взаимодСйствии с Π²Π΅Ρ‰Π΅ΡΡ‚Π²ΠΎΠΌ обнаруТиваСтся Π΅Π³ΠΎ прСрывистая структура, проявляСмая, Π½Π°ΠΏΡ€ΠΈΠΌΠ΅Ρ€, ΠΏΡ€ΠΈ фотоэффСктС. ЀотоэффСктом называСтся Π²Ρ‹Ρ€Ρ‹Π²Π°Π½ΠΈΠ΅ элСктрона вСщСства ΠΏΠΎΠ΄ дСйствиСм свСта. Π Π°Π·Π»ΠΈΡ‡Π°ΡŽΡ‚ Π΄Π²Π° Π²ΠΈΠ΄Π°: внСшний Π·Π°ΠΊΠ»ΡŽΡ‡Π°Π΅Ρ‚ΡΡ Π² ΠΈΡΠΏΡƒΡΠΊΠ°Π½ΠΈΠΈ элСктронов с ΠΏΠΎΠ²Π΅Ρ€Ρ…ности вСщСства, Π²Π½ΡƒΡ‚Ρ€Π΅Π½Π½ΠΈΠΉ связан с ΠΏΠ΅Ρ€Π΅Ρ€Π°ΡΠΏΡ€Π΅Π΄Π΅Π»Π΅Π½ΠΈΠ΅ΠΌ элСктронов Π°Ρ‚ΠΎΠΌΠΎΠ² ΠΏΠΎ ΠΈΡ… ΡΠΎΡΡ‚ΠΎΡΠ½ΠΈΡŽ Π² Ρ‚Π²Π΅Ρ€Π΄ΠΎΠΌ Ρ‚Π΅Π»Π΅, ΠΏΡ€ΠΈ ΠΏΠΎΠ³Π»ΠΎΡ‰Π΅Π½ΠΈΠΈ ΠΈΠΌ ΡΠ»Π΅ΠΊΡ‚Ρ€ΠΎΠΌΠ°Π³Π½ΠΈΡ‚Π½ΠΎΠ³ΠΎ излучСния. УстановлСн Π·Π°ΠΊΠΎΠ½: максимальная ΡΠΊΠΎΡ€ΠΎΡΡ‚ΡŒ Π²Ρ‹Π»Π΅Ρ‚Π°ΡŽΡ‰ΠΈΡ… элСктронов зависит ΠΎΡ‚ Ρ‡Π°ΡΡ‚ΠΎΡ‚Ρ‹ ΠΊΠΎΠ»Π΅Π±Π°Π½ΠΈΠΉ элСктромагнитной Π²ΠΎΠ»Π½Ρ‹ ΠΈ Ρ€Π°ΡΡ‚Π΅Ρ‚ с ΡƒΠ²Π΅Π»ΠΈΡ‡Π΅Π½ΠΈΠ΅ΠΌ частоты. Hv=AΠ²Ρ‹Ρ… + mV2/2, Π³Π΄Π΅ h ΠΏΠΎΡΡ‚оянная Планка.

ΠŸΡ€ΠΈ испускании свСт Π²Π΅Π΄Π΅Ρ‚ сСбя ΠΏΠΎΠ΄ΠΎΠ±Π½ΠΎ ΠΏΠΎΡ‚ΠΎΠΊΡƒ частиц с ΡΠ½Π΅Ρ€Π³ΠΈΠ΅ΠΉ E=hv, зависящСй ΠΎΡ‚ Ρ‡Π°ΡΡ‚ΠΎΡ‚Ρ‹ испускания. Π‘Π°ΠΌΠ° свСтовая частица ΠΏΠΎΠ»ΡƒΡ‡ΠΈΠ»Π° Π½Π°Π·Π²Π°Π½ΠΈΠ΅ Ρ„ΠΎΡ‚ΠΎΠ½, ΠΈΠ»ΠΈ свСтовой ΠΊΠ²Π°Π½Ρ‚. ЭнСргия Ρ„ΠΎΡ‚ΠΎΠ½Π° часто выраТаСтся Ρ‡Π΅Ρ€Π΅Π· Ρ†ΠΈΠΊΠ»ΠΈΡ‡Π΅ΡΠΊΡƒΡŽ частоту w=2Пv h-=h/2ΠΏ=1.05*10^-34 Π”ΠΆ*с Π’ΠΎΠ³Π΄Π° E = hv=h-w E=mc2 Π’ΠΎΠ³Π΄Π° m=hv/c2/ Π€ΠΎΡ‚ΠΎΠ½ Π½Π΅ ΠΈΠΌΠ΅Π΅Ρ‚ массы. Π’Π°ΠΊΠΈΠΌ ΠΎΠ±Ρ€Π°Π·ΠΎΠΌ p=mc=hv/c=h/лямда.

На ΠΎΡΠ½ΠΎΠ²Π΅ внСшнСго фотоэффСкта Ρ€Π°Π±ΠΎΡ‚Π°ΡŽΡ‚ фотоэлСмСнты (Ρ‚ΡƒΡ€Π½ΠΈΠΊΠ΅Ρ‚ Π² ΠΌΠ΅Ρ‚Ρ€ΠΎ) Π€ΠΎΡ‚ΠΎΠ΄ΠΈΠΎΠ΄Ρ‹ для измСрСния распрСдСлСниС Ρ‚Π΅ΠΌΠΏΠ΅Ρ€Π°Ρ‚ΡƒΡ€Ρ‹ слабо Π½Π°Π³Ρ€Π΅Ρ‚Ρ‹Ρ… Ρ‚Π΅Π». Π‘ΠΎΠ»Π½Π΅Ρ‡Π½Ρ‹Π΅ Π±Π°Ρ‚Π°Ρ€Π΅ΠΈ Π² ΠΊΠΎΡΠΌΠΈΡ‡Π΅ΡΠΊΠΈΡ… Π°ΠΏΠΏΠ°Ρ€Π°Ρ‚Π°Ρ….

2. 2. Π­Π»Π΅ΠΊΡ‚Ρ€ΠΎΠ΅ΠΌΠΊΠΎΡΡ‚ΡŒ. ΠšΠΎΠ½Π΄Π΅Π½ΡΠ°Ρ‚ΠΎΡ€Ρ‹. ЭнСргия заряТСнного кондСнсатора. Π‘ΠΎΠ΅Π΄ΠΈΠ½Π΅Π½ΠΈΠ΅ кондСнсаторов.

Π­Π»Π΅ΠΊΡ‚Ρ€ΠΎΠ΅ΠΌΠΊΠΎΡΡ‚ΡŒΡŽ называСтся физичСская Π²Π΅Π»ΠΈΡ‡ΠΈΠ½Π°, опрСдСляСмая ΠΎΡ‚Π½ΠΎΡˆΠ΅Π½ΠΈΠ΅ΠΌ заряда q ΠΎΠ΄Π½ΠΎΠΉ ΠΈΠ· ΠΏΠ»Π°ΡΡ‚ΠΈΠ½ кондСнсатора ΠΊ Π½Π°ΠΏΡ€ΡΠΆΠ΅Π½ΠΈΡŽ ΠΌΠ΅ΠΆΠ΄Ρƒ ΠΎΠ±ΠΊΠ»Π°Π΄ΠΊΠ°ΠΌΠΈ кондСнсатора U: Π‘=q/U. (Π€). Π­Π»Π΅ΠΊΡ‚Ρ€ΠΎΠ΅ΠΌΠΊΠΎΡΡ‚ΡŒ — это ΡΠΏΠΎΡΠΎΠ±Π½ΠΎΡΡ‚ΡŒ ΠΏΡ€ΠΎΠ²ΠΎΠ΄Π½ΠΈΠΊΠ° Π½Π°ΠΊΠ°ΠΏΠ»ΠΈΠ²Π°Ρ‚ΡŒ элСктричСский заряд. Для накоплСния Π·Π½Π°Ρ‡ΠΈΡ‚Π΅Π»ΡŒΠ½Ρ‹Ρ… количСств Ρ€Π°Π·Π½ΠΎΠΈΠΌΠ΅Π½Π½Ρ‹Ρ… элСктричСских зарядов ΠΏΡ€ΠΈΠΌΠ΅Π½ΡΡŽΡ‚ΡΡ кондСнсаторы. ΠšΠΎΠ½Π΄Π΅Π½ΡΠ°Ρ‚ΠΎΡ€ — это систСма ΠΏΡ€ΠΎΠ²ΠΎΠ΄Π½ΠΈΠΊΠΎΠ² ΡΠ»Π΅ΠΊΡ‚Ρ€ΠΎΠ΅ΠΌΠΊΠΎΡΡ‚ΡŒ ΠΊΠΎΡ‚ΠΎΡ€Ρ‹Ρ… Π½Π΅ ΠΌΠ΅Π½ΡΠ΅Ρ‚ся ΠΎΡ‚ Π²Π½Π΅ΡˆΠ½Π΅Π³ΠΎ воздСйствия. НапряТСниС ΠΌΠ΅ΠΆΠ΄Ρƒ ΠΎΠ±ΠΊΠ»Π°Π΄ΠΊΠ°ΠΌΠΈ заряТСнного кондСнсатора U=q/C. Π’ ΠΏΡ€ΠΎΡ†Π΅ΡΡΠ΅ разрядки напряТСниС ΠΏΠ°Π΄Π°Π΅Ρ‚ Π΄ΠΎ Π½ΡƒΠ»Ρ. Π‘Ρ€Π΅Π΄Π½Π΅Π΅ Π·Π½Π°Ρ‡Π΅Π½ΠΈΠ΅ напряТСния Π² ΠΏΡ€ΠΎΡ†Π΅ΡΡΠ΅ разрядки Uср=U/2 = q/2C. Для Ρ€Π°Π±ΠΎΡ‚Ρ‹ А, ΡΠΎΠ²Π΅Ρ€ΡˆΠ°Π΅ΠΌΠΎΠΉ элСктричСским ΠΏΠΎΠ»Π΅ΠΌ ΠΏΡ€ΠΈ разрядкС кондСнсатора A=q*Uср=qU/2 = CU2/2. Π—Π½Π°Ρ‡ΠΈΡ‚ ΠΏΠΎΡ‚Π΅Π½Ρ†ΠΈΠ°Π»ΡŒΠ½Π°Ρ энСргия W=A=CU2/2=q2/2C=qU/2. ЭнСргия кондСнсатора обусловлСна Ρ‚Π΅ΠΌ, Ρ‡Ρ‚ΠΎ элСктричСскоС ΠΏΠΎΠ»Π΅ ΠΌΠ΅ΠΆΠ΄Ρƒ Π΅Π³ΠΎ ΠΎΠ±ΠΊΠ»Π°Π΄ΠΊΠ°ΠΌΠΈ ΠΎΠ±Π»Π°Π΄Π°Π΅Ρ‚ энСргиСй.

ΠŸΡ€ΠΈ ΠΏΠΎΡΠ»Π΅Π΄ΠΎΠ²Π°Ρ‚Π΅Π»ΡŒΠ½ΠΎΠΌ соСдинСнии: 1/C0=1/C1+1/C2. ΠŸΡ€ΠΈ ΠΏΠ°Ρ€Π°Π»Π»Π΅Π»ΡŒΠ½ΠΎΠΌ C0=C1+C2.

Π‘ΠΈΠ»Π΅Ρ‚ 23.

1. 1. МодСль Π°Ρ‚ΠΎΠΌΠ° Π Π΅Π·Π΅Ρ€Ρ„ΠΎΡ€Π΄Π° — Π‘ΠΎΡ€Π°. ΠšΠ²Π°Π½Ρ‚ΠΎΠ²Ρ‹Π΅ постулаты Π‘ΠΎΡ€Π°.

2. 2. ΠŸΠΎΠ»ΡƒΠΏΡ€ΠΎΠ²ΠΎΠ΄Π½ΠΈΠΊΠΎΠ²Ρ‹Π΅ ΠΏΡ€ΠΈΠ±ΠΎΡ€Ρ‹.

3. 3. Π—Π°Π΄Π°Ρ‡Π° Π½Π° Π΄Π²ΠΈΠΆΠ΅Π½ΠΈΠ΅ Ρ‚Π΅Π»Π° ΠΏΠΎ ΠΎΠΊΡ€ΡƒΠΆΠ½ΠΎΡΡ‚ΠΈ.

1. 1. ЭкспСримСнты Π Π΅Π·Π΅Ρ€Ρ„ΠΎΡ€Π΄Π° послуТили основой для создания ΠΏΡ€ΠΎΡ‚ΠΎΠ½Π½ΠΎ — Π½Π΅ΠΉΡ‚Ρ€ΠΎΠ½Π½ΠΎΠΉ ΠΌΠΎΠ΄Π΅Π»ΠΈ Π°Ρ‚ΠΎΠΌΠ°. Π’ Ρ†Π΅Π½Ρ‚Ρ€Π΅ Π°Ρ‚ΠΎΠΌΠ° находится Π°Ρ‚ΠΎΠΌΠ½ΠΎΠ΅ ядро. Π’Π΅ΡΡŒ ΠΎΡΡ‚Π°Π»ΡŒΠ½ΠΎΠΉ объСм это элСктроны. Π’Π½ΡƒΡ‚Ρ€ΠΈ ядра элСктронов Π½Π΅Ρ‚. Π―Π΄Ρ€ΠΎ состоит ΠΈΠ· ΠΏΡ€ΠΎΡ‚ΠΎΠ½ΠΎΠ² ΠΈ Π½Π΅ΠΉΡ‚Ρ€ΠΎΠ½ΠΎΠ². Число элСктронов Ρ€Π°Π²Π½ΠΎ числу ΠΏΡ€ΠΎΡ‚ΠΎΠ½ΠΎΠ² Π² ΡΠ΄Ρ€Π΅. Масса элСктрона ΠΏΡ€ΠΈΠΌΠ΅Ρ€Π½ΠΎ Π² 2000 Ρ€Π°Π· мСньшС массы ΠΏΡ€ΠΎΡ‚ΠΎΠ½Π° ΠΈΠ»ΠΈ Π½Π΅ΠΉΡ‚Ρ€ΠΎΠ½Π°, поэтому ΠΏΠΎΡ‡Ρ‚ΠΈ вся масса находится Π² ΡΠ΄Ρ€Π΅. Π­Π»Π΅ΠΊΡ‚Ρ€ΠΎΠ½Ρ‹ Π»Π΅Ρ‚Π°ΡŽΡ‚ Π²ΠΎΠΊΡ€ΡƒΠ³ Π°Ρ‚ΠΎΠΌΠ° ΠΏΠΎ ΠΎΡ€Π±ΠΈΡ‚Π°ΠΌ. Но Ρ‚Π°ΠΊ ΠΊΠ°ΠΊ Π°Ρ‚ΠΎΠΌ двиТСтся ΠΏΠΎ ΠΎΡ€Π±ΠΈΡ‚Π°ΠΌ с ΡƒΡΠΊΠΎΡ€Π΅Π½ΠΈΠ΅ΠΌ, Ρ‚ΠΎ ΠΎΠ½ Π΄ΠΎΠ»ΠΆΠ΅Π½ ΠΈΡΠΏΡƒΡΠΊΠ°Ρ‚ΡŒ ΡΠ½Π΅Ρ€Π³ΠΈΡŽ, Π° Π·Π½Π°Ρ‡ΠΈΡ‚ Π² ΠΊΠΎΠ½Ρ†Π΅ ΡƒΠΏΠ°ΡΡ‚ΡŒ Π½Π° ΡΠ΄Ρ€ΠΎ, ΠΏΡ€Π΅Π²Ρ€Π°Ρ‚ΠΈΠ²ΡˆΠΈΡΡŒ Π² Π½Π΅ΠΉΡ‚Ρ€ΠΈΠ½ΠΎ. Π‘Ρ‹Π»ΠΈ Π²Π²Π΅Π΄Π΅Π½Ρ‹ постулаты Π‘ΠΎΡ€Π°: 1. Атомная систСма ΠΌΠΎΠΆΠ΅Ρ‚ находится Ρ‚ΠΎΠ»ΡŒΠΊΠΎ Π² ΠΎΡΠΎΠ±Ρ‹Ρ… стационары, ΠΈΠ»ΠΈ ΠΊΠ²Π°Π½Ρ‚ΠΎΠ²Ρ‹Ρ…, состояниях, ΠΊΠ°ΠΆΠ΄ΠΎΠΌΡƒ ΠΈΠ· ΠΊΠΎΡ‚ΠΎΡ€Ρ‹Ρ… соотвСтствуСт опрСдСлСнная энСргия. Π’ ΡΡ‚Π°Ρ†ΠΈΠΎΠ½Π°Ρ€Π½ΠΎΠΌ состоянии Π°Ρ‚ΠΎΠΌ Π½Π΅ ΠΈΠ·Π»ΡƒΡ‡Π°Π΅Ρ‚. Π­Ρ‚ΠΎΡ‚ постулат ΠΏΡ€ΠΎΡ‚ΠΈΠ²ΠΎΡ€Π΅Ρ‡ΠΈΡ‚ классичСской ΠΌΠ΅Ρ…Π°Π½ΠΈΠΊΠ΅, согласно ΠΊΠΎΡ‚ΠΎΡ€ΠΎΠΉ энСргия двиТущСгося элСктрона ΠΌΠΎΠΆΠ΅Ρ‚ Π±Ρ‹Ρ‚ΡŒ любой. ΠŸΡ€ΠΎΡ‚ΠΈΠ²ΠΎΡ€Π΅Ρ‡ΠΈΡ‚ ΠΈ ΡΠ»Π΅ΠΊΡ‚Ρ€ΠΎΠ΄ΠΈΠ½Π°ΠΌΠΈΠΊΠ΅, Ρ‚Π°ΠΊ ΠΊΠ°ΠΊ допускаСт ускорСнного двиТСния Π±Π΅Π· испускания энСргии. 2. ΠŸΡ€ΠΈ ΠΏΠ΅Ρ€Π΅Ρ…ΠΎΠ΄Π΅ Π°Ρ‚ΠΎΠΌΠ° ΠΈΠ· ΡΡ‚Π°Ρ†ΠΈΠΎΠ½Π°Ρ€Π½ΠΎΠ³ΠΎ состояния Π² Π΄Ρ€ΡƒΠ³ΠΎΠ΅ испускаСтся ΠΈΠ»ΠΈ поглощаСтся ΠΊΠ²Π°Π½Ρ‚ элСктромагнитной энСргии. Π­Ρ‚ΠΎΡ‚ постулат Ρ‚ΠΎΠΆΠ΅ ΠΏΡ€ΠΎΡ‚ΠΈΠ²ΠΎΡ€Π΅Ρ‡ΠΈΡ‚ элСктродинамикС МаксвСлла, согласно ΠΊΠΎΡ‚ΠΎΡ€ΠΎΠΉ частота ΠΈΠ·Π»ΡƒΡ‡Π΅Π½Π½ΠΎΠ³ΠΎ свСта Ρ€Π°Π²Π½Π° частотС обращСния элСктрона ΠΏΠΎ ΠΎΡ€Π±ΠΈΡ‚Π΅. По Ρ‚Π΅ΠΎΡ€ΠΈΠΈ Π‘ΠΎΡ€Π° частота Ρ‚ΠΎΠ»ΡŒΠΊΠΎ связана с ΠΈΠ·ΠΌΠ΅Π½Π΅Π½ΠΈΠ΅ΠΌ энСргии Π°Ρ‚ΠΎΠΌΠ°.

2. 2. ΠŸΠΎΠ»ΡƒΠΏΡ€ΠΎΠ²ΠΎΠ΄Π½ΠΈΠΊΠΎΠ²Ρ‹Π΅ ΠΏΡ€ΠΈΠ±ΠΎΡ€Ρ‹.

Π‘ΠΈΠ»Π΅Ρ‚ 24.

1. 1. Бостав ядра Π°Ρ‚ΠΎΠΌΠ°. Π˜Π·ΠΎΡ‚ΠΎΠΏΡ‹. ЭнСргия связи.

2. 2. БобствСнная ΠΈ ΠΏΡ€ΠΈΠΌΠ΅ΡΠ½Π°Ρ ΠΏΡ€ΠΎΠ²ΠΎΠ΄ΠΈΠΌΠΎΡΡ‚ΡŒ ΠΏΠΎΠ»ΡƒΠΏΡ€ΠΎΠ²ΠΎΠ΄Π½ΠΈΠΊΠΎΠ². Π’Π΅Ρ€ΠΌΠΎΠΈ фоторСзисторы.

3. 3. (Π—Π°Π΄Π°Ρ‡Π° Π½Π° Π²Ρ‚ΠΎΡ€ΠΎΠΉ Π·Π°ΠΊΠΎΠ½ ΠΡŒΡŽΡ‚ΠΎΠ½Π°).

1. Богласно ΠΏΡ€ΠΎΡ‚ΠΎΠ½Π½ΠΎ — Π½Π΅ΠΉΡ‚Ρ€ΠΎΠ½Π½ΠΎΠΉ ΠΌΠΎΠ΄Π΅Π»ΠΈ Π°Ρ‚ΠΎΠΌΠ½Ρ‹Π΅ ядра состоят ΠΈΠ· ΡΠ»Π΅ΠΌΠ΅Π½Ρ‚Π°Ρ€Π½Ρ‹Ρ… частиц Π΄Π²ΡƒΡ… Π²ΠΈΠ΄ΠΎΠ²: ΠΏΡ€ΠΎΡ‚ΠΎΠ½ΠΎΠ² ΠΈ Π½Π΅ΠΉΡ‚Ρ€ΠΎΠ½ΠΎΠ². Π˜Π·Π²Π΅ΡΡ‚Π½ΠΎ, Ρ‡Ρ‚ΠΎ заряд элСктрона ΠΏΡ€ΠΈΠΌΠ΅Ρ€Π½ΠΎ Ρ€Π°Π²Π΅Π½ заряду ΠΏΡ€ΠΎΡ‚ΠΎΠ½Π° = 1,6*10^-19 Кл, Π° ΠΌΠ°ΡΡΠ° ΠΏΡ€ΠΎΡ‚ΠΎΠ½Π° =1,6726 * 10^-27 ΠΊΠ³ ΠΈΠ»ΠΈ 1,728 Π°.Π΅.ΠΌ. НСйтрон Π½Π΅ ΠΈΠΌΠ΅Π΅Ρ‚ элСктричСского заряда, Π΅Π³ΠΎ масса Ρ€Π°Π²Π½Π° 1,867 Π°.Π΅.ΠΌ.1 Π°.Π΅.ΠΌ.=1,6605*10^-27 ΠΊΠ³ ΠΈ ΡΠΎΠΎΡ‚вСтствуСт энСргии 931,5 ΠœΡΠ’. Число ΠΏΡ€ΠΎΡ‚ΠΎΠ½ΠΎΠ² Π² ΡΠ΄Ρ€Π΅ Π½Π°Π·Ρ‹Π²Π°ΡŽΡ‚ зарядом ядра. ΠœΠ°ΡΡΠΎΠ²Ρ‹ΠΌ числом ядра, А Π½Π°Π·Ρ‹Π²Π°ΡŽΡ‚ сумму числа ΠΏΡ€ΠΎΡ‚ΠΎΠ½ΠΎΠ² Z ΠΈ Ρ‡ΠΈΡΠ»Π° Π½Π΅ΠΉΡ‚Ρ€ΠΎΠ½ΠΎΠ² N А=Z+N.

Π˜Π·ΠΎΡ‚ΠΎΠΏΡ‹ — ядра с ΠΎΠ΄ΠΈΠ½Π°ΠΊΠΎΠ²ΠΎΠΉ массой.

ΠœΠΈΠ½ΠΈΠΌΠ°Π»ΡŒΠ½ΡƒΡŽ ΡΠ½Π΅Ρ€Π³ΠΈΡŽ, ΠΊΠΎΡ‚ΠΎΡ€ΡƒΡŽ Π½ΡƒΠΆΠ½ΠΎ Π·Π°Ρ‚Ρ€Π°Ρ‚ΠΈΡ‚ΡŒ для раздСлСния Π°Ρ‚ΠΎΠΌΠ½ΠΎΠ³ΠΎ ядра Π½Π° ΡΠΎΡΡ‚Π°Π²Π»ΡΡŽΡ‰ΠΈΠ΅ Π΅Π³ΠΎ Π½ΡƒΠΊΠ»ΠΎΠ½Ρ‹, Π½Π°Π·Ρ‹Π²Π°ΡŽΡ‚ энСргиСй связи. Π­Ρ‚Π° энСргия расходуСтся Π½Π° ΡΠΎΠ²Π΅Ρ€ΡˆΠ΅Π½ΠΈΠ΅ Ρ€Π°Π±ΠΎΡ‚Ρ‹ ΠΏΡ€ΠΎΡ‚ΠΈΠ² дСйствия ядСрных сил притяТСния ΠΌΠ΅ΠΆΠ΄Ρƒ Π½ΡƒΠΊΠ»ΠΎΠ½Π°ΠΌΠΈ. На ΠΎΡΠ½ΠΎΠ²Π΅ Π·Π°ΠΊΠΎΠ½Π° сохранСния энСргии ΠΌΠΎΠΆΠ½ΠΎ ΡƒΡ‚Π²Π΅Ρ€ΠΆΠ΄Π°Ρ‚ΡŒ, Ρ‡Ρ‚ΠΎ ΠΏΡ€ΠΈ ΠΎΠ±Ρ€Π°Π·ΠΎΠ²Π°Π½ΠΈΠΈ ядра ΠΈΠ· ΠΎΡ‚Π΄Π΅Π»ΡŒΠ½Ρ‹Ρ… Π½ΡƒΠΊΠ»ΠΎΠ½ΠΎΠ² выдСляСтся энСргия, равная энСргия связи. E=mc2. ВсякоС ΠΈΠ·ΠΌΠ΅Π½Π΅Π½ΠΈΠ΅ энСргии систСмы Π½Π° Π΄Π΅Π»ΡŒΡ‚Π° Π• ΡΠΎΠΏΡ€ΠΎΠ²ΠΎΠΆΠ΄Π°Π΅Ρ‚ся ΠΈΠ·ΠΌΠ΅Π½Π΅Π½ΠΈΠ΅ΠΌ Π΅Π΅ ΠΌΠ°ΡΡΡ‹ Π΄Π΅Π»ΡŒΡ‚Π° m Π½Π° Π²Π΅Π»ΠΈΡ‡ΠΈΠ½Ρƒ Π΄Π΅Π»ΡŒΡ‚Π°Π•/c2. Π’ΠΎΡ‡Π½Π΅ΠΉΡˆΠΈΠ΅ измСрСния массы ядСр ΠΏΠΎΠΊΠ°Π·Ρ‹Π²Π°ΡŽΡ‚, Ρ‡Ρ‚ΠΎ сумма масс покоя ΠΏΡ€ΠΎΡ‚ΠΎΠ½ΠΎΠ² ΠΈ Π½Π΅ΠΉΡ‚Ρ€ΠΎΠ½ΠΎΠ², ΡΠΎΡΡ‚Π°Π²Π»ΡΡŽΡ‰ΠΈΡ… Π΄Π°Π½Π½ΠΎΠ΅ ядро большС массы покоя этого ядра. Π Π°Π·Π½ΠΎΡΡ‚ΡŒ масс Π΄Π΅Π»ΡŒΡ‚Π° M=Zm+Nm — Mя.

2. БобствСнная ΠΈ ΠΏΡ€ΠΈΠΌΠ΅ΡΠ½Π°Ρ ΠΏΡ€ΠΎΠ²ΠΎΠ΄ΠΈΠΌΠΎΡΡ‚ΡŒ ΠΏΠΎΠ»ΡƒΠΏΡ€ΠΎΠ²ΠΎΠ΄Π½ΠΈΠΊΠΎΠ². Π’Π΅Ρ€ΠΌΠΎΠΈ фоторСзисторы.

ΠŸΠΎΠ»ΡƒΠΏΡ€ΠΎΠ²ΠΎΠ΄Π½ΠΈΠΊ — это класс вСщСств, Π·Π°Π½ΠΈΠΌΠ°ΡŽΡ‰ΠΈΡ… ΠΏΡ€ΠΎΠΌΠ΅ΠΆΡƒΡ‚ΠΎΡ‡Π½ΠΎΠ΅ ΠΏΠΎΠ»ΠΎΠΆΠ΅Π½ΠΈΠ΅ ΠΌΠ΅ΠΆΠ΄Ρƒ вСщСствами, Ρ…ΠΎΡ€ΠΎΡˆΠΎ проводящими элСктричСский Ρ‚ΠΎΠΊ, ΠΈ Π²Π΅Ρ‰Π΅ΡΡ‚Π²Π°ΠΌΠΈ, практичСски Π½Π΅ ΠΏΡ€ΠΎΠ²ΠΎΠ΄ΡΡ‰ΠΈΠΌ элСктричСского Ρ‚ΠΎΠΊΠ°. Для Π½ΠΈΡ… Ρ…Π°Ρ€Π°ΠΊΡ‚Π΅Ρ€Π½Π° сильная Π·Π°Π²ΠΈΡΠΈΠΌΠΎΡΡ‚ΡŒ ΠΈΡ… ΡΠ²ΠΎΠΉΡΡ‚Π² ΠΈ Ρ…арактСристик ΠΎΡ‚ ΠΌΠΈΠΊΡ€ΠΎΡΠΊΠΎΠΏΠΈΡ‡Π΅ΡΠΊΠΈΡ… количСств содСрТащихся Π² Π½ΠΈΡ… примСсСй. Π’Π°ΠΆΠ½ΠΎΠ΅ свойство ΠΏΠΎΠ»ΡƒΠΏΡ€ΠΎΠ²ΠΎΠ΄Π½ΠΈΠΊΠΎΠ² состоит Π² Ρ‚ΠΎΠΌ, Ρ‡Ρ‚ΠΎ элСктричСский Ρ‚ΠΎΠΊ пСрСносится Π² Π½ΠΈΡ… Π½Π΅ Ρ‚ΠΎΠ»ΡŒΠΊΠΎ ΠΎΡ‚Ρ€ΠΈΡ†Π°Ρ‚Π΅Π»ΡŒΠ½Ρ‹ΠΌΠΈ зарядами — элСктронами, Π½ΠΎ ΠΈ Ρ€Π°Π²Π½Ρ‹ΠΌΠΈ ΠΈΠΌ ΠΏΠΎ Π²Π΅Π»ΠΈΡ‡ΠΈΠ΅ ΠΏΠΎΠ»ΠΎΠΆΠΈΡ‚Π΅Π»ΡŒΠ½Ρ‹ΠΌΠΈ зарядами — Π΄Ρ‹Ρ€ΠΊΠ°ΠΌΠΈ. Атомы Π² ΠΊΡ€ΠΈΡΡ‚Π°Π»Π»Π΅ ΠΏΠΎΠ»ΡƒΠΏΡ€ΠΎΠ²ΠΎΠ΄Π½ΠΈΠΊΠ° связаны ΠΌΠ΅ΠΆΠ΄Ρƒ собой с ΠΏΠΎΠΌΠΎΡ‰ΡŒΡŽ элСктронов внСшнСй элСктронной ΠΎΠ±ΠΎΠ»ΠΎΡ‡ΠΊΠΈ. ΠŸΡ€ΠΈ Ρ‚Π΅ΠΏΠ»ΠΎΠ²Ρ‹Ρ… колСбаниях Π°Ρ‚ΠΎΠΌΠΎΠ² тСпловая энСргия Π½Π΅Ρ€Π°Π²Π½ΠΎΠΌΠ΅Ρ€Π½ΠΎ распрСдСляСтся ΠΌΠ΅ΠΆΠ΄Ρƒ элСктронами, ΠΎΠ±Ρ€Π°Π·ΡƒΡŽΡ‰ΠΈΠΌΠΈ связи. ΠžΡ‚Π΄Π΅Π»ΡŒΠ½Ρ‹Π΅ элСктроны ΠΌΠΎΠ³ΡƒΡ‚ ΠΏΠΎΠ»ΡƒΡ‡Π°Ρ‚ΡŒ количСство Ρ‚Π΅ΠΏΠ»ΠΎΠ²ΠΎΠΉ энСргии, достаточноС для Ρ‚ΠΎΠ³ΠΎ, Ρ‡Ρ‚ΠΎΠ±Ρ‹ «ΠΎΡ‚ΠΎΡ€Π²Π°Ρ‚ΡŒΡΡ» ΠΎΡ‚ ΡΠ²ΠΎΠ΅Π³ΠΎ Π°Ρ‚ΠΎΠΌΠ° ΠΈ ΠΏΠΎΠ»ΡƒΡ‡ΠΈΡ‚ΡŒ Π²ΠΎΠ·ΠΌΠΎΠΆΠ½ΠΎΡΡ‚ΡŒ свободно ΠΏΠ΅Ρ€Π΅ΠΌΠ΅Ρ‰Π°Ρ‚ΡŒΡΡ Π² ΠΊΡ€ΠΈΡΡ‚Π°Π»Π»Π΅, Ρ‚Π΅ ΡΡ‚Π°Ρ‚ΡŒ ΠΏΠΎΡ‚Π΅Π½Ρ†ΠΈΠ°Π»ΡŒΠ½Ρ‹ΠΌΠΈ носитСлями Ρ‚ΠΎΠΊΠ°. Π’Π°ΠΊΠΎΠΉ ΡƒΡ…ΠΎΠ΄ элСктрона Π½Π°Ρ€ΡƒΡˆΠ°Π΅Ρ‚ ΡΠ»Π΅ΠΊΡ‚Ρ€ΠΈΡ‡Π΅ΡΠΊΡƒΡŽ Π½Π΅ΠΉΡ‚Ρ€Π°Π»ΡŒΠ½ΠΎΡΡ‚ΡŒ Π°Ρ‚ΠΎΠΌΠ°, Ρƒ ΠΊΠΎΡ‚ΠΎΡ€ΠΎΠ³ΠΎ Π²ΠΎΠ·Π½ΠΈΠΊΠ°Π΅Ρ‚ ΠΏΠΎΠ»ΠΎΠΆΠΈΡ‚Π΅Π»ΡŒΠ½Ρ‹ΠΉ заряд, Ρ€Π°Π²Π½Ρ‹ΠΉ ΠΏΠΎ Π²Π΅Π»ΠΈΡ‡ΠΈΠ½Π΅ заряду ΡƒΡˆΠ΅Π΄ΡˆΠ΅Π³ΠΎ элСктрона. Π­Ρ‚ΠΎ Π²Π°ΠΊΠ°Π½Ρ‚Π½ΠΎΠ΅ мСсто называСтся Π΄Ρ‹Ρ€ΠΊΠΎΠΉ. Π’Π°ΠΊ ΠΊΠ°ΠΊ Π²Π°ΠΊΠ°Π½Ρ‚Π½ΠΎΠ΅ мСсто ΠΌΠΎΠΆΠ΅Ρ‚ Π±Ρ‹Ρ‚ΡŒ занято элСктроном сосСднСй связи, Π΄Ρ‹Ρ€ΠΊΠ° Ρ‚Π°ΠΊΠΆΠ΅ ΠΌΠΎΠΆΠ΅Ρ‚ ΠΏΠ΅Ρ€Π΅ΠΌΠ΅Ρ‰Π°Ρ‚ΡŒΡΡ Π²Π½ΡƒΡ‚Ρ€ΠΈ кристалла ΠΈ ΡΠ²Π»ΡΡ‚ΡŒΡΡ ΡƒΠΆΠ΅ ΠΏΠΎΠ»ΠΎΠΆΠΈΡ‚Π΅Π»ΡŒΠ½Ρ‹ΠΌ носитСлСм Ρ‚ΠΎΠΊΠ°. ЕстСствСнно, Ρ‡Ρ‚ΠΎ элСктроны ΠΈ Π΄Ρ‹Ρ€ΠΊΠΈ ΠΏΡ€ΠΈ этих условиях Π²ΠΎΠ·Π½ΠΈΠΊΠ°ΡŽΡ‚ Π² Ρ€Π°Π²Π½Ρ‹Ρ… количСствах, ΠΈ ΡΠ»Π΅ΠΊΡ‚Ρ€ΠΎΠΏΡ€ΠΎΠ²ΠΎΠ΄Π½ΠΎΡΡ‚ΡŒ Ρ‚Π°ΠΊΠΎΠ³ΠΎ идСального кристалла Π±ΡƒΠ΄Π΅Ρ‚ Π² Ρ€Π°Π²Π½ΠΎΠΉ стСпСни ΠΎΠΏΡ€Π΅Π΄Π΅Π»ΡΡ‚ΡŒΡΡ ΠΊΠ°ΠΊ ΠΏΠΎΠ»ΠΎΠΆΠΈΡ‚Π΅Π»ΡŒΠ½Ρ‹ΠΌΠΈ, Ρ‚Π°ΠΊ ΠΈ ΠΎΡ‚Ρ€ΠΈΡ†Π°Ρ‚Π΅Π»ΡŒΠ½Ρ‹ΠΌΠΈ зарядами. Если Π½Π° ΠΌΠ΅ΡΡ‚ΠΎ Π°Ρ‚ΠΎΠΌΠ° основного ΠΏΠΎΠ»ΡƒΠΏΡ€ΠΎΠ²ΠΎΠ΄Π½ΠΈΠΊΠ° ΠΏΠΎΠΌΠ΅ΡΡ‚ΠΈΡ‚ΡŒ Π°Ρ‚ΠΎΠΌ примСси, Π²ΠΎ Π²Π½Π΅ΡˆΠ½Π΅ΠΉ элСктронной ΠΎΠ±ΠΎΠ»ΠΎΡ‡ΠΊΠ΅ ΠΊΠΎΡ‚ΠΎΡ€ΠΎΠ³ΠΎ содСрТится Π½Π° ΠΎΠ΄ΠΈΠ½ элСктрон большС, Ρ‡Π΅ΠΌ Ρƒ Π°Ρ‚ΠΎΠΌΠ° основного ΠΏΠΎΠ»ΡƒΠΏΡ€ΠΎΠ²ΠΎΠ΄Π½ΠΈΠΊΠ°, Ρ‚ΠΎ Ρ‚Π°ΠΊΠΎΠΉ элСктрон окаТСтся ΠΊΠ°ΠΊ Π±Ρ‹ лишним. Достаточно Π² Π΄Π΅ΡΡΡ‚ΠΊΠΈ Ρ€Π°Π· мСньшС энСргии, Ρ‡Ρ‚ΠΎΠ±Ρ‹ ΠΎΡ‚ΠΎΡ€Π²Π°Ρ‚ΡŒ Π΅Π³ΠΎ ΠΎΡ‚ ΡΠ²ΠΎΠ΅Π³ΠΎ Π°Ρ‚ΠΎΠΌΠ° ΠΈ ΠΏΡ€Π΅Π²Ρ€Π°Ρ‚ΠΈΡ‚ΡŒ Π² ΡΠ²ΠΎΠ±ΠΎΠ΄Π½Ρ‹ΠΉ элСктрон. Π’Π°ΠΊΠΈΠ΅ примСси Π½Π°Π·Ρ‹Π²Π°ΡŽΡ‚ΡΡ Π΄ΠΎΠ½ΠΎΡ€Π½Ρ‹ΠΌΠΈ, Ρ‚Π΅ ΠΎΡ‚Π΄Π°ΡŽΡ‰ΠΈΠΌΠΈ лишний элСктрон.

Π’Π²Π΅Π΄Π΅Π½ΠΈΠ΅

Π² ΠΏΠΎΠ»ΡƒΠΏΡ€ΠΎΠ²ΠΎΠ΄Π½ΠΈΠΊ примСсСй, внСшняя элСктронная ΠΎΠ±ΠΎΠ»ΠΎΡ‡ΠΊΠ° ΠΊΠΎΡ‚ΠΎΡ€ΠΎΠ³ΠΎ содСрТит мСньшСС количСство элСктронов, ΠΏΡ€ΠΈΠ²ΠΎΠ΄ΠΈΡ‚ ΠΊ ΠΏΠΎΡΠ²Π»Π΅Π½ΠΈΡŽ Π½Π΅Π·Π°ΠΏΠΎΠ»Π½Π΅Π½Π½Ρ‹Ρ… связСй, ΠΈ Π΄Ρ‹Ρ€ΠΊΠ° ΠΏΠΎΠ»ΡƒΡ‡Π°Π΅Ρ‚ Π²ΠΎΠ·ΠΌΠΎΠΆΠ½ΠΎΡΡ‚ΡŒ свободно ΠΏΠ΅Ρ€Π΅ΠΌΠ΅Ρ‰Π°Ρ‚ΡŒΡΡ ΠΏΠΎ ΠΊΡ€ΠΈΡΡ‚Π°Π»Π»Ρƒ. Π˜Π½Ρ‹ΠΌΠΈ словами, Π΄Π²ΠΈΠΆΠ΅Π½ΠΈΠ΅ Π΄Ρ‹Ρ€ΠΊΠΈ — это ΠΏΠΎΡΠ»Π΅Π΄ΠΎΠ²Π°Ρ‚Π΅Π»ΡŒΠ½Ρ‹ΠΉ ΠΏΠ΅Ρ€Π΅Ρ…ΠΎΠ΄ элСктронов ΠΈΠ· ΠΎΠ΄Π½ΠΎΠΉ сосСднСй связи Π² Π΄Ρ€ΡƒΠ³ΡƒΡŽ. Π’Π°ΠΊΠΈΠ΅ примСси, ΠΏΡ€ΠΈΠ½ΠΈΠΌΠ°ΡŽΡ‰ΠΈΠ΅ элСктрон, Π½Π°Π·Ρ‹Π²Π°ΡŽΡ‚ΡΡ Π°ΠΊΡ†Π΅ΠΏΡ‚ΠΎΡ€Π½Ρ‹ΠΌΠΈ. Π‘ ΡƒΠ²Π΅Π»ΠΈΡ‡Π΅Π½ΠΈΠ΅ΠΌ количСства примСсСй Ρ‚ΠΎΠ³ΠΎ ΠΈΠ»ΠΈ ΠΈΠ½ΠΎΠ³ΠΎ Ρ‚ΠΈΠΏΠ° ΡΠ»Π΅ΠΊΡ‚Ρ€ΠΎΠΏΡ€ΠΎΠ²ΠΎΠ΄Π½ΠΎΡΡ‚ΡŒ кристалла Π½Π°Ρ‡ΠΈΠ½Π°Π΅Ρ‚ ΠΏΡ€ΠΈΠΎΠ±Ρ€Π΅Ρ‚Π°Ρ‚ΡŒ всС Π±ΠΎΠ»Π΅Π΅ Π²Ρ‹Ρ€Π°ΠΆΠ΅Π½Π½Ρ‹ΠΉ элСктронный ΠΈΠ»ΠΈ Π΄Ρ‹Ρ€ΠΎΡ‡Π½Ρ‹ΠΉ Ρ…Π°Ρ€Π°ΠΊΡ‚Π΅Ρ€. Π­Π»Π΅ΠΊΡ‚Ρ€ΠΎΠΏΡ€ΠΎΠ²ΠΎΠ΄ΠΈΠΌΠΎΡΡ‚ΡŒ Π±Ρ‹Π²Π°Π΅Ρ‚ n-Ρ‚ΠΈΠΏΠ° ΠΈ p-Ρ‚ΠΈΠΏΠ°.

Π‘ΠΈΠ»Π΅Ρ‚ 25.

1. 1. Π Π°Π΄ΠΈΠΎΠ°ΠΊΡ‚ΠΈΠ²Π½ΠΎΡΡ‚ΡŒ. Π’ΠΈΠ΄Π° Ρ€Π°Π΄ΠΈΠΎΠ°ΠΊΡ‚ΠΈΠ²Π½Ρ‹Ρ… ΠΈΠ·Π»ΡƒΡ‡Π΅Π½ΠΈΠΉ ΠΈ ΠΈΡ… ΡΠ²ΠΎΠΉΡΡ‚Π²Π°. БиологичСскоС дСйствиС ΠΈΠΎΠ½ΠΈΠ·ΠΈΡ€ΡƒΡŽΡ‰ΠΈΡ… ΠΈΠ·Π»ΡƒΡ‡Π΅Π½ΠΈΠΉ.

2. 2. Π‘ΠΈΠ»Π° трСния Π² Π±Ρ‹Ρ‚Ρƒ ΠΈ Ρ‚Π΅Ρ…Π½ΠΈΠΊΠ΅. Π˜Π·ΠΌΠ΅Ρ€ΡŒΡ‚Π΅ силу трСния.

3. 3. (Π—Π°Π΄Π°Ρ‡Π° Π½Π° ΠΈΡΠΏΠΎΠ»ΡŒΠ·ΠΎΠ²Π°Π½ΠΈΠ΅ Π·Π°ΠΊΠΎΠ½ΠΎΠ² элСктролиза).

1. 1. Π Π°Π΄ΠΈΠΎΠ°ΠΊΡ‚ΠΈΠ²Π½ΠΎΡΡ‚ΡŒ прСдставляСт собой ΡΠ°ΠΌΠΎΠΏΡ€ΠΎΠΈΠ·Π²ΠΎΠ»ΡŒΠ½Ρ‹ΠΉ процСсс, происходящий Π² Π°Ρ‚ΠΎΠΌΠ°Ρ… Ρ€Π°Π΄ΠΈΠΎΠ°ΠΊΡ‚ΠΈΠ²Π½Ρ‹Ρ… элСмСнтов. Π­Ρ‚ΠΎ явлСниС опрСдСляСтся ΠΊΠ°ΠΊ ΡΠ°ΠΌΠΎΠΏΡ€ΠΎΠΈΠ·Π²ΠΎΠ»ΡŒΠ½ΠΎΠ΅ ΠΏΡ€Π΅Π²Ρ€Π°Ρ‰Π΅Π½ΠΈΠ΅ нСустойчивого ΠΈΠ·ΠΎΡ‚ΠΎΠΏΠ° ΠΎΠ΄Π½ΠΎΠ³ΠΎ химичСского элСмСнта Π² ΠΈΠ·ΠΎΡ‚ΠΎΠΏ Π΄Ρ€ΡƒΠ³ΠΎΠ³ΠΎ; ΠΏΡ€ΠΈ этом происходит испусканиС элСктронов, ΠΏΡ€ΠΎΡ‚ΠΎΠ½ΠΎΠ², Π½Π΅ΠΉΡ‚Ρ€ΠΎΠ½ΠΎΠ² ΠΈΠ»ΠΈ ядСр гСлия. Π’ΠΈΠ΄Ρ‹: Π³Π°ΠΌΠΌΠ°Π»ΡƒΡ‡ΠΈ — это ΠΎΡ‡Π΅Π½ΡŒ ΠΊΠΎΡ€ΠΎΡ‚ΠΊΠΈΠ΅ элСктромагнитныС Π²ΠΎΠ»Π½Ρ‹. Π˜Ρ… Π΄Π»ΠΈΠ½Π° ΠΎΡ‚ 10^-10 Π΄ΠΎ 10^-13 ΠΌ. Π‘ΠΊΠΎΡ€ΠΎΡΡ‚ΡŒ ΠΈΡ… Ρ€Π°ΡΠΏΡ€ΠΎΡΡ‚ранСния ΠΎΠΊΠΎΠ»ΠΎ скорости свСта. Π‘Π΅Ρ‚Π° — Π»ΡƒΡ‡ΠΈ. ΠŸΡ€ΠΈΡ€ΠΎΠ΄Π° Π±Π΅Ρ‚Π° Π»ΡƒΡ‡Π΅ΠΉ Π±Ρ‹Π»Π° установлСна Ρ€Π°Π½ΡŒΡˆΠ΅ всСх — Π² 1899 Π³ΠΎΠ΄Ρƒ. По ΠΈΡ… ΠΎΡ‚ΠΊΠ»ΠΎΠ½Π΅Π½ΠΈΡŽ Π² ΡΠ»Π΅ΠΊΡ‚ричСском ΠΈ ΠΌΠ°Π³Π½ΠΈΡ‚Π½Ρ‹Ρ… полях Π±Ρ‹Π» ΠΈΠ·ΠΌΠ΅Ρ€Π΅Π½ ΡƒΠ΄Π΅Π»ΡŒΠ½Ρ‹ΠΉ заряд. Оказалось, Ρ‡Ρ‚ΠΎ ΠΎΠ½ Ρ‚Π°ΠΊΠΎΠΉ ΠΆΠ΅ ΠΊΠ°ΠΊ Ρƒ ΡΠ»Π΅ΠΊΡ‚Ρ€ΠΎΠ½Π°. Π—Π½Π°Ρ‡ΠΈΡ‚ Π±Π΅Ρ‚Π° Π»ΡƒΡ‡ΠΈ --это элСктроны, двиТущиСся с ΠΎΠ³Ρ€ΠΎΠΌΠ½Ρ‹ΠΌΠΈ скоростями, ΠΎΡ‡Π΅Π½ΡŒ Π±Π»ΠΈΠ·ΠΊΠΈΠΌΠΈ ΠΊ ΡΠΊΠΎΡ€ΠΎΡΡ‚ΠΈ свСта. ΠΠ»ΡŒΡ„Π° — частицы. Π—Π½Π°ΠΊ заряда Ρƒ Π½ΠΈΡ… ΠΏΠΎΠ»ΠΎΠΆΠΈΡ‚Π΅Π»ΡŒΠ½Ρ‹ΠΉ. Π­Ρ‚ΠΎ ядро Π°Ρ‚ΠΎΠΌΠΎΠ² гСлия. Π—Π½Π°Ρ‡ΠΈΡ‚ Π΅Π΅ Π·Π°Ρ€ΡΠ΄ 2Π΅, Π° ΠΌΠ°ΡΡΠ° 4 Π°.Π΅.ΠΌ. Π’Ρ‹Π»Π΅Ρ‚Π°ΡŽΡ‰ΠΈΠ΅ ΠΈΠ· Ρ€Π°Π΄ΠΈΠΎΠ°ΠΊΡ‚ΠΈΠ²Π½Ρ‹Ρ… ядСр Π°Π»ΡŒΡ„Π° частицы ΠΈΠΌΠ΅ΡŽΡ‚ большиС скорости, Π΄ΠΎΡΡ‚ΠΈΠ³Π°ΡŽΡ‰ΠΈΠ΅ дСсятых Π΄ΠΎΠ»Π΅ΠΉ скорости свСта, Π·Π½Π°Ρ‡ΠΈΡ‚ ΠΎΠ±Π»Π°Π΄Π°ΡŽΡ‚ большой энСргиСй. Π˜Ρ… ΡΠ²ΠΎΠΉΡΡ‚Π²Π° — это ΠΏΡ€ΠΎΠ½ΠΈΠΊΠ°ΡŽΡ‰Π°Ρ ΠΈ ΠΈΠΎΠ½ΠΈΠ·ΠΈΡ€ΡƒΡŽΡ‰Π°Ρ.

Π˜Π·Π»ΡƒΡ‡Π΅Π½ΠΈΠ΅ Π²Ρ‹Π·Ρ‹Π²Π°Π΅Ρ‚ ΠΈΠΎΠ½ΠΈΠ·Π°Ρ†ΠΈΡŽ Π°Ρ‚ΠΎΠΌΠΎΠ² ΠΈ ΠΌΠΎΠ»Π΅ΠΊΡƒΠ» ΠΈ ΡΡ‚ΠΎ ΠΏΡ€ΠΈΠ²ΠΎΠ΄ΠΈΡ‚ ΠΊ ΠΈΠ·ΠΌΠ΅Π½Π΅Π½ΠΈΡŽ ΠΈΡ… Ρ…имичСской активности. Для Ρ…Π°Ρ€Π°ΠΊ4тСристики воздСйствия излучСния вводится понятиС поглощСнная Π΄ΠΎΠ·Π° излучСния. D=E/m. (Π³Ρ€Π΅ΠΉ). Π‘Π°ΠΌΠΎΠΉ ΠΏΠ΅Ρ€Π²ΠΎΠΉ Π΅Π΄ΠΈΠ½ΠΈΡ†Π΅ΠΉ Π΄ΠΎΠ·Ρ‹ излучСния Π±Ρ‹Π» Ρ€Π΅Π½Ρ‚Π³Π΅Π½, ΠΎΠ½ ΠΎΠΏΡ€Π΅Π΄Π΅Π»ΡΠ΅Ρ‚ся ΠΏΠΎ ΠΈΠΎΠ½ΠΈΠ·Π°Ρ†ΠΈΠΈ, ΠΏΡ€ΠΎΠΈΠ·Π²ΠΎΠ΄ΠΈΠΌΠΎΠΉ ΠΈΠ·Π»ΡƒΡ‡Π΅Π½ΠΈΠ΅ΠΌ. 1 Ρ€Π΅Π½Ρ‚Π³Π΅Π½ ΠΎΠΏΡ€Π΅Π΄Π΅Π»ΡΡŽΡ‚ΡΡ ΠΊΠ°ΠΊ Π΄ΠΎΠ·Ρƒ рСнтгСновского ΠΈΠ»ΠΈ Π³Π°ΠΌΠΌΠ° — излучСния ΠΏΡ€ΠΈ ΠΊΠΎΡ‚ΠΎΡ€ΠΎΠΉ 1 ΠΊΠ³ Π²ΠΎΠ·Π΄ΡƒΡ…Π° ΠΏΠΎΠ³Π»ΠΎΡ‰Π°Π΅Ρ‚ ΡΠ½Π΅Ρ€Π³ΠΈΡŽ 0,878 * 10^-2. 1Π =0,01 Π“Ρ€.

2. 2. Π‘ΠΈΠ»Π° трСния Π² Π±Ρ‹Ρ‚Ρƒ ΠΈ Ρ‚Π΅Ρ…Π½ΠΈΠΊΠ΅. Π˜Π·ΠΌΠ΅Ρ€ΡŒΡ‚Π΅ силу трСния.

Π‘Π΅Π· этой силы Π³Π²ΠΎΠ·Π΄ΠΈ ΠΈ Π²ΠΈΠ½Ρ‚Ρ‹ Π±Ρ‹ выскакивали ΠΈΠ· ΡΡ‚Π΅Π½, Π½ΠΈ ΠΎΠ΄Π½ΠΎΠΉ Π²Π΅Ρ‰ΠΈ нСльзя Π±Ρ‹Π»ΠΎ Π±Ρ‹ Π΄Π΅Ρ€ΠΆΠ°Ρ‚ΡŒ Π² Ρ€ΡƒΠΊΠ°Ρ…, Π½ΠΈΠΊΠΎΠ³Π΄Π° Π±Ρ‹ Π½Π΅ ΠΏΡ€Π΅ΠΊΡ€Π°Ρ‰Π°Π»ΡΡ Π²ΠΈΡ…Ρ€ΡŒ. Π—Π²ΡƒΠΊ Π½Π΅ ΡƒΠΌΠΎΠ»ΠΊΠ°Π» Π±Ρ‹, Π° Π·Π²ΡƒΡ‡Π°Π» эхом. ΠŸΡ€ΠΈΠΌΠ΅Π½ΡΡŽΡ‚: санки, Π²Ρ‹Π²ΠΎΠ·ΠΊΠ° лСса ΠΏΠΎ Π»Π΅Π΄ΡΠ½ΠΎΠΉ Π΄ΠΎΡ€ΠΎΠ³Π΅ (Π΄ΠΎΡ€ΠΎΠ³ΠΈ с Π»Π΅Π΄ΡΠ½Ρ‹ΠΌΠΈ Ρ€Π΅Π»ΡŒΡΠ°ΠΌΠΈ для санСй: Π΄Π²Π΅ лошади ΠΌΠΎΠ³Π»ΠΈ Ρ‚Π°Ρ‰ΠΈΡ‚ΡŒ 70 Ρ‚ΠΎΠ½ Π±Ρ€Π΅Π²Π΅Π½).

Π‘ΠΈΠ»Π΅Ρ‚ 26.

1. 1. ЦСпная рСакция дСлСния ядСр ΡƒΡ€Π°Π½Π°. Π―Π΄Π΅Ρ€Π½Ρ‹ΠΉ Ρ€Π΅Π°ΠΊΡ‚ΠΎΡ€.

2. 2. ΠœΠ΅Ρ…Π°Π½ΠΈΡ‡Π΅ΡΠΊΠ°Ρ Ρ€Π°Π±ΠΎΡ‚Π° ΠΈ ΠΌΠΎΡ‰Π½ΠΎΡΡ‚ΡŒ. Π˜Π·ΠΌΠ΅Ρ€ΡŒΡ‚Π΅ ΠšΠŸΠ” ΠΏΡ€ΠΈ подъСмС ΠΏΠΎ Π½Π°ΠΊΠ»ΠΎΠ½Π½ΠΎΠΉ плоскости.

3. 3. (Π—Π°Π΄Π°Ρ‡Π° Π½Π° Ρ‚Π΅ΠΏΠ»ΠΎΠ²ΠΎΠ΅ дСйствиС Ρ‚ΠΎΠΊΠ°).

1. РСакция происходит ΠΏΠΎΠ΄ дСйствиСм ΠΌΠ΅Π΄Π»Π΅Π½Π½Ρ‹Ρ… Π½Π΅ΠΉΡ‚Ρ€ΠΎΠ½ΠΎΠ². Π£ Π½Π΅Π΅ Π΄Π²Π΅ Π²Π°ΠΆΠ½Ρ‹Ρ… особСнности: 1. ΠŸΡ€ΠΈ Π΄Π΅Π»Π΅Π½ΠΈΠΈ ΠΊΠ°ΠΆΠ΄ΠΎΠ³ΠΎ ядра выдСляСтся Π·Π½Π°Ρ‡ΠΈΡ‚Π΅Π»ΡŒΠ½Π°Ρ энСргия. 2. ΠšΠ°ΠΆΠ΄Ρ‹ΠΉ Π°ΠΊΡ‚ дСлСния сопровоТдаСтся Π²Ρ‹Π»Π΅Ρ‚ΠΎΠΌ 2- 3 Π²Ρ‚ΠΎΡ€ΠΈΡ‡Π½Ρ‹Ρ… Π½Π΅ΠΉΡ‚Ρ€ΠΎΠ½ΠΎΠ², Ρ‚Π΅ ΡΠ΄Π΅Π»Π°Ρ‚ΡŒ Ρ€Π΅Π°ΠΊΡ†ΠΈΡŽ Ρ†Π΅ΠΏΠ½ΠΎΠΉ — ΡΠ°ΠΌΠΎΠΏΠΎΠ΄Π΄Π΅Ρ€ΠΆΠΈΠ²Π°ΡŽΡ‰Π΅ΠΉΡΡ. УправляСмыС Ρ†Π΅ΠΏΠ½Ρ‹Π΅ ядСрныС Ρ€Π΅Π°ΠΊΡ†ΠΈΠΈ ΠΎΡΡƒΡ‰Π΅ΡΡ‚Π²Π»ΡΡŽΡ‚ΡΡ Π² ΡΠ΄Π΅Ρ€Π½Ρ‹Ρ… Ρ€Π΅Π°ΠΊΡ‚ΠΎΡ€Π°Ρ…. Π’ Π½ΠΈΡ… ΠΈΡΠΏΠΎΠ»ΡŒΠ·ΡƒΡŽΡ‚ΡΡ Π½Π΅ Ρ‡ΠΈΡΡ‚Ρ‹Π΅ ΠΈΠ·ΠΎΡ‚ΠΎΠΏΡ‹, Π° ΠΈΡ… ΡΠΌΠ΅ΡΠΈ, Π½Π°ΠΏΡ€ΠΈΠΌΠ΅Ρ€ ΠΏΡ€ΠΈΡ€ΠΎΠ΄Π½Ρ‹ΠΉ ΡƒΡ€Π°Π½, ΠΎΠ±ΠΎΠ³Π°Ρ‰Π΅Π½Π½Ρ‹ΠΉ ΠΈΠ·ΠΎΡ‚ΠΎΠΏΠ°ΠΌΠΈ ΡƒΡ€Π°Π½Π° 235. Π‘ ΠΏΠΎΠΌΠΎΡ‰ΡŒΡŽ ΡΠΏΠ΅Ρ†ΠΈΠ°Π»ΡŒΠ½Ρ‹Ρ… ΠΏΠΎΠ³Π»ΠΎΡ‚ΠΈΡ‚Π΅Π»Π΅ΠΉ Π½Π΅ΠΉΡ‚Ρ€ΠΎΠ½ΠΎΠ² число Π΄Π΅Π»Π΅Π½ΠΈΠΉ Π² Π΅Π΄ΠΈΠ½ΠΈΡ†Ρƒ объСма Π² Π΅Π΄ΠΈΠ½ΠΈΡ†Ρƒ Π²Ρ€Π΅ΠΌΠ΅Π½ΠΈ поддСрТиваСтся Π½Π° Π·Π°Π΄Π°Π½Π½ΠΎΠΌ ΡƒΡ€ΠΎΠ²Π½Π΅. Для Ρ€Π΅Π°ΠΊΡ†ΠΈΠΈ ΠΏΡ€ΠΈΠ³ΠΎΠ΄Π½Ρ‹ Ρ‚ΠΎΠ»ΡŒΠΊΠΎ ядра ΠΈΠ·ΠΎΡ‚ΠΎΠΏΠΎΠ² ΡƒΡ€Π°Π½Π° с ΠΌΠ°ΡΡΠΎΠ²Ρ‹ΠΌ числом 235. Π―Π΄Ρ€Π° дСлятся ΠΏΠΎΠ΄ дСйствиСм ΠΊΠ°ΠΊ быстрых, Ρ‚Π°ΠΊ ΠΈ ΠΌΠ΅Π΄Π»Π΅Π½Π½Ρ‹Ρ… Π½Π΅ΠΉΡ‚Ρ€ΠΎΠ½ΠΎΠ². Для Π΅Π΅ ΠΎΡΡƒΡ‰Π΅ΡΡ‚влСния Π½Π΅ΠΎΠ±Ρ…ΠΎΠ΄ΠΈΠΌΠΎ, Ρ‡Ρ‚ΠΎΠ±Ρ‹ срСднСС число Π²Ρ‹ΡΠ²ΠΎΠ±ΠΎΠ΄ΠΈΠ²ΡˆΠΈΡ…ΡΡ Π² Π΄Π°Π½Π½ΠΎΠΉ массС Π½Π΅ΠΉΡ‚Ρ€ΠΎΠ½ΠΎΠ² Π½Π΅ ΡƒΠΌΠ΅Π½ΡŒΡˆΠ°Π»ΠΎΡΡŒ с Ρ‚Π΅Ρ‡Π΅Π½ΠΈΠ΅ΠΌ Π²Ρ€Π΅ΠΌΠ΅Π½ΠΈ. Π’Π°ΠΆΠ½ΠΎΠ΅ Π·Π½Π°Ρ‡Π΅Π½ΠΈΠ΅ ΠΈΠΌΠ΅Π΅Ρ‚ Π½Π΅ Π²Ρ‹Π·Ρ‹Π²Π°ΡŽΡ‰ΠΈΠΉ дСлСния Π·Π°Ρ…Π²Π°Ρ‚ Π½Π΅ΠΉΡ‚Ρ€ΠΎΠ½ΠΎΠ² ядрами ΠΈΠ·ΠΎΡ‚ΠΎΠΏΠ° 238. ПослС Π·Π°Ρ…Π²Π°Ρ‚Π° образуСтся Ρ€Π°Π΄ΠΈΠΎΠ°ΠΊΡ‚ΠΈΠ²Π½Ρ‹ΠΉ ΠΈΠ·ΠΎΡ‚ΠΎΠΏ 239/92 с ΠΏΠ΅Ρ€ΠΈΠΎΠ΄ΠΎΠΌ полураспада 23 ΠΌΠΈΠ½ΡƒΡ‚Ρ‹. Распад происходит с ΠΈΡΠΏΡƒΡΠΊΠ°Π½ΠΈΠ΅ΠΌ элСктрона ΠΈ ΠΎΠ±Ρ€Π°Π·ΠΎΠ²Π°Π½ΠΈΠ΅ΠΌ ΠΏΠ΅Ρ€Π²ΠΎΠ³ΠΎ Π·Π°ΡƒΡ€Π°Π½ΠΎΠ²ΠΎΠ³ΠΎ элСмСнта — нСптуния: 239/92U-239/93Np + 0/-1 e.

НСптуний Π² ΡΠ²ΠΎΡŽ ΠΎΡ‡Π΅Ρ€Π΅Π΄ΡŒ Π±Π΅Ρ‚Π° — Ρ€Π°Π΄ΠΈΠΎΠ°ΠΊΡ‚ΠΈΠ²Π΅Π½ с ΠΏΠ΅Ρ€ΠΈΠΎΠ΄ΠΎΠΌ полураспада ΠΎΠΊΠΎΠ»ΠΎ Π΄Π²ΡƒΡ… Π΄Π½Π΅ΠΉ. ΠžΠ±Ρ€Π°Π·ΡƒΠ΅Ρ‚ΡΡ ΠΏΠ»ΡƒΡ‚ΠΎΠ½ΠΈΠΉ. 239/93Ne-239/94Pu + 0/-1 e. ΠŸΠ»ΡƒΡ‚ΠΎΠ½ΠΈΠΉ ΠΎΡ‚Π½ΠΎΡΠΈΡ‚Π΅Π»ΡŒΠ½ΠΎ стабилСн, Ρ‚Π°ΠΊ ΠΊΠ°ΠΊ Π΅Π³ΠΎ ΠΏΠ΅Ρ€ΠΈΠΎΠ΄ полураспада ΠΎΠΊΠΎΠ»ΠΎ 24 000 Π»Π΅Ρ‚.

2. ΠœΠ΅Ρ…Π°Π½ΠΈΡ‡Π΅ΡΠΊΠ°Ρ Ρ€Π°Π±ΠΎΡ‚Π° ΠΈ ΠΌΠΎΡ‰Π½ΠΎΡΡ‚ΡŒ. Π˜Π·ΠΌΠ΅Ρ€ΡŒΡ‚Π΅ ΠšΠŸΠ” ΠΏΡ€ΠΈ подъСмС ΠΏΠΎ Π½Π°ΠΊΠ»ΠΎΠ½Π½ΠΎΠΉ плоскости.

Π­Π»Π΅ΠΌΠ΅Π½Ρ‚Π°Ρ€Π½ΠΎΠΉ Ρ€Π°Π±ΠΎΡ‚ΠΎΠΉ силы Π½Π° ΡΠ»Π΅ΠΌΠ΅Π½Ρ‚Π°Ρ€Π½ΠΎΠΌ ΠΏΠ΅Ρ€Π΅ΠΌΠ΅Ρ‰Π΅Π½ΠΈΠΈ ΠΌΠ°Ρ‚Π΅Ρ€ΠΈΠ°Π»ΡŒΠ½ΠΎΠΉ Ρ‚ΠΎΡ‡ΠΊΠΈ называСтся скалярная физичСская Π²Π΅Π»ΠΈΡ‡ΠΈΠ½Π°. Π—Π½Π°Ρ‡Π΅Π½ΠΈΠ΅ элСмСнтарной Ρ€Π°Π±ΠΎΡ‚Ρ‹ силы зависит ΠΎΡ‚ Π²Ρ‹Π±ΠΎΡ€Π° систСмы отсчСта. Π•Π΄ΠΈΠ½ΠΈΡ†Π° Ρ€Π°Π±ΠΎΡ‚Ρ‹ — Π”ΠΆ. ΠŸΠΎΡ‚Π΅Π½Ρ†ΠΈΠ°Π»ΡŒΠ½Ρ‹ΠΌΠΈ Π½Π°Π·Ρ‹Π²Π°ΡŽΡ‚ΡΡ силы, Ρ€Π°Π±ΠΎΡ‚Π° ΠΊΠΎΡ‚ΠΎΡ€Ρ‹Ρ… зависит ΠΎΡ‚ Π½Π°Ρ‡Π°Π»ΡŒΠ½ΠΎΠ³ΠΎ ΠΈ ΠΊΠΎΠ½Π΅Ρ‡Π½ΠΎΠ³ΠΎ полоТСния двиТущСйся ΠΌΠ°Ρ‚Π΅Ρ€ΠΈΠ°Π»ΡŒΠ½ΠΎΠΉ Ρ‚ΠΎΡ‡ΠΊΠΈ ΠΈΠ»ΠΈ Ρ‚Π΅Π»Π° ΠΈ Π½Π΅ Π·Π°Π²ΠΈΡΠΈΡ‚ ΠΎΡ‚ Ρ„ΠΎΡ€ΠΌΡ‹ Ρ‚Ρ€Π°Π΅ΠΊΡ‚ΠΎΡ€ΠΈΠΈ. ΠŸΡ€ΠΈ Π·Π°ΠΌΠΊΠ½ΡƒΡ‚ΠΎΠΉ Ρ‚Ρ€Π°Π΅ΠΊΡ‚ΠΎΡ€ΠΈΠΈ Ρ€Π°Π±ΠΎΡ‚Π° ΠΏΠΎΡ‚Π΅Π½Ρ†ΠΈΠ°Π»ΡŒΠ½ΠΎΠΉ силы всСгда Ρ€Π°Π²Π½Π° 0. К ΠΏΠΎΡ‚Π΅Π½Ρ†ΠΈΠ°Π»ΡŒΠ½Ρ‹ΠΌ силам относятся силы тяготСния, силы упругости ΠΈ ΡΠ»Π΅ΠΊΡ‚ричСскиС силы. Быстроту выполнСния Ρ€Π°Π±ΠΎΡ‚Ρ‹ Π² Ρ‚Π΅Ρ…Π½ΠΈΠΊΠ΅ Ρ…Π°Ρ€Π°ΠΊΡ‚Π΅Ρ€ΠΈΠ·ΡƒΡŽΡ‚ ΠΌΠΎΡ‰Π½ΠΎΡΡ‚ΡŒΡŽ. Она ΠΏΠΎΠΊΠ°Π·Ρ‹Π²Π²Π°Π΅Ρ‚, какая Ρ€Π°Π±ΠΎΡ‚Π° ΡΠΎΠ²Π΅Ρ€ΡˆΠ°Π΅Ρ‚ΡΡ Ρ‚Π΅Π»ΠΎΠΌ Π² Π΅Π΄Π΅Π½ΠΈΡ†Ρƒ Π²Ρ€Π΅ΠΌΠ΅Π½ΠΈ. Π­Ρ‚ΠΎ ΡΠΊΠΎΡ€ΠΎΡΡ‚ΡŒ ΡΠΎΠ²Π΅Ρ€ΡˆΠ΅Π½ΠΈΡ Ρ€Π°Π±ΠΎΡ‚Ρ‹ N=A/t. Π˜Π·ΠΌΠ΅Ρ€ΡΠ΅Ρ‚ΡΡ Π² Π²Π°Ρ‚Ρ‚Π°Ρ… (Π·Π° 1 с Π²Ρ‹ΠΏΠΎΠ»Π½ΡΠ΅Ρ‚ся Ρ€Π°Π±ΠΎΡ‚Ρ‹ Π² 1 Π”ΠΆ).

Π‘ΠΈΠ»Π΅Ρ‚ 27.

1. 1. ΠŸΠΎΠ²Π΅Ρ€Ρ…Π½ΠΎΡΡ‚Π½ΠΎΠ΅ натяТСниС. Π‘ΠΌΠ°Ρ‡ΠΈΠ²Π°Π½ΠΈΠ΅ ΠΈ ΠΊΠ°ΠΏΠΈΠ»Π»ΡΡ€Π½ΠΎΡΡ‚ΡŒ. ΠŸΠΎΠ²Π΅Ρ€Ρ…Π½ΠΎΡΡ‚Π½ΠΎΠ΅ натяТСниС Π² ΠΏΡ€ΠΈΡ€ΠΎΠ΄Π΅ ΠΈ Ρ‚Π΅Ρ…Π½ΠΈΠΊΠ΅.

2. 2. АктивноС ΠΈ Ρ€Π΅Π°ΠΊΡ‚ΠΈΠ²Π½ΠΎΠ΅ сопротивлСниС. Π—Π°ΠΊΠΎΠ½ Ома для Ρ†Π΅ΠΏΠΈ ΠΏΠ΅Ρ€Π΅ΠΌΠ΅Π½Π½ΠΎΠ³ΠΎ Ρ‚ΠΎΠΊΠ°.

3. 3. «Π˜Π—ΠœΠ•Π Π’Π• Π£Π”Π•Π›Π¬ΠΠžΠ• Π‘ΠžΠŸΠ ΠΠ’Π˜Π’Π›Π•ΠΠ˜Π• ΠŸΠ ΠžΠ’ΠžΠ”ΠΠ˜ΠšΠ».

1. 1. ΠŸΠΎΠ²Π΅Ρ€Ρ…Π½ΠΎΡΡ‚Π½Π°Ρ энСргия — это Π΄ΠΎΠΏΠΎΠ»Π½ΠΈΡ‚Π΅Π»ΡŒΠ½Π°Ρ энСргия, ΠΊΠΎΡ‚ΠΎΡ€ΠΎΠΉ ΠΎΠ±Π»Π°Π΄Π°ΡŽΡ‚ ΠΌΠΎΠ»Π΅ΠΊΡƒΠ»Ρ‹ повСрхности, Π² ΠΎΡ‚Π»ΠΈΡ‡ΠΈΠΈ ΠΎΡ‚ ΠΌΠΎΠ»Π΅ΠΊΡƒΠ» Π²Π½ΡƒΡ‚Ρ€ΠΈ Тидкости. П=сигма*s. Π‘ΠΈΠ³ΠΌΠ° — это коэффициСнт повСрхностного натяТСния. FΡ‚Π½=сигма l (A=F*Π΄Π΅Π»ΡŒΡ‚Π° h A=Π΄Π΅Π»ΡŒΡ‚Π°ΠŸ=сигма l Π΄Π΅Π»ΡŒΡ‚Π° h). УсловиС смачивания: сила взаимодСйствия ΠΌΠΎΠ»Π΅ΠΊΡƒΠ» Ρ‚Π²Π΅Ρ€Π΄Ρ‹Ρ… Ρ‚Π΅Π» Π³ΠΎΡ€Π°Π·Π΄ΠΎ большС силы взаимодСйствия Тидкости. ΠšΠ°ΠΏΠΈΠ»Π»ΡΡ€Ρ‹ — это Ρ‚ΠΎΠ½ΠΊΠΈΠ΅ Ρ‚Ρ€ΡƒΠ±ΠΎΡ‡ΠΊΠΈ, сосуды. h=2сигма/Ρ€ΠΎ g r. (F=mg, сигма l =Ρ€ΠΎ V g, сигма 2 П r = Ρ€ΠΎ (П r2 h) g. Π‘Π΅Ρ€Π΅Π·ΠΎΠ²Ρ‹ΠΉ сок, ΠΏΠΎΠ»ΠΎΡ‚Π΅Π½Ρ†Π΅, ΠΌΡ‹Ρ‚ΡŒΠ΅ ΠΏΠΎΠ»Π°, Π±ΠΎΡ€ΠΎΠ½ΠΈΡ‚ΡŒ, Π³ΠΈΠ΄Ρ€ΠΎΠ·Π°Ρ‰ΠΈΡ‚Π°,. с Π³ΡƒΡΡ, Π²ΠΎΠ΄Π° Π² ΡΠΈΡ‚Π΅. ΠšΠΎΡΡ„Ρ„ΠΈΡ†ΠΈΠ΅Π½Ρ‚ зависит ΠΎΡ‚ Ρ€ΠΎΠ΄Π° Тидкости, ΠΏΡ€ΠΈ Π½Π°Π³Ρ€Π΅Π²Π°Π½ΠΈΠΈ — ΡƒΠΌΠ΅Π½ΡŒΡˆΠ°Π΅Ρ‚ΡΡ.

2. 2. АктивноС ΠΈ Ρ€Π΅Π°ΠΊΡ‚ΠΈΠ²Π½ΠΎΠ΅ сопротивлСниС. Π—Π°ΠΊΠΎΠ½ Ома для Ρ†Π΅ΠΏΠΈ ΠΏΠ΅Ρ€Π΅ΠΌΠ΅Π½Π½ΠΎΠ³ΠΎ Ρ‚ΠΎΠΊΠ°.

Устройства, ΠΏΠΎΠ»Π½ΠΎΡΡ‚ΡŒΡŽ ΠΏΡ€Π΅ΠΎΠ±Ρ€Π°Π·ΡƒΡŽΡ‰ΠΈΠ΅ ΡΠ»Π΅ΠΊΡ‚Ρ€ΠΈΡ‡Π΅ΡΠΊΡƒΡŽ ΡΠ½Π΅Ρ€Π³ΠΈΡŽ Π² Π΄Ρ€ΡƒΠ³ΠΈΠ΅ Π²ΠΈΠ΄Ρ‹ энСргии, Π½Π°Π·Ρ‹Π²Π°ΡŽΡ‚ Π°ΠΊΡ‚ΠΈΠ²Π½ΠΎΠΉ Π½Π°Π³Ρ€ΡƒΠ·ΠΊΠΎΠΉ, Π° ΠΈΡ… ΡΠΎΠΏΡ€ΠΎΡ‚ΠΈΠ²Π»Π΅Π½ΠΈΠ΅ — Π°ΠΊΡ‚ΠΈΠ²Π½Ρ‹ΠΌ сопротивлСниСм. ΠŸΡ€Π΅Π΄ΠΏΠΎΠ»ΠΎΠΆΠΈΠΌ, Ρ‡Ρ‚ΠΎ напряТСниС Π½Π° ΠΊΠΎΠ½Ρ†Π°Ρ… Ρ†Π΅ΠΏΠΈ мСняСтся ΠΏΠΎ Π³Π°Ρ€ΠΌΠΎΠ½ΠΈΡ‡Π΅ΡΠΊΠΎΠΌΡƒ Π·Π°ΠΊΠΎΠ½Ρƒ u=Umcos wt. Как ΠΈ Π² ΡΠ»ΡƒΡ‡Π°Π΅ постоянного Ρ‚ΠΎΠΊΠ°, ΠΌΠ³Π½ΠΎΠ²Π΅Π½Π½ΠΎΠ΅ Π·Π½Π°Ρ‡Π΅Π½ΠΈΠ΅ силы Ρ‚ΠΎΠΊΠ° ΠΏΡ€ΠΎΠΏΠΎΡ€Ρ†ΠΈΠΎΠ½Π°Π»ΡŒΠ½ΠΎ ΠΌΠ³Π½ΠΎΠ²Π΅Π½Π½ΠΎΠΌΡƒ Π·Π½Π°Ρ‡Π΅Π½ΠΈΡŽ напряТСния. ΠŸΠΎΡΡ‚ΠΎΠΌΡƒ примСняСтся Π·Π°ΠΊΠΎΠ½ Ома для участка Ρ†Π΅ΠΏΠΈ: i=U/R=Umcos wt/R = Im cos wt. На Π°ΠΊΡ‚ΠΈΠ²Π½ΠΎΠΌ сопротивлСнии колСбания силы Ρ‚ΠΎΠΊΠ° ΡΠΎΠ²ΠΏΠ°Π΄Π°ΡŽΡ‚ ΠΏΠΎ Ρ„Π°Π·Π΅ с ΠΊΠΎΠ»Π΅Π±Π°Π½ΠΈΡΠΌΠΈ напряТСния.

Π‘ΠΈΠ»Π΅Ρ‚ 3.

1. 1. Масса ΠΈ Π΅Π΅ ΠΈΠ·ΠΌΠ΅Ρ€Π΅Π½ΠΈΠ΅. Π‘ΠΈΠ»Π°. Π‘Π»ΠΎΠΆΠ΅Π½ΠΈΠ΅ сил. Π’Ρ‚ΠΎΡ€ΠΎΠΉ Π·-Π½ ΠΡŒΡŽΡ‚ΠΎΠ½Π°.

2. 2. ЭлСктричСский Ρ‚ΠΎΠΊ Π² Ρ€Π°ΡΡ‚Π²ΠΎΡ€Π°Ρ… ΠΈ Ρ€Π°ΡΠΏΠ»Π°Π²ΠΎΠ² элСктролитов. Π—Π°ΠΊΠΎΠ½ элСктролиза. ΠŸΡ€ΠΈΠΌΠ΅Π½Π΅Π½ΠΈΠ΅ элСктролиза Π² Ρ‚Π΅Ρ…Π½ΠΈΠΊΠ΅.

3. 3. «Π˜Π—ΠœΠ•Π Π•ΠΠ˜Π• ΠŸΠžΠšΠΠ—ΠΠ’Π•Π›Π― ΠŸΠ Π•Π›ΠžΠœΠ›Π•ΠΠ˜Π― Π‘Π’Π•ΠšΠ›Π».

1. 1. Масса — физичСская скалярная Π²Π΅Π»ΠΈΡ‡ΠΈΠ½Π°, ΠΏΡ€ΠΎΠΏΠΎΡ€Ρ†ΠΈΠΎΠ½Π°Π»ΡŒΠ½Π°Ρ ΠΎΡ‚Π½ΠΎΡˆΠ΅Π½ΠΈΡŽ Π²Π΅Π»ΠΈΡ‡ΠΈΠ½Ρ‹ Π΄Π΅ΠΉΡΡ‚Π²ΡƒΡŽΡ‰Π΅ΠΉ Π½Π° Ρ‚Π΅Π»ΠΎ силы ΠΊ ΡΠΎΠΎΠ±Ρ‰Π°Π΅ΠΌΠΎΠΌΡƒ Сю ΡƒΡΠΊΠΎΡ€Π΅Π½ΠΈΡŽ:

m=F/a.

Масса ΠΌΠ΅Ρ€Π° инСртности Ρ‚Π΅Π»Π°. Π§Π΅ΠΌ большС масса, Ρ‚Π΅ΠΌ Ρ‚Π΅Π»ΠΎ Π±ΠΎΠ»Π΅Π΅ ΠΈΠ½Π΅Ρ€Ρ‚Π½ΠΎ. ΠžΠ±ΠΎΠ·Π½Π°Ρ‡Π°Π΅Ρ‚ΡΡ m, Π° ΠΈΠ·ΠΌΠ΅Ρ€ΡΠ΅Ρ‚ся Π² ΠΊΠΈΠ»ΠΎΠ³Ρ€Π°ΠΌΠΌΠ°Ρ… (ΠΊΠ³).

Π‘ΠΈΠ»Π° — физичСская вСкторная Π²Π΅Π»ΠΈΡ‡ΠΈΠ½Π°, Ρ…Π°Ρ€Π°ΠΊΡ‚Π΅Ρ€ΠΈΠ·ΡƒΡŽΡ‰Π°Ρ взаимодСйствиС Ρ‚Π΅Π». Π’ ΡΠΈΡΡ‚Π΅ΠΌΠ΅ Π‘И Π·Π° Π΅Π΄ΠΈΠ½ΠΈΡ†Ρƒ силы ΠΏΡ€ΠΈΠ½ΠΈΠΌΠ°ΡŽΡ‚ силу, которая Ρ‚Π΅Π»Ρƒ массой 1 ΠΊΠ³ сообщаСт ускорСниС 1 ΠΌ/с2. Π­Ρ‚Ρƒ Π΅Π΄ΠΈΠ½ΠΈΡ†Ρƒ Π½Π°Π·Ρ‹Π²Π°ΡŽΡ‚ Π½ΡŒΡŽΡ‚ΠΎΠ½ΠΎΠΌ (Н).

Π’Ρ‚ΠΎΡ€ΠΎΠΉ Π·-Π½ ΠΡŒΡŽΡ‚ΠΎΠ½Π°.

УскорСниС ΠΏΡ€ΠΈΠΎΠ±Ρ€Π΅Ρ‚Π°Π΅ΠΌΠΎΠ΅ Ρ‚Π΅Π»ΠΎΠΌ ΠΏΠΎΠ΄ дСйствиСм Π½Π° Π½Π΅Π³ΠΎ силы ΠΏΡ€ΡΠΌΠΎΠΏΡ€ΠΎΠΏΠΎΡ€Ρ†ΠΈΠΎΠ½Π°Π»ΡŒΠ½ΠΎ этой силС ΠΈ ΠΎΠ±Ρ€Π°Ρ‚Π½ΠΎ Π΅Π³ΠΎ массС.

Π’Ρ‚ΠΎΡ€ΠΎΠΉ Π·-Π½ ΠΈΠΌΠ΅Π΅Ρ‚ ряд особСнностСй:

1. Он ΡΠΏΡ€Π°Π²Π΅Π΄Π»ΠΈΠ² для Π»ΡŽΠ±Ρ‹Ρ… сил;

2. 2. Π‘ΠΈΠ»Π° являСтся ΠΏΡ€ΠΈΡ‡ΠΈΠ½ΠΎΠΉ, Π²Ρ‹Π·Ρ‹Π²Π°ΡŽΡ‰Π΅ΠΉ ускорСниС;

3. 3. Π’Π΅ΠΊΡ‚ΠΎΡ€ ускорСния соноправлСн с Π²Π΅ΠΊΡ‚ΠΎΡ€ΠΎΠΌ силы;

4. 4. Если Π½Π° Ρ‚Π΅Π»ΠΎ дСйствуСт Π½Π΅ ΠΎΠ΄Π½Π° сила, Ρ‚ΠΎ Π±Π΅Ρ€Π΅Ρ‚ся Ρ€Π°Π²Π½ΠΎΠ΄Π΅ΠΉΡΡ‚Π²ΡƒΡŽΡ‰Π°Ρ этих сил;

5. 5. Если всС дСйствия скомпСнсированы, Ρ‚ΠΎ Ρ€Π°Π²Π½ΠΎΠ΄Π΅ΠΉΡΡ‚Π²ΡƒΡŽΡ‰Π°Ρ Ρ€Π°Π²Π½Π° 0, ΡΠ»Π΅Π΄ΠΎΠ²Π°Ρ‚Π΅Π»ΡŒΠ½ΠΎ ускорСниС Ρ€Π°Π²Π½ΠΎ 0, Ρ‚. Π΅. Ρ‚Π΅Π»ΠΎ двиТСтся Ρ€Π°Π²Π½ΠΎΠΌΠ΅Ρ€Π½ΠΎ прямолинСйно ΠΈΠ»ΠΈ покоится.

1 Н — это такая сила, которая сообщаСт Ρ‚Π΅Π»Ρƒ массой 1 ΠΊΠ³ ускорСниС 1 ΠΌ/с2.

ΠŸΡ€Π°Π²ΠΈΠ»ΠΎ слоТСния сил:

1. 1. Если слагаСмыС силы Π½Π°ΠΏΡ€Π°Π²Π»Π΅Π½Ρ‹ ΠΏΠΎ ΠΎΠ΄Π½ΠΎΠΉ прямой, Ρ‚ΠΎ ΠΌΠΎΠ΄ΡƒΠ»ΡŒ Ρ€Π΅Π·ΡƒΠ»ΡŒΡ‚ΠΈΡ€ΡƒΡŽΡ‰Π΅ΠΉ силы Ρ€Π°Π²Π΅Π½ суммС ΠΌΠΎΠ΄ΡƒΠ»Π΅ΠΉ сил, Ссли силы Π½Π°ΠΏΡ€Π°Π²Π»Π΅Π½Ρ‹ Π² ΠΎΠ΄Π½Ρƒ сторону ΠΈ Ρ€Π°Π·Π½ΠΎΡΡ‚ΠΈ ΠΌΠΎΠ΄ΡƒΠ»Π΅ΠΉ сил, Ссли силы Π½Π°ΠΏΡ€Π°Π²Π»Π΅Π½Ρ‹ Π² Ρ€Π°Π·Π½Ρ‹Π΅ стороны. Π’ΠΎ Π²Ρ‚ΠΎΡ€ΠΎΠΌ случаС Ρ€Π΅Π·ΡƒΠ»ΡŒΡ‚ΠΈΡ€ΡƒΡŽΡ‰Π°Ρ Π½Π°ΠΏΡ€Π°Π²Π»Π΅Π½Π° Π² ΡΡ‚ΠΎΡ€ΠΎΠ½Ρƒ большСй силы.

2. 2. Если силы Π½Π°ΠΏΡ€Π°Π²Π»Π΅Π½Ρ‹ ΠΏΠΎΠ΄ ΡƒΠ³Π»ΠΎΠΌ, Ρ‚ΠΎ ΠΌΠΎΠ΄ΡƒΠ»ΡŒ Ρ€Π΅Π·ΡƒΠ»ΡŒΡ‚ΠΈΡ€ΡƒΡŽΡ‰Π΅ΠΉ силы числСнно Ρ€Π°Π²Π΅Π½ Π΄Π»ΠΈΠ½Π΅ Π΄ΠΈΠ°Π³ΠΎΠ½Π°Π»ΠΈ ΠΏΠ°Ρ€Π°Π»Π»Π΅Π»ΠΎΠ³Ρ€Π°ΠΌΠΌΠ°, построСнного Π½Π° ΡΡ‚ΠΈΡ… силах ΠΊΠ°ΠΊ Π½Π° ΡΠΎΡΡ‚Π°Π²Π»ΡΡŽΡ‰ΠΈΡ…. Π Π΅Π·ΡƒΠ»ΡŒΡ‚ΠΈΡ€ΡƒΡŽΡ‰Π°Ρ Π²Ρ‹Ρ…ΠΎΠ΄ΠΈΡ‚ ΠΈΠ· Ρ‚ΠΎΡ‡ΠΊΠΈ прилоТСния слагаСмых сил.

Π‘ΠΊΠ»Π°Π΄Ρ‹Π²Π°Ρ‚ΡŒ силы ΠΏΠΎ ΠΏΡ€Π°Π²ΠΈΠ»Ρƒ Ρ‚Ρ€Π΅ΡƒΠ³ΠΎΠ»ΡŒΠ½ΠΈΠΊΠ° нСльзя, Ρ‚ΠΊ Ρ‚СряСтся Ρ‚ΠΎΡ‡ΠΊΠ° прилоТСния сил.

2. 2. ЭлСктричСский Ρ‚ΠΎΠΊ Π² Ρ€Π°ΡΡ‚Π²ΠΎΡ€Π°Ρ… ΠΈ Ρ€Π°ΡΠΏΠ»Π°Π²ΠΎΠ² элСктролитов. Π—Π°ΠΊΠΎΠ½ элСктролиза. ΠŸΡ€ΠΈΠΌΠ΅Π½Π΅Π½ΠΈΠ΅ элСктролиза Π² Ρ‚Π΅Ρ…Π½ΠΈΠΊΠ΅.

ВСщСства, ΠΊΠΎΡ‚ΠΎΡ€Ρ‹Π΅ проводят элСктричСский Ρ‚ΠΎΠΊ Π½Π°Π·Ρ‹Π²Π°ΡŽΡ‚ΡΡ элСктролитами. ИзмСнСниС химичСского состава раствора ΠΈΠ»ΠΈ расплава ΠΏΡ€ΠΈ ΠΏΡ€ΠΎΡ…ΠΎΠΆΠ΄Π΅Π½ΠΈΠΈ Ρ‡Π΅Ρ€Π΅Π· Π½Π΅Π³ΠΎ элСктричСского Ρ‚ΠΎΠΊΠ°. ΠžΠ±ΡƒΡΠ»ΠΎΠ²Π»Π΅Π½Π½ΠΎΠ΅ ΠΏΠΎΡ‚Π΅Ρ€Π΅ΠΉ ΠΈΠ»ΠΈ присоСдинСнии элСктронов ΠΈΠΎΠ½Π°ΠΌΠΈ, Π½Π°Π·Ρ‹Π²Π°ΡŽΡ‚ элСктролизом.

Майкл Π€Π°Ρ€Π°Π΄Π΅ΠΉ установил, Ρ‡Ρ‚ΠΎ ΠΏΡ€ΠΈ ΠΏΡ€ΠΎΡ…ΠΎΠΆΠ΄Π΅Π½ΠΈΠΈ ΡΠ». Π’ΠΎΠΊΠ° Ρ‡Π΅Ρ€Π΅Π· элСктролит масса вСщСства m, Π²Ρ‹Π΄Π΅Π»ΠΈΠ²ΡˆΠ΅Π³ΠΎΡΡ Π½Π° ΡΠ»Π΅ΠΊΡ‚Ρ€ΠΎΠ΄Π΅, ΠΏΡ€ΠΎΠΏΠΎΡ€Ρ†ΠΈΠΎΠ½Π°Π»ΡŒΠ½Π° заряду q, ΠΏΡ€ΠΎΡˆΠ΅Π΄ΡˆΠ΅Π³ΠΎ Ρ‡Π΅Ρ€Π΅Π· элСктролит:

m=k*q ΠΈΠ»ΠΈ m=k*I*t.

Π—Π°Π²ΠΈΡΠΈΠΌΠΎΡΡ‚ΡŒ, ΠΏΠΎΠ»ΡƒΡ‡Π΅Π½Π½ΡƒΡŽ Π€Π°Ρ€Π°Π΄Π΅Π΅ΠΌ, Π½Π°Π·Ρ‹Π²Π°ΡŽΡ‚ Π·Π°ΠΊΠΎΠ½ΠΎΠΌ элСктролиза. ΠšΠΎΡΡ„Ρ„ΠΈΡ†ΠΈΠ΅Π½Ρ‚ ΠΏΡ€ΠΎΠΏΠΎΡ€Ρ†ΠΈΠΎΠ½Π°Π»ΡŒΠ½ΠΎΡΡ‚ΠΈ k Π½Π°Π·Ρ‹Π²Π°Π΅Ρ‚ся элСктрохимичСским эквивалСнтом.

k=1/e*Na * M/n ==> m=1/e*Na * M/n *I *t.

ΠšΠΎΡΡ„Ρ„ΠΈΡ†ΠΈΠ΅Π½Ρ‚ k Ρ‡ΠΈΡΠ»Π΅Π½Π½ΠΎ Ρ€Π°Π²Π΅Π½ массС Π²Ρ‹Π΄Π΅Π»ΠΈΠ²ΡˆΠ΅Π³ΠΎΡΡ Π½Π° ΡΠ»Π΅ΠΊΡ‚Ρ€ΠΎΠ΄Π°Ρ… вСщСства ΠΏΡ€ΠΈ пСрСносС ΠΈΠΎΠ½Π°ΠΌΠΈ заряда Π² 1 Кл:

k=m/q; [k]=кг/Кл.

ΠŸΡ€ΠΎΠΈΠ·Π²Π΅Π΄Π΅Π½ΠΈΠ΅ заряда элСктрона Π½Π° Ρ‡ΠΈΡΠ»ΠΎ Авогадро называСтся числом ЀарадСя: 96 500 Кл/моль.

Число ЀарадСя это элСктричСский заряд, пСрСносимый вСщСством Π² ΠΊΠΎΠ»ΠΈΡ‡Π΅ΡΡ‚Π²Π΅ 1 моль ΠΏΡ€ΠΈ элСктролизС.

Π’ ΡΠ»Π΅ΠΊΡ‚ричСском ΠΏΠΎΠ»Π΅ ΠΈΠΎΠ½Ρ‹ элСктролита приходят Π² Π΄Π²ΠΈΠΆΠ΅Π½ΠΈΠ΅: ΠΏΠΎΠ»ΠΎΠΆΠΈΡ‚Π΅Π»ΡŒΠ½Ρ‹Π΅ ΠΈΠΎΠ½Ρ‹ двиТутся ΠΊ ΠΊΠ°Ρ‚ΠΎΠ΄Ρƒ, Π° ΠΎΡ‚Ρ€ΠΈΡ†Π°Ρ‚Π΅Π»ΡŒΠ½Ρ‹Π΅ ΠΊ Π°Π½ΠΎΠ΄Ρƒ. Π’Π°ΠΊ Π²ΠΎΠ·Π½ΠΈΠΊΠ°Π΅Ρ‚ элСктричСский Ρ‚ΠΎΠΊ Π² ΡΠ»Π΅ΠΊΡ‚Ρ€ΠΎΠ»ΠΈΡ‚Π΅. ΠŸΡ€ΠΈ встрСчи ΠΏΠΎΠ»ΠΎΠΆΠΈΡ‚Π΅Π»ΡŒΠ½ΠΎΠ³ΠΎ ΠΈ ΠΎΡ‚Ρ€ΠΈΡ†Π°Ρ‚Π΅Π»ΡŒΠ½ΠΎΠ³ΠΎ ΠΈΠΎΠ½ΠΎΠ², происходит ΠΈΡ… ΡΠΎΠ΅Π΄ΠΈΠ½Π΅Π½ΠΈΠ΅ — рСкомбинация.

Π‘ ΠΏΠΎΠΌΠΎΡ‰ΡŒΡŽ элСктролиза ΠΈΠ· ΡΠΎΠ»Π΅ΠΉ ΠΈ ΠΎΠΊΡΠΈΠ΄ΠΎΠ² ΠΏΠΎΠ»ΡƒΡ‡Π°ΡŽΡ‚ ΠΌΠ½ΠΎΠ³ΠΈΠ΅ ΠΌΠ΅Ρ‚Π°Π»Π»Ρ‹. ЭлСктролитичСский способ Π΄Π°Π΅Ρ‚ Π²ΠΎΠ·ΠΌΠΎΠΆΠ½ΠΎΡΡ‚ΡŒ ΠΏΠΎΠ»ΡƒΡ‡Π°Ρ‚ΡŒ вСщСства с ΠΌΠ°Π»Ρ‹ΠΌ количСством примСсСй. ΠŸΡƒΡ‚Π΅ΠΌ элСктролиза ΠΌΠΎΠΆΠ½ΠΎ Π½Π°Π½ΠΎΡΠΈΡ‚ΡŒ Ρ‚ΠΎΠ½ΠΊΠΈΠ΅ слои ΠΌΠ΅Ρ‚Π°Π»Π»ΠΎΠ², эти слои ΠΌΠΎΠ³ΡƒΡ‚ ΡΠ»ΡƒΠΆΠΈΡ‚ΡŒ Π·Π°Ρ‰ΠΈΡ‚ΠΎΠΉ издСлия ΠΎΡ‚ ΠΎΠΊΠΈΡΠ»Π΅Π½ΠΈΡ. Π’Π°ΠΊΠΎΠΉ способ называСтся — Π³Π°Π»ΡŒΠ²Π°Π½ΠΎΡΡ‚Π΅Π³ΠΈΠ΅ΠΉ.

ΠŸΡ€ΠΈ Π΄Π»ΠΈΡ‚Π΅Π»ΡŒΠ½ΠΎΠΌ пропускании Ρ‚ΠΎΠΊΠ°, получаСтся толстый слой ΠΌΠ΅Ρ‚Π°Π»Π»Π°, ΠΊΠΎΡ‚ΠΎΡ€Ρ‹ΠΉ ΠΌΠΎΠΆΠ΅Ρ‚ Π±Ρ‹Ρ‚ΡŒ ΠΎΡ‚Π΄Π΅Π»Π΅Π½ с ΡΠΎΡ…Ρ€Π°Π½Π΅Π½ΠΈΠ΅ΠΌ Ρ„ΠΎΡ€ΠΌΡ‹ — Π³Π°Π»ΡŒΠ²Π°Π½ΠΎΠΏΠ»Π°ΡΡ‚ΠΈΠΊΠ°. Π―Π²Π»Π΅Π½ΠΈΠ΅ элСктролиза Π»Π΅ΠΆΠΈΡ‚ Π² ΠΎΡΠ½ΠΎΠ²Π΅ ΠΏΡ€ΠΈΠ½Ρ†ΠΈΠΏΠ° дСйствия кислотных ΠΈ Ρ‰Π΅Π»ΠΎΡ‡Π½Ρ‹Ρ… аккумуляторов, Π³Π΄Π΅ ΠΈΡΠΏΠΎΠ»ΡŒΠ·ΡƒΡŽΡ‚ ΠΎΠ±Ρ€Π°Ρ‚ΠΈΠΌΠΎΡΡ‚ΡŒ процСсса элСктролиза.

Π‘ΠΈΠ»Π΅Ρ‚ 4.

1. 1. Π—Π°ΠΊΠΎΠ½ всСмирного тяготСния. Π‘ΠΈΠ»Π° тяТСсти. Π‘Π²ΠΎΠ±ΠΎΠ΄Π½ΠΎΠ΅ ΠΏΠ°Π΄Π΅Π½ΠΈΠ΅ Ρ‚Π΅Π». ВСс Ρ‚Π΅Π»Π°. ΠΠ΅Π²Π΅ΡΠΎΠΌΠΎΡΡ‚ΡŒ.

2. 2. Π›ΠΈΠ½Π·Ρ‹. ΠŸΠΎΡΡ‚Ρ€ΠΎΠ΅Π½ΠΈΠ΅ изобраТСния Π² Ρ‚ΠΎΠ½ΠΊΠΈΡ… Π»ΠΈΠ½Π·Π°Ρ…. ΠžΠΏΡ‚ΠΈΡ‡Π΅ΡΠΊΠ°Ρ сила Π»ΠΈΠ½Π·Ρ‹. Π€ΠΎΡ€ΠΌΡƒΠ»Π° Ρ‚ΠΎΠ½ΠΊΠΎΠΉ Π»ΠΈΠ½Π·Ρ‹.

3. 3. Π—Π°Π΄Π°Ρ‡Π° Π½Π° Π΄Π²ΠΈΠΆΠ΅Π½ΠΈΠ΅ заряТСнной частицы Π² ΠΌΠ°Π³Π½ΠΈΡ‚Π½ΠΎΠΌ ΠΏΠΎΠ»Π΅.

1. 1. Π—Π°ΠΊΠΎΠ½ всСмирного тяготСния:

ВсС Ρ‚Π΅Π»Π° ΠΏΡ€ΠΈΡ‚ΡΠ³ΠΈΠ²Π°ΡŽΡ‚ΡΡ Π΄Ρ€ΡƒΠ³ ΠΊ Π΄Ρ€ΡƒΠ³Ρƒ с ΡΠΈΠ»ΠΎΠΉ, прямо ΠΏΡ€ΠΎΠΏΠΎΡ€Ρ†ΠΈΠΎΠ½Π°Π»ΡŒΠ½ΠΎΠΉ ΠΏΡ€ΠΎΠΈΠ·Π²Π΅Π΄Π΅Π½ΠΈΡŽ ΠΈΡ… ΠΌΠ°ΡΡ ΠΈ ΠΎΠ±Ρ€Π°Ρ‚Π½ΠΎ ΠΏΡ€ΠΎΠΏ. ΠšΠ²Π°Π΄Ρ€Π°Ρ‚Ρƒ расстояния ΠΌΠ΅ΠΆΠ΄Ρƒ Π½ΠΈΠΌΠΈ. Π­Ρ‚Ρƒ силу Π½Π°Π·Ρ‹Π²Π°ΡŽΡ‚ силой тяготСния.

F=G*m1*m2/r2, Π³Π΄Π΅ GкоэффициСнт ΠΏΡ€ΠΎΠΏΠΎΡ€Ρ†ΠΈΠΎΠ½Π°Π»ΡŒΠ½ΠΎΡΡ‚ΠΈΠ³Ρ€Π°Π²ΠΈΡ‚Π°Ρ†ΠΈΠΎΠ½Π½Π°Ρ постоянная. [G]=6.67 * 10^-11 Н*ΠΌ2/ΠΊΠ³2.

Π“Ρ€Π°Π½ΠΈΡ†Ρ‹ примСнимости:

1. 1. Ρ‚ΠΎΠ»ΡŒΠΊΠΎ для ΠΌ.Ρ‚.

2. 2. Ρ‚Π΅Π», ΠΈΠΌΠ΅ΡŽΡ‰ΠΈΡ… Ρ„ΠΎΡ€ΠΌΡƒ ΡˆΠ°Ρ€Π°.

3. 3. ΡˆΠ°Ρ€Π° большого радиуса, Π²Π·Π°ΠΈΠΌΠΎΠ΄Π΅ΠΉΡΡ‚Π²ΡƒΡŽΡ‰Π΅Π³ΠΎ с Ρ‚Π΅Π»Π°ΠΌΠΈ, Ρ€Π°Π·ΠΌΠ΅Ρ€Ρ‹ ΠΊΠΎΡ‚ΠΎΡ€Ρ‹Ρ… ΠΌΠ½ΠΎΠ³ΠΎ мСньшС Ρ€Π°Π·ΠΌΠ΅Ρ€ΠΎΠ² ΡˆΠ°Ρ€Π°.

Π—Π°ΠΊΠΎΠ½ Π½Π΅ΠΏΡ€ΠΈΠΌΠ΅Π½ΠΈΠΌ, Π½Π°ΠΏΡ€ΠΈΠΌΠ΅Ρ€, для взаимодСйствия бСсконСчного стСрТня ΠΈ ΡˆΠ°Ρ€Π°.

Π‘ΠΈΠ»Π° тяТСсти — это сила с ΠΊΠΎΡ‚ΠΎΡ€ΠΎΠΉ ЗСмля притягиваСт ΠΊ ΡΠ΅Π±Π΅ Ρ‚Π΅Π»ΠΎ. ΠŸΡ€ΠΎΠΏΠΎΡ€Ρ†ΠΈΠΎΠ½Π°Π»ΡŒΠ½Π° массС Ρ‚Π΅Π»Π° ΠΈ ΡΠΎΠΎΠ±Ρ‰Π°Π΅Ρ‚ Π΅ΠΌΡƒ ускорСниС свободного падСния.

g=G*M/r2, Ρ‚Π΅ g Π½Π΅ зависит ΠΎΡ‚ ΠΌΠ°ΡΡΡ‹, Π½ΠΎ Π·Π°Π²ΠΈΡΠΈΡ‚ ΠΎΡ‚ Π²Ρ‹ΡΠΎΡ‚Ρ‹ Ρ‚Π΅Π»Π° Π½Π°Π΄ Π—Π΅ΠΌΠ»Π΅ΠΉ, ΠΎΡ‚ ΡˆΠΈΡ€ΠΎΡ‚Ρ‹ мСста (ЗСмля Π½Π΅ ΠΈΠ½Π΅Ρ€Ρ†ΠΈΠ°Π»ΡŒΠ½Π°Ρ систСма отсчСта, ΠΎΡ‚ ΠΏΠΎΡ€ΠΎΠ΄Ρ‹ Π·Π΅ΠΌΠ½ΠΎΠΉ ΠΊΠΎΡ€Ρ‹, ΠΎΡ‚ Ρ„ΠΎΡ€ΠΌΡ‹ Π—Π΅ΠΌΠ»ΠΈ.

Π‘ΠΈΠ»Π° тяготСния ΠΈ ΡΠΈΠ»Π° тяТСсти носят Π³Ρ€Π°Π²ΠΈΡ‚Π°Ρ†ΠΈΠΎΠ½Π½Ρ‹ΠΉ Ρ…Π°Ρ€Π°ΠΊΡ‚Π΅Ρ€.

Π‘Π²ΠΎΠ±ΠΎΠ΄Π½ΠΎΠ΅ ΠΏΠ°Π΄Π΅Π½ΠΈΠ΅ Ρ‚Π΅Π»Π° являСтся частным случаСм равноускорСнного двиТСния, ΠΏΡ€ΠΈ условии, Ρ‡Ρ‚ΠΎ ускорСниС Π°.

* * Π‘Π²ΠΎΠ±ΠΎΠ΄Π½Ρ‹ΠΌ ΠΏΠ°Π΄Π΅Π½ΠΈΠΌ называСтся Ρ‚Π°ΠΊΠΎΠ΅ Π΄Π²ΠΈΠΆΠ΅Π½ΠΈΠ΅ Ρ‚Π΅Π»Π°, ΠΏΡ€ΠΈ ΠΊΠΎΡ‚ΠΎΡ€ΠΎΠΌ ΠΌ.Ρ‚. (Ρ‚Π΅Π»ΠΎ) двиТСтся ΠΏΠΎΠ΄ Π΄Π΅ΠΉΡΡ‚Π²ΡƒΡŽΡ‰Π΅ΠΉ Ρ‚ΠΎΠ»ΡŒΠΊΠΎ силы тяТСсти, ΠΏΡ€ΠΈ этом сопротивлСниС Π²ΠΎΠ·Π΄ΡƒΡ…Π° Π½Π΅ ΡƒΡ‡ΠΈΡ‚ываСтся.

ΠŸΡ€ΠΈ Π΄Π²ΠΈΠΆΠ΅Π½ΠΈΠΈ Ρ‚Π΅Π»Π° Π²Π²Π΅Ρ€Ρ… ΠΏΡ€ΠΈΠΌΠ΅Π½ΠΈΠΌΡ‹ всС Ρ„ΠΎΡ€ΠΌΡƒΠ»Ρ‹ для Ρ€Π°Π²Π½ΠΎΠ·Π°ΠΌΠ΅Π΄Π»Π΅Π½Π½ΠΎΠ³ΠΎ двиТСния; всСгда Π΅ΡΡ‚ΡŒ Π½Π°Ρ‡Π°Π»ΡŒΠ½Π°Ρ ΡΠΊΠΎΡ€ΠΎΡΡ‚ΡŒ, Π° ΠΊΠΎΠ½Π΅Ρ‡Π½Π°Ρ ΠΏΡ€ΠΈ Ρ‚Π°ΠΊΠΎΠΌ Π΄Π²ΠΈΠΆΠ΅Π½ΠΈΠΈ обращаСтся Π² О.

ВСс Ρ‚Π΅Π»Π° — это сила. Π‘ ΠΊΠΎΡ‚ΠΎΡ€ΠΎΠΉ Ρ‚Π΅Π»ΠΎ дСйствуСт Π½Π° ΠΎΠΏΠΎΡ€Ρƒ ΠΈΠ»ΠΈ подвСс, вслСдствиС притяТСния Π΅Π³ΠΎ ΠΊ Π—Π΅ΠΌΠ»Π΅.

На ΠΏΠΎΠΊΠΎΡΡ‰Π΅Π΅ΡΡ Ρ‚Π΅Π»ΠΎ дСйствуСт сила тяТСсти ΠΈ ΡΠΈΠ»Π° Ρ€Π΅Π°ΠΊΡ†ΠΈΠΈ ΠΎΠΏΠΎΡ€Ρ‹, эта сила упругости ΠΈ Π΅ΡΡ‚ΡŒ вСс Ρ‚Π΅Π»Π° (ΠΏΠΎ Ρ‚Ρ€Π΅Ρ‚ΡŒΠ΅ΠΌΡƒ Π·-Π½ ΠΡŒΡŽΡ‚ΠΎΠ½Π°).

Когда Ρ‚Π΅Π»ΠΎ ΡΠΎΠ²Π΅Ρ€ΡˆΠ°Π΅Ρ‚ свободноС ΠΏΠ°Π΄Π΅Π½ΠΈΠ΅ (a=g), Ρ‚ΠΎ Π²Π·Π°ΠΈΠΌΠΎΠ΄Π΅ΠΉΡΡ‚Π²ΠΈΠ΅ ΠΌΠ΅ΠΆΠ΄Ρƒ Ρ‚Π΅Π»ΠΎΠΌ ΠΈ ΠΎΠΏΠΎΡ€ΠΎΠΉ отсутствуСт ΠΈ Π²Π΅Ρ Ρ‚Π΅Π»Π° Ρ€Π°Π²Π΅Π½ 0. Π­Ρ‚ΠΎ случай ΠΏΠΎΠ»Π½ΠΎΠΉ нСвСсомости.

ΠœΠΎΠΆΠ΅Ρ‚ Π½Π°Π±Π»ΡŽΠ΄Π°Ρ‚ΡŒΡΡ Π² ΡΠ»Π΅Π΄ΡƒΡŽΡ‰ΠΈΡ… случаях:

1. ΠΏΡ€ΠΈ Π΄Π²ΠΈΠΆΠ΅Π½ΠΈΠΈ ΠΊΠΎΠ³Π΄Π° ΡΠΎΠ²ΠΏΠ°Π΄Π°ΡŽΡ‚ направлСния Π½Π°Ρ‡Π°Π»ΡŒΠ½ΠΎΠΉ скорости ΠΈ ΡƒΡΠΊΠΎΡ€Π΅Π½ΠΈΡ.

2. 2. ΠΏΡ€ΠΈ Π΄Π²ΠΈΠΆΠ΅Π½ΠΈΠΈ, ΠΊΠΎΠ³Π΄Π° Π½Π°Ρ‡Π°Π»ΡŒΠ½Π°Ρ ΡΠΊΠΎΡ€ΠΎΡΡ‚ΡŒ ΠΈ ΡƒΡΠΊΠΎΡ€Π΅Π½ΠΈΠ΅ ΠΏΡ€ΠΎΡ‚ΠΈΠ²ΠΎΠΏΠΎΠ»ΠΎΠΆΠ½Ρ‹.

3. Π΄Π²ΠΈΠΆΠ΅Π½ΠΈΠ΅ спутника ΠΏΠΎ ΠΎΡ€Π±ΠΈΡ‚Π΅.

4. 4. ΠΊΠΎΠ³Π΄Π° Ρ‚Π΅Π»ΠΎ находится ΠΌΠ΅ΠΆΠ΄Ρƒ Π—Π΅ΠΌΠ»Π΅ΠΉ ΠΈ Π›ΡƒΠ½ΠΎΠΉ.

5. Π»ΠΆΠ΅Π½Π΅Π²Π΅ΡΠΎΠΌΠΎΡΡ‚ΡŒ Π½Π°Π±Π»ΡŽΠ΄Π°Π΅Ρ‚ΡΡ Π² Π²ΠΎΠ΄Π΅.

2. 2. Π›ΠΈΠ½Π·ΠΎΠΉ называСтся ΠΏΡ€ΠΎΠ·Ρ€Π°Ρ‡Π½ΠΎΠ΅ Ρ‚Π΅Π»ΠΎ, ΠΎΠ³Ρ€Π°Π½ΠΈΡ‡Π΅Π½Π½ΠΎΠ΅ двумя сфСричСскими повСрхностями.

Π’ΠΎΠ½ΠΊΠΎΠΉ, Ссли Π΅Π΅ Ρ‚ΠΎΠ»Ρ‰ΠΈΠ½Π° ΠΌΠ°Π»Π° ΠΏΠΎ ΡΡ€Π°Π²Π½Π΅Π½ΠΈΡŽ с Ρ€Π°Π΄ΠΈΡƒΡΠ°ΠΌΠΈ ΠΊΡ€ΠΈΠ²ΠΈΠ·Π½Ρ‹ Π΅Π΅ ΠΏΠΎΠ²Π΅Ρ€Ρ…ностСй, Π² ΠΏΡ€ΠΎΡ‚ΠΈΠ²Π½ΠΎΠΌ случаС — толстой.

ΠžΠΏΡ‚ΠΈΡ‡Π΅ΡΠΊΠ°Ρ сила — это Π²Π΅Π»ΠΈΡ‡ΠΈΠ½Π°, обратная фокусному Ρ€Π°ΡΡΡ‚ΠΎΡΠ½ΠΈΡŽ.

D=1/F.

Π˜Π·ΠΌΠ΅Ρ€ΡΠ΅Ρ‚ΡΡ Π² Π΄ΠΈΠΎΠΏΡ‚риях. 1 Π΄ΠΈΠΎΠΏΡ‚Ρ€ΠΈΠΉ — это оптичСская сила Ρ‚Π°ΠΊΠΎΠΉ Π»ΠΈΠ½Π·Ρ‹, фокусноС расстояниС ΠΊΠΎΡ‚ΠΎΡ€ΠΎΠΉ 1 ΠΌ.

Π‘ΠΈΠ»Π΅Ρ‚ 5.

1. 1. Π’Ρ€Π΅Ρ‚ΠΈΠΉ Π·-Π½ ΠΡŒΡŽΡ‚ΠΎΠ½Π°. Π˜ΠΌΠΏΡƒΠ»ΡŒΡ Ρ‚Π΅Π»Π°, Π·-Π½ сохранСния ΠΈΠΌΠΏΡƒΠ»ΡŒΡΠ°. Π Π΅Π°ΠΊΡ‚ΠΈΠ²Π½ΠΎΠ΅ Π΄Π²ΠΈΠΆΠ΅Π½ΠΈΠ΅ Π² ΠΏΡ€ΠΈΡ€ΠΎΠ΄Π΅ ΠΈ Ρ‚Π΅Ρ…Π½ΠΈΠΊΠ΅.

2. 2. Бамоиндукция. Π˜Π½Π΄ΡƒΠΊΡ‚ΠΈΠ²Π½ΠΎΡΡ‚ΡŒ. ЭнСргия ΠΌΠ°Π³Π½ΠΈΡ‚Π½ΠΎΠ³ΠΎ поля ΠΊΠ°Ρ‚ΡƒΡˆΠΊΠΈ с Ρ‚ΠΎΠΊΠΎΠΌ.

3. 3. (ΠŸΠ΅Ρ€Π²Ρ‹ΠΉ Π·-Π½ Ρ‚Π΅Ρ€ΠΌΠΎΠ΄ΠΈΠ½Π°ΠΌΠΈΠΊΠΈ).

Π’Ρ€Π΅Ρ‚ΠΈΠΉ Π·Π°ΠΊΠΎΠ½ ΠΡŒΡŽΡ‚ΠΎΠ½Π°. Π’ ΠΈΠ½Π΅Ρ€Ρ†ΠΈΠ°Π»ΡŒΠ½ΠΎΠΉ систСмС отсчСта силы взаимодСйствия Π΄Π²ΡƒΡ… Ρ‚Π΅Π» Ρ€Π°Π²Π½Ρ‹ ΠΏΠΎ ΠΌΠΎΠ΄ΡƒΠ»ΡŽ ΠΈ Π½Π°ΠΏΡ€Π°Π²Π»Π΅Π½Ρ‹ Π² ΠΏΡ€ΠΎΡ‚ΠΈΠ²ΠΎΠΏΠΎΠ»ΠΎΠΆΠ½Ρ‹Π΅ стороны. Π’Ρ€Π΅Ρ‚ΠΈΠΉ Π·Π°ΠΊΠΎΠ½ ΠΎΡ‚Ρ€Π°ΠΆΠ°Π΅Ρ‚ Ρ„Π°ΠΊΡ‚ равноправия Π²Π·Π°ΠΈΠΌΠΎΠ΄Π΅ΠΉΡΡ‚Π²ΡƒΡŽΡ‰ΠΈΡ… Ρ‚Π΅Π».

Бвойство Ρ‚Π΅Π»Π° ΡΠΎΡ…Ρ€Π°Π½ΡΡ‚ΡŒ свою ΡΠΊΠΎΡ€ΠΎΡΡ‚ΡŒ ΠΏΡ€ΠΈ отсутствии взаимодСйствий с Π΄Ρ€ΡƒΠ³ΠΈΠΌΠΈ Ρ‚Π΅Π»Π°ΠΌΠΈ называСтся ΠΈΠ½Π΅Ρ€Ρ‚Π½ΠΎΡΡ‚ΡŒΡŽ. ЀизичСская Π²Π΅Π»ΠΈΡ‡ΠΈΠ½Π°, ΡΠ²Π»ΡΡŽΡ‰Π°ΡΡΡ ΠΌΠ΅Ρ€ΠΎΠΉ инСртности Ρ‚Π΅Π»Π° Π² ΠΏΠΎΡΡ‚ΡƒΠΏΠ°Ρ‚Π΅Π»ΡŒΠ½ΠΎΠΌ Π΄Π²ΠΈΠΆΠ΅Π½ΠΈΠΈ, называСтся ΠΈΠ½Π΅Ρ€Ρ‚Π½ΠΎΠΉ массой. Π’ ΠΌΠ΅Ρ…Π°Π½ΠΈΠΊΠ΅ ΠΡŒΡŽΡ‚ΠΎΠ½Π° считаСтся, Ρ‡Ρ‚ΠΎ: Π°) масса Ρ‚Π΅Π»Π° Ρ€Π°Π²Π½Π° суммС масс всСх частиц (ΠΈΠ»ΠΈ ΠΌΠ°Ρ‚Π΅Ρ€ΠΈΠ°Π»ΡŒΠ½Ρ‹Ρ… Ρ‚ΠΎΡ‡Π΅ΠΊ), ΠΈΠ· ΠΊΠΎΡ‚ΠΎΡ€Ρ‹Ρ… ΠΎΠ½ΠΎ состоит; Π±) для Π΄Π°Π½Π½ΠΎΠΉ совокупности Ρ‚Π΅Π» выполняСтся Π·Π°ΠΊΠΎΠ½ сохранСния массы: ΠΏΡ€ΠΈ Π»ΡŽΠ±Ρ‹Ρ… процСссах, происходящих Π² ΡΠΈΡΡ‚Π΅ΠΌΠ΅ Ρ‚Π΅Π», Π΅Π΅ ΠΌΠ°ΡΡΠ° остаСтся Π½Π΅ΠΈΠ·ΠΌΠ΅Π½Π½ΠΎΠΉ.

Π‘ΠΈΠ»ΠΎΠΉ называСтся вСкторная физичСская вСкторная Π²Π΅Π»ΠΈΡ‡ΠΈΠ½Π°, ΡΠ²Π»ΡΡŽΡ‰Π°ΡΡΡ ΠΌΠ΅Ρ€ΠΎΠΉ мСханичСского воздСйствия Π½Π° Ρ‚Π΅Π»ΠΎ со ΡΡ‚ΠΎΡ€ΠΎΠ½Ρ‹ Π΄Ρ€ΡƒΠ³ΠΈΡ… Ρ‚Π΅Π» ΠΈΠ»ΠΈ ΠΏΠΎΠ»Π΅ΠΉ.

Π—Π°ΠΊΠΎΠ½Ρ‹ сохранСния Π² ΠΌΠ΅Ρ…Π°Π½ΠΈΠΊΠ΅. Π˜ΠΌΠΏΡƒΠ»ΡŒΡΠΎΠΌ Π²Π΅ΠΊΡ‚ΠΎΡ€Π° Ρ€1 ΠΌΠ°Ρ‚Π΅Ρ€ΠΈΠ°Π»ΡŒΠ½ΠΎΠΉ Ρ‚ΠΎΡ‡ΠΊΠΈ называСтся вСкторная Π²Π΅Π»ΠΈΡ‡ΠΈΠ½Π°, равная ΠΏΡ€ΠΎΠΈΠ·Π²Π΅Π΄Π΅Π½ΠΈΡŽ массы Ρ‚ΠΎΡ‡ΠΊΠΈ Π½Π° ΡΠΊΠΎΡ€ΠΎΡΡ‚ΡŒ Π²Π΅ΠΊΡ‚ΠΎΡ€Π° v Π΅Π΅ Π΄Π²ΠΈΠΆΠ΅Π½ΠΈΡ. Π—Π½Π°Π½ΠΈΠ΅ Π·Π°ΠΊΠΎΠ½Π° сохранСния ΠΈΠΌΠΏΡƒΠ»ΡŒΡΠ° Π΄Π°Π΅Ρ‚ Π²ΠΎΠ·ΠΌΠΎΠΆΠ½ΠΎΡΡ‚ΡŒ Π²Ρ‹ΠΏΠΎΠ»Π½ΠΈΡ‚ΡŒ расчСты Ρ€Π΅Π·ΡƒΠ»ΡŒΡ‚Π°Ρ‚Π° взаимодСйствия Ρ‚Π΅Π», ΠΊΠΎΠ³Π΄Π° значСния Π΄Π΅ΠΉΡΡ‚Π²ΡƒΡŽΡ‰ΠΈΡ… сил нСизвСстны.

Π˜ΠΌΠΏΡƒΠ»ΡŒΡ ΠΌΠΎΠΆΠ΅Ρ‚ ΡΠΎΡ…Ρ€Π°Π½ΡΡ‚ΡŒΡΡ Ρ‚ΠΎΠ»ΡŒΠΊΠΎ Π² ΠΈΠ·ΠΎΠ»ΠΈΡ€ΠΎΠ²Π°Π½Π½ΠΎΠΉ систСмС. Замкнутая систСма Ρ‚Π΅Π» — это ΡΠΎΠ²ΠΎΠΊΡƒΠΏΠ½ΠΎΡΡ‚ΡŒ Ρ‚Π΅Π», Π²Π·Π°ΠΈΠΌΠΎΠ΄Π΅ΠΉΡΡ‚Π²ΡƒΡŽΡ‰ΠΈΡ… ΠΌΠ΅ΠΆΠ΄Ρƒ собой, Π½ΠΎ Π½Π΅ Π²Π·Π°ΠΈΠΌΠΎΠ΄Π΅ΠΉΡΡ‚Π²ΡƒΡŽΡ‰ΠΈΡ… с Π΄Ρ€ΡƒΠ³ΠΈΠΌΠΈ Ρ‚Π΅Π»Π°ΠΌΠΈ. Π—Π°ΠΊΠΎΠ½ сохранСния ΠΈΠΌΠΏΡƒΠ»ΡŒΡΠ° Π² ΠΌΠ΅Ρ…Π°Π½ΠΈΠΊΠ΅ являСтся слСдствиСм Π²Ρ‚ΠΎΡ€ΠΎΠ³ΠΎ ΠΈ Ρ‚Ρ€Π΅Ρ‚ΡŒΠ΅Π³ΠΎ Π·Π°ΠΊΠΎΠ½ΠΎΠ² ΠΡŒΡŽΡ‚ΠΎΠ½Π°.

ГСомСтричСская сумма ΠΈΠΌΠΏΡƒΠ»ΡŒΡΠΎΠ² Ρ‚Π΅Π», ΡΠΎΡΡ‚Π°Π²Π»ΡΡŽΡ‰ΠΈΡ… Π·Π°ΠΌΠΊΠ½ΡƒΡ‚ΡƒΡŽ систСму, остаСтся постоянной ΠΏΡ€ΠΈ Π»ΡŽΠ±Ρ‹Ρ… двиТСниях ΠΈ Π²Π·Π°ΠΈΠΌΠΎΠ΄Π΅ΠΉΡΡ‚Π²ΡƒΡŽΡ‰ΠΈΡ… Ρ‚Π΅Π» этой систСмы.

2. 2. Бамоиндукция. Π˜Π½Π΄ΡƒΠΊΡ‚ΠΈΠ²Π½ΠΎΡΡ‚ΡŒ. ЭнСргия ΠΌΠ°Π³Π½ΠΈΡ‚Π½ΠΎΠ³ΠΎ поля ΠΊΠ°Ρ‚ΡƒΡˆΠΊΠΈ с Ρ‚ΠΎΠΊΠΎΠΌ.

Если ΠΏΠΎ ΠΊΠ°Ρ‚ΡƒΡˆΠΊΠ΅ ΠΈΠ΄Π΅Ρ‚ ΠΏΠ΅Ρ€Π΅ΠΌΠ΅Π½Π½Ρ‹ΠΉ Ρ‚ΠΎΠΊ, Ρ‚ΠΎ ΠΌΠ°Π³Π½ΠΈΡ‚Π½Ρ‹ΠΉ ΠΏΠΎΡ‚ΠΎΠΊ, ΠΏΡ€ΠΎΠ½ΠΈΠ·Ρ‹Π²Π°ΡŽΡ‰ΠΈΠΉ ΠΊΠ°Ρ‚ΡƒΡˆΠΊΡƒ, измСняСтся. ΠŸΠΎΡΡ‚ΠΎΠΌΡƒ Π² ΠΏΡ€ΠΎΠ²ΠΎΠ΄Π½ΠΈΠΊΠ΅, ΠΏΠΎ ΠΊΠΎΡ‚ΠΎΡ€ΠΎΠΌΡƒ ΠΈΠ΄Π΅Ρ‚ ΠΏΠ΅Ρ€Π΅ΠΌΠ΅Π½Π½Ρ‹ΠΉ элСктричСский Ρ‚ΠΎΠΊ, Π²ΠΎΠ·Π½ΠΈΠΊΠ°Π΅Ρ‚ Π­Π”Π‘ ΠΈΠ½Π΄ΡƒΠΊΡ†ΠΈΠΈ. ΠŸΡ€ΠΈ самоиндукции проводящий ΠΊΠΎΠ½Ρ‚ΡƒΡ€ ΠΈΠΌΠ΅Π΅Ρ‚ Π΄Π²ΠΎΡΠΊΡƒΡŽ Ρ€ΠΎΠ»ΡŒ: ΠΏΠΎ Π½Π΅ΠΌΡƒ ΠΏΡ€ΠΎΡ‚Π΅ΠΊΠ°Π΅Ρ‚ Ρ‚ΠΎΠΊ, Π²Ρ‹Π·Ρ‹Π²Π°ΡŽΡ‰ΠΈΠΉ ΠΈΠ½Π΄ΡƒΠΊΡ†ΠΈΡŽ, ΠΈ Π² Π½Π΅ΠΌ ΠΆΠ΅ появляСтся Π­Π”Π‘ ΠΈΠ½Π΄ΡƒΠΊΡ†ΠΈΠΈ. Π˜Π·ΠΌΠ΅Π½ΡΡŽΡ‰Π΅Π΅ΡΡ ΠΌΠ°Π³Π½ΠΈΡ‚Π½ΠΎΠ΅ ΠΏΠΎΠ»Π΅ ΠΈΠ½Π΄ΡƒΡ†ΠΈΡ€ΡƒΠ΅Ρ‚ Π­Π”Π‘ Π² Ρ‚ΠΎΠΌ ΠΏΡ€ΠΎΠ²ΠΎΠ΄Π½ΠΈΠΊΠ΅, ΠΏΠΎ ΠΊΠΎΡ‚ΠΎΡ€ΠΎΠΌΡƒ Ρ‚Π΅Ρ‡Π΅Ρ‚ Ρ‚ΠΎΠΊ, ΡΠΎΠ·Π΄Π°ΡŽΡ‰ΠΈΠΉ это ΠΏΠΎΠ»Π΅. По ΠΏΡ€Π°Π²ΠΈΠ»Ρƒ Π›Π΅Π½Ρ†Π° Π² ΠΌΠΎΠΌΠ΅Π½Ρ‚ нарастания Ρ‚ΠΎΠΊΠ° Π½Π°ΠΏΡ€ΡΠΆΠ΅Π½Π½ΠΎΡΡ‚ΡŒ Π²ΠΈΡ…Ρ€Π΅Π²ΠΎΠ³ΠΎ элСктричСского поля Π½Π°ΠΏΡ€Π°Π²Π»Π΅Π½Π° ΠΏΡ€ΠΎΡ‚ΠΈΠ² Ρ‚ΠΎΠΊΠ°. Π‘Π»Π΅Π΄ΠΎΠ²Π°Ρ‚Π΅Π»ΡŒΠ½ΠΎ. Π’ ΡΡ‚ΠΎΡ‚ ΠΌΠΎΠΌΠ΅Π½Ρ‚ Π²ΠΈΡ…Ρ€Π΅Π²ΠΎΠ΅ ΠΏΠΎΠ»Π΅ прСпятствуСт Π½Π°Ρ€Π°ΡΡ‚Π°Π½ΠΈΡŽ Ρ‚ΠΎΠΊΠ°. И Π½Π°ΠΎΠ±ΠΎΡ€ΠΎΡ‚, Π² ΠΌΠΎΠΌΠ΅Π½Ρ‚ ΡƒΠΌΠ΅Π½ΡŒΡˆΠ΅Π½ΠΈΡ Ρ‚ΠΎΠΊΠ° Π΅Π³ΠΎ ΠΏΠΎΠ΄Π΄Π΅Ρ€ΠΆΠΈΠ²Π°Π΅Ρ‚ Π²ΠΈΡ…Ρ€Π΅Π²ΠΎΠ΅ ΠΏΠΎΠ»Π΅. Π­Ρ‚ΠΎ ΠΏΡ€ΠΈΠ²ΠΎΠ΄ΠΈΡ‚ ΠΊ Ρ‚ΠΎΠΌΡƒ, Ρ‡Ρ‚ΠΎ ΠΏΡ€ΠΈ Π·Π°ΠΌΡ‹ΠΊΠ°Π½ΠΈΠΈ Ρ†Π΅ΠΏΠΈ, содСрТащСй ΠΏΠΎΡΡ‚ΠΎΡΠ½Π½ΡƒΡŽ Π­Π”Π‘, ΠΎΠΏΡ€Π΅Π΄Π΅Π»Π΅Π½Π½ΠΎΠ΅ Π·Π½Π°Ρ‡Π΅Π½ΠΈΠ΅ силы Ρ‚ΠΎΠΊΠ° устанавливаСтся Π½Π΅ ΡΡ€Π°Π·Ρƒ, Π° ΠΏΠΎΡΡ‚Π΅ΠΏΠ΅Π½Π½ΠΎ, с Ρ‚Π΅Ρ‡Π΅Π½ΠΈΠ΅ΠΌ Π²Ρ€Π΅ΠΌΠ΅Π½ΠΈ. Π‘ Π΄Ρ€ΡƒΠ³ΠΎΠΉ стороны, ΠΏΡ€ΠΈ ΠΎΡ‚ΠΊΠ»ΡŽΡ‡Π΅Π½ΠΈΠΈ источника Ρ‚ΠΎΠΊΠ° Π² Π·Π°ΠΌΠΊΠ½ΡƒΡ‚Ρ‹Ρ… ΠΊΠΎΠ½Ρ‚ΡƒΡ€Π°Ρ… прСкращаСтся Π½Π΅ ΠΌΠ³Π½ΠΎΠ²Π΅Π½Π½ΠΎ. Π’ΠΎΠ·Π½ΠΈΠΊΠ°ΡŽΡ‰Π°Ρ Π­Π”Π‘ самоиндукции ΠΌΠΎΠΆΠ΅Ρ‚ ΠΏΡ€Π΅Π²Ρ‹ΡˆΠ°Ρ‚ΡŒ Π­Π”Π‘ источника, Ρ‚Π°ΠΊ ΠΊΠ°ΠΊ ΠΈΠ·ΠΌΠ΅Π½Π΅Π½ΠΈΠ΅ Ρ‚ΠΎΠΊΠ° ΠΈ Π΅Π³ΠΎ ΠΌΠ°Π³Π½ΠΈΡ‚Π½ΠΎΠ³ΠΎ поля ΠΏΡ€ΠΈ ΠΎΡ‚ΠΊΠ»ΡŽΡ‡Π΅Π½ΠΈΠΈ источника происходит ΠΎΡ‡Π΅Π½ΡŒ быстро.

Π―Π²Π»Π΅Π½ΠΈΠ΅ самоиндукции — это частный случай явлСния элСктромагнитной ΠΈΠ½Π΄ΡƒΠΊΡ†ΠΈΠΈ, поэтому Π­Π”Π‘ самоиндукции ΠΌΠΎΠΆΠ½ΠΎ Π²Ρ‹Ρ€Π°Π·ΠΈΡ‚ΡŒ ΠΏΠΎ Ρ„ΠΎΡ€ΠΌΡƒΠ»Π΅: E= - Π€/t. Но ΠΈΠ·ΠΌΠ΅Π½Π΅Π½ΠΈΠ΅ ΠΌΠ°Π³Π½ΠΈΡ‚Π½ΠΎΠ³ΠΎ ΠΏΠΎΡ‚ΠΎΠΊΠ° Π€ ΠΏΡ€ΠΎΠΏΠΎΡ€Ρ†ΠΈΠΎΠ½Π°Π»ΡŒΠ½ΠΎ измСнСнию силы Ρ‚ΠΎΠΊΠ° Π² ΠΊΠΎΠ½Ρ‚ΡƒΡ€Π΅: Π€~I, ΠΈΠ»ΠΈ Π€=L*I, Π³Π΄Π΅ L-ΠΈΠ½Π΄ΡƒΠΊΡ‚ΠΈΠ²Π½ΠΎΡΡ‚ΡŒ. Π‘Π»Π΅Π΄ΠΎΠ²Π°Ρ‚Π΅Π»ΡŒΠ½ΠΎ, Ρ„ΠΎΡ€ΠΌΡƒΠ»Π° для Π­Π”Π‘ самоиндукции Π±ΡƒΠ΄Π΅Ρ‚ ΠΈΠΌΠ΅Ρ‚ΡŒ Π²ΠΈΠ΄: E= -L *I/t, Π³Π΄Π΅ I/t — Π²Π΅Π»ΠΈΡ‡ΠΈΠ½Π°, Ρ…Π°Ρ€Π°ΠΊΡ‚Π΅Ρ€ΠΈΠ·ΡƒΡŽΡ‰Π°Ρ ΡΠΊΠΎΡ€ΠΎΡΡ‚ΡŒ измСнСния силы Ρ‚ΠΎΠΊΠ° Π² Ρ†Π΅ΠΏΠΈ. Π˜Π½Π°Ρ‡Π΅ говоря, Π­Π”Π‘ самоиндукции ΠΏΡ€ΠΎΠΏΠΎΡ€Ρ†ΠΈΠΎΠ½Π°Π»ΡŒΠ½Π° скорости измСнСния силы Ρ‚ΠΎΠΊΠ° Π² Ρ†Π΅ΠΏΠΈ.

Π˜Π½Π΄ΡƒΠΊΡ‚ΠΈΠ²Π½ΠΎΡΡ‚ΡŒ Ρ€Π°Π²Π½Π° ΠΎΡ‚Π½ΠΎΡˆΠ΅Π½ΠΈΡŽ Π­Π”Π‘ самоиндукции ΠΊ ΡΠΊΠΎΡ€ΠΎΡΡ‚ΠΈ измСнСния Ρ‚ΠΎΠΊΠ° Π² Ρ†Π΅ΠΏΠΈ.

L=E/ I/t. Π’ Π‘И Π·Π° Π΅Π΄ΠΈΠ½ΠΈΡ†Ρƒ индуктивности ΠΏΡ€ΠΈΠ½ΠΈΠΌΠ°ΡŽΡ‚ ΠΈΠ½Π΄ΡƒΠΊΡ‚ΠΈΠ²Π½ΠΎΡΡ‚ΡŒ Ρ‚Π°ΠΊΠΎΠ³ΠΎ ΠΏΡ€ΠΎΠ²ΠΎΠ΄Π½ΠΈΠΊΠ°, Π² ΠΊΠΎΡ‚ΠΎΡ€ΠΎΠΌ ΠΏΡ€ΠΈ ΠΈΠ·ΠΌΠ΅Π½Π΅Π½ΠΈΠΈ силы Ρ‚ΠΎΠΊΠ° со ΡΠΊΠΎΡ€ΠΎΡΡ‚ΡŒΡŽ 1 А/с Π²ΠΎΠ·Π½ΠΈΠΊΠ°Π΅Ρ‚ Π­Π”Π‘ самоиндукции Π² 1 Π’. Π­Ρ‚Π° Π΅Π΄ΠΈΠ½ΠΈΡ†Π° индуктивности называСтся 1 Π³Π΅Π½Ρ€ΠΈ (Π“Π½): 1 Π“Π½= Π’/ А/с= Π’*с/А.

ΠžΠΏΡ‹Ρ‚Π½Ρ‹ΠΌ ΠΏΡƒΡ‚Π΅ΠΌ установлСно, Ρ‡Ρ‚ΠΎ ΠΈΠ½Π΄ΡƒΠΊΡ‚ΠΈΠ²Π½ΠΎΡΡ‚ΡŒ зависит ΠΎΡ‚ Π΄Π»ΠΈΠ½Ρ‹ ΠΈ Ρ„ΠΎΡ€ΠΌΡ‹ ΠΏΡ€ΠΎΠ²ΠΎΠ΄Π½ΠΈΠΊΠ°, Π° Ρ‚Π°ΠΊΠΆΠ΅ ΠΎΡ‚ ΠΌΠ°Π³Π½ΠΈΡ‚Π½Ρ‹Ρ… свойств Π»ΠΊΡ€ΡƒΠΆΠ°ΡŽΡ‰Π΅ΠΉ ΠΏΡ€ΠΎΠ²ΠΎΠ΄Π½ΠΈΠΊ срСды.

По Π·Π°ΠΊΠΎΠ½Ρƒ сохранСния энСргии, энСргия Ρ‚ΠΎΠΊΠ° Ρ€Π°Π²Π½Π° Ρ‚ΠΎΠΉ энСргии, ΠΊΠΎΡ‚ΠΎΡ€ΡƒΡŽ Π΄ΠΎΠ»ΠΆΠ΅Π½ Π·Π°Ρ‚Ρ€Π°Ρ‚ΠΈΡ‚ΡŒ источник Ρ‚ΠΎΠΊΠ° Π½Π° ΡΠΎΠ·Π΄Π°Π½ΠΈΠ΅ Ρ‚ΠΎΠΊΠ°. ΠŸΡ€ΠΈ Π·Π°ΠΌΡ‹ΠΊΠ°Π½ΠΈΠΈ элСктричСской Ρ†Π΅ΠΏΠΈ. Π‘ΠΎΠ΄Π΅Ρ€ΠΆΠ°Ρ‰Π΅ΠΉ ΠΏΠΎΡΡ‚ΠΎΡΠ½Π½ΡƒΡŽ ΡΠ»Π΅ΠΊΡ‚Ρ€ΠΎΠ΄Π²ΠΈΠΆΡƒΡ‰ΡƒΡŽΡΡ силу (Π­Π”Π‘), энСргия источника Ρ‚ΠΎΠΊΠ° ΠΏΠ΅Ρ€Π²ΠΎΠ½Π°Ρ‡Π°Π»ΡŒΠ½ΠΎ расходуСтся Π½Π° ΡΠΎΠ·Π΄Π°Π½ΠΈΠ΅ Ρ‚ΠΎΠΊΠ°, Ρ‚ΠΎΠ΅ ΡΡ‚ΡŒ Π½Π° ΠΏΡ€ΠΈΠ²Π΅Π΄Π΅Π½ΠΈΠ΅ элСктронов Π² ΡƒΠΏΠΎΡ€ΡΠ΄ΠΎΡ‡Π½ΠΎΠ΅ Π΄Π²ΠΈΠΆΠ΅Π½ΠΈΠ΅ ΠΈ ΠΎΠ±Ρ€Π°Π·ΠΎΠ²Π°Π½ΠΈΠ΅ связанного с Ρ‚ΠΎΠΊΠΎΠΌ ΠΌΠ°Π³Π½ΠΈΡ‚Π½ΠΎΠ³ΠΎ поля, Π° Ρ‚Π°ΠΊΠΆΠ΅ частично Π½Π° ΡƒΠ²Π΅Π»ΠΈΡ‡Π΅Π½ΠΈΠ΅ Π²Π½ΡƒΡ‚Ρ€Π΅Π½Π½Π΅ΠΉ энСргии ΠΏΡ€ΠΎΠ²ΠΎΠ΄Π½ΠΈΠΊΠ°. ПослС установлСния постоянного значСния силы Ρ‚ΠΎΠΊΠ° энСргия источника расходуСтся ΠΈΡΠΊΠ»ΡŽΡ‡ΠΈΡ‚Π΅Π»ΡŒΠ½ΠΎ Π½Π° Π²Ρ‹Π΄Π΅Π»Π΅Π½ΠΈΠ΅ Ρ‚Π΅ΠΏΠ»ΠΎΡ‚Ρ‹. ЭнСргия Ρ‚ΠΎΠΊΠ° ΠΏΡ€ΠΈ этом ΡƒΠΆΠ΅ Π½Π΅ ΠΈΠ·ΠΌΠ΅Π½ΡΠ΅Ρ‚ся. Для создания Ρ‚ΠΎΠΊΠ° Π½Π΅ΠΎΠ±Ρ…ΠΎΠ΄ΠΈΠΌΠΎ Π·Π°Ρ‚Ρ€Π°Ρ‡ΠΈΠ²Π°Ρ‚ΡŒ ΡΠ½Π΅Ρ€Π³ΠΈΡŽ, Ρ‚. Π΅. ΡΠΎΠ²Π΅Ρ€ΡˆΠ°Ρ‚ΡŒ Ρ€Π°Π±ΠΎΡ‚Ρƒ, Ρ‚Π°ΠΊ ΠΊΠ°ΠΊ ΠΏΡ€ΠΈ Π·Π°ΠΌΡ‹ΠΊΠ°Π½ΠΈΠΈ элСктричСской Ρ†Π΅ΠΏΠΈ, ΠΊΠΎΠ³Π΄Π° сила Ρ‚ΠΎΠΊΠ° Π½Π°Ρ‡ΠΈΠ½Π°Π΅Ρ‚ Π½Π°Ρ€Π°ΡΡ‚Π°Ρ‚ΡŒ, Π² ΠΏΡ€ΠΎΠ²ΠΎΠ΄Π½ΠΈΠΊΠ΅ появляСтся Π²ΠΈΡ…Ρ€Π΅Π²ΠΎΠ΅ элСктричСскоС ΠΏΠΎΠ»Π΅, Π½Π°ΠΏΡ€Π°Π²Π»Π΅Π½Π½ΠΎΠ΅ ΠΏΡ€ΠΎΡ‚ΠΈΠ² Ρ‚ΠΎΠ³ΠΎ элСктричСского ΠΏΠΎΠ»Π΅, ΠΊΠΎΡ‚ΠΎΡ€ΠΎΠ΅ создаСтся Π² ΠΏΡ€ΠΎΠ²ΠΎΠ΄Π½ΠΈΠΊΠ΅ благодаря источнику Ρ‚ΠΎΠΊΠ°. Π§Ρ‚ΠΎΠ±Ρ‹ сила Ρ‚ΠΎΠΊΠ° Ρ€Π°Π²Π½ΠΎΠΉ ΠΎΠΏΡ€Π΅Π΄Π΅Π»Π΅Π½Π½ΠΎΠΌΡƒ Π·Π½Π°Ρ‡Π΅Π½ΠΈΡŽ, источник Ρ‚ΠΎΠΊΠ° Π΄ΠΎΠ»ΠΆΠ΅Π½ ΡΠΎΠ²Π΅Ρ€ΡˆΠΈΡ‚ΡŒ Ρ€Π°Π±ΠΎΡ‚Ρƒ ΠΏΡ€ΠΎΡ‚ΠΈΠ² сил Π²ΠΈΡ…Ρ€Π΅Π²ΠΎΠ³ΠΎ поля. Π­Ρ‚Π° Ρ€Π°Π±ΠΎΡ‚Π° ΠΈ ΠΈΠ΄Π΅Ρ‚ Π½Π° ΡƒΠ²Π΅Π»ΠΈΡ‡Π΅Π½ΠΈΠ΅ энСргии ΠΌΠ°Π³Π½ΠΈΡ‚Π½ΠΎΠ³ΠΎ поля Ρ‚ΠΎΠΊΠ°. ΠŸΡ€ΠΈ Ρ€Π°Π·ΠΌΡ‹ΠΊΠ°Π½ΠΈΠΈ элСктричСской Ρ†Π΅ΠΏΠΈ Ρ‚ΠΎΠΊ исчСзаСт, ΠΈ Π²ΠΈΡ…Ρ€Π΅Π²ΠΎΠ΅ ΠΏΠΎΠ»Π΅ ΡΠΎΠ²Π΅Ρ€ΡˆΠ°Π΅Ρ‚ ΠΏΠΎΠ»ΠΎΠΆΠΈΡ‚Π΅Π»ΡŒΠ½ΡƒΡŽ Ρ€Π°Π±ΠΎΡ‚Ρƒ. ВыдСляСтся запасСнная Ρ‚ΠΎΠΊΠΎΠΌ энСргия. Π­Ρ‚ΠΎ обнаруТиваСтя ΠΏΠΎ ΠΌΠΎΡ‰ΡŒΠ½ΠΎΠΉ искрС, Π²ΠΎΠ·Π½ΠΈΠΊΠ°ΡŽΡ‰Π΅ΠΉ ΠΏΡ€ΠΈ Ρ€Π°Π·ΠΌΡ‹ΠΊΠ°Π½ΠΈΠΈ Ρ†Π΅ΠΏΠΈ с Π±ΠΎΠ»ΡŒΡˆΠΎΠΉ ΠΈΠ½Π΄ΡƒΠΊΡ‚ΠΈΠ²Π½ΠΎΡΡ‚ΡŒΡŽ. По ΠΎΠ½ΠΎΠ»ΠΎΠ³ΠΈΠΈ с ΠΊΠΈΠ½Π΅Ρ‚ичСской энСргиСй, энСргия ΠΌΠ°Π³Π½ΠΈΡ‚Π½ΠΎΠ³ΠΎ поля Ρ‚ΠΎΠΊΠ° Ρ€Π°Π²Π½Π°: W=LI2/2.

Π‘ΠΈΠ»Π΅Ρ‚ 6.

1. 1. ΠšΠΈΠ½Π΅Ρ‚ΠΈΡ‡Π΅ΡΠΊΠ°Ρ ΠΈ ΠΏΠΎΡ‚Π΅Π½Ρ†ΠΈΠ°Π»ΡŒΠ½Π°Ρ энСргия. Π—Π°ΠΊΠΎΠ½ сохранСния энСргии Π² ΠΌΠ΅Ρ…Π°Π½ΠΈΠΊΠ΅. ВычислитС ΠΏΠΎΡ‚Π΅Π½Ρ†ΠΈΠ°Π»ΡŒΠ½ΡƒΡŽ ΡΠ½Π΅Ρ€Π³ΠΈΡŽ Ρ‚Π΅Π»Π° Π² ΠΏΠΎΠ»Π΅ силы тяТСсти Π² Π·Π°Π΄Π°Π½Π½ΠΎΠΉ систСмС отсчСта.

2. 2. НСпрСрывный ΠΈ Π»ΠΈΠ½Π΅ΠΉΡ‡Π°Ρ‚Ρ‹ΠΉ спСктры. Π‘ΠΏΠ΅ΠΊΡ‚Ρ€Ρ‹ испускания ΠΈ ΠΏΠΎΠ³Π»ΠΎΡ‰Π΅Π½ΠΈΡ. Π‘ΠΏΠ΅ΠΊΡ‚Ρ€Π°Π»ΡŒΠ½Ρ‹ΠΉ Π°Π½Π°Π»ΠΈΠ· ΠΈ Π΅Π³ΠΎ ΠΏΡ€ΠΈΠΌΠ΅Π½Π΅Π½ΠΈΠ΅.

3. 3. (Π·Π°Π΄Π°Ρ‡Π° Π½Π° Ρ€Π°Π±ΠΎΡ‚Ρƒ ΠΈ ΠΌΠΎΡ‰Π½ΠΎΡΡ‚ΡŒ Ρ‚ΠΎΠΊΠ°).

1. Π­Π½Π΅Ρ€Π³ΠΈΠ΅ΠΉ — называСтся скалярная физичСская Π²Π΅Π»ΠΈΡ‡ΠΈΠ½Π°, ΡΠ²Π»ΡΡŽΡ‰Π°ΡΡΡ Π΅Π΄ΠΈΠ½ΠΎΠΉ ΠΌΠ΅Ρ€ΠΎΠΉ Π΅Π΄ΠΈΠ½ΠΎΠΉ ΠΌΠ΅Ρ€ΠΎΠΉ Ρ€Π°Π·Π»ΠΈΡ‡Π½Ρ‹Ρ… Ρ„ΠΎΡ€ΠΌ двиТСния ΠΌΠ°Ρ‚Π΅Ρ€ΠΈΠΈ ΠΈ ΠΌΠ΅Ρ€ΠΎΠΉ ΠΏΠ΅Ρ€Π΅Ρ…ΠΎΠ΄Π° двиТСния ΠΌΠ°Ρ‚Π΅Ρ€ΠΈΠΈ ΠΈΠ· ΠΎΠ΄Π½ΠΈΡ… Ρ„ΠΎΡ€ΠΌ Π² Π΄Ρ€ΡƒΠ³ΠΈΠ΅.

ΠšΠΈΠ½Π΅Ρ‚ΠΈΡ‡Π΅ΡΠΊΠ°Ρ энСргия ΠΌΠ°Ρ‚Π΅Ρ€ΠΈΠ°Π»ΡŒΠ½ΠΎΠΉ Ρ‚ΠΎΡ‡ΠΊΠΈ ΠΈΠ»ΠΈ Ρ‚Π΅Π»Π° являСтся ΠΌΠ΅Ρ€ΠΎΠΉ ΠΈΡ… ΠΌΠ΅Ρ…аничСского двиТСния, зависящСй ΠΎΡ‚ ΡΠΊΠΎΡ€ΠΎΡΡ‚Π΅ΠΉ, зависящСй ΠΎΡ‚ ΡΠΊΠΎΡ€ΠΎΡΡ‚Π΅ΠΉ ΠΈΡ… Π΄Π²ΠΈΠΆΠ΅Π½ΠΈΡ Π² Π΄Π°Π½Π½ΠΎΠΉ ΠΈΠ½Π΅Ρ€Ρ†ΠΈΠ°Π»ΡŒΠ½ΠΎΠΉ систСмС отсчСта. ΠšΠΈΠ½Π΅Ρ‚ΠΈΡ‡Π΅ΡΠΊΠ°Ρ энСргия — это энСргия двиТСния, зависящая ΠΎΡ‚ ΠΌΠ°ΡΡΡ‹ Ρ‚Π΅Π»Π° ΠΈ ΡΠΊΠΎΡ€ΠΎΡΡ‚ΠΈ Π΅Π³ΠΎ двиТСния. Π›ΡŽΠ±ΠΎΠ΅ двиТущССся Ρ‚Π΅Π»ΠΎ ΠΎΠ±Π»Π°Π΄Π°Π΅Ρ‚ кинСтичСской энСргиСй. Π­ΠΊΡΠΏΠ΅Ρ€ΠΈΠΌΠ΅Π½Ρ‚Π°Π»ΡŒΠ½ΠΎ установлСно, Ρ‡Ρ‚ΠΎ ΠΏΠΎ ΡΠΎΠ²Π΅Ρ€ΡˆΠ΅Π½Π½ΠΎΠΉ Ρ€Π°Π±ΠΎΡ‚Π΅ ΠΌΠΎΠΆΠ½ΠΎ ΡΡƒΠ΄ΠΈΡ‚ΡŒ ΠΎΠ± ΠΈΠ·ΠΌΠ΅Π½Π΅Π½ΠΈΠΈ кинСтичСской энСргии. На ΠΎΡΠ½ΠΎΠ²Π°Π½ΠΈΠΈ этого утвСрТдСния ΠΌΠΎΠΆΠ½ΠΎ ΠΏΠΎΠ»ΡƒΡ‡ΠΈΡ‚ΡŒ Ρ‚ΠΎΡ‡Π½ΡƒΡŽ ΠΌΠ°Ρ‚Π΅ΠΌΠ°Ρ‚ΠΈΡ‡Π΅ΡΠΊΡƒΡŽ Ρ„ΠΎΡ€ΠΌΡƒΠ»Ρƒ кинСтичСской энСргии. Π‘ΡƒΠ΄Π΅ΠΌ ΡΡ‡ΠΈΡ‚Π°Ρ‚ΡŒ, Ρ‡Ρ‚ΠΎ СдинствСнным Ρ€Π΅Π·ΡƒΠ»ΡŒΡ‚Π°Ρ‚ΠΎΠΌ Ρ€Π°Π±ΠΎΡ‚Ρ‹ силы Π½Π° ΠΏΡƒΡ‚ΠΈ Π±ΡƒΠ΄Π΅Ρ‚ сообщСниС Ρ‚Π΅Π»Ρƒ кинСтичСской энСргии: Π•=А, Π½ΠΎ А=F*S, Π° F= ma. Из ΠΊΠΈΠ½Π΅ΠΌΠ°Ρ‚ΠΈΠΊΠΈ извСстно, Ρ‡Ρ‚ΠΎ ΠΏΡƒΡ‚ΡŒ ΠΏΡ€ΠΈ равноускорСнном Π΄Π²ΠΈΠΆΠ΅Π½ΠΈΠΈ ΠΏΡ€ΠΈ Π½Π°Ρ‡Π°Π»ΡŒΠ½ΠΎΠΉ скорости Ρ€Π°Π²Π½ΠΎΠΉ 0 рассчитываСтся ΠΏΠΎ Ρ„ΠΎΡ€ΠΌΡƒΠ»Π΅ S=V2/2a,.

ΠžΡ‚ ΡΡŽΠ΄Π° ΠΏΠΎΠ»ΡƒΡ‡Π°Π΅ΠΌ E=FS=maV2/2a=mV2/2. ΠšΠΈΠ½Π΅Ρ‚ΠΈΡ‡Π΅ΡΠΊΠ°Ρ энСргия Ρ€Π°Π²Π½Π° Ρ€Π°Π±ΠΎΡ‚Π΅, поэтому измСряСтся Π² Π”ΠΆ.

Π’Π΅ΠΎΡ€Π΅ΠΌΠ° ΠΎ ΠΊΠΈΠ½Π΅Ρ‚ичСской энСргии: ΠΈΠ·ΠΌΠ΅Π½Π΅Π½ΠΈΠ΅ кинСтичСской энСргии Ρ‚Π΅Π»Π° ΠΏΡ€ΠΈ Π΅Π³ΠΎ ΠΏΠ΅Ρ€Π΅Ρ…ΠΎΠ΄Π΅ ΠΈΠ· ΠΎΠ΄Π½ΠΎΠ³ΠΎ состояния Π² Π΄Ρ€ΡƒΠ³ΠΎΠ΅ Ρ€Π°Π²Π½ΠΎ Ρ€Π°Π±ΠΎΡ‚Π΅ всСх сил, Π΄Π΅ΠΉΡΡ‚Π²ΡƒΡŽΡ‰ΠΈΡ… Π½Π° ΡΡ‚ΠΎ Ρ‚Π΅Π»ΠΎ. Π Π°Π±ΠΎΡ‚Π° Π»ΡŽΠ±Ρ‹Ρ… сил являСтся ΠΌΠ΅Ρ€ΠΎΠΉ измСрСния кинСтичСской энСргии Ρ‚Π΅Π»Π°. ДСйствиС сил, Ρ€Π°Π±ΠΎΡ‚Π° ΠΊΠΎΡ‚ΠΎΡ€Ρ‹Ρ… Π½Π° Π΄Π°Π½Π½ΠΎΠΌ участкС Ρ‚Ρ€Π°Π΅ΠΊΡ‚ΠΎΡ€ΠΈΠΈ ΠΏΠΎΠ»ΠΎΠΆΠΈΡ‚Π΅Π»ΡŒΠ½Π°, ΠΏΡ€ΠΈΠ²ΠΎΠ΄ΠΈΡ‚ ΠΊ ΡƒΠ²Π΅Π»ΠΈΡ‡Π΅Π½ΠΈΡŽ кинСтичСской энСргии Ρ‚Π΅Π»Π° (ΠΈ Π½Π°ΠΎΠ±ΠΎΡ€ΠΎΡ‚). ΠŸΠΎΡ‚Π΅Π½Ρ†ΠΈΠ°Π»ΡŒΠ½ΠΎΠΉ энСргиСй называСтся энСргия, которая опрСдСляСтся Π²Π·Π°ΠΈΠΌΠ½Ρ‹ΠΌ располоТСниСм Ρ‚Π΅Π» ΠΈΠ»ΠΈ частСй ΠΎΠ΄Π½ΠΎΠ³ΠΎ Ρ‚Π΅Π»Π°.

ΠŸΠΎΡ‚Π΅Π½Ρ†ΠΈΠ°Π»ΡŒΠ½Π°Ρ энСргия Ρ‚Π΅Π»Π°, поднятого Π½Π°Π΄ Π—Π΅ΠΌΠ»Π΅ΠΉ, — это энСргия взаимодСйствия Ρ‚Π΅Π»Π° ΠΈ Π—Π΅ΠΌΠ»ΠΈ с ΠΏΠΎΠΌΠΎΡ‰ΡŒΡŽ Π³Ρ€Π°Π²ΠΈΡ‚Π°Ρ†ΠΈΠΎΠ½Π½Ρ‹Ρ… сил. ΠŸΠΎΡ‚Π΅Π½Ρ†ΠΈΠ°Π»ΡŒΠ½Π°Ρ энСргия — это Π²Π΅Π»ΠΈΡ‡ΠΈΠ½Π°, равная ΠΏΡ€ΠΎΠΈΠ·Π²Π΅Π΄Π΅Π½ΠΈΡŽ массы Ρ‚Π΅Π»Π° Π½Π° ΠΌΠΎΠ΄ΡƒΠ»ΡŒ ускорСния свободного падСния ΠΈ Π½Π° Π²Ρ‹ΡΠΎΡ‚Ρƒ Ρ‚Π΅Π»Π° Π½Π°Π΄ ΠΏΠΎΠ²Π΅Ρ€Ρ…Π½ΠΎΡΡ‚ΡŒΡŽ Π—Π΅ΠΌΠ»ΠΈ.

ΠŸΠΎΡ‚Π΅Π½Ρ†ΠΈΠ°Π»ΡŒΠ½ΠΎΠΉ энСргиСй называСтся Ρ‡Π°ΡΡ‚ΡŒ мСханичСской энСргии, зависящая ΠΎΡ‚ Π²Π·Π°ΠΈΠΌΠ½ΠΎΠ³ΠΎ располоТСния Π΅Π΅ Ρ‡Π°ΡΡ‚Π΅ΠΉ ΠΈ ΠΈΡ… ΠΏΠΎΠ»ΠΎΠΆΠ΅Π½ΠΈΡ Π²ΠΎ Π²Π½Π΅ΡˆΠ½Π΅ΠΌ силовом ΠΏΠΎΠ»Π΅.

Π—Π°ΠΊΠΎΠ½ сохранСния мСханичСской энСргии: мСханичСская энСргия систСмы, Π² ΠΊΠΎΡ‚ΠΎΡ€ΠΎΠΉ Π΄Π΅ΠΉΡΡ‚Π²ΡƒΡŽΡ‚ ΠΏΠΎΡ‚Π΅Π½Ρ†ΠΈΠ°Π»ΡŒΠ½Ρ‹Π΅ силы, сохраняСтся постоянной Π² ΠΏΡ€ΠΎΡ†Π΅ΡΡΠ΅ двиТСния систСмы.

2. НСпрСрывный ΠΈ Π»ΠΈΠ½Π΅ΠΉΡ‡Π°Ρ‚Ρ‹ΠΉ спСктры. Π‘ΠΏΠ΅ΠΊΡ‚Ρ€Ρ‹ испускания ΠΈ ΠΏΠΎΠ³Π»ΠΎΡ‰Π΅Π½ΠΈΡ. Π‘ΠΏΠ΅ΠΊΡ‚Ρ€Π°Π»ΡŒΠ½Ρ‹ΠΉ Π°Π½Π°Π»ΠΈΠ· ΠΈ Π΅Π³ΠΎ ΠΏΡ€ΠΈΠΌΠ΅Π½Π΅Π½ΠΈΠ΅.

Π‘ΠΏΠ΅ΠΊΡ‚Ρ€ — это Ρ†Π²Π΅Ρ‚Π½Ρ‹Π΅ полосы, ΠΏΠΎΠ»ΡƒΡ‡Π°ΡŽΡ‰ΠΈΠ΅ΡΡ Π² Ρ€Π΅Π·ΡƒΠ»ΡŒΡ‚Π°Ρ‚Π΅ разлоТСния свСта ΠΏΡ€ΠΈΠ·ΠΌΠΎΠΉ) ΠΈΠ»ΠΈ Π΄Ρ€ΡƒΠ³ΠΈΠΌ ΠΏΡ€Π΅Π΄ΠΌΠ΅Ρ‚ΠΎΠΌ) ΠΏΠΎ Π΄Π»ΠΈΠ½Π°ΠΌ Π²ΠΎΠ»Π½. Π Π°Π·Π»ΠΈΡ‡Π½Ρ‹Π΅ вСщСства ΠΏΡ€ΠΈ ΠΏΠΎΠ³Π»ΠΎΡ‰Π΅Π½ΠΈΠΈ энСргии сами становятся источниками свСта ΠΈ ΠΌΠΎΠ³ΡƒΡ‚ ΡΠΎΠ·Π΄Π°Π²Π°Ρ‚ΡŒ излучСния Ρ€Π°Π·Π½ΠΎΠΎΠ±Ρ€Π°Π·Π½Ρ‹Ρ… составов, спСктры ΠΊΠΎΡ‚ΠΎΡ€Ρ‹Ρ… ΠΈΡΡΠ»Π΅Π΄ΡƒΡŽΡ‚ с ΠΏΠΎΠΌΠΎΡ‰ΡŒΡŽ ΡΠΏΠ΅Ρ†ΠΈΠ°Π»ΡŒΠ½Ρ‹Ρ… ΠΏΡ€ΠΈΠ±ΠΎΡ€ΠΎΠ² — спСктроскопов. ВсС спСктры ΠΌΠΎΠΆΠ½ΠΎ Ρ€Π°Π·Π΄Π΅Π»ΠΈΡ‚ΡŒ Π½Π° Ρ‚Ρ€ΠΈ Ρ‚ΠΈΠΏΠ°: Π½Π΅ΠΏΡ€Π΅Ρ€Ρ‹Π²Π½Ρ‹Π΅, Π»ΠΈΠ½Π΅ΠΉΡ‡Π°Ρ‚Ρ‹Π΅ ΠΈ ΡΠΏΠ΅ΠΊΡ‚Ρ€Ρ‹ поглощСния.

НСпрСрывныС спСктры. Π‘ΠΎΠ»Π½Π΅Ρ‡Π½Ρ‹ΠΉ спСктр, спСктр элСктричСской Π»Π°ΠΌΠΏΡ‹ ΡΠ²Π»ΡΡŽΡ‚ΡΡ Π½Π΅ΠΏΡ€Π΅Ρ€Ρ‹Π²Π½Ρ‹ΠΌΠΈ. Π’ ΡΠΏΠ΅ΠΊΡ‚Ρ€Π΅ Π½Π΅Ρ‚ Ρ€Π°Π·Ρ€Ρ‹Π²ΠΎΠ², ΠΈ Π½Π° ΡΠΊΡ€Π°Π½Π΅ ΠΌΠΎΠΆΠ½ΠΎ Π²ΠΈΠ΄Π΅Ρ‚ΡŒ ΡΠΏΠ»ΠΎΡˆΠ½ΡƒΡŽ Ρ†Π²Π΅Ρ‚Π½ΡƒΡŽ полоску. Π’Π°ΠΊΠΈΠ΅ спСктры Π΄Π°ΡŽΡ‚ Ρ‚Π΅Π»Π°, находящиСся Π² Ρ‚Π²Π΅Ρ€Π΄ΠΎΠΌ ΠΈΠ»ΠΈ ΠΆΠΈΠ΄ΠΊΠΎΠΌ состоянии. Для получСния Π²ΠΈΠ΄ΠΈΠΌΠΎΠ³ΠΎ Π½Π΅ΠΏΡ€Π΅Ρ€Ρ‹Π²Π½ΠΎΠ³ΠΎ спСктра Π½ΡƒΠΆΠ½ΠΎ Π½Π°Π³Ρ€Π΅Ρ‚ΡŒ Ρ‚Π΅Π»ΠΎ Π΄ΠΎ Π²Ρ‹ΡΠΎΠΊΠΎΠΉ Ρ‚Π΅ΠΌΠΏΠ΅Ρ€Π°Ρ‚ΡƒΡ€Ρ‹. Π₯Π°Ρ€Π°ΠΊΡ‚Π΅Ρ€ Π½Π΅ΠΏΡ€Π΅Ρ€Ρ‹Π²Π½ΠΎΠ³ΠΎ спСктра Π² ΡΠΈΠ»ΡŒΠ½ΠΎΠΉ стСпСни зависит ΠΎΡ‚ Π²Π·Π°ΠΈΠΌΠΎΠ΄Π΅ΠΉΡΡ‚вия Π°Ρ‚ΠΎΠΌΠΎΠ² Π΄Ρ€ΡƒΠ³ с Π΄Ρ€ΡƒΠ³ΠΎΠΌ.

Π›ΠΈΠ½Π΅ΠΉΡ‡Π°Ρ‚Ρ‹Π΅ спСктры. Π˜Ρ… Π΄Π°ΡŽΡ‚ всС вСщСства, находящиСся Π² Π³Π°Π·ΠΎΠΎΠ±Ρ€Π°Π·Π½ΠΎΠΌ Π°Ρ‚ΠΎΠΌΠ°Ρ€Π½ΠΎΠΌ (Π½ΠΎ Π½Π΅ ΠΌΠΎΠ»Π΅ΠΊΡƒΠ»ΡΡ€Π½ΠΎΠΌ) состоянии. Π’ ΡΡ‚ΠΎΠΌ случаС свСт ΠΈΠ·Π»ΡƒΡ‡Π°ΡŽΡ‚ Π°Ρ‚ΠΎΠΌΡ‹, ΠΊΠΎΡ‚ΠΎΡ€Ρ‹Π΅ практичСски Π½Π΅ Π²Π·Π°ΠΈΠΌΠΎΠ΄Π΅ΠΉΡΡ‚Π²ΡƒΡŽΡ‚ Π΄Ρ€ΡƒΠ³ с Π΄Ρ€ΡƒΠ³ΠΎΠΌ. Π­Ρ‚ΠΎ самый Ρ„ΡƒΠ½Π΄Π°ΠΌΠ΅Π½Ρ‚Π°Π»ΡŒΠ½Ρ‹ΠΉ Ρ‚ΠΈΠΏ спСктров. ΠžΠ±Ρ‹Ρ‡Π½ΠΎ для наблюдСния Π»ΠΈΠ½Π΅ΠΉΡ‡Π°Ρ‚Ρ‹Ρ… спСктров ΠΈΡΠΏΠΎΠ»ΡŒΠ·ΡƒΡŽΡ‚ свСчСниС ΠΏΠ°Ρ€ΠΎΠ² вСщСства Π² ΠΏΠ»Π°ΠΌΠ΅Π½ΠΈ ΠΈΠ»ΠΈ свСчСниС Π³Π°Π·ΠΎΠ²ΠΎΠ³ΠΎ разряда Π² Ρ‚Ρ€ΡƒΠ±ΠΊΠ΅, Π½Π°ΠΏΠΎΠ»Π½Π΅Π½Π½ΠΎΠΉ исслСдуСмым Π³Π°Π·ΠΎΠΌ.

Π‘ΠΏΠ΅ΠΊΡ‚Ρ€Ρ‹ поглощСния. Если ΠΏΡ€ΠΎΠΏΡƒΡΠΊΠ°Ρ‚ΡŒ Π±Π΅Π»Ρ‹ΠΉ свСт сквозь Ρ…ΠΎΠ»ΠΎΠ΄Π½Ρ‹ΠΉ Π½Π΅ΠΈΠ·Π»ΡƒΡ‡Π°ΡŽΡ‰ΠΈΠΉ Π³Π°Π·, Ρ‚ΠΎ Π½Π° Ρ„ΠΎΠ½Π΅ Π½Π΅ΠΏΡ€Π΅Ρ€Ρ‹Π²Π½ΠΎΠ³ΠΎ спСктра источника ΠΏΠΎΡΠ²Π»ΡΡŽΡ‚ΡΡ Ρ‚Π΅ΠΌΠ½Ρ‹Π΅ Π»ΠΈΠ½ΠΈΠΈ. Π“Π°Π· ΠΏΠΎΠ³Π»ΠΎΡ‰Π°Π΅Ρ‚ Π½Π°ΠΈΠ±ΠΎΠ»Π΅Π΅ свСт ΠΊΠ°ΠΊ Ρ€Π°Π· Ρ‚Π΅Ρ… Π΄Π»ΠΈΠ½ Π²ΠΎΠ»Π½, ΠΊΠΎΡ‚ΠΎΡ€Ρ‹Π΅ ΠΎΠ½ ΠΈΡΠΏΡƒΡΠΊΠ°Π΅Ρ‚ Π² ΡΠΈΠ»ΡŒΠ½ΠΎ Π½Π°Π³Ρ€Π΅Ρ‚ΠΎΠΌ состоянии. Π’Π΅ΠΌΠ½Ρ‹Π΅ Π»ΠΈΠ½ΠΈΠΈ Π½Π° Ρ„ΠΎΠ½Π΅ Π½Π΅ΠΏΡ€Π΅Ρ€Ρ‹Π²Π½ΠΎΠ³ΠΎ спСктра — это Π»ΠΈΠ½ΠΈΠΈ поглощСния, ΠΎΠ±Ρ€Π°Π·ΡƒΡŽΡ‰ΠΈΠ΅ Π² ΡΠΎΠ²ΠΎΠΊΡƒΠΏΠ½ΠΎΡΡ‚ΠΈ спСктр поглощСния.

ΠžΠΏΡ‹Ρ‚Π½Ρ‹ΠΌ ΠΏΡƒΡ‚Π΅ΠΌ оказалось, Ρ‡Ρ‚ΠΎ свСтящиСся ΠΏΠ°Ρ€Ρ‹ любого химичСского элСмСнта ΠΈΠ·Π»ΡƒΡ‡Π°ΡŽΡ‚ Ρ‚ΠΎΠ»ΡŒΠΊΠΎ ΠΎΠ΄Π½ΠΎΠΌΡƒ Π΅ΠΌΡƒ свойствСнный спСктр — Π½Π°Π±ΠΎΡ€ монохроматичСских ΠΈΠ·Π»ΡƒΡ‡Π΅Π½ΠΈΠΈ, ΠΊΠ°ΠΆΠ΄ΠΎΠΌΡƒ ΠΈΠ· ΠΊΠΎΡ‚ΠΎΡ€Ρ‹Ρ… Π² ΡΠΏΠ΅ΠΊΡ‚Ρ€Π΅ ΠΏΡ€ΠΈΠ½Π°Π΄Π»Π΅ΠΆΠΈΡ‚ своя линия. Анализ спСктров Π΄Π°Π΅Ρ‚ Π²ΠΎΠ·ΠΌΠΎΠΆΠ½ΠΎΡΡ‚ΡŒ ΠΎΠ±Π½Π°Ρ€ΡƒΠΆΠΈΡ‚ΡŒ ΠΈ ΠΌΠ°Π³Π½ΠΈΡ‚Π½Ρ‹Π΅ поля. ΠŸΡ€ΠΈ воздСйствии ΠΌΠ°Π³Π½ΠΈΡ‚Π½ΠΎΠ³ΠΎ поля Π½Π° ΠΈΠ·Π»ΡƒΡ‡Π°ΡŽΡ‰ΠΈΠ΅ Π°Ρ‚ΠΎΠΌΡ‹ ΠΏΠΎΡΠ²Π»ΡΡŽΡ‚ΡΡ линииспутники. По ΡΠΏΠ΅ΠΊΡ‚Ρ€Π°ΠΌ ΡƒΡ‡Π΅Π½Ρ‹Π΅ ΠΎΠΏΡ€Π΅Π΄Π΅Π»ΡΡŽΡ‚ энСргСтичСскиС ΡƒΡ€ΠΎΠ²Π½ΠΈ Π°Ρ‚ΠΎΠΌΠΎΠ² ΠΈ ΠΌΠΎΠ»Π΅ΠΊΡƒΠ». Π‘ ΠΏΠΎΡΠ²Π»Π΅Π½ΠΈΠ΅ΠΌ Π»Π°Π·Π΅Ρ€Π° появился Π½ΠΎΠ²Ρ‹ΠΉ Ρ€Π°Π·Π΄Π΅Π» спСктроскопии — лазСрная спСктроскопия. Π›Π°Π·Π΅Ρ€Ρ‹ с ΠΏΠ΅Ρ€Π΅ΡΡ‚Ρ€Π°ΠΈΠ²Π°Π΅ΠΌΠΎΠΉ частотой ΠΏΠΎΠ·Π²ΠΎΠ»ΡΡŽΡ‚ ΠΏΠΎΠ΄ΠΎΠ±Ρ€Π°Ρ‚ΡŒ Ρ‚Π°ΠΊΡƒΡŽ частоту излучСния, ΠΏΡ€ΠΈ ΠΊΠΎΡ‚ΠΎΡ€ΠΎΠΉ Π±ΡƒΠ΄Π΅Ρ‚ Π²ΠΎΠ·Π±ΡƒΠΆΠ΄Π΅Π½ Π²ΠΏΠΎΠ»Π½Π΅ ΠΎΠΏΡ€Π΅Π΄Π΅Π»Π΅Π½Π½Ρ‹ΠΉ ΡƒΡ€ΠΎΠ²Π΅Π½ΡŒ ΠΈΠ·ΡƒΡ‡Π°Π΅ΠΌΠΎΠ³ΠΎ Π°Ρ‚ΠΎΠΌΠ° ΠΈΠ»ΠΈ ΠΌΠΎΠ»Π΅ΠΊΡƒΠ»Ρ‹. Π›Π°Π·Π΅Ρ€Ρ‹ с ΠΏΠ΅Ρ€Π΅ΡΡ‚Ρ€Π°ΠΈΠ²Π°Π΅ΠΌΠΎΠΉ частотой Π΄Π°ΡŽΡ‚ Π²ΠΎΠ·ΠΌΠΎΠΆΠ½ΠΎΡΡ‚ΡŒ Π΄ΠΎΡΡ‚ΠΈΠ³Π½ΡƒΡ‚ΡŒ ΠΏΡ€Π΅Π΄Π΅Π»ΡŒΠ½ΠΎΠΉ Ρ‡ΡƒΠ²ΡΡ‚Π²ΠΈΡ‚Π΅Π»ΡŒΠ½ΠΎΡΡ‚ΠΈ ΡΠΏΠ΅ΠΊΡ‚Ρ€Π°Π»ΡŒΠ½ΠΎΠ³ΠΎ Π°Π½Π°Π»ΠΈΠ·Π° — ΠΎΠ±Π½Π°Ρ€ΡƒΠΆΠΈΠ²Π°Ρ‚ΡŒ Π°Ρ‚ΠΎΠΌΡ‹ элСмСнта с ΠΊΠΎΠ½Ρ†Π΅Π½Ρ‚Ρ€Π°Ρ†ΠΈΠ΅ΠΉ, скаТСм, Π² 100 Π°Ρ‚ΠΎΠΌΠΎΠ² Π½Π° 1 ΡΠΌ3 объСм Π³Π°Π·Π°.

Π‘ΠΏΠ΅ΠΊΡ‚Ρ€Π°Π»ΡŒΠ½Ρ‹ΠΉ Π°Π½Π°Π»ΠΈΠ· — это ΠΌΠ΅Ρ‚ΠΎΠ΄ опрСдСлСния химичСского состава вСщСства ΠΏΠΎ Π΅Π³ΠΎ спСктру.

Π‘ΠΈΠ»Π΅Ρ‚ 7.

1. 1. ΠžΡΠ½ΠΎΠ²Π½Ρ‹Π΅ полоТСния МКВ ΠΈ ΠΈΡ… ΠΎΠΏΡ‹Ρ‚Π½ΠΎΠ΅ обоснованиС. Масса ΠΈ Ρ€Π°Π·ΠΌΠ΅Ρ€Ρ‹ ΠΌΠΎΠ»Π΅ΠΊΡƒΠ». ΠŸΠΎΡΡ‚ΠΎΡΠ½Π½Π°Ρ Авогадро.

2. 2. ΠšΠΎΠ»Π΅Π±Π°Ρ‚Π΅Π»ΡŒΠ½Ρ‹Π΅ двиТСния. ГармоничСскиС колСбания. Амплитуда, ΠΏΠ΅Ρ€ΠΈΠΎΠ΄, частота ΠΈ Ρ„Π°Π·Π° ΠΊΠΎΠ»Π΅Π±Π°Π½ΠΈΠΉ. Π˜Π·ΠΌΠ΅Ρ€ΡŒΡ‚Π΅ ΡΠΎΠ±ΡΡ‚Π²Π΅Π½Π½ΡƒΡŽ частоту ΠΏΡ€Π΅Π΄Π»ΠΎΠΆΠ΅Π½Π½ΠΎΠΉ ΠΊΠΎΠ»Π΅Π±Π°Ρ‚Π΅Π»ΡŒΠ½ΠΎΠΉ систСмы.

3. 3. «ΠŸΠΠ ΠΠ›Π›Π•Π›Π¬ΠΠžΠ• Π‘ΠžΠ•Π”Π˜ΠΠ•ΠΠ˜Π• ΠŸΠ ΠžΠ’ΠžΠ”Π˜ΠšΠžΠ’».

1. МоТно Π²Ρ‹Π΄Π΅Π»ΠΈΡ‚ΡŒ Ρ‚Ρ€ΠΈ основных полоТСния молСкулярно-кинСтичСской Ρ‚Π΅ΠΎΡ€ΠΈΠΈ, которая ΠΎΠ±ΡŠΡΡΠ½ΡΠ΅Ρ‚ свойства Ρ‚Π΅Π», состоящих ΠΈΠ· ΠΎΠ³Ρ€ΠΎΠΌΠ½ΠΎΠ³ΠΎ числа ΠΌΠΎΠ»Π΅ΠΊΡƒΠ», Π° Ρ‚Π°ΠΊΠΆΠ΅ особСнности Ρ‚Π΅ΠΏΠ»ΠΎΠ²Ρ‹Ρ… процСссов, Π² Π½ΠΈΡ… ΠΏΡ€ΠΎΡ‚Π΅ΠΊΠ°ΡŽΡ‰ΠΈΡ…:

1. 1. вСщСство состоит ΠΈΠ· ΠΎΡ‚Π΄Π΅Π»ΡŒΠ½Ρ‹Ρ… ΠΌΠ΅Π»ΡŒΡ‡Π°ΠΉΡˆΠΈΡ… частиц, Π½Π°Π·Ρ‹Π²Π°Π΅ΠΌΡ‹Ρ… ΠΌΠΎΠ»Π΅ΠΊΡƒΠ»Π°ΠΌΠΈ; ΠΌΠΎΠ»Π΅ΠΊΡƒΠ»Π° — это наимСньшая элСктричСски Π½Π΅ΠΉΡ‚Ρ€Π°Π»ΡŒΠ½Π°Ρ частица вСщСства, ΠΎΠ±Π»Π°Π΄Π°ΡŽΡ‰Π°Ρ всСми Π΅Π³ΠΎ химичСскими свойствами ΠΈ ΠΌΠΎΠ³ΡƒΡ‰Π°Ρ ΡΡƒΡ‰Π΅ΡΡ‚Π²ΠΎΠ²Π°Ρ‚ΡŒ ΡΠ°ΠΌΠΎΡΡ‚ΠΎΡΡ‚Π΅Π»ΡŒΠ½ΠΎ;

2. 2. ΠΌΠΎΠ»Π΅ΠΊΡƒΠ»Ρ‹ находятся Π² Π±Π΅ΡΠΏΡ€ΠΈΡΡ‚растном, хаотичСском Π΄Π²ΠΈΠΆΠ΅Π½ΠΈΠΈ;

3. 3. ΠΌΠΎΠ»Π΅ΠΊΡƒΠ»Ρ‹ Π²Π·Π°ΠΈΠΌΠΎΠ΄Π΅ΠΉΡΡ‚Π²ΡƒΡŽΡ‚ Π΄Ρ€ΡƒΠ³ с Π΄Ρ€ΡƒΠ³ΠΎΠΌ.

РСальноС сущСствованиС ΠΌΠΎΠ»Π΅ΠΊΡƒΠ» ΠΏΠΎΠ΄Ρ‚Π²Π΅Ρ€ΠΆΠ΄Π°Π΅Ρ‚ ΠΎΠ³Ρ€ΠΎΠΌΠ½ΠΎΠ΅ количСство ΡΠΊΡΠΏΠ΅Ρ€ΠΈΠΌΠ΅Π½Ρ‚Π°Π»ΡŒΠ½Ρ‹Ρ… Ρ„Π°ΠΊΡ‚ΠΎΠ². Π’Π°ΠΊ, всСм извСстно, Ρ‡Ρ‚ΠΎ Ρ‚Π²Π΅Ρ€Π΄ΠΎΠ΅ вСщСство ΠΌΠΎΠΆΠ½ΠΎ Ρ€Π°Π·Π΄Ρ€ΠΎΠ±ΠΈΡ‚ΡŒ Π»ΠΈΠ±ΠΎ Ρ€Π°ΡΡ‚Π²ΠΎΡ€ΠΈΡ‚ΡŒ Π² Π²ΠΎΠ΄Π΅ ΠΈΠ»ΠΈ Π΄Ρ€ΡƒΠ³ΠΈΡ… растворитСлях. ΠœΡ‹ Π·Π½Π°Π΅ΠΌ Ρ‡Ρ‚ΠΎ Π³Π°Π·Ρ‹ ΠΌΠΎΠ³ΡƒΡ‚ Ρ€Π°ΡΡˆΠΈΡ€ΡΡ‚ΡΡ ΠΈΠ»ΠΈ ΡΠΆΠΈΠΌΠ°Ρ‚ΡŒΡΡ. БроуновскоС Π΄Π²ΠΈΠΆΠ΅Π½ΠΈΠ΅ ΠΈΠ»ΠΈ диффузия ΡΠ²ΠΈΠ΄Π΅Ρ‚Π΅Π»ΡŒΡΡ‚Π²ΡƒΡŽΡ‚ ΠΎ Ρ‚ΠΎΠΌ. Π§Ρ‚ΠΎ ΠΌΠ΅ΠΆΠ΄Ρƒ ΠΌΠΎΠ»Π΅ΠΊΡƒΠ»Π°ΠΌΠΈ ΠΎΠ΄Π½ΠΎΠ³ΠΎ ΠΈ Ρ‚ΠΎΠ³ΠΎ ΠΆΠ΅ вСщСства Π΅ΡΡ‚ΡŒ ΠΏΡ€ΠΎΠΌΠ΅ΠΆΡƒΡ‚ΠΊΠΈ.

ΠœΠΎΠ»Π΅ΠΊΡƒΠ»Ρ‹ Π² Π²Π΅Ρ‰Π΅ΡΡ‚Π²Π΅ Π²Π·Π°ΠΈΠΌΠΎΠ΄Π΅ΠΉΡΡ‚Π²ΡƒΡŽΡ‚ Π΄Ρ€ΡƒΠ³ с Π΄Ρ€ΡƒΠ³ΠΎΠΌ: Π½Π°Π»ΠΈΡ‡ΠΈΠ΅ сил притяТСния ΠΏΠΎΠ΄Ρ‚Π²Π΅Ρ€ΠΆΠ΄Π°Π΅Ρ‚ Ρ‚ΠΎΡ‚ Ρ„Π°ΠΊΡ‚, Ρ‡Ρ‚ΠΎ Ρ‚Π΅Π»Π° сами ΠΏΠΎ ΡΠ΅Π±Π΅ Π½Π΅ Ρ€Π°ΡΠΏΠ°Π΄Π°ΡŽΡ‚ся Π½Π° ΠΌΠΎΠ»Π΅ΠΊΡƒΠ»Ρ‹, Π° Π΄Π»Ρ Ρ€Π°Π·Ρ€Ρ‹Π²Π°, Π½Π°ΠΏΡ€ΠΈΠΌΠ΅Ρ€, Ρ‚Π²Π΅Ρ€Π΄ΠΎΠ³ΠΎ Ρ‚Π΅Π»Π° трСбуСтся усилиС. О Π½Π°Π»ΠΈΡ‡ΠΈΠΈ сил притяТСния ΠΌΠΎΠΆΠ½ΠΎ ΡΡƒΠ΄ΠΈΡ‚ΡŒ ΠΏΠΎ Ρ‚ΠΎΠΌΡƒ, Ρ‡Ρ‚ΠΎ Π΄Π²Π΅ Π±Π»ΠΈΠ·ΠΊΠΎ располоТСнныС ΠΊΠ°ΠΏΠ»ΠΈ Тидкости ΡΠ»ΠΈΠΏΠ°ΡŽΡ‚ΡΡ.

Π’Π²Π΅Ρ€Π΄Ρ‹Π΅ Ρ‚Π΅Π»Π° ΠΈ ΠΆΠΈΠ΄ΠΊΠΎΡΡ‚ΠΈ практичСски нСсТимаСмы. Π‘Π°ΠΌΠΎ ΠΆΠ΅ сущСствованиС Ρ‚Π²Π΅Ρ€Π΄Ρ‹Ρ… Ρ‚Π΅ ΠΈ ΠΆΠΈΠ΄ΠΊΠΎΡΡ‚Π΅ΠΉ ΡΠ²ΠΈΠ΄Π΅Ρ‚Π΅Π»ΡŒΡΡ‚Π²ΡƒΠ΅Ρ‚ ΠΎ Ρ‚ΠΎΠΌ, Ρ‡Ρ‚ΠΎ силы отталкивания ΡƒΠ±Ρ‹Π²Π°ΡŽΡ‚ с ΡƒΠ²Π΅Π»ΠΈΡ‡Π΅Π½ΠΈΠ΅ΠΌ расстояния быстрСС, Ρ‡Π΅ΠΌ силы притяТСния. Если Π±Ρ‹ послСдниС ΡƒΠ±Ρ‹Π²Π°Π»ΠΈ быстрСС сил отталкивания, Ρ‚ΠΎ Π² ΠΏΡ€ΠΈΡ€ΠΎΠ΄Π΅ просто Π½Π΅ Π±Ρ‹Π»ΠΎ Π±Ρ‹ Π±ΠΎΠ»ΡŒΡˆΠΈΡ… устойчивых совокупностСй ΠΌΠΎΠ»Π΅ΠΊΡƒΠ», Ρ‚Π°ΠΊ ΠΊΠ°ΠΊ ΠΌΠΎΠ»Π΅ΠΊΡƒΠ»Ρ‹ Ρ€Π°Π·Π»Π΅Ρ‚Π΅Π»ΠΈΡΡŒ Π±Ρ‹ ΠΏΠΎΠ΄ дСйствиСм ΠΏΠΎΠ΄ дСйствиСм сил отталкивания.

ΠœΠΎΠ»Π΅ΠΊΡƒΠ»Π° — это наимСньшая частица вСщСства, ΠΎΠ±Π»Π°Π΄Π°ΡŽΡ‰Π°Ρ всСми Π΅Π³ΠΎ химичСскими свойствами. ΠœΠΎΠ»Π΅ΠΊΡƒΠ»Π° способна с ΡΠ°ΠΌΠΎΡΡ‚ΠΎΡΡ‚Π΅Π»ΡŒΠ½ΠΎΠΌΡƒ ΡΡƒΡ‰Π΅ΡΡ‚Π²ΠΎΠ²Π°Π½ΠΈΡŽ. Она ΠΌΠΎΠΆΠ΅Ρ‚ ΡΠΎΡΡ‚ΠΎΡΡ‚ΡŒ ΠΈΠ· ΠΎΠ΄ΠΈΠ½Π°ΠΊΠΎΠ²Ρ‹Ρ… Π°Ρ‚ΠΎΠΌΠΎΠ² ΠΈ Ρ€Π°Π·Π»ΠΈΡ‡Π½Ρ‹Ρ…. Π‘ΡƒΡ‰Π½ΠΎΡΡ‚ΡŒ ΠΌΠΎΠ»Π΅ΠΊΡƒΠ»Ρ‹ ΠΌΠΎΠΆΠ½ΠΎ ΠΎΠΏΠΈΡΠ°Ρ‚ΡŒ ΠΈ Ρ Π΄Ρ€ΡƒΠ³ΠΎΠΉ Ρ‚ΠΎΡ‡ΠΊΠΈ зрСния: ΠΌΠΎΠ»Π΅ΠΊΡƒΠ»Π° --это устойчивая систСма, состоящая ΠΈΠ· Π°Ρ‚ΠΎΠΌΠ½Ρ‹Ρ… ядСр ΠΈ ΠΎΠΊΡ€ΡƒΠΆΠ°ΡŽΡ‰ΠΈΡ… элСктронов, ΠΏΡ€ΠΈΡ‡Π΅ΠΌ химичСскиС свойства ΠΌΠΎΠ»Π΅ΠΊΡƒΠ» ΠΎΠΏΡ€Π΅Π΄Π΅Π»ΡΡŽΡ‚ΡΡ элСктронами Π²Π½Π΅ΡˆΠ½ΠΈΡ… ΠΎΠ±ΠΎΠ»ΠΎΡ‡Π΅ΠΊ Π² Π°Ρ‚ΠΎΠΌΠ°Ρ…. Атомы ΠΎΠ±ΡŠΠ΅Π΄ΠΈΠ½ΡΡŽΡ‚ΡΡ Π² ΠΌΠΎΠ»Π΅ΠΊΡƒΠ»Ρ‹ Π² Π±ΠΎΠ»ΡŒΡˆΠΈΠ½ΡΡ‚Π²Π΅ случаСв химичСскими связями. ΠžΠ±Ρ‹Ρ‡Π½ΠΎ такая связь создаСтся ΠΎΠ΄Π½ΠΎΠΉ, двумя ΠΈΠ»ΠΈ трСмя ΠΏΠ°Ρ€Π°ΠΌΠΈ элСктронов, ΠΊΠΎΡ‚ΠΎΡ€Ρ‹ΠΌΠΈ Π²Π»Π°Π΄Π΅ΡŽΡ‚ сообща Π΄Π²Π° Π°Ρ‚ΠΎΠΌΠ°. ΠœΠΎΠ»Π΅ΠΊΡƒΠ»Ρ‹ Ρ…Π°Ρ€Π°ΠΊΡ‚Π΅Ρ€ΠΈΠ·ΡƒΡŽΡ‚ΡΡ ΠΎΠΏΡ€Π΅Π΄Π΅Π»Π΅Π½Π½Ρ‹ΠΌ Ρ€Π°Π·ΠΌΠ΅Ρ€ΠΎΠΌ ΠΈ Ρ„ΠΎΡ€ΠΌΠΎΠΉ. Если извСстны молСкулярный вСс ΠΈ ΠΏΠ»ΠΎΡ‚Π½ΠΎΡΡ‚ΡŒ Π΄Π°Π½Π½ΠΎΠ³ΠΎ вСщСства. Π’ΠΎ Π²Ρ‹Ρ‡ΠΈΡΠ»ΠΈΡ‚ΡŒ Ρ€Π°Π·ΠΌΠ΅Ρ€ Π΅Π³ΠΎ ΠΌΠΎΠ»Π΅ΠΊΡƒΠ» нСслоТно. Для этого Π½Π°Π΄ΠΎ объСм, Π·Π°Π½ΠΈΠΌΠ°Π΅ΠΌΡ‹ΠΉ Π³Ρ€Π°ΠΌΠΌ — ΠΌΠΎΠ»Π΅ΠΊΡƒΠ»ΠΎΠΉ вСщСства, Ρ€Π°Π·Π΄Π΅Π»ΠΈΡ‚ΡŒ Π½Π° Ρ‡ΠΈΡΠ»ΠΎ Авогадро (6,02*1023 1/моль). Зная Π΄ΠΈΠ°ΠΌΠ΅Ρ‚Ρ€ ΠΌΠΎΠ»Π΅ΠΊΡƒΠ»Ρ‹ ΠΈ ΠΏΠ»ΠΎΡ‚Π½ΠΎΡΡ‚ΡŒ вСщСства ΠΌΠΎΠΆΠ½ΠΎ ΠΎΠΏΡ€Π΅Π΄Π΅Π»ΠΈΡ‚ΡŒ массу ΠΌΠΎΠ»Π΅ΠΊΡƒΠ»Ρ‹ m=p*V.

Из Π·Π°ΠΊΠΎΠ½ΠΎΠ² Авогадро:

— Π’ Ρ€Π°Π²Π½Ρ‹Ρ… ΠΎΠ±ΡŠΠ΅ΠΌΠ°Ρ… любого Π³Π°Π·Π° ΠΏΡ€ΠΈ ΠΎΠ΄ΠΈΠ½Π°ΠΊΠΎΠ²Ρ‹Ρ… условиях содСрТится ΠΎΠ΄ΠΈΠ½Π°ΠΊΠΎΠ²ΠΎΠ΅ количСство ΠΌΠΎΠ»Π΅ΠΊΡƒΠ».

— ΠœΠΎΠ»ΡŒ любого вСщСства содСрТит ΠΎΠ΄Π½ΠΎ ΠΈ Ρ‚ΠΎΠΆΠ΅ число ΠΌΠΎΠ»Π΅ΠΊΡƒΠ».

Π­Ρ‚ΠΎ число ΠΏΠΎΠ»ΡƒΡ‡ΠΈΠ»ΠΎ Π½Π°Π·Π²Π°Π½ΠΈΠ΅ число Авогадро. Число Авогадро опрСдСляСтся Ρ€Π°Π·Π»ΠΈΡ‡Π½Ρ‹ΠΌΠΈ ΠΌΠ΅Ρ‚ΠΎΠ΄Π°ΠΌΠΈ, Π½Π΅ΠΊΠΎΡ‚ΠΎΡ€Ρ‹Π΅ ΠΈΠ· Π½ΠΈΡ… основаны Π½Π° ΠΈΠ·ΡƒΡ‡Π΅Π½ΠΈΠΈ броуновского двиТСния.

2. 2. ΠšΠΎΠ»Π΅Π±Π°Ρ‚Π΅Π»ΡŒΠ½Ρ‹Π΅ двиТСния. ГармоничСскиС колСбания. Амплитуда, ΠΏΠ΅Ρ€ΠΈΠΎΠ΄, частота ΠΈ Ρ„Π°Π·Π° ΠΊΠΎΠ»Π΅Π±Π°Π½ΠΈΠΉ. Π˜Π·ΠΌΠ΅Ρ€ΡŒΡ‚Π΅ ΡΠΎΠ±ΡΡ‚Π²Π΅Π½Π½ΡƒΡŽ частоту ΠΏΡ€Π΅Π΄Π»ΠΎΠΆΠ΅Π½Π½ΠΎΠΉ ΠΊΠΎΠ»Π΅Π±Π°Ρ‚Π΅Π»ΡŒΠ½ΠΎΠΉ систСмы.

КолСбаниС — это процСсс, ΠΊΠΎΡ‚ΠΎΡ€Ρ‹ΠΉ Ρ‚ΠΎΡ‡Π½ΠΎ ΠΈΠ»ΠΈ ΠΏΡ€ΠΈΠ±Π»ΠΈΠ·ΠΈΡ‚Π΅Π»ΡŒΠ½ΠΎ повторяСтся Π²ΠΎ Π²Ρ€Π΅ΠΌΠ΅Π½ΠΈ. ΠœΠ΅Ρ…Π°Π½ΠΈΡ‡Π΅ΡΠΊΠΈΠ΅ колСбания — это мСханичСскиС двиТСния, ΠΊΠΎΡ‚ΠΎΡ€Ρ‹ΠΉ Ρ‚ΠΎΡ‡Π½ΠΎ ΠΈΠ»ΠΈ ΠΏΡ€ΠΈΠ±Π»ΠΈΠ·ΠΈΡ‚Π΅Π»ΡŒΠ½ΠΎ повторяСтся Π²ΠΎ Π²Ρ€Π΅ΠΌΠ΅Π½ΠΈ. КолСбания, происходящиС ΠΏΠΎ Π·Π°ΠΊΠΎΠ½Ρƒ синуса ΠΈΠ»ΠΈ косинуса — гармоничСскиС. ΠžΡΠ½ΠΎΠ²Π½Ρ‹Π΅ характСристики ΠΊΠΎΠ»Π΅Π±Π°Π½ΠΈΠΉ — это ΠΏΠ΅Ρ€ΠΈΠΎΠ΄, частота, Π°ΠΌΠΏΠ»ΠΈΡ‚ΡƒΠ΄Π°, Ρ‚. Π΅. физичСскиС Π²Π΅Π»ΠΈΡ‡ΠΈΠ½Ρ‹, ΠΊΠΎΡ‚ΠΎΡ€Ρ‹Π΅ относятся ΠΊ ΠΊΠΎΠ»Π΅Π±Π°Π½ΠΈΡΠΌ Π² Ρ†Π΅Π»ΠΎΠΌ ΠΈ Π½Π΅ ΠΌΠ΅Π½ΡΡŽΡ‚ся Π½Π° ΠΏΡ€ΠΎΡ‚яТСнии всСго колСбания. Амплитуда — это наибольшСС смСщСниС ΠΊΠΎΠ»Π΅Π±Π»ΡŽΡ‰Π΅Π³ΠΎΡΡ Ρ‚Π΅Π»Π° ΠΎΡ‚ ΠΏΠΎΠ»ΠΎΠΆΠ΅Π½ΠΈΡ равновСсия (срСднСго полоТСния). Π•Π΄Π΅Π½. Π˜Π·ΠΌΠ΅Ρ€Π΅Π½ΠΈΡ — ΠΌ. ΠŸΠ΅Ρ€ΠΈΠΎΠ΄ — врСмя ΠΎΠ΄Π½ΠΎΠ³ΠΎ колСбания (измСряСтся Π² с). Частота — количСство ΠΊΠΎΠ»Π΅Π±Π°Π½ΠΈΠΉ Π·Π° 1 с (Π“Ρ†, Π’=1/v, Π“Ρ†=1/с). ЦикличСская частота — … Π·Π° 2П сСкунды (Ρ€Π°Π΄/с, w=2Пv, w=2ΠΏ/v). Π€Π°Π·Π° — это Π²Ρ‹Ρ€Π°ΠΆΠ΅Π½ΠΈΠ΅, стоящиС ΠΏΠΎΠ΄ Π·Π½Π°ΠΊΠΎΠΌ синуса ΠΈΠ»ΠΈ косинуса (ΠΈΠΌΠ΅Π½Π½ΠΎ ΠΎΠ½Π° опрСдСляСт состояниС систСмы (wt + Ρ„ΠΈ0). ΠΠ°Ρ‡Π°Π»ΡŒΠ½Π°Ρ Ρ„Π°Π·Π° — это Π·Π½Π°Ρ‡Π΅Π½ΠΈΠ΅ Ρ„Π°Π·Ρ‹ Π² Π½Π°Ρ‡Π°Π»ΡŒΠ½Ρ‹ΠΉ ΠΌΠΎΠΌΠ΅Π½Ρ‚ Π²Ρ€Π΅ΠΌΠ΅Π½ΠΈ. (измСряСтся Π² Ρ€Π°Π΄ΠΈΠ°Π½Π°Ρ…). Π‘Π²ΠΎΠ±ΠΎΠ΄Π½Ρ‹Π΅ колСбания — ΡΠΎΠ²Π΅Ρ€ΡˆΠ°ΡŽΡ‰ΠΈΠ΅ΡΡ ΠΏΠΎΠ΄ дСйствиСм Π²Π½ΡƒΡ‚Ρ€Π΅Π½Π½ΠΈΡ… сил (условиС: внутрСнняя сила Π½Π°ΠΏΡ€Π°Π²Π»Π΅Π½Π° ΠΊ ΠΏΠΎΠ»ΠΎΠΆΠ΅Π½ΠΈΡŽ равновСсия, ΠΎΡ‡Π΅Π½ΡŒ нСбольшая сила трСния).

Π‘ΠΈΠ»Π΅Ρ‚ 8.

1. 1. Π Π°Π±ΠΎΡ‚Π° Π² Ρ‚Π΅Ρ€ΠΌΠΎΠ΄ΠΈΠ½Π°ΠΌΠΈΠΊΠ΅. ВнутрСнняя энСргия ΠΈ ΡΠΏΠΎΡΠΎΠ±Ρ‹ Π΅Π΅ ΠΈΠ·ΠΌΠ΅Ρ€Π΅Π½ΠΈΡ. ΠŸΠ΅Ρ€Π²Ρ‹ΠΉ Π·-Π½ Ρ‚Π΅Ρ€ΠΌΠΎΠ΄ΠΈΠ½Π°ΠΌΠΈΠΊΠΈ.

2. 2. Π“Π΅Π½Π΅Ρ€Π°Ρ‚ΠΎΡ€ ΠΏΠ΅Ρ€Π΅ΠΌΠ΅Π½Π½ΠΎΠ³ΠΎ Ρ‚ΠΎΠΊΠ°. Врансформатор. ΠŸΡ€ΠΎΠΈΠ·Π²ΠΎΠ΄ΡΡ‚Π²ΠΎ, ΠΏΠ΅Ρ€Π΅Π΄Π°Ρ‡Π° ΠΈ ΠΈΡΠΏΠΎΠ»ΡŒΠ·ΠΎΠ²Π°Π½ΠΈΠ΅ элСктричСской энСргии.

3. 3. (Π—Π°ΠΊΠΎΠ½ Π½Π° ΠΏΡ€ΠΈΠΌΠ΅Π½Π΅Π½ΠΈΠ΅ Π·Π°ΠΊΠΎΠ½Π° сохранСния ΠΈΠΌΠΏΡƒΠ»ΡŒΡΠ°).

Π Π°Π±ΠΎΡ‚Π° Π² Ρ‚Π΅Ρ€ΠΌΠΎΠ΄ΠΈΠ½Π°ΠΌΠΈΠΊΠ΅ ΠΈΠ»ΠΈ Ρ€Π°Π±ΠΎΡ‚Π° ΠΏΡ€ΠΈ ΠΈΠ·ΠΌΠ΅Π½Π΅Π½ΠΈΠΈ объСма Π³Π°Π·Π°: Π Π°Π±ΠΎΡ‚Π° ΠΏΡ€ΠΈ ΠΈΠ·ΠΎΠ±Π°Ρ€Π½ΠΎΠΌ Ρ€Π°ΡΡˆΠΈΡ€Π΅Π½ΠΈΠΈ Π³Π°Π·Π°. А=F * l = p* S* l = p* V, Π³Π΄Π΅ Ρ€ — Π΄Π°Π²Π»Π΅Π½ΠΈΠ΅ Π³Π°Π·Π°, V — ΠΈΠ·ΠΌΠ΅Π½Π΅Π½ΠΈΠ΅ Π΅Π³ΠΎ объСма.

Π˜Π·ΠΎΡ…ΠΎΡ€ΠΈΡ‡Π΅ΡΠΊΠΈΠΉ: V=0 => A=0, U =Q.

Π˜Π·ΠΎΡ‚Π΅Ρ€ΠΌΠΈΡ‡Π΅ΡΠΊΠΈΠΉ Π’=0 => U=0 Q=A.

Π˜Π·ΠΎΠ±Π°Ρ€ΠΈΡ‡Π΅ΡΠΊΠΈΠΉ Q= U+A.

АдиабатичСский Q=0 (быстро) — U=A (работая ΠΎΡΡ‚Ρ‹Π²Π°ΡŽ).

ΠŸΠ΅Ρ€Π²Ρ‹ΠΉ Π·Π°ΠΊΠΎΠ½ Ρ‚Π΅Ρ€ΠΌΠΎΠ΄ΠΈΠ½Π°ΠΌΠΈΠΊΠΈ ΡƒΡ‚Π²Π΅Ρ€ΠΆΠ΄Π°Π΅Ρ‚. Π§Ρ‚ΠΎ всякоС Ρ‚Π΅Π»ΠΎ ΠΎΠ±Π»Π°Π΄Π°Π΅Ρ‚ Π²Π½ΡƒΡ‚Ρ€Π΅Π½Π½Π΅ΠΉ энСргиСй U, ΠΏΡ€ΠΈΡ‡Π΅ΠΌ внутрСнняя энСргия ΠΌΠΎΠΆΠ΅Ρ‚ ΡƒΠΌΠ΅Π½ΡŒΡˆΠ°Ρ‚ΡŒΡΡ, Ссли Ρ‚Π΅Π»ΠΎ ΡΠΎΠ²Π΅Ρ€ΡˆΠ°Π΅Ρ‚ Ρ€Π°Π±ΠΎΡ‚Ρƒ А, ΠΈ ΡƒΠ²Π΅Π»ΠΈΡ‡ΠΈΠ²Π°Ρ‚ΡŒΡΡ, Ссли Π΅ΠΌΡƒ ΡΠΎΠΎΠ±Ρ‰Π°ΡŽΡ‚ Ρ‚Π΅ΠΏΠ»ΠΎΡ‚Ρƒ Q. U = Q-A, Π³Π΄Π΅, А — это Ρ€Π°Π±ΠΎΡ‚Π° ΡΠΎΠ²Π΅Ρ€ΡˆΠ°Π΅ΠΌΠ°Ρ самой тСрмодинамичСской систСмой Π½Π°Π΄ внСшними Ρ‚Π΅Π»Π°ΠΌΠΈ.

Π’ Π½Π΅ΠΈΠ·ΠΎΠ»ΠΈΡ€ΠΎΠ²Π°Π½Π½ΠΎΠΉ тСрмодинамичСской систСмС ΠΈΠ·ΠΌΠ΅Π½Π΅Π½ΠΈΠ΅ Π²Π½ΡƒΡ‚Ρ€Π΅Π½Π½Π΅ΠΉ энСргии Π΄Π΅Π»ΡŒΡ‚Π° U Ρ€Π°Π²Π½ΠΎ разности ΠΌΠ΅ΠΆΠ΄Ρƒ ΠΏΠΎΠ»ΡƒΡ‡Π΅Π½Π½Ρ‹ΠΌ количСством Ρ‚Π΅ΠΏΠ»ΠΎΡ‚Ρ‹ Q ΠΈ Ρ€Π°Π±ΠΎΡ‚ΠΎΠΉ А, ΡΠΎΠ²Π΅Ρ€ΡˆΠ°Π΅ΠΌΠΎΠΉ самой систСмой.

Или.

Π’ Π½Π΅ΠΈΠ·ΠΎΠ»ΠΈΡ€ΠΎΠ²Π°Π½Π½ΠΎΠΉ тСрмодинамичСской систСмС ΠΈΠ·ΠΌΠ΅Π½Π΅Π½ΠΈΠ΅ Π²Π½ΡƒΡ‚Ρ€Π΅Π½Π½Π΅ΠΉ энСргии Π΄Π΅Π»ΡŒΡ‚Π° U Ρ€Π°Π²Π½ΠΎ суммС количСства Ρ‚Π΅ΠΏΠ»ΠΎΡ‚Ρ‹ Q, ΠΏΠ΅Ρ€Π΅Π΄Π°Π½Π½ΠΎΠΉ систСмС, ΠΈ Ρ€Π°Π±ΠΎΡ‚Ρ‹, А Π²Π½Π΅ΡˆΠ½ΠΈΡ… сил: Π΄Π΅Π»ΡŒΡ‚Π° U=Q+A1; А1=-А.

ΠŸΠ΅Ρ€Π²Ρ‹ΠΉ Π·Π°ΠΊΠΎΠ½ Ρ‚Π΅Ρ€ΠΌΠΎΠ΄ΠΈΠ½Π°ΠΌΠΈΠΊΠΈ — это всСобщий Π·Π°ΠΊΠΎΠ½ сохранСния ΠΈ ΠΏΡ€Π΅Π²Ρ€Π°Ρ‰Π΅Π½ΠΈΡ энСргии. Из Π½Π΅Π³ΠΎ слСдуСт, Ρ‡Ρ‚ΠΎ Ссли внутрСнняя энСргия постоянна ΠΈ Ρ‚Π΅Π»ΠΎ Π½Π΅ ΠΏΠΎΡƒΡ‡Π°Π΅Ρ‚ ΠΈ Π½Π΅ ΠΎΡ‚Π΄Π°Π΅Ρ‚ Ρ‚Π΅ΠΏΠ»Π°, Ρ‚ΠΎ ΠΎΠ½ΠΎ Π½Π΅ ΠΌΠΎΠΆΠ΅Ρ‚ ΡΠΎΠ²Π΅Ρ€ΡˆΠΈΡ‚ΡŒ Ρ€Π°Π±ΠΎΡ‚Ρƒ Ρ€Π°Π²Π½ΡƒΡŽ 0. Π’Π°ΠΊΠΈΠΌ ΠΎΠ±Ρ€Π°Π·ΠΎΠΌ, нСльзя ΠΏΠΎΠ»ΡƒΡ‡ΠΈΡ‚ΡŒ Ρ€Π°Π±ΠΎΡ‚Ρƒ ΠΈΠ· Π½ΠΈΡ‡Π΅Π³ΠΎ ΠΈΠ»ΠΈ ΠΏΡ€Π΅Π²Ρ€Π°Ρ‚ΠΈΡ‚ΡŒ Π΅Π΅ Π½ΠΈ Π²ΠΎ Ρ‡Ρ‚ΠΎ.

Π‘ ΠΏΠΎΠ·ΠΈΡ†ΠΈΠΈ молСкулярно — кинСтичСской Ρ‚Π΅ΠΎΡ€ΠΈΠΈ внутрСнняя энСргияэто сумма ΠΏΠΎΡ‚Π΅Π½Ρ†ΠΈΠ°Π»ΡŒΠ½ΠΎΠΉ энСргии взаимодСйствия частиц, ΡΠΎΡΡ‚Π°Π²Π»ΡΡŽΡ‰ΠΈΡ… Ρ‚Π΅Π»ΠΎ, ΠΈ ΠΊΠΈΠ½Π΅Ρ‚ичСской энСргии ΠΈΡ… Π±Π΅ΡΠΏΠΎΡ€ΡΠ΄ΠΎΡ‡Π½ΠΎΠ³ΠΎ Ρ‚Π΅ΠΏΠ»ΠΎΠ²ΠΎΠ³ΠΎ двиТСния. ВнутрСнняя энСргия — это Π΅Ρ‰Π΅ ΠΎΠ΄ΠΈΠ½ Π²ΠΈΠ΄ энСргии, ΠΏΡ€ΠΈΡ‡Π΅ΠΌ эта энСргия сущСствуСт всСгда, ΠΎΠ½Π° Π½Π΅ Π·Π°Π²ΠΈΡΠΈΡ‚ ΠΎΡ‚ ΠΌΠ΅Ρ…аничСского двиТСния Ρ‚Π΅Π»Π°, Π½ΠΈ ΠΎΡ‚ ΠΏΠΎΠ»ΠΎΠΆΠ΅Π½ΠΈΡ этого Ρ‚Π΅Π»Π° ΠΎΡ‚Π½ΠΎΡΠΈΡ‚Π΅Π»ΡŒΠ½ΠΎ Π΄Ρ€ΡƒΠ³ΠΈΡ… Ρ‚Π΅Π». Она зависит ΠΎΡ‚ Ρ‚Π΅ΠΌΠΏΠ΅Ρ€Π°Ρ‚ΡƒΡ€Ρ‹, давлСния ΠΈ ΠΎΠ±ΡŠΠ΅ΠΌΠ°. ΠœΠΎΠ»Π΅ΠΊΡƒΠ»Ρ‹ — это слоТныС частицы, состоящиС ΠΈΠ· Π²Π·Π°ΠΈΠΌΠΎΠ΄Π΅ΠΉΡΡ‚Π²ΡƒΡŽΡ‰ΠΈΡ… ΠΈ ΠΊΠΎΠ»Π΅Π±Π»ΡŽΡ‰ΠΈΡ…ся Π°Ρ‚ΠΎΠΌΠΎΠΌ, ΠΊΠΎΡ‚ΠΎΡ€Ρ‹Π΅ Ρ‚ΠΎΠΆΠ΅ ΠΎΠ±Π»Π°Π΄Π°ΡŽΡ‚ энСргиСй. Π­ΠΉΠ½ΡˆΡ‚Π΅ΠΉΠ½ Π΄ΠΎΠΊΠ°Π·Π°Π», Ρ‡Ρ‚ΠΎ ΠΏΠΎΠ»Π½ΡƒΡŽ ΡΠ½Π΅Ρ€Π³ΠΈΡŽ Ρ‚Π΅Π»Π° ΠΌΠΎΠΆΠ½ΠΎ Π²Ρ‹Ρ‡ΠΈΡΠ»ΠΈΡ‚ΡŒ ΠΏΠΎ Ρ„ΠΎΡ€ΠΌΡƒΠ»Π΅ Π•=m*с2, Π³Π΄Π΅ с=300 000 ΠΊΠΌ/с. Π˜Π·ΠΌΠ΅Π½ΠΈΡ‚ΡŒ Π΅Π΅ ΠΌΠΎΠΆΠ½ΠΎ ΡΠΎΠ²Π΅Ρ€ΡˆΠ΅Π½ΠΈΠ΅ΠΌ мСханичСской Ρ€Π°Π±ΠΎΡ‚Ρ‹ ΠΈΠ»ΠΈ Ρ‚Π΅ΠΏΠ»ΠΎΠΏΠ΅Ρ€Π΅Π΄Π°Ρ‡Π΅ΠΉ. ВнутрСнняя энСргия идСального Π³Π°Π·Π°: Ссли ΠΏΠΎΡ‚Π΅Π½Ρ†ΠΈΠ°Π»ΡŒΠ½Π°Ρ энСргия взаимодСйствия ΠΌΠΎΠ»Π΅ΠΊΡƒΠ» Ρ€Π°Π²Π½Π° 0, внутрСнняя энСргия идСального Π³Π°Π·Π° Ρ€Π°Π²Π½Π° суммС кинСтичСских энСргий хаотичСского Ρ‚Π΅ΠΏΠ»ΠΎΠ²ΠΎΠ³ΠΎ двиТСния всСх ΠΌΠΎΠ»Π΅ΠΊΡƒΠ» U=N*E=v*Na=3/2m/M RT, Π³Π΄Π΅ v — количСство вСщСства Π² ΠΌΠΎΠ»ΡΡ…, ΠΊ — постоянная Π‘ΠΎΠ»ΡŒΡ†ΠΌΠ°Π½Π°, Π’ — Π°Π±ΡΠΎΠ»ΡŽΡ‚Π½Π°Ρ Ρ‚Π΅ΠΌΠΏΠ΅Ρ€Π°Ρ‚ΡƒΡ€Π°, m — масса Π³Π°Π·Π°, М — молярная масса Π³Π°Π·Π°, R — ΡƒΠ½ΠΈΠ²Π΅Ρ€ΡΠ°Π»ΡŒΠ½Π°Ρ газовая постоянная. P*V=m/M RT => U =3/2Pv.

2. 2. Π“Π΅Π½Π΅Ρ€Π°Ρ‚ΠΎΡ€ ΠΏΠ΅Ρ€Π΅ΠΌΠ΅Π½Π½ΠΎΠ³ΠΎ Ρ‚ΠΎΠΊΠ°. Врансформатор. ΠŸΡ€ΠΎΠΈΠ·Π²ΠΎΠ΄ΡΡ‚Π²ΠΎ, ΠΏΠ΅Ρ€Π΅Π΄Π°Ρ‡Π° ΠΈ ΠΈΡΠΏΠΎΠ»ΡŒΠ·ΠΎΠ²Π°Π½ΠΈΠ΅ элСктричСской энСргии.

БоврСмСнная Ρ‚Π΅Ρ…Π½ΠΈΠΊΠ° нСмыслима Π±Π΅Π· ΠΈΠ½Π΄ΡƒΠΊΡ†ΠΈΠΎΠ½Π½Ρ‹Ρ… Π³Π΅Π½Π΅Ρ€Π°Ρ‚ΠΎΡ€ΠΎΠ² ΠΏΠ΅Ρ€Π΅ΠΌΠ΅Π½Π½ΠΎΠ³ΠΎ Ρ‚ΠΎΠΊΠ°. Π’ ΡΡ‚ΠΈΡ… Π³Π΅Π½Π΅Ρ€Π°Ρ‚ΠΎΡ€Π°Ρ… мСханичСская энСргия прСвращаСтся Π² ΡΠ»Π΅ΠΊΡ‚Ρ€ΠΈΡ‡Π΅ΡΠΊΡƒΡŽ, Π° ΠΈΡ… Π΄Π΅ΠΉΡΡ‚Π²ΠΈΠ΅ основано Π½Π° ΡΠ»Π΅ΠΊΡ‚Ρ€ΠΎΠΌΠ°Π³Π½ΠΈΡ‚Π½ΠΎΠΉ ΠΈΠ½Π΄ΡƒΠΊΡ†ΠΈΠΈ. ΠŸΠΎΡ‚ΠΎΠΊ ΠΌΠ°Π³Π½ΠΈΡ‚Π½ΠΎΠΉ ΠΈΠ½Π΄ΡƒΠΊΡ†ΠΈΠΈ Π€, ΠΏΡ€ΠΎΠ½ΠΈΠ·Ρ‹Π²Π°ΡŽΡ‰ΠΈΠΉ ΠΏΡ€ΠΎΠ²ΠΎΠ»ΠΎΡ‡Π½ΡƒΡŽ Ρ€Π°ΠΌΠΊΡƒ ΠΏΠ»ΠΎΡ‰Π°Π΄ΡŒΡŽ S, ΠΏΡ€ΠΎΠΏΠΎΡ€Ρ†ΠΈΠΎΠ½Π°Π»Π΅Π½ ΠΌΠ°Π³Π½ΠΈΡ‚Π½ΠΎΠΉ ΠΈΠ½Π΄ΡƒΠΊΡ†ΠΈΠΈ Π’, ΠΏΠ»ΠΎΡ‰Π°Π΄ΠΈ Ρ€Π°ΠΌΠΊΠΈ ΠΈ ΠΊΠΎΡΠΈΠ½ΡƒΡΠ° ΡƒΠ³Π»Π° Π°Π»ΡŒΡ„Π° ΠΌΠ΅ΠΆΠ΄Ρƒ Π½ΠΎΡ€ΠΌΠ°Π»ΡŒΡŽ ΠΊ Ρ€Π°ΠΌΠΊΠ΅ ΠΈ Π²Π΅ΠΊΡ‚ΠΎΡ€ΠΎΠΌ ΠΌΠ°Π³Π½ΠΈΡ‚Π½ΠΎΠΉ ΠΈΠ½Π΄ΡƒΠΊΡ†ΠΈΠΈ Π’:

Π€=Π’*S*cos (Π°Π»ΡŒΡ„Π°). ΠŸΡ€ΠΈ Π²Ρ€Π°Ρ‰Π΅Π½ΠΈΠΈ Ρ€Π°ΠΌΠΊΠΈ с ΠΏΠΎΡΡ‚оянной ΡƒΠ³Π»ΠΎΠ²ΠΎΠΉ ΡΠΊΠΎΡ€ΠΎΡΡ‚ΡŒΡŽ, ΡƒΠ³ΠΎΠ» Π°Π»ΡŒΡ„Π° увСличиваСтся ΠΏΡ€ΡΠΌΠΎΠΏΡ€ΠΎΠΏΠΎΡ€Ρ†ΠΈΠΎΠ½Π°Π»ΡŒΠ½ΠΎ Π²Ρ€Π΅ΠΌΠ΅Π½ΠΈ Π°Π»ΡŒΡ„Π°=wt. По Π·Π°ΠΊΠΎΠ½Ρƒ элСктромагнитной ΠΈΠ½Π΄ΡƒΠΊΡ†ΠΈΠΈ Π­Π”Π‘ ΠΈΠ½Π΄ΡƒΠΊΡ†ΠΈΠΈ Π² Ρ€Π°ΠΌΠΊΠ΅ Ρ€Π°Π²Π½Π° скорости измСнСния ΠΏΠΎΡ‚ΠΎΠΊΠ° ΠΌΠ°Π³Π½ΠΈΡ‚Π½ΠΎΠΉ ΠΈΠ½Π΄ΡƒΠΊΡ†ΠΈΠΈ, взятой со Π·Π½Π°ΠΊΠΎΠΌ минус, Ρ‚. Π΅. ΠΏΡ€ΠΎΠΈΠ·Π²ΠΎΠ΄Π½ΠΎΠΉ ΠΏΠΎΡ‚ΠΎΠΊΠ° ΠΌΠ°Π³Π½ΠΈΡ‚Π½ΠΎΠΉ ΠΈΠ½Π΄ΡƒΠΊΡ†ΠΈΠΈ ΠΏΠΎ Π²Ρ€Π΅ΠΌΠ΅Π½ΠΈ: Π΅=-dΠ€/dt = -BwSsinwt=e1sinwt, Π³Π΄Π΅ Π΅1=BSw — Π°ΠΌΠΏΠ»ΠΈΡ‚ΡƒΠ΄Π° Π­Π”Π‘ ΠΈΠ½Π΄ΡƒΠΊΡ†ΠΈΠΈ. Π’Π°ΠΊ ΠΊΠ°ΠΊ угловая ΡΠΊΠΎΡ€ΠΎΡΡ‚ΡŒ w=2Пv=2П/Π’? Ρ‚ΠΎ ΠΏΠΎΠ»ΡƒΡ‡Π°Π΅ΠΌ Π΅=Π΅1*sin (2П/Π’)*t. Π§Ρ‚ΠΎΠ±Ρ‹ ΠΏΠΎΠ»ΡƒΡ‡ΠΈΡ‚ΡŒ большой ΠΌΠ°Π³Π½ΠΈΡ‚Π½Ρ‹ΠΉ ΠΏΠΎΡ‚ΠΎΠΊ, Π² Π³Π΅Π½Π΅Ρ€Π°Ρ‚ΠΎΡ€Π°Ρ… ΠΏΡ€ΠΈΠΌΠ΅Π½ΡΡŽΡ‚ ΠΏΠΎΡ‡Ρ‚ΠΈ Π·Π°ΠΌΠΊΠ½ΡƒΡ‚ΡƒΡŽ ΠΌΠ°Π³Π½ΠΈΡ‚Π½ΡƒΡŽ систСму, ΡΠΎΡΡ‚ΠΎΡΡ‰ΡƒΡŽ ΠΈΠ· Π΄Π²ΡƒΡ… сСрдСчников, сдСланных ΠΈΠ· ΡΠΏΠ΅Ρ†ΠΈΠ°Π»ΡŒΠ½ΠΎΠΉ элСктротСхничСской стали. ΠžΠ±ΠΌΠΎΡ‚ΠΊΠΈ, ΡΠΎΠ·Π΄Π°ΡŽΡ‰ΠΈΠ΅ ΠΌΠ°Π³Π½ΠΈΡ‚Π½ΠΎΠ΅ ΠΏΠΎΠ»Π΅, располоТСны Π² ΠΏΠ°Π·Π°Ρ… ΠΎΠ΄Π½ΠΎΠ³ΠΎ ΠΈΠ· ΡΠ΅Ρ€Π΄Π΅Ρ‡Π½ΠΈΠΊΠΎΠ², Π° ΠΎΠ±ΠΌΠΎΡ‚ΠΊΠΈ, Π² ΠΊΠΎΡ‚ΠΎΡ€Ρ‹Ρ… индуцируСтся Π­Π”Π‘, — Π² ΠΏΠ°Π·Π°Ρ… Π΄Ρ€ΡƒΠ³ΠΎΠ³ΠΎ. Один ΠΈΠ· ΡΠ΅Ρ€Π΄Π΅Ρ‡Π½ΠΈΠΊΠΎΠ² вмСстС со ΡΠ²ΠΎΠ΅ΠΉ ΠΎΠ±ΠΌΠΎΡ‚ΠΊΠΎΠΉ вращаСтся Π²ΠΎΠΊΡ€ΡƒΠ³ Π³ΠΎΡ€ΠΈΠ·ΠΎΠ½Ρ‚Π°Π»ΡŒΠ½ΠΎΠΉ ΠΈΠ»ΠΈ Π²Π΅Ρ€Ρ‚ΠΈΠΊΠ°Π»ΡŒΠ½ΠΎΠΉ оси. ΠŸΠΎΡΡ‚ΠΎΠΌΡƒ Π΅Π³ΠΎ Π½Π°Π·Ρ‹Π²Π°ΡŽΡ‚ Ρ€ΠΎΡ‚ΠΎΡ€ΠΎΠΌ, Π΄Ρ€ΡƒΠ³ΠΎΠΉ статором. Π’ΠΎΠΊ ΠΊ Ρ€ΠΎΡ‚ΠΎΡ€Ρƒ подводят с ΠΏΠΎΠΌΠΎΡ‰ΡŒΡŽ ΡΠΊΠΎΠ»ΡŒΠ·ΡΡ‰ΠΈΡ… ΠΊΠΎΠ½Ρ‚Π°ΠΊΡ‚ΠΎΠ². Для этого Π΅Π³ΠΎ ΡΠ½Π°Π±ΠΆΠ°ΡŽΡ‚ ΠΊΠΎΠ½Ρ‚Π°ΠΊΡ‚Π½Ρ‹ΠΌΠΈ ΠΊΠΎΠ»ΡŒΡ†Π°ΠΌΠΈ, присоСдинСнными ΠΊ ΠΊΠΎΠ½Ρ†Π°ΠΌ ΠΎΠ±ΠΌΠΎΡ‚ΠΊΠΈ. НСподвиТныС пластины — Ρ‰Π΅Ρ‚ΠΊΠΈ — ΠΏΡ€ΠΈΠΆΠ°Ρ‚Ρ‹ ΠΊ ΠΊΠΎΠ»ΡŒΡ†Π°ΠΌ ΠΈ ΠΎΡΡƒΡ‰Π΅ΡΡ‚Π²Π»ΡΡŽΡ‚ связь ΠΎΠ±ΠΌΠΎΡ‚ΠΊΠΈ Ρ€ΠΎΡ‚ΠΎΡ€Π° с Π²Π½Π΅ΡˆΠ½Π΅ΠΉ Ρ†Π΅ΠΏΡŒΡŽ. Π‘ΠΈΠ»Π° Ρ‚ΠΎΠΊΠ° Π² ΠΎΠ±ΠΌΠΎΡ‚ΠΊΠ°Ρ… элСктромагнита, ΡΠΎΠ·Π΄Π°ΡŽΡ‰Π΅Π³ΠΎ ΠΌΠ°Π³Π½ΠΈΡ‚Π½ΠΎΠ΅ ΠΏΠΎΠ»Π΅, Π·Π½Π°Ρ‡ΠΈΡ‚Π΅Π»ΡŒΠ½ΠΎ мСньшС силы Ρ‚ΠΎΠΊΠ°, ΠΎΡ‚Π΄Π°Π²Π°Π΅ΠΌΠΎΠ³ΠΎ Π³Π΅Π½Π΅Ρ€Π°Ρ‚ΠΎΡ€ΠΎΠΌ Π²ΠΎ Π²Π½Π΅ΡˆΠ½ΡŽΡŽ Ρ†Π΅ΠΏΡŒ. ΠŸΠΎΡΡ‚ΠΎΠΌΡƒ Π³Π΅Π½Π΅Ρ€ΠΈΡ€ΡƒΠ΅ΠΌΡ‹ΠΉ Ρ‚ΠΎΠΊ ΡƒΠ΄ΠΎΠ±Π½Π΅Π΅ ΡΠ½ΠΈΠΌΠ°Ρ‚ΡŒ с Π½Π΅ΠΏΠΎΠ΄Π²ΠΈΠΆΠ½Ρ‹Ρ… ΠΎΠ±ΠΌΠΎΡ‚ΠΎΠΊ, Π° Ρ‡Π΅Ρ€Π΅Π· ΡΠΊΠΎΠ»ΡŒΠ·ΡΡ‰ΠΈΠ΅ ΠΊΠΎ7Ρ‚Π°ΠΊΡ‚Ρ‹ ΠΏΠΎΠ΄Π²ΠΎΠ΄ΠΈΡ‚ΡŒ ΡΡ€Π°Π²Π½ΠΈΡ‚Π΅Π»ΡŒΠ½ΠΎ слабый Ρ‚ΠΎΠΊ ΠΊ Π²Ρ€Π°Ρ‰Π°ΡŽΡ‰Π΅ΠΌΡƒΡΡ элСктромагниту. Π­Ρ‚ΠΎΡ‚ Ρ‚ΠΎΠΊ вырабатываСтся ΠΎΡ‚Π΄Π΅Π»ΡŒΠ½Ρ‹ΠΌ Π³Π΅Π½Π΅Ρ€Π°Ρ‚ΠΎΡ€ΠΎΠΌ постоянного Ρ‚ΠΎΠΊΠ° (Π²ΠΎΠ·Π±ΡƒΠ΄ΠΈΡ‚Π΅Π»Π΅ΠΌ), располоТСнным Π½Π° Ρ‚ΠΎΠΌ ΠΆΠ΅ Π²Π°Π»Ρƒ. Если Π³Π΅Π½Π΅Ρ€Π°Ρ‚ΠΎΡ€ приводится Π² Π΄Π΅ΠΉΡΡ‚Π²ΠΈΠ΅ ΠΏΠ°Ρ€ΠΎΠ²ΠΎΠΉ ΠΈΠ»ΠΈ Π³Π°Π·ΠΎΠ²ΠΎΠΉ Ρ‚ΡƒΡ€Π±ΠΈΠ½ΠΎΠΉ, Ρ‚ΠΎ ΠΎΠ½ Π½Π°Π·Ρ‹Π²Π°Π΅Ρ‚ся Ρ‚ΡƒΡ€Π±ΠΎΠ³Π΅Π½Π΅Ρ€Π°Ρ‚ΠΎΡ€ΠΎΠΌ. Если Ρ€ΠΎΡ‚ΠΎΡ€ вращаСтся ΠΎΡ‚ Π”Π’Π‘, Ρ‚ΠΎ Π΄ΠΈΠ·Π΅Π»ΡŒ — Π³Π΅Π½Π΅Ρ€Π°Ρ‚ΠΎΡ€.

Π’Ρ‹Π³ΠΎΠ΄Π½ΠΎΠ΅ ΠΎΡ‚Π»ΠΈΡ‡ΠΈΠ΅ ΠΏΠ΅Ρ€Π΅ΠΌΠ΅Π½Π½ΠΎΠ³ΠΎ Ρ‚ΠΎΠΊΠ° ΠΎΡ‚ ΠΏΠΎΡΡ‚оянного, Π² Ρ‚ΠΎΠΌ, Ρ‡Ρ‚ΠΎ ΠΌΠΎΠΆΠ½ΠΎ ΡΡ€Π°Π²Π½ΠΈΡ‚Π΅Π»ΡŒΠ½ΠΎ Π»Π΅Π³ΠΊΠΎ ΠΈΠ·ΠΌΠ΅Π½ΡΡ‚ΡŒ Π΅Π³ΠΎ силу. Аппараты, ΠΏΡ€Π΅ΠΎΠ±Ρ€Π°Π·ΡƒΡŽΡ‰ΠΈΠ΅ ΠΏΠ΅Ρ€Π΅ΠΌΠ΅Π½Π½Ρ‹ΠΉ Ρ‚ΠΎΠΊ ΠΎΠ΄Π½ΠΎΠ³ΠΎ напряТСния Π² Π΄Ρ€ΡƒΠ³ΠΎΠ΅ — Π½Π°Π·Ρ‹Π²Π°ΡŽΡ‚ΡΡ элСктричСскими трансформаторами. Бостоит ΠΈΠ· Π½Π΅ΡΠΊΠΎΠ»ΡŒΠΊΠΈΡ… ΠΊΠ°Ρ‚ΡƒΡˆΠ΅ΠΊ ΠΈΠ·ΠΎΠ»ΠΈΡ€ΠΎΠ²Π°Π½Π½ΠΎΠ³ΠΎ ΠΏΡ€ΠΎΠ²ΠΎΠ΄Π°, Ρ€Π°Π·ΠΌΠ΅Ρ‰Π΅Π½Π½Ρ‹Ρ… Π½Π° ΠΌΠ°Π³Π½ΠΈΡ‚ΠΎΠΏΡ€ΠΎΠ²ΠΎΠ΄Π΅ ΠΈΠ· Ρ‚ΠΎΠ½ΠΊΠΈΡ… пластин ΡΠΏΠ΅Ρ†ΠΈΠ°Π»ΡŒΠ½ΠΎ элСктротСхничСской стали. ΠŸΠ΅Ρ€Π΅ΠΌΠ΅Π½Π½Ρ‹ΠΉ Ρ‚ΠΎΠΊ, Ρ‚Π΅ΠΊΡƒΡ‰ΠΈΠΉ ΠΏΠΎ ΠΎΠ΄Π½ΠΎΠΉ ΠΈΠ· ΠΎΠ±ΠΌΠΎΡ‚ΠΎΠΊ (ΠΏΠ΅Ρ€Π²ΠΈΡ‡Π½ΠΎΠΉ). Π‘ΠΎΠ·Π΄Π°Π΅Ρ‚ Π²ΠΎΠΊΡ€ΡƒΠ³ Π½Π΅Π΅ ΠΈ Π² ΠΌΠ°Π³Π½ΠΈΡ‚ΠΎΠΏΡ€ΠΎΠ²ΠΎΠ΄Π΅ ΠΏΠ΅Ρ€Π΅ΠΌΠ΅Π½Π½ΠΎΠΉ ΠΌΠ°Π³Π½ΠΈΡ‚Π½ΠΎΠ΅ ΠΏΠΎΠ»Π΅, ΠΏΠ΅Ρ€Π΅ΡΠ΅ΠΊΠ°ΡŽΡ‰Π΅Π΅ Π²ΠΈΡ‚ΠΊΠΈ Π΄Ρ€ΡƒΠ³ΠΎΠΉ (Π²Ρ‚ΠΎΡ€ΠΈΡ‡Π½ΠΎΠΉ), Π²ΠΎΠ·Π±ΡƒΠΆΠ΄Π°Π΅Ρ‚ Π² Π½Π΅ΠΉ ΠΏΠ΅Ρ€Π΅ΠΌΠ΅Π½Π½ΡƒΡŽ ΡΠ»Π΅ΠΊΡ‚Ρ€ΠΎΠ΄Π²ΠΈΠΆΡƒΡ‰ΡƒΡŽ силу. Если ΠΎΠ±Π΅ ΠΎΠ±ΠΌΠΎΡ‚ΠΊΠΈ ΠΈΠΌΠ΅ΡŽΡ‚ Ρ€Π°Π²Π½ΠΎΠ΅ количСство Π²ΠΈΡ‚ΠΊΠΎΠ², Ρ‚ΠΎ Π² Π½Π΅ΠΉ навСдСтся Ρ‚Π°ΠΊΠΎΠ΅ ΠΆΠ΅ напряТСниС, ΠΊΠ°ΠΊΠΎΠ΅ Π² ΠΏΠ΅Ρ€Π²ΠΈΡ‡Π½ΠΎΠΉ. Если Π½Π΅ Ρ€Π°Π²Π½ΠΎΠ΅ количСство, Ρ‚ΠΎ Ρ‚рансформатор ΠΌΠΎΠΆΠ΅Ρ‚ Π±Ρ‹Ρ‚ΡŒ ΠΏΠΎΠ²Ρ‹ΡˆΠ°ΡŽΡ‰ΠΈΠΌ (Π²ΠΎ Π²Ρ‚ΠΎΡ€ΠΈΡ‡Π½ΠΎΠΉ ΠΎΠ±ΠΌΠΎΡ‚ΠΊΠ΅ большС Π²ΠΈΡ‚ΠΊΠΎΠ²), ΠΏΠΎΠ½ΠΈΠΆΠ°ΡŽΡ‰ΠΈΠΌ — Π½Π°ΠΎΠ±ΠΎΡ€ΠΎΡ‚. ДСйствиС основано Π½Π° ΡΠ²Π»Π΅Π½ΠΈΠΈ элСктромагнитной ΠΈΠ½Π΄ΡƒΠΊΡ†ΠΈΠΈ. ΠŸΡ€ΠΈ ΠΏΡ€ΠΎΡ…ΠΎΠΆΠ΄Π΅Π½ΠΈΠΈ ΠΏΠ΅Ρ€Π΅ΠΌΠ΅Π½Π½ΠΎΠ³ΠΎ Ρ‚ΠΎΠΊΠ° ΠΏΠΎ ΠΏΠ΅Ρ€Π²ΠΈΡ‡Π½ΠΎΠΉ ΠΎΠ±ΠΌΠΎΡ‚ΠΊΠ΅ Π² ΡΠ΅Ρ€Π΄Π΅Ρ‡Π½ΠΈΠΊΠ΅ появляСтся ΠΏΠ΅Ρ€Π΅ΠΌΠ΅Π½Π½Ρ‹ΠΉ ΠΌΠ°Π³Π½ΠΈΡ‚Π½Ρ‹ΠΉ ΠΏΠΎΡ‚ΠΎΠΊ, ΠΊΠΎΡ‚ΠΎΡ€Ρ‹ΠΉ Π²ΠΎΠ·Π±ΡƒΠΆΠ΄Π°Π΅Ρ‚ Π­Π”Π‘ ΠΈΠ½Π΄ΡƒΠΊΡ†ΠΈΠΈ Π² ΠΊΠ°ΠΆΠ΄ΠΎΠΉ ΠΎΠ±ΠΌΠΎΡ‚ΠΊΠ΅. Π‘Π΅Ρ€Π΄Π΅Ρ‡Π½ΠΈΠΊ ΠΈΠ· Ρ‚рансформаторной стали ΠΊΠΎΠ½Ρ†Π΅Π½Ρ‚Ρ€ΠΈΡ€ΡƒΠ΅Ρ‚ ΠΌΠ°Π³Π½ΠΈΡ‚Π½ΠΎΠ΅ ΠΏΠΎΠ»Π΅, Ρ‚Π°ΠΊ Ρ‡Ρ‚ΠΎ ΠΌΠ°Π³Π½ΠΈΡ‚Π½Ρ‹ΠΉ ΠΏΠΎΡ‚ΠΎΠΊ практичСски сущСствуСт Ρ‚ΠΎΠ»ΡŒΠΊΠΎ Π²Π½ΡƒΡ‚Ρ€ΠΈ сСрдСчника ΠΈ ΠΎΠ΄ΠΈΠ½Π°ΠΊΠΎΠ² Π²ΠΎ Π²ΡΠ΅Ρ… Π΅Π³ΠΎ сСчСниях.

U1/U2 = I2/I1, U1/U2 = E1/E2 = n1/n2 = К, Π³Π΄Π΅ К — коэффициСнт трансформации, ΠΏΡ€ΠΈ ΠΊ>0 -ΠΏΠΎΠ½ΠΈΠΆΠ°ΡŽΡ‰ΠΈΠΉ… ΠŸΠΈΡ€ Ρ€Π°Π·ΠΎΠΌΠΊΠ½ΡƒΡ‚ΠΎΠΉ Π²Ρ‚ΠΎΡ€ΠΈΡ‡Π½ΠΎΠΉ ΠΎΠ±ΠΌΠΎΡ‚ΠΊΠΈ трансформатор с ΠΌΠ°Π»Ρ‹ΠΌ Π°ΠΊΡ‚ΠΈΠ²Π½Ρ‹ΠΌ сопротивлСниСм ΠΏΠ΅Ρ€Π²ΠΈΡ‡Π½ΠΎΠΉ ΠΎΠ±ΠΌΠΎΡ‚ΠΊΠΈ ΠΏΠΎΡ‡Ρ‚ΠΈ Π½Π΅ ΠΏΠΎΡ‚рСбляСт ΡΠ½Π΅Ρ€Π³ΠΈΡŽ ΠΈΠ· ΡΠ΅Ρ‚ΠΈ, Ρ‚Π°ΠΊ ΠΊΠ°ΠΊ Π²Π΅Π»ΠΈΠΊΠΎ ΠΈΠ½Π΄ΡƒΠΊΡ‚ΠΈΠ²Π½ΠΎΠ΅ сопротивлСниС Π½Π΅Π½Π°Π³Ρ€ΡƒΠΆΠ΅Π½Π½ΠΎΠΉ ΠΎΠ±ΠΌΠΎΡ‚ΠΊΠΈ трансформатора. Если ΠΊ ΠΊΠΎΠ½Ρ†Π°ΠΌ Π²Ρ‚ΠΎΡ€ΠΈΡ‡Π½ΠΎΠΉ ΠΎΠ±ΠΌΠΎΡ‚ΠΊΠΈ ΠΏΡ€ΠΈΡΠΎΠ΅Π΄ΠΈΠ½ΠΈΡ‚ΡŒ Ρ†Π΅ΠΏΡŒ, Ρ‚ΠΎ ΡΠΈΠ»Π° Ρ‚ΠΎΠΊΠ° Π²ΠΎ Π²Ρ‚ΠΎΡ€ΠΈΡ‡Π½ΠΎΠΉ ΠΎΠ±ΠΌΠΎΡ‚ΠΊΠ΅ ΡƒΠΆΠ΅ Π½Π΅ Π±ΡƒΠ΄Π΅Ρ‚ Ρ€Π°Π²Π½Π° 0. Появившийся Ρ‚ΠΎΠΊ создаСт Π² ΡΠ΅Ρ€Π΄Π΅Ρ‡Π½ΠΈΠΊΠ΅ свой ΠΏΠ΅Ρ€Π΅ΠΌΠ΅Π½Π½Ρ‹ΠΉ ΠΌΠ°Π³Π½ΠΈΡ‚Π½Ρ‹ΠΉ ΠΏΠΎΡ‚ΠΎΠΊ, ΠΊΠΎΡ‚ΠΎΡ€Ρ‹ΠΉ ΠΏΠΎ ΠΏΡ€Π°Π²ΠΈΠ»Ρƒ Π›Π΅Π½Ρ†Π° Π΄ΠΎΠ»ΠΆΠ΅Π½ ΡƒΠΌΠ΅Π½ΡŒΡˆΠΈΡ‚ΡŒ измСнСния ΠΌΠ°Π³Π½ΠΈΡ‚Π½ΠΎΠ³ΠΎ ΠΏΠΎΡ‚ΠΎΠΊΠ° Π² ΡΠ΅Ρ€Π΄Π΅Ρ‡Π½ΠΈΠΊΠ΅. Но ΡƒΠΌΠ΅Π½ΡŒΡˆΠ΅Π½ΠΈΠ΅ Π°ΠΌΠΏΠ»ΠΈΡ‚ΡƒΠ΄Ρ‹ ΠΏΠΎΡ‚ΠΎΠΊΠ° Π΄ΠΎΠ»ΠΆΠ½ΠΎ ΡƒΠΌΠ΅Π½ΡŒΡˆΠΈΡ‚ΡŒ Π­Π”Π‘. Однако это Π½Π΅Π²ΠΎΠ·ΠΌΠΎΠΆΠ½ΠΎ, Ρ‚Π°ΠΊ ΠΊΠ°ΠΊ ΠΌΠΎΠ΄ΡƒΠ»ΠΈ U1=e1. ΠŸΠΎΡΡ‚ΠΎΠΌΡƒ ΠΏΡ€ΠΈ Π·Π°ΠΌΡ‹ΠΊΠ°Π½ΠΈΠΈ Ρ†Π΅ΠΏΠΈ Π²Ρ‚ΠΎΡ€ΠΈΡ‡Π½ΠΎΠΉ ΠΎΠ±ΠΌΠΎΡ‚ΠΊΠΈ автоматичСски увСличиваСтся сила Ρ‚ΠΎΠΊΠ° Π² ΠΏΠ΅Ρ€Π²ΠΈΡ‡Π½ΠΎΠΉ. Π£Π²Π΅Π»ΠΈΡ‡Π΅Π½ΠΈΠ΅ силы Ρ‚ΠΎΠΊΠ° Π² ΠΏΠ΅Ρ€Π²ΠΈΡ‡Π½ΠΎΠΉ Ρ†Π΅ΠΏΠΈ (ΠΏΠΎ Π·Π°ΠΊΠΎΠ½Ρƒ сохранСния энСргии) ΡƒΠ²Π΅Π»ΠΈΡ‡ΠΈΡ‚ силу Ρ‚ΠΎΠΊΠ° Π²ΠΎ Π²Ρ‚ΠΎΡ€ΠΈΡ‡Π½ΠΎΠΉ.

Врансформаторы находят ΡˆΠΈΡ€ΠΎΠΊΠΎΠ΅ ΠΏΡ€ΠΈΠΌΠ΅Π½Π΅Π½ΠΈΠ΅ Π² ΠΏΡ€ΠΎΠΌΡ‹ΡˆΠ»Π΅Π½Π½ΠΎΡΡ‚ΠΈ ΠΈ Π±Ρ‹Ρ‚Ρƒ. Π‘ΠΈΠ»ΠΎΠ²Ρ‹Π΅ элСктричСскиС трансформаторы Π΄Π°ΡŽΡ‚ Π²ΠΎΠ·ΠΌΠΎΠΆΠ½ΠΎΡΡ‚ΡŒ ΠΏΠ΅Ρ€Π΅Π΄Π°Π²Π°Ρ‚ΡŒ ΠΏΠ΅Ρ€Π΅ΠΌΠ΅Π½Π½Ρ‹ΠΉ Ρ‚ΠΎΠΊΠΎΠΌ линиям элСктропСрСдачи Π½Π° Π±ΠΎΠ»ΡŒΡˆΠΈΠ΅ расстояния с ΠΌΠ°Π»Ρ‹ΠΌΠΈ потСрями энСргии. Для этого напряТСниС ΠΏΠ΅Ρ€Π΅ΠΌΠ΅Π½Π½ΠΎΠ³ΠΎ Ρ‚ΠΎΠΊΠ°, Π²Ρ‹Ρ€Π°Π±Π°Ρ‚Ρ‹Π²Π°Π΅ΠΌΠΎΠ³ΠΎ Π³Π΅Π½Π΅Ρ€Π°Ρ‚ΠΎΡ€Π°ΠΌΠΈ элСктростанции, с ΠΏΠΎΠΌΠΎΡ‰ΡŒΡŽ трансформаторов ΠΏΠΎΠ²Ρ‹ΡˆΠ°ΡŽΡ‚ Π΄ΠΎ Π½Π΅ΡΠΊΠΎΠ»ΡŒΠΊΠΈΡ… сотСн тысяч Π²ΠΎΠ»ΡŒΡ‚ ΠΈ ΠΏΠΎΡΡ‹Π»Π°ΡŽΡ‚ ΠΏΠΎ Π›Π­ΠŸ. Π’ ΠΌΠ΅ΡΡ‚Π΅ потрСблСния напряТСниС ΠΏΠΎΠ½ΠΈΠΆΠ°ΡŽΡ‚ трансформаторами.

Π‘ΠΈΠ»Π΅Ρ‚ 9.

1. 1. Π’Π΅ΠΌΠΏΠ΅Ρ€Π°Ρ‚ΡƒΡ€Π° ΠΈ Π΅Π΅ Ρ„изичСский смысл. Π˜Π·ΠΌΠ΅Ρ€Π΅Π½ΠΈΠ΅ Ρ‚Π΅ΠΌΠΏΠ΅Ρ€Π°Ρ‚ΡƒΡ€Ρ‹. Π˜Π·ΠΌΠ΅Ρ€ΡŒΡ‚Π΅ Ρ‚Π΅ΠΌΠΏΠ΅Ρ€Π°Ρ‚ΡƒΡ€Ρƒ Π² ΠΊΠ»Π°ΡΡΠ΅.

2. 2. ВСрмоэлСктронная эмиссия, Π΅Π΅ ΠΈΡΠΏΠΎΠ»ΡŒΠ·ΠΎΠ²Π°Π½ΠΈΠ΅ Π² ΡΠ»Π΅ΠΊΡ‚Ρ€ΠΎΠ²Π°ΠΊΡƒΡƒΠΌΠ½Ρ‹Ρ… ΠΏΡ€ΠΈΠ±ΠΎΡ€Π°Ρ…. Π­Π»Π΅ΠΊΡ‚Ρ€ΠΎΠ½Π½ΠΎ — лучСвая Ρ‚Ρ€ΡƒΠ±ΠΊΠ°.

3. 3. «Π˜Π—ΠœΠ•Π Π•ΠΠ˜Π• ΠŸΠ›ΠžΠ’ΠΠžΠ‘Π’Π˜ Π’Π’Π•Π Π”ΠžΠ“Πž ВЕЛА».

Π’Π΅Ρ€ΠΌΠΎΠΌΠ΅Ρ‚Ρ€ — это Ρ‚Π΅Π»ΠΎ. ΠšΠΎΡ‚ΠΎΡ€ΠΎΠ΅ находится Π² Ρ‚Π΅ΠΏΠ»ΠΎΠ²ΠΎΠΌ равновСсии с Ρ‚Π΅Π»ΠΎΠΌ, Ρ‚Π΅ΠΌΠΏΠ΅Ρ€Π°Ρ‚ΡƒΡ€Ρƒ ΠΊΠΎΡ‚ΠΎΡ€ΠΎΠ³ΠΎ ΠΈΠ·ΠΌΠ΅Ρ€ΡΡŽΡ‚. ЀизичСский ΠΏΠ°Ρ€Π°ΠΌΠ΅Ρ‚Ρ€, ΠΎΠ΄ΠΈΠ½Π°ΠΊΠΎΠ²Ρ‹ΠΉ Π²ΠΎ Π²ΡΠ΅Ρ… частях систСмы Ρ‚Π΅Π», находящихся Π² ΡΠΎΡΡ‚оянии Ρ‚Π΅ΠΏΠ»ΠΎΠ²ΠΎΠ³ΠΎ равновСсия, Π½Π°Π·Ρ‹Π²Π°ΡŽΡ‚ Ρ‚Π΅ΠΌΠΏΠ΅Ρ€Π°Ρ‚ΡƒΡ€ΠΎΠΉ Ρ‚Π΅Π»Π°. Π’ ΠΏΠΎΠ²ΡΠ΅Π΄Π½Π΅Π²Π½ΠΎΠΉ ΠΏΡ€Π°ΠΊΡ‚ΠΈΠΊΠ΅ ΡˆΠΈΡ€ΠΎΠΊΠΎ распространСн способ измСрСния Ρ‚Π΅ΠΌΠΏΠ΅Ρ€Π°Ρ‚ΡƒΡ€Ρ‹ с ΠΏΠΎΠΌΠΎΡ‰ΡŒΡŽ ΠΆΠΈΠ΄ΠΊΠΎΠ³ΠΎ Ρ‚Π΅Ρ€ΠΌΠΎΠΌΠ΅Ρ‚Ρ€Π°. Π’ Π΅Π³ΠΎ устройствС ΠΈΡΠΏΠΎΠ»ΡŒΠ·ΡƒΠ΅Ρ‚ΡΡ свойство Ρ€Π°ΡΡˆΠΈΡ€Π΅Π½ΠΈΡ Тидкости ΠΏΡ€ΠΈ Π½Π°Π³Ρ€Π΅Π²Π°Π½ΠΈΠΈ. Π’ ΠΊΠ°Ρ‡Π΅ΡΡ‚Π²Π΅ Ρ€Π°Π±ΠΎΡ‡Π΅Π³ΠΎ Ρ‚Π΅Π»Π° ΠΎΠ±Ρ‹Ρ‡Π½ΠΎ ΠΏΡ€ΠΈΠΌΠ΅Π½ΡΡŽΡ‚ Ρ€Ρ‚ΡƒΡ‚ΡŒ, спирт, Π³Π»ΠΈΡ†Π΅Ρ€ΠΈΠ½. Π§Ρ‚ΠΎΠ±Ρ‹ ΠΈΠ·ΠΌΠ΅Ρ€ΠΈΡ‚ΡŒ Ρ‚Π΅ΠΌΠΏΠ΅Ρ€Π°Ρ‚ΡƒΡ€Ρƒ Ρ‚Π΅Π»Π°, Ρ‚Π΅Ρ€ΠΌΠΎΠΌΠ΅Ρ‚Ρ€ приводят Π² ΠΊΠΎΠ½Ρ‚Π°ΠΊΡ‚ с ΡΡ‚ΠΈΠΌ Ρ‚Π΅Π»ΠΎΠΌ; ΠΌΠ΅ΠΆΠ΄Ρƒ Ρ‚Π΅Ρ€ΠΌΠΎΠΌΠ΅Ρ‚Ρ€ΠΎΠΌ ΠΈ Ρ‚Π΅Π»ΠΎΠΌ Π±ΡƒΠ΄Π΅Ρ‚ ΠΎΡΡƒΡ‰Π΅ΡΡ‚Π²Π»ΡΡ‚ΡŒΡΡ Ρ‚Π΅ΠΏΠ»ΠΎΠΏΠ΅Ρ€Π΅Π΄Π°Ρ‡Π° Π΄ΠΎ ΡƒΡΡ‚ановлСния Ρ‚Π΅ΠΏΠ»ΠΎΠ²ΠΎΠ³ΠΎ равновСсия. Масса Ρ‚Π΅Ρ€ΠΌΠΎΠΌΠ΅Ρ‚Ρ€Π° Π΄ΠΎΠ»ΠΆΠ½Π° Π±Ρ‹Ρ‚ΡŒ ΠΌΠ½ΠΎΠ³ΠΎ мСньшС массы Ρ‚Π΅Π»Π°, Ρ‚Π°ΠΊ ΠΊΠ°ΠΊ Π² ΠΏΡ€ΠΎΡ‚ΠΈΠ²Π½ΠΎΠΌ случаС процСсс измСрСния ΠΌΠΎΠΆΠ΅Ρ‚ сущСствСнно ΠΈΠ·ΠΌΠ΅Π½ΠΈΡ‚ΡŒ Ρ‚Π΅ΠΌΠΏΠ΅Ρ€Π°Ρ‚ΡƒΡ€Ρƒ Ρ‚Π΅Π»Π°.

2. 2. ВСрмоэлСктронная эмиссия, Π΅Π΅ ΠΈΡΠΏΠΎΠ»ΡŒΠ·ΠΎΠ²Π°Π½ΠΈΠ΅ Π² ΡΠ»Π΅ΠΊΡ‚Ρ€ΠΎΠ²Π°ΠΊΡƒΡƒΠΌΠ½Ρ‹Ρ… ΠΏΡ€ΠΈΠ±ΠΎΡ€Π°Ρ…. Π­Π»Π΅ΠΊΡ‚Ρ€ΠΎΠ½Π½ΠΎ — лучСвая Ρ‚Ρ€ΡƒΠ±ΠΊΠ°.

Π―Π²Π»Π΅Π½ΠΈΠ΅ Π²Ρ‹Ρ…ΠΎΠ΄Π° элСктронов ΠΈΠ· ΠΌΠ΅Ρ‚Π°Π»Π»Π° ΠΏΡ€ΠΈ Π΅Π³ΠΎ Π½Π°Π³Ρ€Π΅Π²Π°Π½ΠΈΠΈ называСтся тСрмоэлСктронной эмиссиСй. Π­Ρ‚ΠΎ явлСниС ΠΈΡΠΏΠΎΠ»ΡŒΠ·ΡƒΠ΅Ρ‚ΡΡ Π² Ρ€Π°Π·Π»ΠΈΡ‡Π½Ρ‹Ρ… элСктронных ΠΏΡ€ΠΈΠ±ΠΎΡ€Π°Ρ…. ΠŸΡ€ΠΎΡΡ‚Π΅ΠΉΡˆΠΈΠΉ — элСктровакуумный Π΄ΠΈΠΎΠ΄. Π­Ρ‚ΠΎΡ‚ ΠΏΡ€ΠΈΠ±ΠΎΡ€ состоит ΠΈΠ· ΡΡ‚Склянного Π±ΠΎΠ»ΠΎΠ½Π°, Π² ΠΊΠΎΡ‚ΠΎΡ€ΠΎΠΌ находятся Π΄Π²Π° элСктрода: ΠΊΠ°Ρ‚ΠΎΠ΄ ΠΈ Π°Π½ΠΎΠ΄. Анод ΠΈΠ·Π³ΠΎΡ‚ΠΎΠ²Π»Π΅Π½ ΠΈΠ· ΠΌΠ΅Ρ‚алличСской пластины, ΠΊΠ°Ρ‚ΠΎΠ΄ — ΠΈΠ· Ρ‚ΠΎΠ½ΠΊΠΎΠΉ мСталличСской ΠΏΡ€ΠΎΠ²ΠΎΠ»ΠΎΠΊΠΈ, свСрнутой Π² ΡΠΏΠΈΡ€Π°Π»ΡŒ. ΠšΠΎΠ½Ρ†Ρ‹ спирали ΡƒΠΊΡ€Π΅ΠΏΠ»Π΅Π½Ρ‹ Π½Π° ΠΌΠ΅Ρ‚алличСских стСрТнях, ΠΈΠΌΠ΅ΡŽΡ‰ΠΈΡ… Π΄Π²Π° Π²Ρ‹Π²ΠΎΠ΄Π° для ΠΏΠΎΠ΄ΠΊΠ»ΡŽΡ‡Π΅Π½ΠΈΡ Π² ΡΠ»Π΅ΠΊΡ‚Ρ€ΠΈΡ‡Π΅ΡΠΊΡƒΡŽ Ρ†Π΅ΠΏΡŒ. Π‘ΠΎΠ΅Π΄ΠΈΠ½ΠΈΠ² Π²Ρ‹Π²ΠΎΠ΄Ρ‹ ΠΊΠ°Ρ‚ΠΎΠ΄Π° с ΠΈΡΡ‚ΠΎΡ‡Π½ΠΈΠΊΠΎΠΌ Ρ‚ΠΎΠΊΠ°, ΠΌΠΎΠΆΠ½ΠΎ Π²Ρ‹Π·Π²Π°Ρ‚ΡŒ Π½Π°Π³Ρ€Π΅Π²Π°Π½ΠΈΠ΅ ΠΏΡ€ΠΎΠ²ΠΎΠ»ΠΎΡ‡Π½ΠΎΠΉ спирали ΠΊΠ°Ρ‚ΠΎΠ΄Π° проходящим Ρ‚ΠΎΠΊΠΎΠΌ Π΄ΠΎ Π²Ρ‹ΡΠΎΠΊΠΎΠΉ Ρ‚Π΅ΠΌΠΏΠ΅Ρ€Π°Ρ‚ΡƒΡ€Ρ‹. ΠŸΡ€ΠΎΠ²ΠΎΠ»ΠΎΡ‡Π½ΡƒΡŽ ΡΠΏΠΈΡ€Π°Π»ΡŒ Π½Π°Π³Ρ€Π΅Π²Π°Π΅ΠΌΡƒΡŽ элСктричСским Ρ‚ΠΎΠΊΠΎΠΌ, Π½Π°Π·Ρ‹Π²Π°ΡŽΡ‚ Π½ΠΈΡ‚ΡŒΡŽ Π½Π°ΠΊΠ°Π»Π° Π»Π°ΠΌΠΏΡ‹.

Если Π² Π°Π½ΠΎΠ΄Π΅ элСктронной Π»Π°ΠΌΠΏΡ‹ ΡΠ΄Π΅Π»Π°Ρ‚ΡŒ отвСрстиС, Ρ‚ΠΎ Ρ‡Π°ΡΡ‚ΡŒ элСктронов, ускорСнных элСктричСским ΠΏΠΎΠ»Π΅ΠΌ, ΠΏΡ€ΠΎΠ»Π΅Ρ‚ΠΈΡ‚ Π² ΡΡ‚ΠΎ отвСрстиС, образуя Π·Π° Π°Π½ΠΎΠ΄ΠΎΠΌ элСктронный ΠΏΡƒΡ‡ΠΎΠΊ. Π­Π»Π΅ΠΊΡ‚Ρ€ΠΎΠ½Π½Ρ‹ΠΉ ΠΏΡƒΡ‡ΠΎΠΊ, попадая Π½Π° Ρ‚Π΅Π»ΠΎ, Π²Ρ‹Π·Ρ‹Π²Π°Π΅Ρ‚ ΠΈΡ… Π½Π°Π³Ρ€Π΅Π²Π°Π½ΠΈΠ΅. ΠŸΡ€ΠΈ Ρ‚ΠΎΡ€ΠΌΠΎΠΆΠ΅Π½ΠΈΠΈ быстрых элСктронов, Ρ€Π°Π·ΠΎΠ³Π½Π°Π½Π½Ρ‹Ρ… Π΄ΠΎ Π±ΠΎΠ»ΡŒΡˆΠΈΡ… скоростСй элСктричСским ΠΏΠΎΠ»Π΅ΠΌ ΠΈ ΠΏΠΎΠΏΠ°Π΄Π°ΡŽΡ‰ΠΈΡ… Π½Π° Π²Π΅Ρ‰Π΅ΡΡ‚Π²ΠΎ. Π’ΠΎΠ·Π½ΠΈΠΊΠ°Π΅Ρ‚ рСнтгСновскоС ΠΈΠ·Π»ΡƒΡ‡Π΅Π½ΠΈΠ΅. НСкоторыС вСщСства, Π±ΠΎΠΌΠ±Π°Ρ€Π΄ΠΈΡ€ΡƒΠ΅ΠΌΡ‹Π΅ элСктронами свСтятся (Π»ΡŽΠΌΠΈΠ½ΠΎΡ„ΠΎΡ€Ρ‹). Π­Π»Π΅ΠΊΡ‚Ρ€ΠΎΠ½Π½Ρ‹Π΅ ΠΏΡƒΡ‡ΠΊΠΈ ΠΎΡ‚ΠΊΠ»Π°Π½ΡΡŽΡ‚ΡΡ элСктричСскими ΠΈ ΠΌΠ°Π³Π½ΠΈΡ‚Π½Ρ‹ΠΌΠΈ полями. Π­Π»Π΅ΠΊΡ‚Ρ€ΠΎΠ½Π½ΠΎ — лучСвая Ρ‚Ρ€ΡƒΠ±ΠΊΠ° — это Π²Π°ΠΊΡƒΡƒΠΌΠ½Ρ‹ΠΉ элСктронный ΠΏΡ€ΠΈΠ±ΠΎΡ€, ΠΏΠΎΠ·Π²ΠΎΠ»ΡΡŽΡ‰ΠΈΠΉ ΠΏΡ€Π΅ΠΎΠ±Ρ€Π°Π·ΠΎΠ²Ρ‹Π²Π°Ρ‚ΡŒ элСктричСскиС сигналы Π² Π²ΠΈΠ΄ΠΈΠΌΠΎΠ΅ ΠΈΠ·ΠΎΠ±Ρ€Π°ΠΆΠ΅Π½ΠΈΠ΅. Π’ Π΄Π»ΠΈΠ½Π½ΠΎΠΌ Π±Π°Π»Π»ΠΎΠ½Π΅ Ρ‚Ρ€ΡƒΠ±ΠΊΠΈ создан высокий Π²Π°ΠΊΡƒΡƒΠΌ. Π’Π½ΡƒΡ‚Ρ€ΠΈ Π±Π°Π»Π»ΠΎΠ½Π° имССтся систСма элСктродов. ΠŸΠΎΠ·Π²ΠΎΠ»ΡΡŽΡ‰Π°Ρ ΠΏΠΎΠ»ΡƒΡ‡Π°Ρ‚ΡŒ ΠΎΡ‡Π΅Π½ΡŒ Ρ‚ΠΎΠ½ΠΊΠΈ ΠΈ ΠΎΡ‡Π΅Π½ΡŒ Π΄Π»ΠΈΠ½Π½Ρ‹ΠΉ ΠΏΡƒΡ‡ΠΎΠΊ элСктронов. Π­Ρ‚Ρƒ ΡΠΎΠ²ΠΎΠΊΡƒΠΏΠ½ΠΎΡΡ‚ΡŒ элСктродов Π½Π°Π·Ρ‹Π²Π°ΡŽΡ‚ элСктронной ΠΏΡƒΡˆΠΊΠΎΠΉ (ΠΏΡ€ΠΎΠΆΠ΅ΠΊΡ‚ΠΎΡ€ΠΎΠΌ). Она состоит ΠΈΠ· ΠΏΠΎΠ΄ΠΎΠ³Ρ€Π΅Π²Π°Π΅ΠΌΠΎΠ³ΠΎ ΠΊΠ°Ρ‚ΠΎΠ΄Π°, ΡƒΠΏΡ€Π°Π²Π»ΡΡŽΡ‰Π΅Π³ΠΎ элСктрода, ΠΏΠ΅Ρ€Π²ΠΎΠ³ΠΎ ΠΈ Π²Ρ‚ΠΎΡ€ΠΎΠ³ΠΎ Π°Π½ΠΎΠ΄ΠΎΠ². ΠšΠ°Ρ‚ΠΎΠ΄ прСдставляСт собой ΡƒΠ·ΠΊΠΈΠΉ Ρ†ΠΈΠ»ΠΈΠ½Π΄Ρ€, Π²Π½ΡƒΡ‚Ρ€ΠΈ ΠΊΠΎΡ‚ΠΎΡ€ΠΎΠ³ΠΎ находится Π½Π°Π³Ρ€Π΅Π²Π°Ρ‚Π΅Π»ΡŒ. Π‘Π½Π°Ρ€ΡƒΠΆΠΈ ΠΊΠ°Ρ‚ΠΎΠ΄ ΠΏΠΎΠΊΡ€Ρ‹Ρ‚ ΡΠΏΠ΅Ρ†ΠΈΠ°Π»ΡŒΠ½Ρ‹ΠΌ вСщСством с ΠΌΠ°Π»ΠΎΠΉ Ρ€Π°Π±ΠΎΡ‚ΠΎΠΉ Π²Ρ‹Ρ…ΠΎΠ΄Π° элСктронов. Π£ΠΏΡ€Π°Π²Π»ΡΡŽΡ‰ΠΈΠΉ элСктродом ΠΏΡ€Π΅Π΄Π½Π°Π·Π½Π°Ρ‡Π΅Π½ для Ρ€Π΅Π³ΡƒΠ»ΠΈΡ€ΠΎΠ²ΠΊΠΈ интСнсивности элСктронного ΠΏΡƒΡ‡ΠΊΠ°. Он ΠΈΠΌΠ΅Π΅Ρ‚ Ρ†ΠΈΠ»ΠΈΠ½Π΄Ρ€ΠΈΡ‡Π΅ΡΠΊΡƒΡŽ Ρ„ΠΎΡ€ΠΌΡƒ ΠΈ ΠΎΠΊΡ€ΡƒΠΆΠ°Π΅Ρ‚ ΠΊΠ°Ρ‚ΠΎΠ΄. Π§Π΅Ρ€Π΅Π· отвСрстиС Π² ΠΎΡΠ½ΠΎΠ²Π°Π½ΠΈΠΈ этого Ρ†ΠΈΠ»ΠΈΠ½Π΄Ρ€Π° ΠΏΡ€ΠΎΠ»Π΅Ρ‚Π°ΡŽΡ‚ элСктроны. Π˜ΡΠΏΡƒΡΠΊΠ°Π΅ΠΌΡ‹Π΅ ΠΊΠ°Ρ‚ΠΎΠ΄ΠΎΠΌ. На ΡƒΠΏΡ€Π°Π²Π»ΡΡŽΡ‰ΠΈΠΉ элСктрод ΠΏΠΎΠΏΠ°Π΄Π°Π΅Ρ‚ нСбольшой ΠΎΡ‚Ρ€ΠΈΡ†Π°Ρ‚Π΅Π»ΡŒΠ½Ρ‹ΠΉ ΠΏΠΎΡ‚Π΅Π½Ρ†ΠΈΠ°Π». ИзмСняя ΠΏΠΎΡ‚Π΅Π½Ρ†ΠΈΠ°Π» ΡƒΠΏΡ€Π°Π²Π»ΡΡŽΡ‰Π΅Π³ΠΎ элСктрода ΠΈ, ΡΠ»Π΅Π΄ΠΎΠ²Π°Ρ‚Π΅Π»ΡŒΠ½ΠΎ, измСняя ΡΡ€ΠΊΠΎΡΡ‚ΡŒ пятна Π½Π° ΡΠΊΡ€Π°Π½Π΅. Π’Π°ΠΊΠΆΠ΅ ΠΈΠΌΠ΅ΡŽΡ‚ΡΡ Π²Π΅Ρ€Ρ‚ΠΈΠΊΠ°Π»ΡŒΠ½Ρ‹Π΅ ΠΈ Π³ΠΎΡ€ΠΈΠ·ΠΎΠ½Ρ‚Π°Π»ΡŒΠ½Ρ‹Π΅ ΠΎΡ‚ΠΊΠ»ΠΎΠ½ΡΡŽΡ‰ΠΈΠ΅ пластины.

ΠŸΠΎΠΊΠ°Π·Π°Ρ‚ΡŒ вСсь тСкст
Π—Π°ΠΏΠΎΠ»Π½ΠΈΡ‚ΡŒ Ρ„ΠΎΡ€ΠΌΡƒ Ρ‚Π΅ΠΊΡƒΡ‰Π΅ΠΉ Ρ€Π°Π±ΠΎΡ‚ΠΎΠΉ